A expressão 1 cossec x e o mesmo que


2738   

$z = 2x + y$ é a equação do plano tangente ao gráfico de $f(x,y)$ no ponto $(1,1,3)$. Calcule $\dfrac{\partial f}{\partial x}(1,1)$ e $\dfrac{\partial f}{\partial y}(1,1).$


$\displaystyle \frac{\partial f}{\partial x} (1,1) = 2$ e  $\displaystyle \frac{\partial f}{\partial y} (1,1) = 1.$


1979   

Determine o domínio da curva de equação vetorial
$$\textbf{r}(t) = \left( \sqrt{\dfrac{t - 2}{t + 1}}, \ln{(5 - t^2)}, e^{-t} \right).$$


2483   

Use a integral tripla para determinar o volume do sólido dado.

  1.  $x^{2}+y^{2}\leq 1$ e $x^{2}+z^{2}\leq 1.$

  2.  $(x-a)^{2}+y^{2}\leq a^{2}$, $x^{2}+y^{2}+z^{2}\leq 4a^{2}$, $z\geq 0$ $(a>0).$

  3.  $x^{2}+y^{2}\leq a^{2}$ e $x^{2}+z^{2}\leq a^{2}$ $(a>0).$

  4.  $x^{2}+y^{2}+z^{2}\leq a^{2}$ e $z\geq \dfrac{a}{2}$ $(a>0).$


  1.  $\dfrac{16}{3}.$

  2.  $\dfrac{16a^3}{3} \left(\dfrac{\pi}{2} - \dfrac{2}{3}\right).$

  3.  $\dfrac{16a^3}{3}.$

  4.  $\dfrac{5\pi a^3}{24}.$


3039   

Esboce a região cuja área é dada pela integral e calcule-a, sendo: $\displaystyle\int_{0}^{\pi/2}\int_{0}^{4\cos{\theta}}   r \,drd\theta.$


$2\pi;$ região de integração:

A expressão 1 cossec x e o mesmo que


2166   

Use o Teorema de Green para calcular $\int_{C}\mathbf{F} \cdot d\mathbf{r}$, onde $\mathbf{F}(x,y) = (2x+y)\mathbf{i} + (3x-y)\mathbf{j}$, $C$ é uma curva fechada, simples, $C^1$ por partes, orientada no sentido positivo, cuja imagem é a fronteira de um compacto $B$ com área $\alpha$. (Verifique a orientação da curva antes de aplicar o Teorema.)


$2\times$(Área de $B$).


2823   

Determine os valores máximo e mínimo absolutos de $f$ no conjunto $D.$

$f(x,y)=x^{2}-2xy+2y^{2}$ em $D=\{(x,y)\in \mathbb{R}^2: \;|x|+|y|\leq 1\}.$


Valor máximo: $\displaystyle  2;$ valor mínimo: $0.$


2233   

Seja $\mathbf{r} = x\mathbf{i} + y\mathbf{j} + z\mathbf{k}$ e $r=|\mathbf{r}|$. Verifique a identidade $\nabla^2r^3=12r$.


$\nabla^2r^3= \dfrac{\partial}{\partial x} \left[ \dfrac{3}{2} \sqrt{x^{2} + y^{2} + z^{2}} (2x) \right] + \dfrac{\partial}{\partial y} \left[ \dfrac{3}{2} \sqrt{x^{2} + y^{2} + z^{2}} (2y) \right]\\ + \dfrac{\partial}{\partial z} \left[ \dfrac{3}{2} \sqrt{x^{2} + y^{2} + z^{2}} (2z) \right].$ (Note que: $r = \sqrt{x^{2} + y^{2} + z^{2}}.$)


2389   

Determine as equações do plano tangente e da reta normal à superfície dada, no ponto dado.

$x^2 - 2y^2 + z^2 + yz = 2$ em $(2,1,-1).$


Plano tangente: $4x - 5y - z = 4$,
Reta normal: $(x,y,z) = (2,1,-1) + \lambda (4,-5,-1),$ $\lambda \in \mathbb{R}.$


2195   

Seja ${\bf F}=(z tg^{-1}(y^{2}),z^{3}\ln(x^{2}+1),z).$ Determine o fluxo de ${\bf F}$ através da parte do parabolóide $x^{2}+y^{2}+z=2$ que está acima do plano $z=1$ e está orientada para cima. (Observe que a superfície acima não é fechada.)


2140   

Demonstre a identidade $\displaystyle\iint\limits_{S}\mbox{rot}\, {\bf F}\cdot dS=0$, supondo que $S$ e $E$ satisfaçam as condições do Teorema do Divergente e que as funções escalares e as componentes dos campos vetoriais tenham derivadas parciais de segunda ordem contínuas.



Pelo Teorema do Divergente, temos
$\displaystyle\iint\limits_{ S}\mbox{rot} {\bf F}\cdot dS = \iiint\limits_{ E}\mbox{div} (\mbox{rot} {\bf F})\,dV,$
em que $E$ é o sólido que tem $S$ como fronteira. Observe que
\begin{align*}
&\mbox{div} (\mbox{rot} {\bf F})  =\\ & \frac{\partial}{\partial x}(R_y - Q_z) + \frac{\partial}{\partial y}(P_z - R_x) + \frac{\partial}{\partial z}(Q_x - P_y) \\ & R_{xy} - Q_{xz} + P_{yz} - R_{yx} + Q_{zx} - P_{zy} = 0,
\end{align*}
pois, como as derivadas de segunda ordem são contínuas, temos, pelo Teorema de Clairaut, que $P_{yz} = P_{zy}$, $Q_{zx} = Q_{xz}$ e $R_{xy} = R_{yx}$. Portanto,

$\displaystyle\iint\limits_{S}\mbox{rot}{\bf F}\cdot dS=0.$


2758   

Verifique que a função $f(x,y) = \ln{(1 + x^2 + y^2)}$ é diferenciável.


As derivadas parciais $\frac{\partial f}{\partial x}$ e $\frac{\partial f}{\partial y}$ de cada função $f$ existem e são contínuas em todos os pontos do domínio.


3115   

Seja \(G\) a caixa retangular definida pelas desigualdades \(a\leq x\leq b\),  \(c\leq y\leq d\) e \(k\leq z\leq l\). Mostre que \[\iiint\limits_G f(x)g(y)h(z)\,dV = \left[\int_a^bf(x)\,dx\right]\left[\int_c^dg(y)\,dy\right]\left[\int_k^lh(z)\,dz\right].\]


2105   

Suponha que ${\bf F}$ seja um campo vetorial inverso do quadrado, ou seja,

$${\bf F}({\bf r})=\frac{c{\bf r}}{|{\bf r}|^{3}}$$

para alguma constante $c$, onde ${\bf r}=x\,{\bf i}+y\,{\bf j}+z\,{\bf k}.$ Determine o trabalho realizado por ${\bf F}$ ao mover um objeto de um ponto $P_{1}$ por um caminho para um ponto $P_{2}$ em termos da distância $d_{1}$ e $d_{2}$ desses pontos à origem.


$c\left(\dfrac{1}{d_{1}} - \dfrac{1}{d_{2}}\right).$


2241   

As equações de Maxwell relacionam o campo elétrico $\mathbf{E}$ e o campo magnético $\mathbf{H}$, quando eles variam com o tempo em uma região que não contenha carga nem corrente, como segue:

$$\text{div }{\mathbf{E}} = 0,\text{     }\text{div }{\mathbf{H}} = 0$$

$$\text{rot }{\mathbf{E}} = -\dfrac{1}{c}\dfrac{\partial\mathbf{H}}{\partial t},\text{     }\text{rot }{\mathbf{H}}  =  \dfrac{1}{c}\dfrac{\partial\mathbf{E}}{\partial t},$$

em que $c$ é a velocidade da luz. Use essas equações para demonstrar o seguinte:

  1. $\nabla \times (\nabla \times \mathbf{E}) = - \dfrac{1}{c^2}\dfrac{\partial^2\mathbf{E}}{\partial t^2}$,

  2. $\nabla \times (\nabla \times \mathbf{H}) = - \dfrac{1}{c^2}\dfrac{\partial^2\mathbf{H}}{\partial t^2}$,

  3. $\nabla^2{\mathbf{E}} =  \dfrac{1}{c^2}\dfrac{\partial^2\mathbf{E}}{\partial t^2}$,

  4. $\nabla^2{\mathbf{H}} =  \dfrac{1}{c^2}\dfrac{\partial^2\mathbf{H}}{\partial t^2}$.


  1. $\nabla \times (\nabla \times \mathbf{E}) = \nabla \times (\text{rot } \mathbf{E}) = \nabla \times \left( -\dfrac{1}{c} \dfrac{\partial \mathbf{H}}{\partial t} \right) = -\dfrac{1}{c} \dfrac{\partial}{\partial t} \text{rot } \mathbf{H} = -\dfrac{1}{c} \dfrac{\partial}{\partial t} \left(\dfrac{1}{c} \dfrac{\partial \mathbf{E}}{\partial t} \right)$

  2. Análogo ao item 1.

  3. Note que $\nabla^2{\mathbf{E}} =  \nabla \text{div } \mathbf{E} - \text{rot } \text{rot }(\mathbf{E}).$

  4. Análogo ao item 3.


2242   

Calcule $\int_{C}\mathbf{F} \cdot \mathbf{n} \, ds$ ($\mathbf{n}$ é unitário, onde $\mathbf{F}(x,y) = x\mathbf{i} + y\mathbf{j}$, $C$ dada por $\mathbf{r}(t) = (\cos{t},\sin{t})$, $0 \leq t \leq 2\pi$ e $\mathbf{n}$ a normal exterior.


$2\pi.$


2714   

Determine uma equação do plano tangente à superfície no ponto especificado.

$z = \sqrt{xy}, \quad (1,1,1)$.


$x + y - 2z = 0$.


2769   

Considere a superfície dada implicitamente por

$$x^{2}+2y^{2}+2z^{2}=-4xyz.$$

  1. Calcule as derivadas $\dfrac{\partial z}{\partial x}$ e $\dfrac{\partial z}{\partial y}$ em um ponto genérico.

  2. Quais os pontos nos quais as derivadas parciais calculadas no item anterior não estão definidas?


  1. $\displaystyle \frac{\partial z}{\partial x} = -\frac{x + 2yz}{2(z + xy)} \;\;\;\text{e}\;\;\; \frac{\partial z}{\partial y} = -\frac{y + xz}{z + xy}.$

  2. $\left\lbrace (x,y,z) \in \mathbb{R}^{3};\; z = -xy \right\rbrace$.


3155   

A fórmula de Taylor de primeira ordem para $f(\vec{a} + \vec{v})$ pode ser escrita como $ f(\vec{a}) + \nabla f(\vec{a}) \cdot \vec{v}$, já desconsiderando o termo de erro. Calcule-a para $f(x,y) = x^2/2 + y$, $\vec{a} = (0,0)$ e $\vec{v} = (1/2,1/2)$. Calcule também o erro cometido, dizendo se é um erro pequeno ou grande e por quê.


2453   

Integre $g(x,y,z)=x+y+z$ sobre a porção do plano $2x+2y+z=2$ que está no primeiro octante.


$2.$


2199   

 Se $f(u,v,w)$ é diferenciável, $u=x-y$, $v=y-z$ e $w=z-x$, mostre que 
$$\frac{\partial f}{\partial x}+\frac{\partial f}{\partial y}+\frac{\partial f}{\partial z}=0.$$


Note que $\displaystyle \frac{\partial f}{\partial x} = \frac{\partial f}{\partial u} - \frac{\partial f}{\partial w}, $$\displaystyle \frac{\partial f}{\partial y} = -\frac{\partial f}{\partial u} + \frac{\partial f}{\partial v}$ e $\displaystyle \frac{\partial f}{\partial z} = -\frac{\partial f}{\partial v} + \frac{\partial f}{\partial w}.$


2552   

Explique por que cada função é contínua ou descontínua.

  1. A temperatura externa como função da latitude, da longitude e do tempo.

  2. A altura acima do nível do mar como função da longitude, da latitude e do tempo.

  3. O custo da tarifa do táxi como função da distância percorrida e do tempo gasto.


  1. Contínua.

  2. Descontínua.

  3. Descontínua.


3053   

Esboce o sólido cujo volume é dado pela integral iterada.

$\displaystyle\int_{0}^{1}\int_{0}^{1-x}\int_{0}^{2-2z}\;dy dz dx$


A expressão 1 cossec x e o mesmo que


2334   

Determine a taxa de variação máxima de $f$ no ponto dado e a direção em que isso ocorre.

$f(x,y,z) = \tan{(x + 2y + 3z)},  (-5,1,1).$


$\sqrt{14}.$


3136   

Em 1831, o físico Michael Faraday descobriu que uma corrente elétrica pode ser produzida variando-se o fluxo magnético através de um arco condutor. Suas experiências mostraram que a força eletromotriz \(\mathbf{E}\) está relacionada com a indução magnética \(\mathbf{B}\) pela equação \[ \oint_C\mathbf{E}\cdot\,d\mathbf{r} = - \iint\limits_\sigma\dfrac{\partial\mathbf{B}}{\partial t}\cdot\mathbf{n}\,dS.\] Use este resultado para fazer uma conjectura acerca da relação entre \(\mathrm{rot\,}\mathbf{E}\) e \(\mathbf{B}\). Explique seu raciocínio.


2706   

Se $z=\sin(x+\sin{y})$, mostre que $\dfrac{\partial z}{\partial x} \;\dfrac{\partial^{2} z}{\partial x \partial y}=\dfrac{\partial z}{\partial y}\;\dfrac{\partial^{2}z}{\partial x^{2}}$.


$\begin{aligned}[t]\frac{\partial z}{\partial x} &= \cos(x + \sin y),\;\;\; \frac{\partial z}{\partial y} = \cos(x + \sin y) \cos y,\\\frac{\partial z^{2}}{\partial x\partial y} &= -\sin (x + \sin y) \cos y\;\;\text{e}\;\; \frac{\partial^{2} z}{\partial x^{2}} = -\sin (x + \sin y).\end{aligned}$


2679   

Seja $s = f(x,y,z,w)$ dada por $s = e^{\frac{x}{y} - \frac{z}{w}}$. Verifique que

$$x\dfrac{\partial s}{\partial x} + y \dfrac{\partial s}{\partial y} + z \dfrac{\partial s}{\partial z} + w \dfrac{\partial s}{\partial w} = 0.$$


$\begin{aligned}[t]\frac{\partial s}{\partial x} &= \frac{1}{y} e^{\frac{x}{y} - \frac{z}{w}},\;\;\;\;\;\frac{\partial s}{\partial y} = -\frac{x}{y^{2}} e^{\frac{x}{y} - \frac{z}{w}},\\\frac{\partial s}{\partial z} &= -\frac{1}{w} e^{\frac{x}{y} - \frac{z}{w}}\;\;\;\;\;\text{e}\;\;\;\;\; \frac{\partial s}{\partial w} = \frac{z}{w^{2}} e^{\frac{x}{y} - \frac{z}{w}}.\end{aligned}$


2085   

Calcule $\displaystyle\int_{(-1,0)}^{(1,0)}\dfrac{x}{x^{2}+y^{2}}\,dx+\dfrac{y}{x^{2}+y^{2}}\,dy$.


$\displaystyle \dfrac{\pi}{4} + \arctan\left( \dfrac{2}{3} \right).$


2991   

Calcule a integral, efetuando uma mudança de variáveis apropriada. $\displaystyle\iint\limits_{R} \sin{(4x^2 + y^2)} \, dA$, em que $R$ é o cojunto de todos $(x,y)$ tais que $4x^2 + y^2 \leq 1$ e $y \geq 0$.


$\dfrac{\pi}{4}(1 - \cos(1)).$


2897   

Determine o plano tangente à superfície $\frac{x^2}{4} + \frac{y^2}{9} + \frac{z^2}{16} = 1$, com $x > 0$, $y > 0$ e $z > 0$, que forma com os planos coordenados um tetraedro de volume mínimo. (Dica: O volume do tetraedro formado pelos planos coordenados e o plano $ax + by + cz = d$ no primeiro octante é dado por $V = d^3/(6abc)$.)


$6x + 4y + 3z = 12\sqrt{3}.$


2321   

  1. Determine uma representação paramétrica ${\bf r}:D\subset \mathbb{R}^{2}\rightarrow \mathbb{R}^{3}$ do paraboloide elíptico $z=\dfrac{x^{2}}{a^{2}}+\dfrac{y^{2}}{b^{2}}.$

  2. Calcule a equação do plano tangente à superfície paramétrica dada no item (a) no ponto $(-a\pi,0,\pi^{2}).$


  1. $x = u,$ $y = v,$ $z = \dfrac{u^{2}}{a^{2}}+\dfrac{v^{2}}{b^{2}},$ onde $u,v \in \mathbb{R}.$

  2. $2\pi(x + a\pi) + a(z - \pi^{2}) = 0.$


3081   

Verifique que a função \(\displaystyle u(x,t)=\sin(x-ct)\) é uma solução da equação da onda unidimensional \[ \dfrac{\partial^2u}{\partial t^2} = c^2\dfrac{\partial^2u}{\partial x^2}, \] onde \(c\) é uma constante que depende das características da onda.



Calculando diretamente as derivadas parciais da função dada, temos

\[\begin{array}{ll} \dfrac{\partial u}{\partial x} = \cos(x-ct),   & \dfrac{\partial^2u}{\partial x^2}= -\sin(x-ct) \\ \dfrac{\partial u}{\partial t} = -c\cos(x-ct), & \dfrac{\partial^2 u}{\partial t^2}= -c^2\sin(x-ct). \end{array}\] Assim, podemos ver que \(u(x,t)\) satisfaz a equação dada.


2236   

Seja $\mathbf{r} = x\mathbf{i} + y\mathbf{j} + z\mathbf{k}$ e $r=|\mathbf{r}|$. Verifique a identidade $\nabla\left(\dfrac{1}{r}\right) = -\dfrac{\mathbf{r}}{r^3}$.


$\nabla\left(\dfrac{1}{r}\right) =- \dfrac{\frac{1}{2\sqrt{x^2 + y^2 + z^2}} (2x)}{x^2 + y^2 + z^2} \mathbf{i} - \dfrac{-\frac{1}{2\sqrt{x^2 + y^2 + z^2}} (2y)}{x^2 + y^2 + z^2} \mathbf{j} - \dfrac{-\frac{1}{2\sqrt{x^2 + y^2 + z^2}} (2z)}{x^2 + y^2 + z^2} \mathbf{k}.$ (Note que: $r = \sqrt{x^{2} + y^{2} + z^{2}}.$)


2205   

Suponha que $w=f(x,y)$ é diferenciável e que exista uma constante $\alpha$ tal que 
$x=u\cos(\alpha)-v\sin(\alpha)$
$y=u\sin(\alpha)+v\cos(\alpha).$
Mostre que 
$$\bigg(\frac{\partial w}{\partial u}\bigg)^{2}+\bigg(\frac{\partial w}{\partial v}\bigg)^{2}=\bigg(\frac{\partial w}{\partial x}\bigg)^{2}+\bigg(\frac{\partial w}{\partial y}\bigg)^{2}.$$


Note que $\displaystyle \frac{\partial w}{\partial u} = \cos(\alpha) \frac{\partial w}{\partial x} + \sin(\alpha) \frac{\partial w}{\partial y}$ e $\displaystyle \frac{\partial w}{\partial v} = -\sin(\alpha) \frac{\partial w}{\partial x} + \cos(\alpha) \frac{\partial w}{\partial y}.$


2064   

Dados ${\bf F}(x,y)=xy^{2}\,{\bf i}+x^{2}y\,{\bf j}$, $C: {\bf r}(t)=(t+\sin\frac{1}{2}\pi t, t+\cos \frac{1}{2}\pi t)$, $0\leq t\leq 1.$

  1. Determine uma função $f$ tal que ${\bf F}=\nabla f$.

  2. Use o resultado anterior para calcular $\int_{C}{\bf F}\cdot d{\bf r}$ sobre a curva $C$ dada.


  1. $f(x,y) = \dfrac{x^{2}y^{2}}{2};$

  2. $2.$


3005   

Determine os momentos de inércia da lâmina que ocupa a região $D$ e tem função densidade $\rho$ quando: $D$ é a região triangular delimitada pelas retas $x = 0, \ y = x$ e   $2x + y = 6; \quad \rho(x,y) = x^2$.


$\displaystyle I_{x} = \dfrac{1}{16}(e^4 - 1),$ $I_{y} = \dfrac{1}{16}(e^4 - 1)$ e $I_{0} = \dfrac{1}{16}(e^4 + 2e^2 - 3).$


2535   

Ache o centro de massa de $E$, em que:

  1.  A densidade de um ponto $P$ de um sólido cúbico $E$ de aresta $a$ é diretamente proporcional ao quadrado da distância de $P$ a um vértice fixo do cubo.

  2.  $E$ é o tetraedro delimitado pelos planos coordenados e o plano  $2x+5y+z=10$ e a densidade em $P(x,y,z)$ é diretamente proporcional $\grave{a}$ distância do plano $xz$ a $P.$


  1.  $\displaystyle \left( \dfrac{7a}{12},\dfrac{7a}{12},\dfrac{7a}{12} \right).$

  2.  $\displaystyle \left( 1,\dfrac{4}{5},2 \right).$


2609   

Use o Teorema de Stokes para calcular $\displaystyle\int_C {\bf F}\cdot d{\bf R}$. $C$ é orientada no sentido anti-horário quando vista de cima.

  • ${\bf F}(x,y,z) = (x+y^2){\bf i} + (y+z^2){\bf j} + (z+x^2){\bf k}$, $C$ é o triângulo com vértices $(1,0,0)$, $(0,1,0)$, $(0,0,1)$.


$1$.


1976   

Trace a curva com equações paramétricas
\begin{eqnarray}
x & = & \sqrt{1 - 0,25 \cos^210t} \cos{t} \nonumber \\
y & = & \sqrt{1 - 0,25 \cos^210t} \sin{t} \nonumber \\
z & = & 0,5 \cos{10t}. \nonumber
\end{eqnarray} 
Explique a aparência da curva, mostrando que ela está em uma esfera.


2311   

Determine a derivada direcional da função no ponto dado e na direção do vetor $\bf{v}$.

$f(x,y) = 1 + 2x\sqrt{y},  (3,4),  \bf{v} = \left(4, -3\right).$


$\displaystyle \frac{23}{10}.$


2721   

Determine a diferencial da função $m = p^5q^3$.


$dm = 5p^{4}q^{3} dp + 3p^{5}q^{2} dq$.


2408   

Inverta a ordem de integração.

  1. $\displaystyle\int_{0}^{1}\bigg[\int_{e^{y-1}}^{e^{y}}f(x,y)\,dx\bigg]dy$

  2.  $\displaystyle\int_{0}^{1}\bigg[\int_{2x}^{x+1}f(x,y)\,dy\bigg]dx$

  3. $\displaystyle\int_{0}^{\frac{\pi}{4}}\bigg[\int_{0}^{\tan(x)}f(x,y)\,dy\bigg]dx$


  1.  $\displaystyle\int_{e^{-1}}^{1}\bigg[\int_{0}^{1 + \ln(x)}f(x,y) \ , dy\bigg]dx + \displaystyle\int_{1}^{e}\bigg[\int_{\ln(x)}^{1}f(x,y)\,dy\bigg]dx$

  2.  $\displaystyle\int_{0}^{1}\bigg[\int_{0}^{y/2}f(x,y)\,dx\bigg]dy + \int_{1}^{2}\bigg[\int_{y - 1}^{y/2}f(x,y)\,dx\bigg]dy$

  3.  $\displaystyle \int_{0}^{1}\bigg[\int_{0}^{\arctan(y)}f(x,y)\,dx \bigg]dy $


2828   

Determine os pontos do cone $z^{2}=x^{2}+y^{2}$ que estão mais próximos do ponto $(4,2,0).$


$(2,1,\sqrt{5})$ e $(2,1,-\sqrt{5}).$


2243   

Calcule $\int_{C}\mathbf{F} \cdot \mathbf{n} \, ds$ ($\mathbf{n}$ é unitário), onde $\mathbf{F}(x,y) = y\mathbf{j}$, $C$ a fronteira do quadrado de vértices $(0,0)$, $(1,0)$, $(1,1)$, $(0,1)$ e $\mathbf{n}$ a normal que aponta para fora do quadrado, sendo $C$ orientada no sentido anti-horário.


$1.$


2529   

Represente graficamente o domínio da função $z=f(x,y)$ dada por $f(x,y)=\dfrac{x-y}{\sqrt{1-x^{2}-y^{2}}}$.


$\left\lbrace (x,y); x^{2} + y^{2} < 1 \right\rbrace$

A expressão 1 cossec x e o mesmo que


2857   

Seja

$$f(x,y)=k(x-y)^{2}+\frac{y^{4}}{2}-\frac{y^{2}}{2},\;\;k\neq 0.$$

  1. Encontre os pontos críticos da função $f$.

  2. Classifique os pontos críticos da função $f$ no caso em que $k>0$.


  1. $(0,0), (1,1)$ e $(-1,-1).$

  2. Pontos de mínimo: $(1,1)$ e $(-1,-1);$ ponto de sela: $(0,0).$


2350   

Determine o valor médio de $f(x,y)=e^{y}\sqrt{x+e^{y}}$ sobre o retângulo  $R=[0,4]\times [0,1].$


$\dfrac{(4 + e)^{5/2} - e^{5/2} - 5^{5/2} + 1}{15}.$


2789   

Determine os valores máximos e mínimos locais e pontos de sela da função $f(x,y)=9-2x+4y-x^{2}-4y^{2}$.


Ponto de máximo: $\displaystyle \left( -1, \frac{1}{2} \right).$


3099   

Mostre (verifique) que as integrais abaixo podem ser calculadas como:


1. \[ \int_1^5\int_2^{y/2}6x^2y\,dxdy = \int_1^5\left(\dfrac{1}{4}y^4-16y\right)\,dy \]


2. \[ \int_1^5\int_2^{x/2}6x^2y\,dydx = \int_1^5\left(\dfrac{3}{4}x^4-12x^2\right)\,dx \]


2914   

Considere a integral dada em coordenadas polares por $$\int_{0}^{\pi/4}   \int_{0}^{2\cos{\theta}}r\,dr d\theta,$$ a qual representa a área de uma região $R$ do plano $xy.$

  1. Escreva a região $R$ em coordenadas cartesianas.

  2. Faça um esboço da região $R.$

  3. Calcule a área da região $R.$


  1.  $R = \left\lbrace (x,y);   (x - 1)^2 + y^2 \leq 1,\quad x \leq y,\quad x \geq 0,\quad y \geq 0 \right\rbrace.$

  2. (...)

  3.  $\dfrac{\pi + 2}{4}.$


2019   

Uma piscina de 8 por 12 metros está cheia de água. A profundidade é medida em intervalos de 2 metros, começando em um canto da piscina, e os valores foram registrados na tabela. Estime o volume de água na piscina.

$$ \begin{array}{|c|c|c|c|c|c|c|c|} \hline  & 0 & 2   & 4   & 6   & 8   & 10  & 12  \\ \hline  0      & 1 & 1,5  & 2   & 2,4 & 2,8 & 3   & 3   \\ 2       & 1 & 1,5 & 2   & 2,8 & 3   & 3,6 & 3   \\ 4        & 1 & 1,8 & 2,7 & 3   & 3,6 & 4   & 3,2 \\ 6        & 1 & 1,5 & 2   & 2,3 & 2,7 & 3   & 2,5 \\   8     & 1 & 1   & 1   & 1   & 1,5 & 2   & 2   \\ \hline\end{array}$$


$\approx 227.$


3124   

  1.  Mostre que se \(R\) for a região triangular de vértices \((0,0)\), \((1,0)\) e  \((0,1)\), então   \[\iint\limits_R f(x+y)\,dA = \int_0^1 uf(u)\,du.\]

  2.  Use o resultado anterior para calcular a integral \[ \iint\limits_R e^{x+y}\,dA. \]


2130   

$f(t)$ e $g(x,y)$ são funções diferenciáveis tais que $g(t,f(t))=0$ para todo $t$. Suponha $f(0)=1$, 
$\dfrac{\partial g}{\partial x}(0,1)=2$ e $\dfrac{\partial g}{\partial y}(0,1)=4$. Determine a equação da reta tangente a $\gamma(t)=(t,f(t))$, 
no ponto $\gamma(0).$


$\displaystyle (x,y) = (0,1) + \lambda \left(1, - \frac{1}{2}\right),$ $\lambda \in \mathbb{R}.$


3102   

A reta \(y=2-x\) intersecta a parábola \(y=x^2\) nos pontos \((-2,4)\) e \((1,1)\). Mostre que, se \(R\) denotar a região englobada por \(y=2-x\) e \(y=x^2\), então \[ \iint_R\left(1+2y\right)\,dA = \int_{-2}^1\int_{x^2}^{2-x}\left(1+2y\right)\,dydx = 18,9 \]


2207   

Calcule as integrais iteradas.

  1. $\displaystyle\int_{0}^{\pi/2}\int_{0}^{\cos{\theta}}e^{\sin{\theta}}\,dr d\theta$

  2. $\displaystyle\int_{0}^{1}\int_{0}^{v}\sqrt{1-v^{2}}\,du dv$


  1. $e - 1.$

  2.  $\dfrac{1}{3}.$


2384   

Calcule a integral iterada.

  1.  $\displaystyle\int_{0}^{1}\!\!\int_{0}^{1}(u-v)^{5}\,du dv$

  2. $\displaystyle\int_{0}^{2}\!\!\int_{0}^{\pi}r\sin^{2}{\theta}\,d\theta dr$


  1. $0.$

  2. $\pi.$


2542   

Encontre o fluxo do campo ${\bf F}$ ao longo da porção da superfície dada no sentido especificado.

  • ${\bf F}(x,y,z)=yx^{2}{\bf i}-2{\bf j}+xz{\bf k}$; $S$ é a superfície retangular $y=0$, $-1\leq x \leq 2$, $2\leq z \leq 7$, sentido $-{\bf j}.$


$30.$


2416   

Determine o volume do sólido descrito abaixo.

  1.  Limitado pelo cilindro $x^{2}+y^{2}=1$ e pelos planos $y=z$, $x=0$ e $z=0$, no primeiro octante.

  2.  Cuja base é a região no plano $xy$ que é limitada pela parábola $y=4-x^{2}$ e pela reta $y=3x$, enquanto o topo do sólido é limitado pelo plano $z=x+4.$

  3.  No primeiro octante limitado pelos planos coordenados, pelo cilindro   $x^{2}+y^{2}=4$ e pelo plano $z+y=3.$


  1.  $\dfrac{1}{3}.$

  2.  $\dfrac{625}{12}.$

  3.  $\dfrac{9\pi - 8}{3}.$


2621   

Suponha que $S$ e $C$ satisfaçam as hipóteses do Teorema de Stokes e $f$ e $g$ tenham derivadas parciais de segunda ordem contínuas. Demonstre que $\displaystyle\int_C (f\nabla g + g\nabla f) \cdot d{\bf R} = 0$


Note que $\mbox{rot} (f\nabla g + g\nabla f) = {\bf 0}.$


2375   

A temperatura em um ponto $(x,y,z)$ é dada por

$$T(x,y,z) = 200e^{-x^2 - 3y^2 - 9z^2},$$

em que $T$ é medido em °C  e $x,y$ e $z$ em metros.

  1.  Determine a taxa de variação da temperatura no ponto $P = (2,-1,2)$ em direção ao ponto $(3,-3,3)$.
  2.  Qual é a direção de maior crescimento da temperatura em $P$?
  3.  Encontre a taxa máxima de crescimento em $P$.


  1.  $\displaystyle \frac{5200\sqrt{6}}{3e^{43}}$ ºC/m.
  2.  $400 e^{-43} (-2,3,-18).$
  3.  $400 e^{-43}\sqrt{337}$ ºC/m.


2220   

Seja $f$ um campo escalar e $\mathbf{F}$ um campo vetorial. Diga se cada expressão tem significado. Em caso negativo, explique por quê. Em caso afirmativo, diga se é um campo vetorial ou escalar.  

  1. $\text{div }{(\text{grad }{f})}$;

  2. $\text{rot }{(\text{rot }{\mathbf{F}})}$;

  3. $(\text{grad }{f}) \times (\text{div }{\mathbf{F}})$.


  1. $\text{div }{(\text{grad }{f})}$ é um campo escalar.

  2. $\text{rot }{(\text{rot }{\mathbf{F}})}$ é um campo vetorial.

  3. $(\text{grad }{f}) \times (\text{div }{\mathbf{F}})$  não tem significado pois $\text{div } \bf{F}$ é um campo escalar.


1931   

Calcule a integral de linha, onde $C$ é a curva dada.

$\displaystyle\int_{C}xy^{4}\,ds$,   $C$ é a metade direita do círculo $x^{2}+y^{2}=16.$


$\dfrac{2^{13}}{5}.$


2697   

Calcule todas as derivadas parciais de $2^{\underline{a}}$ ordem de $z=\ln(1+x^{2}+y^{2})$.


$\begin{aligned}[t]\frac{\partial^{2} z}{\partial x^{2}} &= \frac{2 + 2y^{2} - 2x^{2}}{(1 + x^{2} + y^{2})^{2}},\;\;\;\;\; \frac{\partial^{2} z}{\partial y^{2}}= \frac{2 + 2x^{2} - 2y^{2}}{(1 + x^{2} + y^{2})^{2}} \;\;\;\;\;\text{e}\\\frac{\partial^{2} z}{\partial x\partial y} &= \frac{\partial^{2} z}{\partial y\partial x}= \frac{-4xy}{(1 + x^{2} + y^{2})^{2}}.C\end{aligned}$


2965   

Dentre as coordenadas cilíndricas ou esféricas, utilize a que lhe parecer mais apropriada para determinaretermine o volume da menor cunha esférica cortada de uma esfera de raio $a$ por dois planos que se interceptam ao longo de um diâmetro com um ângulo de $\pi/6.$


$\dfrac{\pi a^3}{9}.$


2466   

Calcule a integral de superfície $\displaystyle\iint \limits_{ S}{\bf F}\cdot d{\bf S}$ para o campo vetorial ${\bf F}$ e superfície orientada $S$ dados abaixo. Em outras palavras, determine o fluxo de ${\bf F}$ através de $S$. Para superfícies fechadas, use a orientação positiva (para fora).

  • ${\bf F}(x,y,z)=x^{2}{\bf i}+y^{2}{\bf j}+z^{2}{\bf k}$ e $S$ é a fronteira do semicilindro sólido $0 \leq z \leq \sqrt{1-y^{2}}$, $0 \leq x \leq 2.$


$2\pi + \dfrac{8}{3}.$


3119   

Use coordenadas esféricas para encontrar o volume do sólido: contido no interior do cone \(\phi=\pi/4\), entre as esferas \(\rho=1\) e \(\rho=2\).


2223   

Determine a derivada direcional de $f$ no ponto dado e na direção indicada pelo ângulo $\theta$.

$f(x,y) = x^2y^3 - y^4,  (2,1),$  $\theta = \pi/4.$


 $6\sqrt{2}.$


2022   

Considere a função
$$f(x,y) = \begin{cases}
x + y, & \quad \text{se } xy = 0,\\
1, & \quad \text{caso contrário}.
\end{cases}$$
Mostre que $f$ não possui derivada direcional em $(0,0)$ na direção de um vetor $\bf{v} = (a,b)$ com $a^2 + b^2 = 1$ e $ab \neq 0$.



Seja $\bf{v} = (a,b)$ um vetor unitário (isto é, $a^2+b^2 = 1$), em que $ab \neq 0$. A derivada direcional em $(0,0)$ na direção do vetor unitário $\bf{v}$ existe se o limite

$$\lim_{h \to 0} \frac{f(0+ah,0+bh)-f(0,0)}{h}$$

existir. Para $h \neq 0$, temos $(ah)(bh) \neq 0$. Logo $f(ah,bh) = 1$. Assim, o limite em questão se reduz a

$$\lim_{h \to 0} \frac{1}{h},$$

e esse limite não existe. Como o vetor $\bf{v}$ satisfazendo as hipóteses foi tomado arbitrariamente, concluímos que $f$ não possui derivada direcional em $(0,0)$ na direção de nenhum vetor $\bf{v} = (a,b)$ que satisfaça $a^2 + b^2 = 1$ e $ab \neq 0$.


3122   

Uma esfera astroidal tem equação \(x^{2/3}+y^{2/3}+z^{2/3}=a^{2/3}\). Encontre o volume do sólido compreendido por uma esfera astroidal usando uma integral tripla e a transformação \begin{align*}  x & = \rho (\sin\phi\cos\theta)^3, \\  y & = \rho (\sin\phi\sin\theta)^3, \\  z & = \rho (\cos\phi)^3, \end{align*} para a qual \(0\leq\rho\leq a\), \(0\leq\phi\leq\pi\), \(0\leq\theta\leq 2\pi\).


\(\dfrac{4}{35}\pi a^3\)


2409   

Calcule a integral de superfície $\displaystyle\iint\limits_{S}\dfrac{z}{\sqrt{1+4x^{2}+4y^{2}}}dS$, onde $S$ é a parte do parabolóide

$z=1-x^{2}-y^{2}$ que se encontra dentro do cilindro $x^{2}+y^{2}\leq 2y.$



Parametrizando a superfície $S$, temos as equações paramétricas:

$x=u, y=v \, \mbox{e} \, z=1-u^{2}-v^{2}.$
Então,
${\bf r}(u,v)=u{\bf i}+v{\bf j}+(1-u^{2}-v^{2}){\bf k}.$
Logo,
$f({\bf r}(u,v))=\dfrac{1-u^{2}-v^{2}}{\sqrt{1-4u^{2}-4v^{2}}},$ ${\bf r}_{u}={\bf i}+0{\bf j}-2u{\bf k}$ e ${\bf r}_{v}=0{\bf i}+{\bf j}-2v{\bf k}.$
Temos que

${\bf r}_{u}\times {\bf r}_{v}=\left| \begin{array}{ccc} {\bf i} & {\bf j} & {\bf k}\\ 1 & 0 & -2u\\ 0 & 1 & -2v \end{array} \right| = 2u{\bf i}+2v{\bf j}+{\bf k}$,

implicando que $|{\bf r}_{u}\times {\bf r}_{v}|=\sqrt{(2u)^{2}+(2v)^{2}+1^{2}}=\sqrt{1+4u^{2}+4v^{2}}.$ Assim,

$\displaystyle\iint\limits_{S}\dfrac{z}{\sqrt{1+4x^{2}+4y^{2}}}dS=\displaystyle\iint\limits_{D} f({\bf r}(u.v))|{\bf r}_{u}\times {\bf r}_{v}| du dv$ $=\displaystyle\iint\limits_{D} \frac{1-u^{2}-v^{2}}{\sqrt{1-4u^{2}-4v^{2}}} \sqrt{1+4u^{2}+4v^{2}} du dv=\displaystyle\iint\limits_{D}(1-u^{2}-v^{2})du dv$.
Notemos que
$D=\{(u,v)| u^{2}+v^{2}\leq 2v\}=\{(u,v)|u^{2}+(v-1)^{2}\leq 1\}.$
Em coordenadas polares teremos que
$u=r\cos \theta, v-1=r\sin \theta,$
$du dv=\left| \begin{array}{cc}
\dfrac{\partial u}{\partial r} & \dfrac{\partial u}{\partial \theta}\\
\dfrac{\partial v}{\partial r} & \dfrac{\partial v}{\partial \theta}
\end{array} \right|$, $ dr d\theta=\left| \begin{array}{cc} \cos \theta & -r\sin \theta\\ \sin \theta & r\cos \theta \end{array} \right| \, e \, du dv=r dr d\theta.$

Como $u^{2}+u^{2}=2u \Rightarrow r^{2}\cos^{2}\theta+r^{2}\sin^{2}\theta=r\sin \theta \Rightarrow r=2\sin \theta,$ então $0\leq r \leq 2\sin \theta \, \mbox{e} \, 0 \leq \theta \leq \pi.$
Logo
$\displaystyle\iint\limits_{S}\dfrac{z} {\sqrt{1+4x^{2}+4y^{2}}}dS=\displaystyle\int_{0}^{\pi}\displaystyle\int_{0}^{2\sin \theta}(1-r^{2}\cos^{2} \theta-r^{2}\sin^{2}\theta)r dr d\theta$

$\displaystyle\int_{0}^{\pi}\displaystyle\int_{0}^{2\sin \theta}(1-r^{2})r dr d\theta=\displaystyle\int_{0}^{\pi}\int_{0}^{2\sin \theta}(r-r^{3})dr d\theta$ $=\displaystyle\int_{0}^{\pi}(2\sin^{2}\theta-4\sin^{4}\theta)\bigg|_{0}^{2\sin \theta}d\theta=2\int_{0}^{\pi}\sin^{2}\theta d\theta-4\int_{0}^{\pi}\sin^{4}\theta$

$=2\cdot\left(\dfrac{\theta}{2}-\frac{1}{4}\sin 2\theta\right)\bigg|_{0}^{\pi}-4\cdot \left(-\dfrac{1}{4}\sin^{3}
\theta \cos \theta+\dfrac{3}{8}\theta-\dfrac{3}{16}\sin 2\theta\right)\bigg|_{0}^{\pi}$
$=2\cdot \dfrac{\pi}{2}-4\cdot\left(\dfrac{3}{8}\pi\right)=-\dfrac{\pi}{2}.$


2507   

Faça o mapa de contorno da função $f(x,y)=y-\ln{x}$ mostrando várias de suas curvas de nível.


$y = \ln(x) + C.$

A expressão 1 cossec x e o mesmo que


3120   

Usando coordenadas esféricas, calcule a massa da esfera sólida de raio \(a\) com densidade proporcional à distância ao centro (tomando \(k\) como a constante de proporcionalidade).


 \(k\pi a^4\)


2748   

Explique por que a função é diferenciável no ponto dado. $f(x,y) = \dfrac{x}{x+y}, \quad (2,1)$.


As derivadas $f_{x}$ e $f_{y}$ de cada $f$ existem e são contínuas no ponto dado, logo $f$ é diferenciável.


3026   

Calcule $\int_{0}^{1}\!\int_{x}^{1}3y^{4}\cos(xy^{2})\,dy dx$. Esboce a região de integração.


$1 - \cos(1).$

A expressão 1 cossec x e o mesmo que


2015   

Calcule $\int_{C}{\bf E}\cdot d{\bf l}$, onde ${\bf E}(x,y)=\dfrac{1}{x^{2}+y^{2}}\dfrac{x\,{\bf i}+y\,{\bf j}}{\sqrt{x^{2}+y^{2}}}$ e $C$ é a curva dada por $x=2\,\cos t$, $y=\sin t$, com $0\leq t\leq \dfrac{\pi}{2}.$


$-\dfrac{1}{2}.$


2034   

Utilize a Regra da Cadeia para determinar $\mathrm{\partial}z/\mathrm{\partial} s$ e $\mathrm{\partial}z/ \mathrm{\partial}t.$

$z=e^{r}\cos{\theta}$, $r=st$, $\theta=\sqrt{s^{2}+t^{2}}$.


$\displaystyle \frac{\partial z}{\partial s} = e^{r} \left( t\cos(\theta) - \frac{s}{\sqrt{s^{2} + t^{2}}} \sin(\theta) \right) $ e $\displaystyle \frac{\partial z}{\partial t} = e^{r} \left( s\cos(\theta) - \frac{t}{\sqrt{s^{2} + t^{2}}} \sin(\theta) \right).$


2925   

Mude o ponto $(1,\sqrt{3},2\sqrt{3})$ dado em coordenadas retangulares para esféricas.


$\displaystyle \left( 4, \dfrac{\pi}{3}, \dfrac{\pi}{6} \right).$


2174   

Calcule a área da região $R$ delimitada pela cardioide $\mathbf{r}(t) = (x(t),y(t))$, em que $x(t) = 2\cos{t}-\cos{2t}$ e $y(t) = 2\sin{t}-\sin{2t}$, $t \in [0,2\pi]$.


$6\pi.$


2761   

Determine o maior conjunto de pontos em que a função $f(x,y) = \begin{cases}\dfrac{xy}{x^2 + y^2}, & \quad \text{se } (x,y) \neq (0,0),\\0, & \quad \text{se } (x,y) = (0,0)\end{cases}$ é diferenciável. Justifique.


$\mathbb{R}^{2} \setminus \left\lbrace (0,0) \right\rbrace$.


3145   

A Lei de Coulomb afirma que a força eletrostática \(\mathbf{F}(\mathbf{r})\) que uma partícula com carga \(Q\) exerce sobre outra partícula com carga \(q\) é dada pela fórmula \[ \mathbf{F}(\mathbf{r}) = \dfrac{q\,Q}{4\pi\epsilon_0\|\mathbf{r}\|^3}\mathbf{r}, \] onde \(\mathbf{r}\) é o vetor posição da carga \(q\) em relação a \(Q\) e \(\epsilon_0\) é uma constante positiva (chamada permissividade do meio).

  1.  Expresse o campo vetorial \(\mathbf{F}(\mathbf{r})\) em forma de coordenadas \(\mathbf{F}(x,y,z)\) com \(Q\) na origem.

  2.  Calcule o trabalho realizado pelo campo vetorial \(\mathbf{F}\) sobre uma carga \(q\) que se move ao longo de um segmento de reta de \((3,0,0)\) para \((3,1,5)\).


2754   

A função $f(x,y) = \begin{cases}\dfrac{x^4}{x^2 + y^2}, & \quad \text{se } (x,y) \neq (0,0),\\0, & \quad \text{se } (x,y) = (0,0)\\\end{cases}$ é diferenciável em $(0,0)$? Justifique.


2035   

Utilize a Regra da Cadeia para determinar $\mathrm{\partial}z/\mathrm{\partial} s$ e $\mathrm{\partial}z/ \mathrm{\partial}t.$

$z=\tan(u/v)$, $u=2s+3t$, $v=3s-2t$.


$\displaystyle \frac{\partial z}{\partial s} =  \frac{2u - 3v}{v^{2}} \sec^{2}\left(\frac{u}{v} \right)$ e $\displaystyle \frac{\partial z}{\partial t} = \frac{2u + 3v}{v^{2}} \sec^{2}\left(\frac{u}{v} \right))$.


2295   

Determine uma representação paramétrica para a superfície descrita a seguir. A porção no primeiro octante do cone $z=\sqrt{x^{2}+y^{2}}/2$ entre os planos $z=0$ e $z=3.$


$x = r \cos(\theta),$ $y = r \sin(\theta),$ $z = \dfrac{r}{2},$ onde $0 \leq r \leq 6$ e $0\leq \theta \leq \dfrac{\pi}{2}.$


2013   

Calcule $\displaystyle \int_{C}{\bf E}\cdot d{\bf l}$, onde ${\bf E}(x,y)=\dfrac{1}{x^{2}+y^{2}}\dfrac{x\,{\bf i}+y\,{\bf j}}{\sqrt{x^{2}+y^{2}}}$ e $C: {\bf r}(t)=(t,1)$, $-1\leq t\leq 1.$ ( O ${\bf l}$ desempenha aqui o mesmo papel que ${\bf r}:{\bf l}(t)={\bf r}(t).$)


$0.$


2648   

São mostradas as curvas de nível de uma função $f.$ Determine se as seguintes derivadas parciais são positivas ou negativas no ponto $P.$

A expressão 1 cossec x e o mesmo que

  1. $f_{x}$

  2. $f_{xx}$

  3. $f_{yy}$$f_{y}$

  4. $f_{xy}$


  1. Negativa

  2. Positiva

  3. Positiva

  4. Negativa

  5. Positiva


2157   

Calcule a integral de linha $\displaystyle\oint_{C} (x-y) dx + (x+y)dy$, $C$ é o círculo com centro na origem e raio 2, por dois métodos:

  1. diretamente; e

  2. utilizando o Teorema de Green.


$8\pi.$


3051   

A figura mostra a região da integral

$$\int_{0}^{1}\int_{0}^{1-x^{2}}\int_{0}^{1-x}f(x,y,z)\;dy dz dx.$$

Reescreva essa integral como uma integral iterada equivalente nas cinco outras ordens.

A expressão 1 cossec x e o mesmo que


$\int_{0}^{1}\int_{\sqrt{x}}^{1}\int_{0}^{1-y}f(x,y,z)\;dz dy dx = \int_{0}^{1}\int_{0}^{y^2}\int_{0}^{1-y}f(x,y,z)\;dz dx dy $
$= \int_{0}^{1}\int_{0}^{1 - z}\int_{0}^{y^2}f(x,y,z)\;dx dy dz = \int_{0}^{1}\int_{0}^{1 - y}\int_{0}^{y^2}f(x,y,z)\;dx dz dy $
$= \int_{0}^{1}\int_{0}^{1 - \sqrt{x}}\int_{\sqrt{x}}^{1-z}f(x,y,z)\;dy dz dx = \int_{0}^{1}\int_{0}^{(1 - z)^2}\int_{\sqrt{x}}^{1-z}f(x,y,z)\;dy dx dz.$


3146   

Sejam \(\alpha\) e \(\beta\) dois ângulos que satisfazem \(\displaystyle 0<\beta-\alpha\leq 2\pi\) e suponha que \( r= f(\theta)\) seja uma curva polar lisa com \(f(\theta)>0\) no intervalo \([\alpha,\beta]\). Use a fórmula \[ A = \dfrac{1}{2}\int_C-y\,dx+x\,dy \] para encontrar a área da região \(R\) englobada pela curva \(r=f(\theta)\) e os raios \(\theta=\alpha\) e \(\theta=\beta\).


2237   

Seja $\mathbf{r} = x\mathbf{i} + y\mathbf{j} + z\mathbf{k}$ e $r=|\mathbf{r}|$. Verifique a identidade $\nabla{\ln{r}} = \dfrac{\mathbf{r}}{r^2}$.


$\nabla{\ln{r}} = \dfrac{1}{2} \nabla \ln (x^{2} + y^{2} + z^{2}).$ (Note que: $r = \sqrt{x^{2} + y^{2} + z^{2}}.$)


2968   

Calcule a integral, transformando para coordenadas esféricas. $\displaystyle\int_{0}^{1}\int_{0}^{\sqrt{1-x^{2}}}\int_{\sqrt{x^{2}+y^{2}}}^{\sqrt{2-x^{2}-y^{2}}}xy\,dzdydx$.


$\dfrac{(4\sqrt{2} - 5)}{15}.$


2891   

Passe para coordenadas polares e calcule: $\displaystyle\int_{-1}^{1} \int_{-\sqrt{1-y^{2}}}^{\sqrt{1-y^{2}}}\ln(x^{2}+y^{2}+1)\,dx dy$


$\displaystyle \pi (\ln(4) - 1).$


2975   

Determine o jacobiano da transformação dada por: $x = \dfrac{u}{u+v}, \quad y = \dfrac{v}{u-v}$.


$0.$


2840   

Calcule a integral dupla usando coordenadas polares: $\displaystyle\iint\limits_{R}\arctan\left(\dfrac{y}{x}\right)\,dA$, onde $R$ é a região do primeiro quadrante limitada pelo círculo $x^{2}+y^{2}=25.$


$\displaystyle \frac{25 \pi^2}{16}.$


2561   

Determine $h(x,y) = g(f(x,y))$ e o conjunto no qual $h$ é contínua, em que

$$g(t) = t^2 + \sqrt{t}, \ \ \ f(x,y) = 2x + 3y - 6.$$


$h(x,y) = (2x+3y-6)^{2} + \sqrt{2x + 3y - 6}$ é contínua em $\left\lbrace (x,y);\; y \geq -\frac{2x}{3} + 2 \right\rbrace.$   


2044   

Utilize a Equação
$$ \dfrac{dy}{dx}=-\dfrac{\dfrac{\partial F}{\partial x}}{\dfrac{\partial F}{\partial y}}=-\dfrac{F_x}{F_y}$$
para determinar $\mathrm{d}y/\mathrm{d}x$.
$\sqrt{xy}=1+x^{2}y$


$\displaystyle \frac{dy}{dx} = \frac{4(xy)^{3/2} - y}{x - 2x^{2}\sqrt{xy}} .$


2448   

Calcule $\displaystyle\iint\limits_{S}g(x,y,z)dS,$ sendo $g(x,y,z)=x^{2}+y^{2}+z^{2}$ e $S$ a parte do plano $z=y+4$ interior ao cilindro $x^{2}+y^{2}=4.$


$76\pi \sqrt{2}.$


2969   

Calcule a integral, transformando para coordenadas esféricas. $\displaystyle\int_{0}^{2}\int_{0}^{\sqrt{4-y^{2}}}\int_{0}^{\sqrt{4-x^{2}-y^{2}}}\dfrac{1}{x^{2}+y^{2}+z^{2}}\,dzdxdy$.


$\pi.$


3091   

Encontre todos os extremos relativos de \(x^2y^2\) sujeitos à restrição \(4x^2+y^2=8\). Faça-o de duas maneiras: primeiro, usando restrições para eliminar uma variável e, em seguida, utilizando multiplicadores de Lagrange como variáveis auxiliares.


Ocorre máximo absoluto de \(4\) em \((\pm 1,\pm 2)\); mínimo absoluto de valor \(0\) em \((\pm\sqrt{2},0)\) e \((0,\pm 2\sqrt{2})\).


2249   

Seja ${\bf F}(x,y,z)=(x+y+z^{2})\,{\bf k}$ e seja $S$ a fronteira do cilindro $x^{2}+y^{2}\leq 4$ e $0\leq z \leq 3.$ Calcule $\displaystyle\iint \limits_{S}{\bf F}\cdot {\bf n}\,dS$ onde ${\bf n}$ é a normal exterior, isto é, ${\bf n}$ é a normal que aponta para fora do cilindro.


2168   

A função diferenciável $z=z(x,y)$ é dada implicitamente pela equação $f\bigg(\dfrac{x}{y},\dfrac{z}{x^{\lambda}}\bigg)=0$ ($\lambda\neq 0$ um número real fixo), onde 

$f(u,v)$ é suposta diferenciável e $\dfrac{\partial f}{\partial v}(u,v)\neq 0$. Verifique que 

$$x\frac{\partial z}{\partial x}+y\dfrac{\partial z}{\partial y}=\lambda z.$$


Note que $\displaystyle \frac{\partial z}{\partial x} =   \frac{\lambda z}{x} -\frac{x^{\lambda}}{y} \frac{\partial f}{\partial u} \left(\frac{x}{y},\frac{z}{x^{\lambda}} \right)\left(\frac{\partial f}{\partial v}\left(\frac{x}{y},\frac{z}{x^{\lambda}} \right)\right)^{-1} $  e $\displaystyle \frac{\partial z}{\partial y} =  \frac{x^{\lambda + 1}}{y^{2}} \frac{\partial f}{\partial u} \left(\frac{x}{y},\frac{z}{x^{\lambda}} \right)\left(\frac{\partial f}{\partial v}\left(\frac{x}{y},\frac{z}{x^{\lambda}} \right)\right)^{-1}$.


2741   

Determine a equação do plano que é tangente ao paraboloide $z = 2x^2 + 3y^2$ e paralelo ao plano $4x - 3y - z = 10$.


$4x - 3y - z = -\frac{11}{4}$.


2837   

Calcule a integral dupla usando coordenadas polares: $\displaystyle\iint\limits_{R}\frac{x}{x^{2}+y^{2}}\,dA$, onde $R=\{(x,y)\in \mathbb{R}^{2}| x^{2}+y^{2}\leq 4, x\geq 1\}.$


$2\sqrt{3}.$


2214   

Determine o rotacional e o divergente do campo vetorial $\mathbf{F}(x,y,z) = xyz\mathbf{i} - x^2y\mathbf{k}$.


$\text{rot } \mathbf{F} = -x^2 \mathbf{i} + 3xy \mathbf{j} -xz \mathbf{k}.$ $\text{div } \mathbf{F} = yz.$


2669   

A função $p=p(V,T)$ é dada implicitamente pela equação $pV=nRT$, onde $n$ e $R$ são constantes não-nulas (Lei dos Gases Ideais). Calcule $\dfrac{\partial p}{\partial V}$ e $\dfrac{\partial p}{\partial T}.$


$\displaystyle \frac{\partial p}{\partial V} = -\frac{nRT}{V^{2}}\;\;\;\;\;\text{e}\;\;\;\;\; \frac{\partial p}{\partial T} = \frac{nR}{V}.$


3135   

  1.  Seja \(\sigma\) a superfície de um sólido \(G\) com vetor normal unitário \(\mathbf{n}\) orientado para fora de \(\sigma\). Suponha que \(\mathbf{F}\) seja um campo vetorial com derivadas parciais de primeira ordem contínuas em \(\sigma\). Prove que \[\iint\limits_\sigma (\mathrm{rot\,}\mathbf{F})\cdot\mathbf{n}\,dS = 0.\] [Sugestão: tome \(C\) uma curva fechada simples em \(\sigma\) que separa a superfície em duas subsuperfícies \(\sigma_1\) e \(\sigma_2\) com fronteira comum \(C\). Aplique o Teorema de Stokes a \(\sigma_1\) e a \(\sigma_2\) e some os resultados.]

  2.  O campo vetorial \(\mathrm{rot\,}\mathbf{F}\) é denominado campo rotacional de \(\mathbf{F}\). Em palavras, interprete a fórmula do item anterior como uma afirmação sobre o fluxo do campo rotacional.


2318   

Determine uma equação do plano tangente à superfície parametrizada dada no ponto especificado. ${\bf r}(u,v)=(u,v,u^{2}+v^{2})$, no ponto ${\bf r}(1,1).$


$(x,y,z) = (1,1,2) + s(1,0,2) + t(0,1,2),$ $s,t \in \mathbb{R}.$


1930   

Calcule a integral de linha, onde $C$ é a curva dada.

$\displaystyle\int_{C}y^{3}\,ds$,   $C:\,x=t^{3},\, y=t,\, 0\leq t\leq 2.$


$\displaystyle \frac{1}{54}\left(145^{3/2} - 1 \right).$


2701   

Verifique que $x\;\dfrac{\partial ^{2}z}{\partial x \partial y}+y\;\dfrac{\partial ^{2}z}{\partial y^{2}}=0$, onde $z=(x+y)e^{x/y}.$


$\displaystyle \frac{\partial^{2} z}{\partial x \partial y}= \frac{-3xy - x^{2}}{y^{3}}e^{\frac{x}{y}}  \;\;\;\;\;\text{e}\;\;\;\;\; \frac{\partial^{2} z}{\partial y^{2}}= \frac{3x^{2}y + x^{3}}{y^{4}}e^{\frac{x}{y}}.$


2075   

Seja $R$ o retângulo $1\leq x\leq 2$, $0\leq y\leq 1$. Calcule $\iint\limits_{R} f(x,y)\,dxdy$, sendo $f(x,y)$ igual a

  1.  $ye^{xy}$

  2.  $xy^{2}$


  1.  $\dfrac{(e - 1)^{2}}{2}.$

  2.  $\dfrac{1}{2}.$


2527   

Seja $f(x,y)=\dfrac{x-y}{x+2y}$.

  1. Determine o domínio.

  2. Calcule $f(2u+v,v-u).$


  1. $\left\lbrace (x,y);\; x \neq -2y \right\rbrace$

  2. $\frac{u}{v}.$


2478   

Determine e faça o esboço do domínio da função $f(x,y,z)=\ln(16-4x^{2}-4y^{2}-z^{2})$.


$\left\lbrace (x,y);\; \frac{x^{2}}{4} + \frac{y^{2}}{4} + \frac{z^{2}}{16} < 1\right\rbrace.$

A expressão 1 cossec x e o mesmo que


2471   

Seja $f(x,y,z)=e^{\sqrt{z-x^{2}-y^{2}}}.$

  1. Calcule $f(2,-1,6).$

  2. Determine o domínio de $f$.

  3. Determine a imagem de $f$.


  1. $e.$

  2. $\left\lbrace (x,y,z): \;z \geq x^{2} + y^{2} \right\rbrace.$

  3. $[1,\infty).$


2457   

Considere a função

$$f(x,y)=\sqrt{x+y^{2}-3}$$

  1. Faça um esboço das curvas de nível de $f$ nos níveis $c=0$, $c=1$ e $c=3.$
  2. Quantas curvas de nível de $f$ passam pelo ponto $(3,-1)$?



  1. As curvas de níveis de $f$ são

    $$\sqrt{x+y^{2}-3}=c\,\,\,\,\mbox{ou}\,\,\,\,x+y^{2}-3=c^2\,\,\,\,\mbox{ou}\,\,\,\,x=3+c^2-y^{2},$$

    ou seja, uma família de parábolas com concavidade para a esquerda. As três curvas de níveis pedidas, obtidas considerando respectivamente $c=0$, $c=1$ e $c=3$, são

    $x=3-y^{2}$, $x=4-y^{2}$ e $x=12-y^{2}.$ Elas estão apresentadas na figura abaixo.

    A expressão 1 cossec x e o mesmo que

  2. Pelo ponto $(3,-1)$ passa uma única curva de nível, isto é, $f(x,y)=1.$ Pois caso contrário o ponto $(3,-1)$ teria duas alturas diferentes, o que é impossível.


2762   

Determine o maior conjunto de pontos em que a função $f(x,y) = \begin{cases}\dfrac{x^3}{x^2 + y^2}, & \quad \text{se } (x,y) \neq (0,0),\\0, & \quad \text{se } (x,y) = 0\end{cases}$ é diferenciável. Justifique.


$\mathbb{R}^{2} \setminus \left\lbrace (0,0) \right\rbrace$.


2807   

Determine os valores máximos e mínimos locais e pontos de sela da função $f(x,y)=x^{3}+2xy+y^{2}-5$.


Ponto de mínimo : $\displaystyle \left( \frac{5}{3}, -\frac{5}{3}\right);$ ponto de sela: $\displaystyle \left(-1,1\right).$


1996   

Seja ${\bf F}:\mathbb{R}^{2} \to \mathbb{R}^{2}$ um campo vetorial contínuo tal que, para todo $(x,y)$, ${\bf F}(x,y)$ é paralelo ao vetor $x\,{\bf i}+y\,{\bf j}$. Calcule $\int_{C}{\bf F}\cdot d{\bf r}$, onde ${\bf r}:[a,b]\to \mathbb{R}^{2}$ é uma curva de classe $C^{1}$, cuja imagem está contida na circunferência de centro na origem e raio $r>0$. Interprete geometricamente.


$0.$


2666   

Considere a função $z=\dfrac{xy^{2}}{x^{2}+y^{2}}.$ Verifique que $x\dfrac{\partial z}{\partial x}+y\dfrac{\partial z}{\partial y}=z.$


$\displaystyle \frac{\partial z}{\partial x} = \frac{y^{4} - x^{2}y^{2}}{(x^{2} + y^{2})^{2}}\;\;\;\;\;\;\text{e}\;\;\;\;\; \frac{\partial z}{\partial y} = \frac{2x^{3}y}{(x^{2} + y^{2})^{2}}.$


2783   

Calcule a integral dupla usando coordenadas polares: $\displaystyle\iint\limits_{R}\frac{x^{2}}{x^{2}+y^{2}}\,dA$, onde $R$ é a região anular limitada por $x^{2}+y^{2}=a^{2}$ e $x^{2}+y^{2}=b^{2}$, $0< a< b.$


$\displaystyle \frac{\pi}{2}(b^2 - a^2).$


2379   

Determine a equação da reta tangente à curva $\gamma$ no ponto $\gamma(t_0) = (2,5)$ sabendo-se que $\gamma'(t) \neq \bf{0}$ e que sua imagem está contida na curva de nível $xy = 10$. Qual a equação da reta normal a $\gamma$, neste ponto?


 Reta tangente: $(x,y) = (2,5) + \lambda (-2,5),$ $\lambda \in \mathbb{R},$
Reta normal: $(x,y) = (2,5) + \lambda (5,2),$ $\lambda \in \mathbb{R}.$


3086   

Dado que \(\displaystyle x^3+y^2x-3=0\), determine \(\dfrac{dy}{dx}\) usando derivação implícita.



Derivando implicitamente a equação dada, temos que \(3x^2+y^2+x(2yy')-0=0\). Ou seja,

\[ \frac{dy}{dx}= -\frac{3x^2+y^2}{2xy}.\]


2660   

Determine as derivadas parciais de $f(x,y)=(4xy-3y^{3})^{3}+5x^{2}y$.


$\displaystyle \frac{\partial f}{\partial x} = 12 y (4xy - 3y^{3})^{2} + 10xy\;\;\;\;\;\;\text{e}\;\;\;\;\; \frac{\partial f}{\partial y} = 3(4xy - 3y^{2})^{2}(4x - 9y^{2}) + 5x^{2}.$


2197   

Encontre os valores de $\partial z/ \partial x$ e $\partial z/\partial y$ no ponto indicado.
$\dfrac{1}{x}+\dfrac{1}{y}+\dfrac{1}{z}-1=0$,  $(2,3,6).$


$\displaystyle \frac{\partial z}{\partial x}(2,3,6) = -9$ e $\displaystyle \frac{\partial z}{\partial x}(2,3,6) = -4.$


2517   

Dada a expressão $g(x,y)=f(x,y)+2$, escreva como o gráfico de $g$ é obtido a partir do gráfico de $f.$


Gráfico de $f$ deslocado para cima por duas unidades.


2939   

Determine a massa e o centro de massa da lâmina que ocupa a região $D$ e tem função densidade $\rho$ quando: $D$ é a região triangular delimitada pelas retas $x = 0, \ y = x$ e   $2x + y = 6; \quad \rho(x,y) = x^2$.


Massa: $4;$ centro de massa: $\displaystyle \left(\frac{6}{5},\frac{12}{5} \right).$


2524   

Esboce o gráfico da função $f(x,y)=\sin(\sqrt{x^{2}+y^{2}})$ .Em geral, se $g$ é uma função de uma variável, como saber o gráfico de $f(x,y)=g(\sqrt{x^{2}+y^{2}})$ a partir do gráfico de $g$?


O gráfico de $f(x,y) = g(\sqrt{x^{2} + y^{2}})$ pode ser obtido rotacionando o gráfico de $g$ no plano $xz$ ao redor do eixo $z.$

A expressão 1 cossec x e o mesmo que


2929   

Esboce o sólido cujo volume é dado pela integral abaixo e calcule-a.

$$\int_{0}^{\pi/6}\!\!\int_{0}^{\pi/2}\!\!\int_{0}^{3}\rho^{2}\sin{\phi}\;d\rho d\theta d\phi$$


A expressão 1 cossec x e o mesmo que


2948   

Calcule utilizando coordenadas esféricas. $\displaystyle\iiint\limits_{B}z\,dxdydz$, onde $B$ é o conjunto $z\geq \sqrt{x^{2}+y^{2}}$ e $x^{2}+y^{2}+z^{2}\leq 1.$


$\dfrac{\pi}{8}.$


2581   

Determine o conjunto dos pontos de continuidade da função $f(x,y) = \dfrac{x - y}{\sqrt{1 - x^2 - y^2}}$. Justifique sua resposta.


$\left\lbrace (x,y);\; x^{2} + y^{2} < 1 \right\rbrace.$


3111   

As equações paramétricas \[\begin{array}{lll} x=u, & y=u\cos v, & z=u\sin v \end{array}\] representam o cone que resulta quando a reta \(y=x\) do plano \(xy\) é girada em torno do eixo \(x\). Determine a área de superfície da parte do cone para a qual \(0\leq u\leq 2\) e \(0\leq v\leq 2\pi\).



Sendo \(\displaystyle\{\mathbf{i},\mathbf{j},\mathbf{k}\}\) a base canônica do espaço, a superfície pode ser representada vetorialmente como \[ \mathbf{r}=u\mathbf{i}+u\cos v\mathbf{j}+u\sin v\mathbf{k} \ \  \left(0\leq u\leq 2,\ 0\leq v\leq 2\pi\right). \] Assim, teremos  \begin{align*} \dfrac{\partial\mathbf{r}}{\partial u} & = \mathbf{i} + \cos v\mathbf{j} + \sin v\mathbf{k} \\ \dfrac{\partial\mathbf{r}}{\partial v} & = - u\sin v\mathbf{j} + u\cos v\mathbf{k} \\ \dfrac{\partial\mathbf{r}}{\partial u}\times\dfrac{\partial\mathbf{r}} {\partial v} & = \left|\begin{array}{ccc} \mathbf{i} & \mathbf{j} & \mathbf{k} \\ 1 & \cos v & \sin v \\ 0 & -u\sin v & u\cos v \end{array} \right| = u\mathbf{i} -u\cos v\mathbf{j} - u\sin v\mathbf{k} \\ \|\dfrac{\partial\mathbf{r}}{\partial u}\times \dfrac{\partial\mathbf{r}}{\partial v}\| & = \sqrt{u^2+(-u\cos v)^2+(-u\sin v)^2} = |u|\sqrt{2} = u\sqrt{2}. \end{align*} Segue, portanto, que \[ S = \iint\limits_R\|\dfrac{\partial\mathbf{r}}{\partial u}\times \dfrac{\partial\mathbf{r}}{\partial v}\|\,dA = \int_0^{2\pi}\int_0^2\sqrt{2}u\,dudv = 2\sqrt{2}\int_0^{2\pi}\,dv = 4\pi\sqrt{2}. \]


3090   

Sejam \(\alpha\), \(\beta\) e \(\gamma\) os ângulos de um triângulo.

  1.  Use multiplicadores de Lagrange para determinar o valor máximo de \(f(\alpha,\beta,\gamma)=\cos\alpha\cos\beta\cos\gamma\) e determine os ângulos para os quais o máximo ocorre.

  2.  Expresse \(f(\alpha,\beta,\gamma)\) como uma função apenas de \(\alpha\) e \(\beta\) e use um software de apoio computacional para fazer o gráfico dessa função de duas variáveis. Confirme que o resultado obtido no item anterior é consistente com o gráfico.


  1. \(\alpha = \beta=\gamma=\pi/3\) e valor máximo \(=1/8\).


3078   

Verifique que a função \(f(x,y)=\sqrt{1-x^2-y^2}\) é contínua no disco unitário fechado \(x^2+y^2\leq 1\).



O domínio de \(f\) é o disco unitário fechado \(x^2+y^2\leq 1\). Para todo ponto \((x_0,y_0)\) na fronteira do disco, temos \[ \lim_{(x,y)\to(x_0,y_0)}\sqrt{1-x^2-y^2} = \sqrt{1-x_0^2-y_0^2} = 0.\] Como o mesmo vale também para pontos interiores ao disco, temos que \(f\) é contínua no disco fechado.


2896   

Utilize coordenadas polares para determinar o volume do sólido dado: delimitado pelo hiperboloide $-x^2-y^2+z^2=1$ e acima do plano $xy.$


$\displaystyle \frac{4\pi}{3}.$


2224   

Determine a derivada direcional de $f$ no ponto dado e na direção indicada pelo ângulo $\theta$.

$f(x,y) = ye^{-x},  (0,4),$ $\theta = 2\pi/3$.


$2 + \frac{\sqrt{3}}{2}.$


2690   

Passe para coordenadas polares e calcule.     

  1.  $\displaystyle\int_{0}^{a} \int_{0}^{x}\sqrt{x^{2}+y^{2}}\,dy dx$, em que $a>0$.

  2.  $\displaystyle\iint\limits_{ D}x\,dA$, onde $D$ é a região do primeiro quadrante compreendida entre os círculos x^2+y^2=4$ e  $x^2+y^2=2x.$ 



  1.  

    Temos que a região de integração é: $$ R=\{(x,y)\in \mathbb{R}^{2}|\,0\leq x \leq a,\, 0\leq y \leq x\}.$$

    A expressão 1 cossec x e o mesmo que

    Passando para coordenadas polares temos que $$\left\{ \begin{array}{cc} x=r\,\cos \theta\\ y=r\,\sin \theta\\ dy\,dx=r\,dr\,d\theta\\ \end{array} \right. $$ Como $0\leq x \leq a$, temos que $0\leq r\leq \dfrac{a}{\cos \theta}$ e também $0\leq \theta \leq \dfrac{\pi}{4}.$ Então, $$\int_{0}^{a}\int_{0}^{x}\sqrt{x^{2}+y^{2}}\,dy\,dx= \int_{0}^{\frac{\pi}{4}}\int_{0}^{\frac{a}{\cos \theta}}\sqrt{r^{2}\,\cos^{2}\theta +r^{2}\,\sin^{2}\theta}\,r\,dr\,d\theta$$ $$=\int_{0}^{\frac{\pi} {4}}\int_{0}^\frac{a}{\cos\theta}r^{2}\,dr\,d\theta=\int_{0}^\frac{\pi}{4}\frac{r^3}{3}\bigg|_{0}^{\frac{a}{\cos \theta}}d\theta $$ $$=\frac{a^{3}}{3}\int_{0}^{\frac{\pi}{4}}\frac{1}{\cos^{3}\theta}d\theta=\frac{a^{3}}{3}\int_{0}^{\frac{\pi}{4}}\sec^{3}\theta d\theta$$ $$=\frac{a^{3}}{3}\bigg(\frac{1}{2}\sec \theta \tan \theta+\frac{1}{2}\ln|\sec \theta+\tan \theta|\bigg)\bigg|_{0}^{\frac{\pi}{4}}$$ $$=\frac{a^{3}}{6}\bigg[\bigg(\sec\frac{\pi}{4}\cdot \tan\frac{\pi}{4}+\ln\bigg|\sec\frac{\pi}{4}+\tan\frac{\pi}{4}\bigg|\bigg)- \bigg(\sec 0\cdot \tan 0+\ln|\sec 0+\tan 0|\bigg)\bigg]$$ $$=\frac{a^{3}}{6}\bigg(\sqrt{2}+\ln(\sqrt{2}+1)\bigg)$$

  2.  

    A região  de integração $R$ é descrita na figura seguinte

    A expressão 1 cossec x e o mesmo que

    Notemos que $x^{2}+y^{2}=2x\Leftrightarrow (x-1)^{2}+y^{2}=1.$ Assim, $$\iint\limits_{ R}x\,dA=\underbrace{\iint\limits_{   \substack{x^{2}+y^{2}\leq 4\\ x\geq 0\\ y\geq 0}}x\,dA}_{(1)} -\,\,\underbrace{\iint\limits_{\substack{(x-1)^{2}+y^{2}\leq 1 \\ y\geq 0}}x\,dA}_{(2)}$$ Para a integral $(1)$ temos em coordenadas polares que $$r^{2}\cos^{2}\theta+r^{2}\sin^{2}\theta=4\Rightarrow r^{2}=4\Rightarrow r=\pm 2.$$ Logo, $0\leq r\leq 2$ e $0\leq \theta \leq \dfrac{\pi}{2}.$ Para a integral $(2)$ temos em coordenadas polares que $$(r-\cos \theta-1)^{2}+r^{2}\sin^{2}\theta=1\Rightarrow r^{2}\cos^{2}\theta-2r\cos \theta+1+r^{2}\sin^{2}\theta=1$$ $$\Rightarrow r^{2}-2r\cos\theta=0\Rightarrow r(r-2\cos \theta)=0\Rightarrow r=0   \mbox{ou}    r=2\cos \theta.$$ Logo, $0\leq r\leq 2\cos \theta$ e $0\leq \theta \leq \dfrac{\pi}{2}.$ Assim,  $$\iint\limits_{       R}x\,dA=\int_{0}^{\frac{\pi}{2}}\int_{0}^{2}r\,\cos \theta \cdot r \,dr\,d\theta- \int_{0}^{\frac{\pi}{2}}\int_{0}^{2\cos \theta}r\cos \theta\cdot r\, dr\,d \theta$$ $$=\int_{0}^{\frac{\pi}{2}}\int_{0}^{2}r^{2}\,\cos \theta\,dr\,d\theta-\int_{0}^{\frac{\pi}{2}}\int_{0}^{2\cos \theta}r^{2}\,\cos \theta\,dr\,d\theta$$ $$=\int_{0}^{\frac{\pi}{2}}\cos\,d\theta \cdot \int_{0}^{2}r^{2}\,dr-\int_{0}^{\frac{\pi}{2}}\frac{r^{3}}{3}\cos \theta \bigg|_{0}^{2\cos \theta}\,d\theta$$ $$=\bigg(\sin\theta \bigg|_{0}^{\frac{\pi}{2}}\bigg)\cdot \bigg(\frac{r^{3}}{3}\bigg|_{0}^{2}\bigg)-\frac{8}{3}\int_{0}^{\frac{\pi}{2}}\cos^{4}\theta\,d\theta$$ $$=\bigg(\sin \frac{\pi}{2}-\sin 0\bigg)\cdot \bigg(\frac{8}{3}-0\bigg)-\frac{8}{3}\bigg(\frac{1}{4}\cos^{3}\theta\,\sin \theta+\frac{3}{8}\theta+\frac{3}{16}\sin 2 \theta\bigg)\bigg|_{0}^{\frac{\pi}{2}}$$ $$=\frac{8}{3}-\frac{8}{3}\bigg[\bigg(\frac{1}{4}\cos^{3}\frac{\pi}{2}\sin \frac{\pi}{2}+\frac{3}{8}\cdot\frac{\pi}{2}+\frac{3}{16}\sin2\cdot \frac{\pi}{2}\bigg) -\bigg(\frac{1}{4}\cos^{3}0\sin 0+\frac{3}{8}\cdot 0+\frac{3}{16}\sin 0\bigg)\bigg]$$ $$=\frac{8}{3}-\frac{8}{3}\cdot \bigg(\frac{3\pi}{16}\bigg)=\frac{8}{3}-\frac{\pi}{2}=\frac{16-3\pi}{6}.$$


2746   

Considere a função

$$f(x,y)=\begin{cases}\dfrac{xy}{x^{2}+y^{2}}, & \quad \text{se } (x,y) \neq (0,0),\\0, & \quad \text{se } (x,y)=(0,0).\\\end{cases}$$

  1. A função $f$ é contínua em $(0,0)$? Justifique sua resposta.

  2. Calcule as derivadas parciais $\dfrac{\partial f}{\partial x}(0,0)$ e $\dfrac{\partial f}{\partial y}(0,0).$

  3. Determine $\dfrac{\partial f}{\partial x}(x,y)$ e $\dfrac{\partial f}{\partial y}(x,y)$ para $(x,y)\neq (0,0).$

  4. $f$ é diferenciável em $(0,0)$? Justifique sua resposta.


  1. Não, pois $\displaystyle \lim_{(x,y) \to (0,0)} f(x,y)$ não existe.

  2. $\displaystyle \frac{\partial f}{\partial x}(0,0) = \frac{\partial f}{\partial y}(0,0) = 0$.

  3. $\displaystyle \frac{\partial f}{\partial x} = \frac{y^{3} - x^{2}y}{(x^{2} + y^{2})^{2}}\;\;\;\text{e}\;\;\; \frac{\partial f}{\partial y} = \frac{x^{3} - xy^{2}}{(x^{2} + y^{2})^{2}}$.

  4. Não, pois $f$ não é contínua em $(0,0)$ (ou: pois suas derivadas parciais não são contínuas em $(0,0)$).


2981   

Utilize a transformação dada para calcular a integral. $\displaystyle \iint\limits_{R}(x - 3y) \, dA$, em que $R$ é a região triangular de vértices $(0,0)$, $(2,1)$ e $(1,2)$; $x = 2u + v$, $y = u + 2v$.


$-3.$


1945   

Calcule a integral de linha, onde $C$ é a curva dada.

$\displaystyle\int_{C}dx+xy\,dy+z\,dz$, $C$ é a interseção de $x^{2}+y^{2}+z^{2}=2$, $x\geq 0$, $y\geq 0$ e $z\geq 0$, com o plano $y=x$; o sentido de percurso é do ponto $(0,0,\sqrt{2})$ para $(1,1,0).$


$\displaystyle \frac{1}{3}.$


2020   

Utilize a Regra da Cadeia para determinar $\mathrm{\partial}z/\mathrm{\partial} s$ e $\mathrm{\partial}z/ \mathrm{\partial}t,$ onde
$$z=\sin{\theta}\cos{\phi}, \quad \theta=st^{2}, \quad \phi=s^{2}t.$$



Utilizando a Regra de Cadeia, obtemos
\begin{eqnarray*}
\frac{\partial z}{\partial s} & = & \frac{\partial z}{\partial \theta}\frac{\partial \theta}{\partial s}+\frac{\partial z}{\partial \phi}\frac{\partial \phi}{\partial s} \\
& = & (\cos{\theta}\cos{\phi})(t^2) + (\sin{\theta}(-\sin{\phi}))(2st) \\
& = & t^2\cos(st^2)\cos(s^2t) - 2st\sin(st^2)\sin(s^2t)
\end{eqnarray*}
e
\begin{eqnarray*}
\frac{\partial z}{\partial t} & = & \frac{\partial z}{\partial \theta}\frac{\partial \theta}{\partial t}+\frac{\partial z}{\partial \phi}\frac{\partial \phi}{\partial t} \\
& = & (\cos{\theta}\cos{\phi})(2st) + (\sin{\theta}(-\sin{\phi}))(s^2) \\
& = & 2st\cos(st^2)\cos(s^2t) - s^2\sin(st^2)\sin(s^2t).
\end{eqnarray*}


2287   

Determine uma representação paramétrica para a superfície descrita a seguir. O plano que passa pelo ponto $(1,2,-3)$ e contém os  vetores ${\bf i}+{\bf j}-{\bf k}$ e ${\bf i}-{\bf j}+{\bf k}.$


$x= 1 + u + v,$ $y = 2 + u - v,$ $z = 3 - u + v.$


2882   

O plano $4x - 3y + 8z = 5$ intercepta o cone $z^2 = x^2 + y^2$ em uma elipse.

  1. Faça os gráficos do cone, do plano e da elipse.

  2. Use os multiplicadores de Lagrange para achar os pontos mais alto e mais baixo da elipse.


  1. Gráficos.

  2. Ponto mais alto: $\displaystyle \left( -\frac{4}{3}, 1,\frac{5}{3} \right)$ e ponto mais baixo: $\displaystyle \left( \frac{4}{13}, -\frac{3}{13},\frac{5}{13} \right).$


2881   

O plano $x + y + 2z = 2$ intercepta o paraboloide $z = x^2 + y^2$ em uma elipse. Determine os pontos dessa elipse que estão mais próximo e mais longe da origem.


Mais próximo: $\displaystyle \left( \frac{1}{2}, \frac{1}{2},\frac{1}{2} \right)$ e mais distante: $\displaystyle \left( -1,-1,2 \right).$


2727   

Se $R$ é a resistência equivalente de três resistores conectados em paralelo, com resistências $R_1, R_2, R_3$, então

$$\dfrac{1}{R} = \dfrac{1}{R_1} + \dfrac{1}{R_2} + \dfrac{1}{R_3}.$$

Se as resistências medem, em ohms, $R_1 = 25  \Omega$, $R_2 = 40 \Omega$, $R_3 = 50 \Omega$, com margem de erro de $0,5\%$ em cada uma, estime o erro máximo no valor calculado de $R$.


$\Delta R \approx 0.059 \Omega$.


2737   

Determine o plano que é paralelo ao plano $z = 2x + y$ e tangente ao gráfico de $f(x,y) = x^2 + y^2$.


$z = 2x + y - \frac{5}{4}$.


1954   

Calcule a integral de linha $\int_{C}{\bf F}\cdot d{\bf r}$, onde $C$ é dada pela função vetorial ${\bf r}(t).$

${\bf F}(x,y)=x^{2}\,{\bf j}$, ${\bf r}(t)=(t^{2},3)$, $-1\leq t\leq 1.$


$0.$


2072   

Seja $R$ o retângulo $1\leq x\leq 2$, $0\leq y\leq 1$. Calcule $\iint\limits_{R} f(x,y)\,dxdy$, sendo $f(x,y)$ igual a

  1. $\sqrt{x+y}$

  2. $\dfrac{1}{x+y}$


  1. $\dfrac{4(9\sqrt{3} - 8\sqrt{2} + 1)}{15}.$

  2. $\ln\left( \dfrac{27}{16}\right).$


2364   

Determine as direções em que a derivada direcional da função \linebreak $f(x,y) = x^2 + \sin{xy}$ no ponto $(1,0)$ tem valor 1.


As direções são dadas pelos vetores $(1,0)$ e $\displaystyle \left( \frac{4}{5}, -\frac{3}{5}\right).$


2244   

Calcule $\int_{C}\mathbf{F} \cdot \mathbf{n} \, ds$ ($\mathbf{n}$ é unitário), onde $\mathbf{F}(x,y) = x^2\mathbf{i}$, $C$ dada por $\mathbf{r}(t) = (2\cos{t},\sin{t})$, $0 \leq t \leq 2\pi$ e $\mathbf{n}$ a normal que aponta para fora da região $x^2/4+y^2\leq 1$.


$0$.


2779   

Calcule a integral dupla usando coordenadas polares: $\displaystyle\iint\limits_{R}(x^{2}+2y)\,dx dy$, onde $R$ é o círculo $x^{2}+y^{2}\leq 4.$


$4\pi.$


2912   

Use a integral dupla em coordenadas polares para deduzir a fórmula $$A=\int_{\alpha}^{\beta}\frac{1}{2} r^{2}\,d\theta$$ para a área da região em formato de leque entre a origem e a curva polar $r=f(\theta)$, $\alpha\leq \theta \leq \beta.$


Note que $\displaystyle A = \int_{\alpha}^{\beta}\int_{0}^{f(\theta)} r  dr d\theta. $


2452   

Integre $g(x,y,z)=xyz$ sobre a superfície do sólido retangular cortado do primeiro octante pelos planos $x=a$, $y=b$ e $z=c.$


$\dfrac{abc(ab+ac+bc)}{4}.$


2585   

Encontre o volume da região sólida limitada abaixo pelo plano $z = 0$, lateralmente pelo cilindro $x^2 + y^2 = 1$ e acima pelo paraboloide $z = x^2 + y^2$.



Temos que a região sólida $E$ está acima do plano $z=0$, abaixo do paraboloide $z=x^{2}+y^{2}$ e limitado lateralmente pelo cilindro $x^{2}+y^{2}=1$. Notemos que podemos dividir a região sólida em quatro porções simétricas. Assim, levando em consideração a porção da região sólida $E$ que está no primeiro octante, temos em coordenadas cilíndricas $$0\leq \theta \leq \frac{\pi}{2},\, 0\leq r \leq 1\,\, \mbox{e}\,\, 0\leq z\leq x^{2}+y^{2}=r^{2}.$$ Assim, o volume da região sólida $E$ é: $$V=\iiint\limits_{  E}1\,dV=4\int_{0}^{\frac{\pi}{2}}\int_{0}^{1}\int_{0}^{r^{2}}1\,r\,dz\,dr\,d\theta$$ $$=4\int_{0}^{\frac{\pi}{2}}\int_{0}^{1}zr\,\bigg|_{0}^{r^{2}}\,dr\,d\theta=4\int_{0}^{\frac{\pi}{2}}\int_{0}^{1}r^{3}\,dr\,d\theta$$ $$=4\int_{0}^{\frac{\pi}{2}}\,d\theta\cdot \int_{0}^{1}r^{3}\,dr=4\cdot \theta\bigg|_{0}^{\frac{\pi}{2}}\cdot \frac{r^{4}}{4}\bigg|_{0}^{1}$$ $$=4\cdot \frac{\pi}{2}\cdot \frac{1}{4}=\frac{\pi}{2}.$$


2363   

Seja $f: \mathbb{R} \to \mathbb{R}$ uma função diferenciável de uma variável. Defina

$$g(x,y) = f(r),  r = \sqrt{x^2 + y^2}.$$

Calcule a derivada direcional da função $g$ no ponto $(x,y) \neq (0,0)$ e na direção do vetor $(x,y)$.


$(f'(r))^{2}.$


2586   

Determine o volume do sólido que está acima do plano $xy$, abaixo do paraboloide $z = x^2 + y^2$ e que se encontra dentro do cilindro $x^2 + y^2 = 2x$ e fora do cilindro $x^2 + y^2 = 1.$



Temos que $0\leq z\leq x^{2}+y^{2}$. Como o sólido se encontra dentro do cilindro $x^{2}+y^{2}=2x$ e fora do cilindro $x^{2}+y^{2}=1$, devemos fazer a interseção desses dois cilindros, isto é, $$\left\{\begin{array}{cc} x^{2}+y^{2}=2x\\ x^{2}+y^{2}=1\\ \end{array} \right.\Rightarrow 2x=1\Leftrightarrow x=\frac{1}{2}$$ Em coordenadas cilíndricas temos que \begin{eqnarray*} x&=&r\cos \theta\\ y&=&r\sin \theta\\ z&=&z\\ dz\,dy\,dx&=&r\,dz\,dr\,d\theta \end{eqnarray*} Da equação $x^{2}+y^{2}=1$ temos que $$r^{2}=1\Longrightarrow r=\pm 1,$$ como devemos ter $r\geq 0$, então nesse caso $r=1.$ Da equação $x^{2}+y^{2}=2x$ temos que $$r^{2}=2r\,\cos \theta \Rightarrow r=2\cos \theta.$$  Agora, sendo $x=\frac{1}{2}$ e $r=1$ temos que $$\cos \theta=\frac{1}{2}\Rightarrow \theta=\pm \frac{\pi}{3}.$$ Assim, em coordenadas cilíndricas temos que o sólido $E$ é dado por $$E=\{(\theta,\,r,\,z)|\, -\frac{\pi}{3}\leq \theta \leq \frac{\pi}{3},\, 1\leq r\leq 2 \cos \theta,\,0\leq z\leq r^{2}\}.$$ Então, $$V=\iiint\limits_{  E}1\,dV= \int_{-\frac{\pi}{3}}^{\frac{\pi}{3}}\int_{1}^{2\cos \theta}\int_{0}^{r^{2}}1\,r\,dz\,dr\,d\theta= \int_{-\frac{\pi}{3}}^{\frac{\pi}{3}}\int_{1}^{2\cos \theta}zr\bigg|_{0}^{r}\,dr\,d\theta$$ $$=\int_{-\frac{\pi}{3}}^{\frac{\pi}{3}}\int_{1}^{2\cos \theta}r^{3}\,dr\,d\theta= \int_{-\frac{\pi}{3}}^{\frac{\pi}{3}}\frac{r^{4}}{4}\bigg|_{1}^{2\cos \theta}\,d\theta =\int_{-\frac{\pi}{3}}^{\frac{\pi}{3}}\bigg(4\cos^{4}\theta-\frac{1}{4}\bigg)\,d\theta$$ $$=4\int_{-\frac{\pi}{3}}^{\frac{\pi}{3}}\underbrace{\cos^{4}\theta}_{\mbox{função   par}}\,d\theta-\int_{-\frac{\pi}{3}}^{\frac{\pi}{3}}\underbrace{\frac{1}{4}}_{\mbox{função    par}}\,d\theta =8\int_{0}^{\frac{\pi}{3}}\cos^{4}\theta\,d\theta-2\int_{0}^{\frac{\pi}{3}}\frac{1}{4}\,d\theta$$ $$=8\bigg[\frac{3}{8}\theta+\frac{1}{4}\sin(2\theta)+\frac{1}{32}\sin(4\theta)\bigg]\bigg|_{0}^{\frac{\pi}{3}} -\bigg(\frac{1}{2}\theta\bigg)\bigg|_{0}^{\frac{\pi}{3}}$$ $$=8\bigg[\frac{3}{8}\cdot \frac{\pi}{3}+\frac{1}{4}\sin\bigg(\frac{2\pi}{3}\bigg)+\frac{1}{32}\sin\bigg(\frac{4\pi}{3}\bigg)\bigg]-\frac{1}{2}\cdot \frac{\pi}{3}$$ $$=\pi+\sqrt{3}-\frac{\sqrt{3}}{8}-\frac{\pi}{6}=\frac{5\pi}{6}+\frac{7\sqrt{3}}{8}.$$


1991   

Calcule $\dfrac{\mathrm{d}{\bf r}}{\mathrm{d}t}$ e $\dfrac{\mathrm{d}^{2}{\bf r}}{\mathrm{d}t^{2}}.$

  1. ${\bf r}(t)=(3t^{2},e^{-t},\ln(t^{2}+1))$
  2. ${\bf r}(t)=\sqrt[3]{t^{2}}{\bf i}+\cos(t^{2}){\bf j}+3t{\bf k}$
  3. ${\bf r}(t)=\sin(5t){\bf i}+\cos(4t){\bf j}-e^{-2t}{\bf k}$


2021   

Seja $g(x,y)=f(x^{2}+y^{2})$, onde $f:\mathbb{R}\rightarrow \mathbb{R}$ é uma função diferenciável. Mostre que 
$$y\frac{\partial g}{\partial x}-x\frac{\partial g}{\partial y}=0.$$



Observe que $f$ é uma função de uma variável. Logo, utilizando a Regra da Cadeia para funções de uma variável, obtemos
$$\frac{\partial g}{\partial x}(x,y) = f'(x^2+y^2) (2x)$$
e
$$\frac{\partial g}{\partial y}(x,y) = f'(x^2+y^2) (2y).$$
Portanto
$$y\frac{\partial g}{\partial x}-x\frac{\partial g}{\partial y}=0.$$


2187   

Use o Teorema do Divergente para calcular o fluxo de ${\bf F}$ através de $S,$ onde ${\bf F}(x,y,z)=3x\,{\bf i}+xz\,{\bf j}+z^{2}\,{\bf k}$ e $S$ é a superfície da região delimitada pelo parabolóide $z=4-x^{2}-y^{2}$ e o plano-$xy.$


2071   

Seja $R$ o retângulo $1\leq x\leq 2$, $0\leq y\leq 1$. Calcule $\iint\limits_{ R} f(x,y)\,dxdy$, sendo $f(x,y)$ igual a

  1.  $x+2y$

  2.  $x-y$


  1.  $\dfrac{5}{2}.$

  2.  $1.$


3079   

Mostre que os limites não existem, considerando que \((x,y)\rightarrow (0,0) \) ao longo dos eixos coordenados.

  1.  \[ \lim_{(x,y)\to(0,0)}\dfrac{3}{x^2+2y^2} \]

  2.  \[ \lim_{(x,y)\to(0,0)}\dfrac{x+y}{2x^2+y^2} \]


3074   

Mostre (verifique) que o domínio da função $f(x,y)=\ln(x^2-y)$ consiste em todos os pontos abaixo da curva $y<x^2$.


A função $(x,y)\longmapsto\ln(x^2-y)$ só está definida para \( 0<x^2-y \), ou seja, $y<x^2$. Assim, primeiro esboçamos a parábola $y=x^2$ (como uma curva tracejada, por exemplo). A região $y<x^2$ consiste em todos os pontos abaixo dessa curva.


2662   

Determine as derivadas parciais de $g(x,y)=x^{y}$.


$\displaystyle \frac{\partial g}{\partial x} = yx^{y - 1}\;\;\;\;\;\;\text{e}\;\;\;\;\; \frac{\partial g}{\partial y} = x^{y} \ln x.$


2040   

Utilize a Regra da Cadeia para determinar as derivadas parciais indicadas.
$z=x^{2}+xy^{3}$, $x=uv^{2}+w^{3}$, $y=u+ue^{w}$;
$\dfrac{\partial z}{\partial u}$, $\dfrac{\partial z}{\partial v}$, $\dfrac{\partial z}{\partial w}$ quando $u=2$,  $v=1$, $w=0$.


$\dfrac{\partial z}{\partial u} = 85$, $\dfrac{\partial z}{\partial v} = 178$, $\dfrac{\partial z}{\partial w} = 54.$


2407   

Inverta a ordem de integração.

  1.  $\displaystyle\int_{-1}^{1}\bigg[\int_{x^{2}}^{\sqrt{2-x^{2}}}f(x,y)\,dy\bigg]dx$

  2.  $\displaystyle\int_{0}^{1}\bigg[\int_{y-1}^{2-2y}f(x,y)\,dx\bigg]dy$

  3.  $\displaystyle\int_{0}^{1}\bigg[\int_{x^{2}}^{1}f(x,y)\,dy\bigg]dx$


  1.  $\displaystyle\int_{0}^{1}\bigg[\int_{-\sqrt{y}}^{\sqrt{y}}f(x,y)\,dx\bigg]dy +  \displaystyle\int_{1}^{\sqrt{2}}\bigg[\int_{-\sqrt{2 - y^{2}}}^{\sqrt{2-y^{2}}}f(x,y)\,dx\bigg]dy$

  2.  $\displaystyle\int_{-1}^{0}\bigg[\int_{0}^{x + 1}f(x,y)\,dy \bigg] dx + \int_{0}^{2}\bigg[\int_{0}^{\frac{2-x}{2}}f(x,y)\,dy \bigg] dx$

  3.  $\displaystyle\int_{0}^{1}\bigg[\int_{0}^{\sqrt{y}}f(x,y)\,dx\bigg]dy$


2332   

Determine a taxa de variação máxima de $f$ no ponto dado e a direção em que isso ocorre.

$f(x,y,z) = \dfrac{x + y}{z},  (1,1,-1).$


$\sqrt{6}.$


2582   

Considere a função

$$f(x,y) = \begin{cases}\dfrac{x^2 - xy}{x^2 + y^2}, & \quad \text{se } (x,y) \neq (0,0), \\0, & \quad \text{se } (x,y) = (0,0).\end{cases}$$

  1. Calcule o limite $\displaystyle \lim_{(x,y) \to (0,0)}f(x,y)$ ou mostre que esse limite não existe.

  2. Calcule o limite $\displaystyle \lim_{(x,y) \to (1,1)}f(x,y)$ ou mostre que esse limite não existe.

  3. $f$ é contínua em $(0,0)$? Justifique.

  4. $f$ é contínua em $(1,1)$? Justifique.



  1. O limite não existe.

  2. $0.$

  3. Não.

  4. Sim.


2312   

Determine a derivada direcional da função no ponto dado e na direção do vetor $\bf{v}$.

$g(p,q) = p^4 - p^2q^3,  (2,1),  \bf{v}= \left(-1,2\right).$


 $\displaystyle -\frac{4\sqrt{10}}{5}.$


2169   

Calcule as integrais iteradas.

  1. $\displaystyle\int_{0}^{1}\int_{0}^{x^{2}}(x+2y)\,dy dx$

  2. $\displaystyle\int_{0}^{1}\int_{x^{2}}^{x}(1+2y)\,dy dx$


  1.  $\dfrac{9}{20}.$

  2.  $\dfrac{3}{10}.$


3126   

Verifique que para o vetor posição \(\mathbf{r}=x\mathbf{i}+y\mathbf{j}+z\mathbf{k}\) valem as seguintes propriedades

  1.  \(\displaystyle \mathrm{rot\,}\mathbf{r} = \mathbf{0}\)

  2.  \(\displaystyle \nabla\|\mathbf{r}\| = \dfrac{\mathbf{r}}{\|\mathbf{r}\|} \)


3092   

  1.  Seja \(f(x,y)=x-2y\) e considere uma subdivisão uniforme do retângulo \(R=[0,2]\times[0,2]\) em \(16\) retângulos menores. Tome \((x_k^\ast,y_k^\ast)\) como sendo o centro do \(k\)-ésimo  retângulo e aproxime a integral dupla de \(f\) sobre \(R\) pela soma de Riemann resultante.

  2.  Compare o resultado obtido no item anterior com o valor exato da integral.


2429   

Calcule a integral tripla.

  1.  $\displaystyle\iiint\limits_{  E}x\;dx dy dz$, onde $E$ é o conjunto $0\leq x \leq 1$, $0\leq y \leq 1$ e \\ $x+y\leq z \leq x+y+1.$

  2.  $\displaystyle\iiint\limits_{  E}\sqrt{1-z^{2}}\;dx dy dz$, onde $E$ é o conjunto $0 \leq x \leq 1$, $0\leq z\leq 1$ e $0\leq y \leq z.$


  1.  $\dfrac{1}{2}.$

  2.  $\dfrac{1}{3}.$


3121   

Usando coordenadas esféricas, calcule a massa do sólido compreendido entre as esferas \(x^2+y^2+z^2=1\) e \(x^2+y^2+z^2=4\), com densidade \(\delta(x,y,z)=(x^2+y^2+z^2)^{-1/2}.\)


2234   

Seja $\mathbf{r} = x\mathbf{i} + y\mathbf{j} + z\mathbf{k}$ e $r=|\mathbf{r}|$. Verifique a identidade $\nabla{r} = \dfrac{\mathbf{r}}{r}$.


$\nabla{r} = \left(\dfrac{x}{\sqrt{x^2 + y^2 + z^2}},\dfrac{y}{\sqrt{x^2 + y^2 + z^2}} ,\dfrac{z}{\sqrt{x^2 + y^2 + z^2}} \right).$ (Note que: $r = \sqrt{x^{2} + y^{2} + z^{2}}.$)


2615   

Verifique que o Teorema de Stokes é verdadeiro para o campo vetorial ${\bf F}$ dado e a superfície $S$.

  • ${\bf F}(x,y,z) = y^2{\bf i} + x{\bf j} + z^2{\bf k}$, $S$ é a parte do parabolóide $z = x^2 + y^2$ que está acima do plano $z = 1$, orientado para cima.


$\displaystyle\int_{C} {\bf F} \cdot d{\bf R} = \displaystyle\iint_{S} \mbox{rot} {\bf F} \cdot d{\bf S} = \pi.$


2688   

Encontre $f_{x}$, $f_{y}$ e $f_{z}$ para $f(x,y,z)=(x^{2}+y^{2}+z^{2})^{-1/2}$.


$\begin{aligned}[t]f_{x} &= -x(x^{2} + y^{2} + z^{2})^{-3/2},\;\; f_{y} = -y(x^{2} + y^{2} + z^{2})^{-3/2}\;\;\text{e}\\f_{z} &= -z(x^{2} + y^{2} + z^{2})^{-3/2}.\end{aligned}$


3129   

Dado um campo vetorial \(\mathbf{F}\), uma curva \(C\) é chamada de linha de fluxo deste campo se \(\mathbf{F}\) for um vetor tangente a \(C\) em cada ponto ao longo de \(C\).

  1.  Sejam \(C\) uma linha de fluxo de \(\mathbf{F}(x,y)=-y\mathbf{i}+x\mathbf{j}\) e \((x,y)\) um ponto em \(C\) para o qual \(y\neq 0\). Mostre que as linhas de fluxo satisfazem a equação diferencial \[ \dfrac{dy}{dx} = -\dfrac{x}{y}. \]

  2. Resolva a equação diferencial do item anterior, por separação de variáveis, e mostre que as linhas de fluxo são círculos concêntricos centrados na origem, ou seja, da forma \(x^2+y^2=K\).


2442   

Calcule a integral de superfície $\displaystyle\iint\limits_{S}y dS$, onde $S$ é a parte do parabolóide $y=x^{2}+z^{2}$ que está dentro do cilindro $x^{2}+z^{2}=4.$


$\dfrac{\pi(391\sqrt{17}+1)}{60}.$


2279   

Defina gradiente de uma função de três variáveis. Calcule $\nabla f(x,y,z)$.

$f(x,y,z) = \sqrt{x^2 + y^2 + z^2}$


 $\displaystyle \nabla f(x,y,z) = \frac{1}{\sqrt{x^{2} + y^{2} + z^{2}}}.$


2136   

Mostre que cada a equação a seguir define implicitamente pelo menos uma função diferenciável $z=z(x,y)$. 
Expresse $\partial z /\partial x$ e $\partial z/\partial y$ em termos de $x$, $y$ e $z.$
$x^{3}+y^{3}+z^{3}=x+y+z$


 $\displaystyle \frac{\partial z}{\partial x} = -\frac{3x^{2} - 1}{3z^{2} - 1}$ e $\displaystyle \frac{\partial z}{\partial y} = -\frac{3y^{2} - 1}{3z^{2} - 1}.$


2750   

Mostre que a função $f(x,y) = xy - 5y^2$ é diferenciável achando os valores $\varepsilon_1$ e $\varepsilon_2$ que satisfaçam a Definição $7$ da Seção $14.4$ do Stewart.


$\epsilon_{1} = \Delta y$ e $\epsilon_{2} = -5\Delta y$.


2036   

Seja $W(s,t)=F(u(s,t),v(s,t))$, onde $F$, $u$ e $v$ são diferenciáveis, e $u(1,0)=2$, $u_{s}(1,0)=-2$, $u_{t}(1,0)=6$, $F_{u}(2,3)=-1$, $v(1,0)=3$, $v_{s}(1,0)=5$, $v_{t}(1,0)=4$, $F_{v}(2,3)=10.$ Determine $W_{s}(1,0)$ e $W_{t}(1,0).$


$W_{s}(1,0) = 52$ e $W_{t}(1,0) = 34.$


2373   

Considere a função

$$f(x,y) = \ln{(x^2 + y^2)}.$$

  1. Determine a taxa de variação máxima de $f$ em $(1,1)$ e a direção em que isso ocorre.
  2.  Determine a derivada direcional de $f$ em $(1,1)$ na direção do vetor $\bf{v} = (3,4)$.


  1. Na direção do vetor $(1,1).$ O valor da taxa máxima é $\sqrt{2}.$ 
  2.  $ \displaystyle \frac{7}{5}.$


2922   

Utilize a integral dupla para determinar a área da região: cortada do primeiro quadrante pela curva $r=2(2-\sin(2\theta))^{1/2}.$


$2(\pi - 1).$


2446   

Calcule a integral de superfície $\displaystyle\iint\limits_{S}y dS$, onde $S$ é a superfície com equações paramétricas $x=u$, $y=v$, $z=1-u^{2}$, $0\leq u\leq 1$, $0\leq v\leq \sqrt{u}.$


2920   

Utilize a integral dupla para determinar a área da região: um laço da rosácea $r=\cos(3\theta).$


$\displaystyle \frac{\pi}{12}.$


2592   

Considere um escoamento com velocidade ${\bf v}(x,y,z)$ e densidade $\rho(x,y,z)$, tal que ${\bf u}=\rho {\bf v}$ seja dado por ${\bf u}=x{\bf i}+y{\bf j}-2z{\bf k}$. Seja $S$ a superfície $x^{2}+y^{2}+z^{2}=4$, $z\geq \sqrt{2}$, e seja ${\bf n}$ a normal com componente $z>0$. Calcule o fluxo de ${\bf u}$ através de $S$. (Observe que, neste caso, o fluxo tem dimensões $MT^{-1}$ (massa por unidade de tempo).)


2554   

Determine o limite, se existir, ou mostre que o limite não existe.

$\displaystyle \lim_{(x,y) \to (2,1)}\dfrac{4 - xy}{x^2 + 3y^2}$.


$\frac{2}{7}.$


2619   

Suponha que $S$ e $C$ satisfaçam as hipóteses do Teorema de Stokes e $f$ e $g$ tenham derivadas parciais de segunda ordem contínuas. Demonstre que $\displaystyle\int_C (f\nabla g)\cdot d{\bf R} = \displaystyle\iint_{S} (\nabla f \times \nabla g)\cdot d{\bf S}$


Note que $\mbox{rot} (f\nabla g) = \nabla f \times \nabla g.$


1962   

Calcule a integral de linha $\displaystyle\int_{C}\,dx+\,dy$, onde $C$ é a poligonal de vértices $A_{0}=(0,0)$, $A_{1}=(1,2)$, $A_{2}=(-1,3)$, $A_{3}=(-2,1)$ e $A_{4}=(-1,-1)$, sendo $C$ orientada de $A_{0}$ para $A_{4}.$


$\displaystyle -2.$


1978   

Determine a função vetorial que representa a curva obtida pela intersecção do cone $z = \sqrt{x^2 + y^2}$ com o plano $z = 1 + y$.


2944   

Calcule utilizando coordenadas esféricas. $\displaystyle\iiint\limits_{E}xyz\,dV$, onde $E$ está entre as esferas $\rho=2$ e $\rho=4$ e acima do cone $\phi=\pi/3.$


$0.$


2267   

Determine a área da superfície dada pela parte do paraboloide hiperbólico $z=y^{2}-x^{2}$ que está entre os cilindros $x^{2}+y^{2}=1$ e $x^{2}+y^{2}=4.$



Temos que $z=f(x,y)=y^{2}-x^{2}$ com $1\leq x^{2}+y^{2}\leq 4$. Então,

$$A(S)=\iint\limits_{ D}\sqrt{1+\bigg(\frac{\partial z}{\partial x}\bigg)^{2}+\bigg(\frac{\partial z}{\partial y}\bigg)^{2}}\,dA$$

$$=\iint\limits_{ D}\sqrt{1+(2y)^{2}+(-2x)^{2}}\,dA=\iint\limits_{ D}\sqrt{1+4y^{2}+4x^{2}}\,dA.$$

Usando coordenadas polares temos que

$$x=r\,\cos \theta,\,\,\,\,\, y=r\,\sin \theta \Rightarrow 0\leq \theta\leq \frac{\pi}{2}\,\, \mbox{e}\,\, 1\leq r \leq 2.$$

Assim,

$$A(S)=\int_{0}^{2\pi}\int_{1}^{2}\sqrt{1+4r^{2}}\,r\,dr\,d\theta=\int_{0}^{2\pi}d\theta \cdot \underbrace{\int_{1}^{2}\sqrt{1+4r^{2}}r\,dr}_{\substack{u=1+4r^{2}\\ du=8r\,dr}}$$

$$=\theta\bigg|_{0}^{2\pi}\cdot \int_{5}^{17}u^{1/2}\cdot r\cdot \frac{du}{8r}=2\pi\cdot \frac{1}{8}\int_{5}^{17}u^{1/2}\,du=\frac{\pi}{4}\cdot \frac{2}{3}u^{3/2}\bigg|_{5}^{17}$$

$$=\frac{\pi}{6}\cdot(17^{3/2}-5^{3/2}).$$


2319   

Determine uma equação do plano tangente à superfície parametrizada dada no ponto especificado. ${\bf r}(u,v)=(\arctan (uv),e^{u^{2}-v^{2}},u-v)$, no ponto ${\bf r}(1,-1).$


$(x,y,z) = \left(-\dfrac{\pi}{4},1,2\right) + s\left(-\dfrac{1}{2},2,1\right) + t\left(\dfrac{1}{2},2,-1\right),$ $s,t \in \mathbb{R}.$


3037   

Uma região $R$ é mostrada na figura abaixo. Decida se você deve usar coordenadas polares ou retangulares e escreva $\iint \limits_{R}f(x,y)\,dA$ como uma integral iterada, onde $f$ é uma função qualquer contínua em $R.$


A expressão 1 cossec x e o mesmo que


$\displaystyle \int_{-\frac{\pi}{2}}^{\frac{\pi}{2}} \int_{3}^{6} f(r\cos(\theta),r\sin(\theta)) r  d r d \theta.$


1939   

Calcule a integral de linha, onde $C$ é a curva dada.

$\displaystyle\int_{C}x\,dx+y\,dy+z\,dz$, $C$ é o segmento de extremidades $(0,0,0)$ e $(1,2,1)$, percorrido no sentido de $(1,2,1)$ para $(0,0,0).$


$-3.$


2404   

Considere a integral iterada dada por $$\int_{0}^{1} \int_{x}^{\sqrt{x}}\frac{e^{y}}{y}\,dy dx.$$

  1.  Desenhe a região de integração no plano $xy.$

  2.  Calcule a integral acima.


  1.  (...)

  2.  $e - 2.$


2645   

Determine $\partial z/\partial x$ e $\partial z/\partial y$, sendo $z=f(x)+g(y)$.


$\displaystyle \frac{\partial z}{\partial x} =  f'(x)$

$\frac{\partial z}{\partial y} = g'(y)$.


2829   

Determine os pontos da superfície $y^{2}=9+xz$ que estão mais próximos da origem.


$(0,3,0)$ e $(0,-3,0).$


2432   

Calcule a integral tripla.

  1.  $\displaystyle\iiint\limits_{  E}y\;dx dy dz$, onde $E$ é o conjunto $x^{2}+4y^{2}\leq 1$ e $0\leq z \leq 1.$

  2.  $\displaystyle\iiint\limits_{  E}x\;dx dy dz$, onde $E$ é o conjunto $x^{2}+y^{2}\leq 4$, $x\geq 0$ e \\ $x+y\leq z \leq x+y+1.$


  1.  $0.$

  2.  $\dfrac{16}{3}.$


2861   

Estude com relação a máximos e mínimos a função dada com as restrições dadas.

$f(x,y) = 3x + y$ e $x^2 + 2y^2 \leq 1.$


Ponto de máximo: $\displaystyle \left( \frac{6}{\sqrt{38}}, \frac{1}{\sqrt{38}} \right)$; ponto de mínimo: $\displaystyle \left( -\frac{6}{\sqrt{38}}, -\frac{1}{\sqrt{38}} \right)$.


1990   

 Se ${\bf u}(t)=(\sin{t}, \cos{t}, t)$ e ${\bf v}(t)=(t,t\cos{t},\sin{t})$, use a Fórmula $$\dfrac{d}{dt}\left[{\bf u}(t)\times{\bf v}(t)\right]={\bf u}'(t)\times{\bf v}(t)+{\bf u}(t)\times{\bf v}'(t)$$ para  encontrar $$\frac{\mathrm{d}}{\mathrm{d}t}[{\bf u}(t)\times {\bf v}(t)].$$


$\left\{t^2 \sin (t)-\sin ^2(t)+\cos ^2(t)-2 t \cos (t),2 t-2 \sin (t) \cos (t),-t \sin^2(t)+t \sin (t)+t \cos ^2(t)-\cos (t)+\sin (t) \cos (t)\right\}$


2131   

$f(x,y,z)$ e $g(x,y)$ são funções diferenciáveis tais que, para todo $(x,y)$ no domínio de $g,f(x,y,g(x,y))=0$. 
Suponha $g(1,1)=3$, $\dfrac{\partial f}{\partial x}(1,1,3)=2$, $\dfrac{\partial f}{\partial y}(1,1,3)=5$ e $\dfrac{\partial f}{\partial z}(1,1,3)=10.$ 
Determine a equação do plano tangente ao gráfico de $g$ no ponto $(1,1,3).$


$\displaystyle z - 3 = -\frac{1}{5}(x - 1) - \frac{1}{2} (y-1).$


2323   

Calcule a área da superfície dada por: ${\bf r}(u,v)=(u,v,2-u-v)$ e $u^{2}+v^{2}\leq 1.$. (Sugerimos ao leitor desenhar a imagem da superfície.)


$\pi \sqrt{3}.$


2212   

Existe um campo vetorial $\bf{G}$ em $\mathbb{R}^3$ tal que $\text{rot }{\bf{G}} = (x\sin{y},\cos{y},z-xy)$? Justifique.



Suponha que existe um campo vetorial $\bf G$ tal que $\text{rot } G= (x\,\sin y, \cos y,z-xy)$. Vamos calcular $\text{div } \text{rot } {\bf G}.$

Temos que

$$\text{div } \text{rot } {\bf G}=\frac{\partial (x\,\sin y) }{\partial x}+\frac{\partial (\cos{y})}{\partial y}+\frac{\partial (z-xy)}{\partial z}$$

$$=\sin y- \sin y +1=1.$$


Sabemos que se ${\bf F}=P\,{\bf i}+Q\,{\bf j}+R\,{\bf k}$ é um campo vetorial sobre $\mathbb{R}^{3}$ e $P$, $Q$ e $R$ têm derivadas parciais de segunda ordem contínuas, então $\text{div } \text{rot } {\bf F}=0.$

Como $\text{div } \text{rot } {\bf G}\neq 0$, pela contrapositiva do resultado acima, temos que ${\bf G}$ não é um campo vetorial do $\mathbb{R}^{3}.$


2900   

Use o método dos multiplicadores de Lagrange para determinar o ponto sobre a parábola $y = x^2$ que se encontra mais próximo do ponto $(0,1) \in \mathbb{R}^2.$


$\displaystyle \left(\frac{1}{\sqrt{2}}, \frac{1}{2} \right)$ e $\displaystyle \left(-\frac{1}{\sqrt{2}}, \frac{1}{2} \right).$


1974   

Mostre que a curva com equações paramétricas $x = t \cos{t}, \ y = t \sin{t}, \ z = t$ está no cone $z^2 = x^2 + y^2$ e use esse fato para esboçar a curva.


2880   

  1. Mostre que o valor máximo de $a^2b^2c^2$ sobre uma esfera de raio $r$ centrada na origem de um sistema de coordenadas cartesianas $(a,b,c)$ é $(r^2/3)^3$.

  2. Usando o item anterior, mostre que, para números não negativos $a$, $b$ e $c$,

    $$(abc)^{\frac{1}{3}} \leq \frac{a + b + c}{3},$$

    isto é, a média geométrica de três números não negativos é menor que ou igual à média aritmética.


  1. Use multiplicadores de Lagrange para maximizar $f(a,b,c) = a^{2}b^{2}c^{2}$ sujeita a restrição $a^{2} + b^{2} + c^{2} = r^{2}.$

  2. Como $(\sqrt{a},\sqrt{b},\sqrt{c})$ está na esfera $a + b + c = r^{2},$ pelo item 1 segue que $abc = f(\sqrt{a},\sqrt{b},\sqrt{c}) \leq \left(\dfrac{r^{2}}{3}\right)^{3} = \left(\dfrac{a + b + c}{3}\right)^{3}.$


2027   

Use a Regra da Cadeia para determinar $\mathrm{d}z/\mathrm{d} t$ ou $\mathrm{d}w/ \mathrm{d}t.$

$z=\sqrt{x^{2}+y^{2}}$, $x=e^{2t}$, $y=e^{-2t}$.


$\displaystyle \frac{dz}{dt} = \frac{2xe^{2t} - 2ye^{2t}}{\sqrt{x^{2} + y^{2}}}.$


2272   

No item abaixo :

  1.  determine o gradiente de $f$; 
  2.  calcule o gradiente no ponto $P$; e 
  3.  determine a taxa de variação de $f$ em $P$ na direção do vetor $\bf{u}$.

$f(x,y) = y \ln{x},  P = (1, -3),  \bf{u} = \left(-\frac{4}{5}, \frac{3}{5} \right)$.


  1. $\nabla f(x,y) = (y/x,\ln(x)).$
  2. $\nabla f(1,-3) = (-3,0).$
  3. $\displaystyle \frac{12}{5}.$


3012   

Encontre o centro de massa de uma lâmina em forma de triângulo retângulo isósceles, com os lados iguais tendo comprimento $a$, se a densidade em qualquer ponto for proporcional ao quadrado da distância do vértice oposto à hipotenusa.


$\displaystyle \left(\frac{2a}{5}, \frac{2a}{5} \right).$


2872   

Utilize os multiplicadores de Lagrange para determinar os valores máximo e mínimo da função sujeita à(s) restrição(ões) dada(s).

$f(x,y) = x^2y; \quad x^2 + 2y^2 = 6.$


Valor máximo: $4;$ valor mínimo: $-4.$


2847   

Passe para coordenadas polares e calcule: $\displaystyle\iint\limits_{R}\cos(x^{2}+y^{2})\,dA$, onde $R$ é a região acima do eixo do $x$ e dentro da circunferência $x^{2}+y^{2}=9.$


$\displaystyle \frac{\pi}{2} \sin(9).$


2903   

Determine os pontos da superfície $xyz = 1$ que estão mais próximos da origem.


$(1,1,1),$ $(1,-1,-1),$ $(-1,1,-1)$ e $(-1,-1,1).$


2865   

Estude com relação a máximos e mínimos a função dada com as restrições dadas.

$f(x,y) = x^2 + 2xy + y^2$ e $x + 2y - 1 = 0.$


Ponto de mínimo: $\displaystyle \left(-1,1 \right)$


1973   

 Esboce o gráfico da curva cuja equação vetorial é dada. Indique com setas a direção na qual o parâmetro cresce.

  1. $\textbf{r}(t) = (t, \cos{2t}, \sin{2t})$
  2. $\textbf{r}(t) = (1 + t, 3t, -t)$


2574   

Calcule $\displaystyle \lim_{(x,y) \to (0,0)} \dfrac{xy}{y - x^3}$, caso exista.


Não existe.


1969   

Calcule o limite $\displaystyle \lim_{t \rightarrow 0}\left( \arctan(t), e^{-2t}, \dfrac{\ln{t}}{t} \right)$.



Tomando
${\bf r}(t)=\left(arctg(t), e^{-2t},\frac{\ln t}{t}\right)$, temos que
$\lim\limits_{t\to 0}{\bf r}(t)=\lim\limits_{t\to 0}\left(arctg(t),e^{-2t},\frac{\ln t}{t}\right)=\left(\lim\limits_{t\to 0}arctg(t),\lim\limits_{t\to 0}e^{-2t},\lim\limits_{t\to 0}\frac{\ln t}{t}\right)$
Assim,
$\bullet \lim\limits_{t\rightarrow 0}arctg(t)=0$.
$\bullet \lim\limits_{t\to 0}e^{-2t}=1$.
$\bullet \lim\limits_{t\to 0}\dfrac{\ln t}{t}$ não existe.

Portanto, $\lim\limits_{t\to 0}\left(arctg(t),e^{-2t},\frac{\ln t}{t}\right)$ não existe.


2933   

Calcule a integral em coordenadas esféricas. $\displaystyle\int_{0}^{3\pi/2}\int_{0}^{\pi}\int_{0}^{1}5\rho^{3}\sin^{3}{\phi}\,d\rho d\phi d\theta$.


$\dfrac{5\pi}{2}.$


2463   

Calcule a integral de superfície $\displaystyle\iint \limits_{ S}{\bf F}\cdot d{\bf S}$ para o campo vetorial ${\bf F}$ e superfície orientada $S$ dados abaixo. Em outras palavras, determine o fluxo de ${\bf F}$ através de $S$. Para superfícies fechadas, use a orientação positiva (para fora).

  • ${\bf F}(x,y,z)=x{\bf i}+y{\bf j}+z{\bf k}$, $S$ é a esfera $x^{2}+y^{2}+z^{2}=9.$


$108\pi.$


2488   

Esboce o gráfico da função $f(x,y)=y^{2}+1$.


$z = y^{2} + 1$

A expressão 1 cossec x e o mesmo que


2303   

Identifique e faça um esboço da imagem da superfície parametrizada dada por ${\bf r}(u,v)=(v\,\cos u,v\sin u,v)$, $0\leq u\leq 2\pi$,\, $0\leq v \leq h$, onde $h>0$ é um real dado.


Face lateral do cone $\sqrt{x^{2} + y^{2}} \leq z \leq h$.


3147   

  1.  Use o Teorema de Green para provar que\[ \int_Cf(x)\,dx + g(y)\,dy = 0\] se \(f\) e \(g\) forem funções diferenciáveis e \(C\) for uma curva fechada simples lisa por partes.

  2.  O que isso nos diz sobre o campo vetorial \[ \mathbf{F}(x,y) = f(x)\mathbf{i}+g(y)\mathbf{j}?\]


2469   

Calcule a integral de superfície $\displaystyle\iint \limits_{ S}{\bf F}\cdot d{\bf S}$ para o campo vetorial ${\bf F}$ e superfície orientada $S$ dados abaixo. Em outras palavras, determine o fluxo de ${\bf F}$ através de $S$. Para superfícies fechadas, use a orientação positiva (para fora).

  • ${\bf F}(x,y,z)=(x+y){\bf i}+z{\bf j}+xz{\bf k}$ e $S$ é a superfície do cubo de vértices $(\pm 1,\pm 1, \pm 1).$


$8.$


3141   

Supondo que \(\sigma\) e \(G\) satisfaçam as hipóteses do Teorema da Divergência e que \(f\) e \(g\) sejam funções suficientemente regulares, prove as seguintes identidades (de Green):

  1.  \[\iint\limits_\sigma\left(f\nabla g\right)\cdot\mathbf{n}\,dS = \iiint\limits_G\left( f\Delta g+\nabla f\cdot\nabla g\right)\,dV, \]

  2.  \[\iint\limits_\sigma\left(f\nabla g-g\nabla f\right)\cdot\mathbf{n}\,dS = \iiint\limits_G\left( f\Delta g- g\Delta f\right)\,dV, \] onde \(\displaystyle \Delta f= \dfrac{\partial^2f}{\partial x^2}+\dfrac{\partial^2f}{\partial y^2}+\dfrac{\partial^2f}{\partial z^2}\) é denominado Laplaciano de \(f\).


2802   

Determine os valores máximos e mínimos locais e pontos de sela da função $f(x,y)=x^{2}+y^{3}+xy-3x-4y+5$.


Ponto de mínimo : $\displaystyle \left( 1,1\right);$ ponto de sela: $\displaystyle \left(\frac{23}{12},-\frac{5}{6}\right).$


2541   

Encontre o fluxo do campo ${\bf F}$ ao longo da porção da superfície dada no sentido especificado.

  • ${\bf F}(x,y,z)=-{\bf i}+2{\bf j}+3{\bf k}$; $S$ é a superfície retangular $z=0$, $0\leq x\leq 2$, $0\leq y \leq 3$, sentido ${\bf k}.$


3013   

A função densidade conjunta para um par de variáveis aleatórias $X$ e $Y$ é $$f(x,y) = \begin{cases} Cx(1 + y), & \quad \text{se } 0 \leq x \leq 1, \ 0 \leq y \leq 2,\\ 0, & \quad \text{caso contrário}.
\end{cases}$$

  1.  Determine a constante $C$.

  2.  Determine $P(X \leq 1, \ Y \leq 1)$.

  3.  Determine $P(X + Y \leq 1)$.


  1. $\dfrac{1}{2}.$

  2. $\dfrac{3}{8}.$

  3. $\dfrac{5}{48}$.


2970   

Calcule a integral, transformando para coordenadas esféricas. $\displaystyle\int_{-a}^{a}\int_{-\sqrt{a^{2}-y^{2}}}^{\sqrt{a^{2}-y^{2}}}\int_{-\sqrt{a^{2}-x^{2}-y^{2}}}^{\sqrt{a-x^{2}-y^{2}}}(x^{2}z+y^{2}z+z^{3})\,dzdxdy$.


$0.$


2682   

Encontre $\partial f/\partial x$ e $\partial f/\partial y$ para $f(x,y)=1/(x+y)$.


$\displaystyle \frac{\partial f}{\partial x} = \frac{\partial f}{\partial y} = -\frac{1}{(x^{2} + y^{2})^{2}}$.


2752   

A função $f(x,y) = \begin{cases}\dfrac{x^2 - y^2}{x^2 + y^2}, & \quad \text{se } (x,y) \neq (0,0),\\0, & \quad \text{se } (x,y) = (0,0)\\\end{cases}$ é diferenciável em $(0,0)$? Justifique.


Não.


3018   

Esboce a região de integração para a integral iterada $\displaystyle\int_{0}^{1}\!\!\int_{\sqrt{y}}^{3\sqrt{y}}f(x,y)\,dx dy$.


A expressão 1 cossec x e o mesmo que


3093   

Use um software de apoio computacional para mostrar que o volume \(V\) sob a superfície \(z=xy^3\sin(xy)\) e acima do retângulo \([0,\pi]\times[0,1]\) no plano \(xy\)  é dado por \(V=3/\pi\).


2346   

Encontre a área da parte da esfera $x^{2}+y^{2}+z^{2}=a^{2}$ que está dentro do cilindro $x^{2}+y^{2}=ax.$


$2a^2 (\pi - 2).$


2665   

Determine as derivadas parciais de $z=\dfrac{x\sin{y}}{\cos(x^{2}+y^{2})}$.


$\begin{aligned}[t]\frac{\partial z}{\partial x} &= \frac{\sin y ( \cos(x^{2} + y^{2}) + 2x^{2} \sin(x^{2} + y^{2}))}{(\cos(x^{2} + y^{2}))^{2}}\;\;\;\;\;\;\text{e}\\\frac{\partial z}{\partial y} &= \frac{x \cos y \cos(x^{2} + y^{2}) + 2xy \sin y \sin(x^{2} + y^{2})}{(\cos(x^{2} + y^{2}))^{2}}.\end{aligned}$


2643   

Determine a derivada parcial $f_{x}(3,4)$, onde $f(x,y)=\ln(x+\sqrt{x^{2}+y^{2}}).$


$f_{x}(3,4) = \frac{1}{5}$.


2519   

Dada a expressão $g(x,y)=-f(x,y)$, escreva como o gráfico de $g$ é obtido a partir do gráfico de $f.$


Gráfico de $f$ refletido sobre o plano $xy.$


2038   

 Utilize um diagrama em árvore para escrever a Regra da Cadeia para o caso dado. Suponha que todas as funções sejam diferenciáveis.

$w=f(r,s,t)$, onde $r=r(x,y)$, $s=s(x,y)$, $t=t(x,y)$.


$\displaystyle \frac{\partial w}{\partial x} = \frac{\partial w}{\partial r}\frac{\partial r}{\partial x} + \frac{\partial w}{\partial s}\frac{\partial s}{\partial x} + \frac{\partial w}{\partial t}\frac{\partial t}{\partial x}$ e $\displaystyle \frac{\partial w}{\partial y} = \frac{\partial w}{\partial r}\frac{\partial r}{\partial y} + \frac{\partial w}{\partial s}\frac{\partial s}{\partial y} + \frac{\partial w}{\partial t}\frac{\partial t}{\partial y}$


2400   

 Determine os pontos da superfície $x^2 + 2y^2 + 3z^2 = 1$ nos quais o plano tangente é paralelo ao plano $3x - y + 3z = 1$.


$\displaystyle \left(\frac{3\sqrt{2}}{5}, - \frac{1}{5\sqrt{2}}, \frac{\sqrt{2}}{5} \right)$ e $\displaystyle \left(-\frac{3\sqrt{2}}{5}, \frac{1}{5\sqrt{2}}, -\frac{\sqrt{2}}{5} \right).$


2024   

Se $f$ é uma função constante, $f(x,y) = k$, e $R = [a,b] \times [c,d],$ mostre que $\iint \limits_{R} k \, dA = k(b-a)(d-c).$


Note que se $R$ for dividida em $mn$ subretângulos, vale $$ \sum^{m}_{i = 1} \sum^{n}_{j = 1} f(x_{ij}^{*}, y_{ij}^{*}) \Delta A = k \sum^{m}_{i = 1} \sum^{n}_{j = 1} \Delta A = k (b - a) (d - c), $$ independentemente dos pontos amostrais $(x_{ij}^{*}, y_{ij}^{*})$ escolhidos.


3044   

Calcule a área sob um arco da cicloide $x = t-\sin{t}$, $y = 1-\cos{t}$.


Queremos determinar a área da região $R$ mostrada na figura abaixo.

A expressão 1 cossec x e o mesmo que

Sabemos que, se $y = f(x)$, então a integral $\int_{a}^{b}f(x)dx$ calcula a área que está abaixo do gráfico de $f$ e acima do eixo $x$, com $x$ variando entre $a$ e $b$. A princípio, poderíamos tentar encontrar uma expressão que relacionasse $x(t)$ e $y(t)$ na parametrização da cicloide, mas esse parece ser um trabalho difícil. Usaremos então o que foi provado no exercício anterior. Temos que
$$A(R) = \oint_{C}x\, dy,$$
em que $C = C_1 \cup C_2$ é a curva descrita na figura a seguir.

A expressão 1 cossec x e o mesmo que

Uma parametrização de $C_1$ é $r_1(t) = (x_1(t),y_1(t)) = (t,0)$, em que $0 \leq t \leq 2\pi$. Nesse caso, $y_1'(t) = 0$. Logo,
$$\oint_{C_1}x\, dy = \int_{0}^{2\pi}(t)(0)\, dt = 0.$$
Uma parametrização de $C_2$ é $r_2(t) = (x_2(t),y_2(t)) = (t-\sin{t},1-\cos{t})$, em que $t$ varia de $2\pi$ a $0$. Nesse caso, $y_2'(t) = \sin{t}$. Logo,
$$\begin{array}{rcl}\displaystyle \oint_{C_2}x\, dy & = & \displaystyle \int_{2\pi}^{0}(t-\sin{t}) (\sin{t})\, dt \\ & = & \displaystyle \int_{0}^{2\pi}(\sin^2{t} - t\sin{t}) \, dt \\ & = & \displaystyle \int_{0}^{2\pi}\frac{1-\cos(2t)}{2}\, dt - \int_{0}^{2\pi}t\sin{t} \, dt \\ & = & \pi + 2\pi = 3\pi.\end{array}$$
Portanto, a área da região é $3\pi$.
(Observe que, para resolver a integral $\int_{0}^{2\pi}t\sin{t} \, dt$, usamos integração por partes com $u=t$ e $dv=\sin{t}\,dt$.)


2764   

Determine o maior conjunto de pontos em que a função $f(x,y) = \begin{cases}e^{\dfrac{1}{x^2 + y^2 - 1}}, & \quad \text{se } x^2 + y^2 < 1,\\0, & \quad \text{se } x^2 + y^2 \geq 1\end{cases}$ é diferenciável. Justifique.


$\mathbb{R}^{2}$.


2460   

Seja $f(x,y)=x^{2}e^{3xy}$.

  1. Calcule $f(2,0).$

  2. Determine o domínio de $f$.

  3. Determine a imagem de $f$.


  1. $4.$

  2. $\mathbb{R}^{2}.$

  3. $[0,\infty).$



  1. Queremos calcular $f(2,0)$ sabendo que $f(x,y)=x^{2}e^{3xy}$. Basta substituir os valores na expressão, assim temos
    \[
    f(2,0)=2^{2}e^{3 \cdot 2 \cdot 0}=4e^{0}=4 \cdot 1=4.
    \]
  2. Por definição, o domínio da função $f$ é o conjunto dos pontos de $\mathbb{R}^{2}$ em que a função está bem definida. Em nosso caso, o domínio é o conjunto de pontos em que a função $f(x,y)=x^{2}e^{3xy}$ está bem definida. Portanto, o domínio de $f$ é $\mathbb{R}^2$.
  3. A imagem da função $f$ é o conjunto dos pontos $\{ z\in \mathbb{R} | z = f(x,y) \text{ e } (x,y)\in D\}$, onde $D$ é o domínio de $f$. Observe que $x^{2} \geq 0$ e $e^{3xy}> 0$ para todo $(x,y)\in \mathbb{R}^{2}$, logo $x^{2}e^{3xy}\geq 0$ para todo $(x,y)\in \mathbb{R}^{2}$. Por exemplo, se fixarmos $x=1$ temos que a imagem da função são os pontos da forma $e^{3y}$ com $y\in \mathbb{R}$, ou seja, é todo o intervalo $(0,\infty)$. Agora, quando colocamos $x=0$ a imagem é $0$. Portanto, a imagem de $f$ é o conjunto $[0,\infty]$.


2078   

Calcule a integral dupla.

  1. $\displaystyle\iint\limits_{R} x\sin(x+y)\,dA, \quad R=[0,\pi/6]\times [0,\pi/3].$

  2.  $\displaystyle\iint\limits_{R} xye^{x^{2}y}\,dA, \quad R=[0,1]\times [0,2].$


  1. $\dfrac{\pi}{12}.$

  2. $\dfrac{(e^{2} - 3)}{2}.$


1948   

Calcule o limite $\displaystyle \lim_{t \rightarrow 0}\left(\dfrac{e^t - 1}{t}, \dfrac{\sqrt{1+t}-1}{t}, \dfrac{3}{t+1}\right)$.



Consideremos ${\bf r}(t)=\bigg(\frac{e^{t}-1}{t},\frac{\sqrt{1+t}-1}{t},\frac{3}{t+1}\bigg).$

Temos que o limite de ${\bf r}$ é o vetor cujas componentes são os limites das funções componentes de ${\bf r}$, se esses limites existirem.

Então,

$\lim\limits_{t \to 0}{\bf r}(t)=\lim\limits_{t \to 0}\left(\frac{e^{t}-1}{t},\frac{\sqrt{1+t}-1}{t},\frac{3}{t+1}\right)=\left(\lim\limits_{ t\to 0}\frac{e^{t}-1}{t},\lim\limits_{t \to 0}\frac{\sqrt{1+t}-1}{t},\lim\limits_{t \to 0}\frac{3}{t+1}\right)$

Assim,

$\bullet \lim\limits_{t\rightarrow 0}\dfrac{e^{t}-1}{t}=1$.

$\bullet \lim\limits_{t\to 0}\frac{\sqrt{1+t}-1}{t}=\lim\limits_{t\to 0}\frac{(\sqrt{1+t}-1)(\sqrt{1+t}+1)}{t(\sqrt{1+t}+1)}=\lim\limits_{t\to 0}\frac{1+t-1}{t(\sqrt{1+t}+1)}=\lim\limits_{t\to 0}\frac{t}{t(\sqrt{1+t}+1)}=\lim\limits_{t\to 0}\frac{1}{\sqrt{1+t}+1}=\frac{1}{2}$.

$\bullet \lim\limits_{t\to 0}\dfrac{3}{t+1}=3$.

Portanto,

$\lim\limits_{t\to 0}\bigg(\frac{e^{t}-1}{t},\frac{\sqrt{1+t}-1}{t},\frac{3}{t+1}\bigg)=\bigg(1,\frac{1}{2},3\bigg).$


2500   

Dada a função $f(x,y)=e^{-(x^{2}+y^{2})}$.

  1. Encontre o domínio da função.

  2. Encontre a imagem da função.

  3. Descreva as curvas de nível da função.


  1. $D_{f} = \mathbb{R}^{2}$.

  2. $Im(f) = \left\lbrace z \in \mathbb{R};\; 0 < z \leq 1 \right\rbrace.$

  3. As curvas de nível são as os círculos $x^{2} + y^{2} = C$ com $C > 0$ e a origem.


2491   

Dada a função $f(x,y)=y-x$.

  1. Encontre o domínio da função.

  2. Encontre a imagem da função.

  3. Descreva as curvas de nível da função.


  1. $D_{f} = \mathbb{R}^{2}$.

  2. $Im(f) = \mathbb{R}.$

  3. As curvas de nível são as retas $y - x = C.$


1992   

Determine ${\bf r}(t)$ sabendo que

  1. ${\bf r}'(t)=t{\bf i}+2{\bf k}$ e ${\bf r}(0)={\bf i}+{\bf j}.$
  2. ${\bf r}'(t)=\sin(t){\bf i}+\cos(2t){\bf j}+\dfrac{1}{t+1}{\bf k}$, $t\geq 0$ e ${\bf r}(0)={\bf i}-{\bf j}+2{\bf k}.$
  3. ${\bf r}'(t)=\dfrac{1}{1+4t^{2}}{\bf i}+e^{-t}{\bf j}+{\bf k}$ e ${\bf r}(0)={\bf k}.$


2440   

Calcule a integral de superfície $\displaystyle\iint\limits_{S}\sqrt{1+x^{2}+y^{2}}dS$, onde $S$ é o helicóide com equação vetorial ${\bf r}(u,v)=u\cos v{\bf i}+u\sin v{\bf j}+v{\bf k}$, $0 \leq u \leq 1$, $0 \leq v \leq \pi.$


$\dfrac{4\pi}{3}.$


2054   

Determine se ${\bf F}(x,y)=e^{x}\,\cos y\,{\bf i}+e^{x}\,\sin y\,{\bf j}.$ é ou não um campo vetorial conservativo. Se for, determine uma função $f$ tal que ${\bf F}=\nabla f.$


Não.


2947   

Calcule utilizando coordenadas esféricas. $\displaystyle\iiint\limits_{B}\sqrt{x+y}\sqrt[3]{x+2y-z}\,dxdydz$, onde $B$ é a região $1\leq x+y\leq 2$, $0\leq x+2y-z\leq 1$ e $0\leq z\leq 1.$


$\sqrt{2} - \dfrac{1}{2}.$


2421   

Calcule a integral tripla $\displaystyle\iiint\limits_{B}xyz^{2}\,dV$, onde $B$ é a caixa retangular dada por $B=\{(x,y,z) \in \mathbb{R}^3|\;0\leq x\leq 1,\;-1\leq y\leq 2,\;0\leq z\leq 3\}$, integrando primeiro em relação a $y$, depois a $z$ e então a $x$.


$\dfrac{27}{4}.$


2093   

Seja

$${\bf F}(x,y)=\bigg(\frac{-y}{x^{2}+y^{2}},\frac{x}{x^{2}+y^{2}}+3y\bigg)$$

um campo vetorial em $\mathbb{R}^{2}$. Calcule a integral de linha do campo ${\bf F}$ ao longo das curvas

$C_{1}$ e $C_{2}$, orientadas no sentido anti-horário, onde:

  1. $C_{1}$ é a circunferência de equação $x^{2}+y^{2}=4.$

  2. $C_{2}$ é a fronteira do retângulo $R=\{(x,y)\in \mathbb{R}^{2}|\,-\pi \leq x \leq \pi,-3 \leq y \leq 3\}.$


  1. $0.$

  2. $0.$


2639   

Determine as derivadas parciais de primeira ordem da função $f(x,y)=\dfrac{x-y}{x+y}$.


$\displaystyle \frac{\partial f}{\partial x} = \frac{2y}{(x + y)^{2}}\;\;\;\text{e}\;\;\; \frac{\partial f}{\partial y} = -\frac{2x}{(x + y)^{2}}$.


2785   

Calcule a integral dupla usando coordenadas polares: $\displaystyle\iint\limits_{R}\sqrt{x^{2}+y^{2}}\,dA$, onde $R$ é limitado pelo círculo $y=\sqrt{2x-x^{2}}$ e pela reta $y=x.$


$\displaystyle \frac{8}{9}(2 - \frac{5}{4}\sqrt{2}).$


2380   

Determine $\int_{0}^{5}f(x,y)\,dx$ e $\int_{0}^{1}f(x,y)\,dy$, sendo $f(x,y)=12x^{2}y^{3}.$


$\int_{0}^{5} 12x^{2}y^{3} \,dx = 500y^{3}$ e $\int_{0}^{1} 12x^{2}y^{3} \,dy = 3x^{2}.$


1966   

Calcule a integral de linha $\int_{C}{\bf F}\cdot d{\bf r}$, onde ${\bf F}(x,y)=e^{x-1}\,{\bf i}+xy\,{\bf j}$ e $C$ é dada por ${\bf r}(t)=t^{2}\,{\bf i}+t^{3}\,{\bf j},   0\leq t\leq 1.$


$\displaystyle \frac{11}{8} - \frac{1}{e}.$


2048   

Determine se o conjunto $\{(x,y)|\,x\neq 0\}$ é ou não:

  1. aberto;

  2. conexo; e

  3. simplesmente conexo.



Temos que o conjunto $D=\{(x,y)|\,x\neq 0\}$ consiste de todos os pontos, exceto para aqueles que encontram-se sobre o eixo y. Então:

  1. $D$ é aberto.

  2. Os pontos em lados opostos do eixo $y$ não podem ser conectados por um caminho que se encontra totalmente em $D$, então $D$ não é conexo.

  3. $D$ não é simplesmente conexo, pois não é conexo.


2084   

Calcule $\displaystyle\int_{C}y\,dx+x^{2}\,dy$, onde $C$ é a curva cuja imagem é o segmento de extremidades $(1,1)$ e $(2,2)$, orientada de $(1,1)$ para $(2,2).$


$\dfrac{23}{6}.$


2962   

Usando coordenadas esféricas, determine o centroide e o momento de inércia em relação a um diâmetro de sua base do hemisfério sólido homogêneo de raio $a.$


Centróide: $\left(0,0,\dfrac{3a}{8} \right);$ momento de inércia: $\dfrac{4 K a^5 \pi}{15},$ onde $K$ é a densidade constante.


2352   

Calcule o volume do conjunto dado.

  1.  $0\leq y\leq 1-x^{2}$ e $0\leq z\leq 1-x^{2}.$

  2.  $x^{2}+y^{2}+3\leq z\leq 4.$


  1.  $\dfrac{16}{15}.$

  2.  $\dfrac{\pi}{2}.$


2740   

Considere a função $f(x,y) = x \ \phi\left(\frac{x}{y}\right)$, em que $\phi(u)$ é uma função derivável de uma variável. Mostre que os planos tangentes ao gráfico de $f$ passam pela origem.


Note que $x \frac{\partial f}{\partial x} (x,y) + y \frac{\partial f}{\partial y}(x,y) = f(x,y).$


2055   

Determine se ${\bf F}(x,y)=(e^{x}\,\sin y)\,{\bf i}+(e^{x}\,\cos y)\,{\bf j}.$ é ou não um campo vetorial conservativo. Se for, determine uma função $f$ tal que ${\bf F}=\nabla f.$


Sim. $f(x,y) = e^{x}\sin(y) + K.$


2381   

Calcule a integral iterada.

  1. $\displaystyle\int_{1}^{3} \!\! \int_{0}^{1}(1+4xy)\, dx dy$

  2. $\displaystyle\int_{2}^{4}\!\!\int_{-1}^{1}(x^{2}+y^{2})\,dy dx$


  1.  $10.$

  2.  $\dfrac{116}{3}.$


2210   

Considere a integral

$$\int_{0}^{2}\int_{\frac{y}{2}}^{1}ye^{x^{3}}\,dx dy.$$

  1.  Faça um esboço da região de integração.

  2.  Calcule a integral sendo explícito se vai precisar mudar a ordem de integração.


  1. ...

  2.  $\dfrac{2(e - 1)}{3}.$


2450   

Calcule $\displaystyle\iint\limits_{S}g(x,y,z)dS,$ onde $g(x,y,z)=(x^{2}+y^{2}+z^{2})^{1/2}$ e $S$ é a porção do parabolóide $2z=x^{2}+y^{2}$ interior ao cilindro $x^{2}+y^{2}=2y.$


$\dfrac{5\pi}{2}.$


2405   

Inverta a ordem de integração.

  1.  $\displaystyle\int_{0}^{1}\bigg[\displaystyle\int_{0}^{x}f(x,y)\,dy\bigg]dx$

  2.  $\displaystyle\int_{0}^{1}\bigg[\int_{x^{2}}^{x}f(x,y)\,dy\bigg]dx$

  3.  $\displaystyle\int_{0}^{1}\bigg[\int_{-\sqrt{y}}^{\sqrt{y}}f(x,y)\,dx\bigg]dy$


  1.  $\displaystyle\int_{0}^{1}\bigg[\displaystyle\int_{y}^{1}f(x,y)\,dx\bigg]dy$

  2.  $\displaystyle\int_{0}^{1}\bigg[\int_{y}^{\sqrt{y}}f(x,y)\,dx\bigg]dy$

  3.  $\displaystyle\int_{-1}^{1}\bigg[\int_{x^2}^{1}f(x,y)\,dy\bigg]dx$


2366   

Em que direção e sentido a função dada cresce mais rapidamente no ponto dado? E em que direção e sentido decresce mais rapidamente?

$f(x,y) = x^2 + xy + y^2$ em $(1,1)$.


Cresce: $(3,3)$; descresce: $(-3,-3).$


2526   

Seja $f(x,y)=3x+2y.$ Calcule:

  1. $f(1,-1)$;

  2. $f(a,x)$;

  3. $\dfrac{f(x+h,y)-f(x,y)}{h}$;

  4. $\dfrac{f(x,y+k)-f(x,y)}{k}$.


  1. $1.$

  2. $3a + 2x.$

  3. $3.$

  4. $2.$


2569   

Calcule $\displaystyle \lim_{(x,y) \to (0,0)} \dfrac{x}{\sqrt{x^2 + y^2}}$, caso exista.


Não existe.


2266   

Calcule $\displaystyle\iint\limits_{B}f(x,y)\,dx dy$ sendo dados:

  1.  $f(x,y)=1$ e $B$ a região compreendida entre os gráficos de $y=\sin{x}$ e $y=1-\cos{x}$, com $0\leq x\leq \dfrac{\pi}{2}.$

  2.  $f(x,y)=\sqrt{1+y^{3}}$ e $B=\{(x,y)\in \mathbb{R}^{2}|\;\sqrt{x}\leq y\leq 1 \}.$

  3.  $f(x,y)=x$ e $B$ é o conjunto de todos $(x,y)$ tais que $y\geq x^{2}$ e   $x\leq y\leq x+2.$

  4.  $f(x,y)=\dfrac{y}{x+y^{2}}$ e $B$ o conjunto de todos $(x,y)$ tais que $1\leq x\leq 4$ e $0\leq y\leq \sqrt{x}.$


  1.  $2 - \dfrac{\pi}{2}.$

  2.  $\dfrac{2(2\sqrt{2} - 1)}{9}.$

  3.  $\dfrac{13}{6}.$

  4.  $\dfrac{3 \ln(2)}{2}.$


2990   

Calcule a integral, efetuando uma mudança de variáveis apropriada. $\displaystyle\iint\limits_{R} x^2 \, dA$, em que $R$ é o conjunto de todos $(x,y)$ tais que $4x^2 + y^2 \leq 1$.


$\dfrac{\pi}{32}.$


2100   

Considere o campo vetorial

$${\bf F}(x,y)=(\ln(y^{2}+1))\,{\bf i}+\bigg(\frac{2y(x-1)}{y^{2}+1}\bigg)\,{\bf j}.$$

  1. Determine se $F$ é ou não um campo conservativo.

  2. Determine o trabalho realizado pelo campo vetorial ${\bf F}$ ao mover uma partícula desde o ponto $(-1,1)$ até o ponto $(2,3).$


  1. Sim.

  2. $\ln(10) + 2\ln(2).$


2076   

Seja $R$ o retângulo $1\leq x\leq 2$, $0\leq y\leq 1$. Calcule $\iint\limits_{R} f(x,y)\,dxdy$, sendo $f(x,y)$ igual a

  1. $\dfrac{1}{(x+y)^{2}}$

  2. $\dfrac{1}{1+x^{2}+2xy+y^{2}}$


  1.  $\dfrac{3}{\pi}.$

  2.  $3\arctan(3) - 4\arctan(2) - \ln(2) + \dfrac{\ln(5)}{2} + \dfrac{\pi}{4}.$


3157   

Considerando um campo vetorial $\mathbf{F}$ que representa a velocidade em um fluido, a interpretação do rotacional é que partículas em um ponto $(x,y,z)$ de um fluido tendem a rodar em torno do eixo que aponta na direção de $\text{rot }\mathbf{F}(x,y,z)$. Se $\mathbf{F}$ representar a velocidade da corrente de um rio calmo, que corre somente da direção montante à jusante, podemos dizer que $\text{rot } \mathbf{F}$ é igual ou diferente de zero? Por quê? Para auxiliar na interpretação, faça um esboço do gráfico de $\mathbf{F}$ assumindo que ele não varia na direção $z$.


Igual a zero.


3040   

  1. Definimos a integral imprópria (sobre todo o plano $\mathbb{R}^{2}$) $$I=\displaystyle\iint\limits_{ \mathbb{R}^{2}}e^{-(x^{2}+y^{2})}\,dA=\int_{-\infty}^{\infty} \int_{-\infty}^{\infty}e^{-(x{^2}+y^{2})}\,dy dx= \lim_{a\rightarrow\infty}\displaystyle\iint\limits_{D_{a}}e^{-(x^{2}+y^{2})}\,dA,$$ onde $D_{a}$ é o disco com raio $a$ e centro na origem. Mostre que $$\int_{-\infty}^{\infty} \int_{-\infty}^{\infty}e^{-(x^{2}+y^{2})}\,dA=\pi.$$

  2. Uma definição equivalente da integral imprópria da parte (a) é $$\iint\limits_{\mathbb{R}^{2}}e^{-(x^{2}+y^{2})}\,dA=\lim_{a\rightarrow\infty}\displaystyle\iint\limits_{S_{a}}e^{-(x^{2}+y^{2})}\,dA,$$ onde $S_{a}$ é o quadrado com vértices $(\pm a,\pm a)$. Use esse resultado para mostrar que $$\int_{-\infty}^{\infty}e^{-x^{2}}\,dx\,\int_{-\infty}^{\infty}e^{-y^{2}}\,dy=\pi.$$

  3. Deduza que $$\int_{-\infty}^{\infty}e^{-x^{2}}\,dx=\sqrt{\pi}.$$

  4. Fazendo a mudança de variável $t=\sqrt{2} x$, mostre que $$\int_{-\infty}^{\infty}e^{-x^{2}/2} dx=\sqrt{2\pi}.$$

    (Este é um resultado fundamental em probabilidade e estatística.)


  1. Note que $$\displaystyle\iint\limits_{D_{a}}e^{-(x^{2}+y^{2})}\,dA =\int_{0}^{2\pi}\int_{0}^{a} r e^{-r^2} dr d\theta = \pi (1 - e^{-a^2})$$ para cada $a.$

  2. Note que $$\int\limits_{S_{a}}e^{-(x^{2}+y^{2})}\,dA = \int_{-a}^{a} \int_{-a}^{a} e^{-x^2} e^{-y^2} dxdy = \left(\int_{-a}^{a} e^{-x^2} dx\right) \left(\int_{-a}^{a} e^{-y^2} dy\right) $$ para cada $a.$

  3. Troque $y$ por $x$ no item (b).

  4. Note que fazendo a mudança de variável sugerida, $$\int_{-\infty}^{\infty}e^{-x^{2}/2} dx= \frac{1}{\sqrt{2}}\int_{-\infty}^{\infty}e^{-t^{2}/2} dt = \sqrt{\pi}.$$


1953   

Calcule a integral de linha $\int_{C}{\bf F}\cdot d{\bf r}$, onde $C$ é dada pela função vetorial ${\bf r}(t).$

${\bf F}(x,y,z)=(x+y+z)\,{\bf k}$, ${\bf r}(t)=(t,t,-t^{2})$, $0\leq t\leq 1.$


$-\dfrac{11}{6}.$


2254   

Demonstre a identidade abaixo, supondo que $S$ e $E$ satisfaçam as condições do Teorema do Divergente e que as funções escalares e as componentes dos campos vetoriais tenham derivadas parciais de segunda ordem contínuas.

  1. $\displaystyle\iint\limits_{S} D_{n}f\,dS=\displaystyle\iiint\limits_{E}\nabla^{2}f\,dV.$



Lembre que $D_{n} f = \nabla f \cdot {\bf b}$ e $\mbox{div} (\nabla f) = \nabla^{2} f.$


2961   

Usando coordenadas esféricas, determine o volume do sólido que está dentro da esfera $x^{2}+y^{2}+z^{2}=4$, acima do plano $xy$ e abaixo do cone $z=\sqrt{x^{2}+y^{2}}.$


$\dfrac{8\sqrt{2}\pi}{3}.$


2640   

Determine as derivadas parciais de primeira ordem da função $u=\sqrt{x_{1}^{2}+x_{2}^{2}+\cdot \cdot \cdot +x_{n}^{2}}$.


$\displaystyle \frac{\partial u}{\partial x_{i}}= \frac{x_{i}}{\sqrt{x_{1}^{2}+x_{2}^{2}+\cdot \cdot \cdot +x_{n}^{2}}}$ para todo $i = 1, \cdots, n$.


2555   

Determine o limite, se existir, ou mostre que o limite não existe.

$\displaystyle \lim_{(x,y) \to (0,0)} \dfrac{y^4}{x^4 + 3y^4}$.


Não existe.


2522   

Esboce o gráfico da função $f(x,y)=e^{\sqrt{x^{2}+y^{2}}}$. Em geral, se $g$ é uma função de uma variável, como saber o gráfico de $f(x,y)=g(\sqrt{x^{2}+y^{2}})$ a partir do gráfico de $g$?


O gráfico de $f(x,y) = g(\sqrt{x^{2} + y^{2}})$ pode ser obtido rotacionando o gráfico de $g$ no plano $xz$ ao redor do eixo $z.$

A expressão 1 cossec x e o mesmo que


2344   

  1. Determine, mas não calcule, a integral dupla da área da superfície com as equações paramétricas $x=au\cos v$, $y=bu\sin v$, $z=u^{2}$, $0\leq u\leq 2$, $0\leq v\leq 2\pi.$

  2. Elimine os parâmetros para mostrar que a superfície é um paraboloide elíptico e escreva outra integral dupla que forneça sua área.


  1. $\displaystyle \int^{2\pi}_{0}\int_{0}^{2} \sqrt{4b^2 u^4 \cos^{2}v + 4a^2 u^4 \sin^{2} v + a^2 b^2 u^2} dudv.$

  2. $\displaystyle \int_{-2a}^{2a} \int^{b \sqrt{4 - \frac{x^2}{a^2}}}_{-b \sqrt{4 - \frac{x^2}{a^2}}} \sqrt{1 + \left(2\frac{x}{a^2}\right)^{2} + \left(2\frac{y}{b^2} \right)^{2}} dydx.$


2378   

É dada uma curva $\gamma$ que passa pelo ponto $\gamma(t_0) = (1,3)$ e cuja imagem está contida na curva de nível $x^2 + y^2 = 10$. Suponha $\gamma'(t_0) \neq \bf{0}$.

  1.  Determine a equação da reta tangente a $\gamma$ no ponto $(1,3)$.
  2.  Determine uma curva $\gamma(t)$ satisfazendo as condições acima.


  1. $(x,y) = (1,3) + \lambda (-6,2),$ $\lambda \in \mathbb{R}.$
  2. $\gamma(t) = (\sqrt{10} \cos(t), \sqrt{10} \sin(t)).$


2118   

Calcule $\mathrm{d} z/\mathrm{d} t$ por dois processos:

  1. substituindo as expressões para $x(t)$ e $y(t)$ em $z$ e depois derivando diretamente com relação a $t$
  2. aplicando a Regra da Cadeia: $\frac{dz}{dt}=\frac{\partial z}{\partial x}\frac{dx}{dt}+\frac{\partial z}{\partial y }\frac{dy}{dt}$.

$z=x^{2}+3y^{2}$,$x=\sin{t}$ e  $y=\cos{t}.$


$\displaystyle \frac{dz}{dt} (t) = -4\sin(t)\cos(t).$


2222   

Seja $f$ um campo escalar e $\mathbf{F}$ um campo vetorial. Diga se cada expressão tem significado. Em caso negativo, explique por quê. Em caso afirmativo, diga se é um campo vetorial ou escalar. 

  1. $\text{grad }{(\text{div }{f})}$;

    $\text{div }{(\text{div }{\mathbf{F}})}$;

    $\text{div }{(\text{rot }{(\text{grad }{f})})}$.


  1. $\text{grad }{(\text{div }{f})}$ não tem significado, pois $f$ é um campo escalar.

  2. $\text{div }{(\text{div }{\mathbf{F}})}$ não tem significado pois $\text{div } \bf{F}$ é um campo escalar.

  3. $\text{div }{(\text{rot }{(\text{grad }{f})})}$ é um campo escalar.


3139   

Prove a seguinte identidade \[ \iint\limits_\sigma\mathrm{rot\,}\mathbf{F}\cdot\mathbf{n}\,dS = 0, \] supondo que \(\mathbf{F}\) e \(\sigma\) satisfaçam as hipóteses do Teorema da Divergência.


2228   

Determine se o campo vetorial $\mathbf{F}(x,y,z) = y\cos{xy}\mathbf{i} + x\cos{xy}\mathbf{j} - \sin{z}\mathbf{k}$ é conservativo ou não. Se for conservativo, determine uma função $f$ tal que $\mathbf{F} = \nabla{f}$.


$\mathbf{F}$ é conservativo. $f(x,y,z) = \sin(xy) + \cos(z).$


3020   

Esboce a região de integração e calcule a integral $\displaystyle\int_{0}^{3}\!\!\int_{0}^{2}(4-y^{2})\,dy dx$.


$16.$

A expressão 1 cossec x e o mesmo que


2286   

Identifique a superfície que tem equação paramétrica ${\bf r}(u,v)=2\,\sin u\,{\bf i}+3\,\cos u\,{\bf j}+v\,{\bf k}$, $0\leq v\leq 2.$.


$\dfrac{x^2}{4} + \dfrac{y^{2}}{9} = 1,$ com $0\leq z \leq 2.$


3038   

Esboce a região cuja área é dada pela integral e calcule-a, sendo: $\displaystyle\int_{\pi}^{2\pi}  \int_{4}^{7}   r \, dr d\theta.$


$\displaystyle \frac{33\pi}{2};$ região de integração:

A expressão 1 cossec x e o mesmo que


2196   

Se ${\bf F}=(xz,yz,2)$ e $E$ é a região dada por $x^{2}+y^{2}\leq 1$ e $0\leq z \leq 1,$ mostre que o Teorema do Divergente é verdadeiro neste caso. Calcule as duas integrais do enunciado do Teorema e mostre que elas têm o mesmo valor.


2191   

Calcule $\displaystyle\iint \limits_{S}{\bf u}\cdot {\bf n}\,dS$, sendo $B=\{(x,y,z)\in \mathbb{R}^{3}|\,x^{2}+y^{2}+z^{2}\leq 1$ e $z\geq x+y\}$ e ${\bf u}=-2xy\,{\bf i}+y^{2}\,{\bf j}+3z\,{\bf k}.$


2094   

Considere o campo vetorial

$${\bf F}(x,y)=\bigg(e^{x}\ln(y)-\frac{e^{y}}{x}\bigg)\,{\bf i}+\bigg(\frac{e^{x}}{y}-e^{y}\ln(x)\bigg)\,{\bf j}.$$

  1. O campo ${\bf F}$ é conservativo? Justifique sua resposta.

  2. Calcule $\int_{C}{\bf F}\cdot d{\bf r}$, onde $C$ é qualquer caminho ligando o ponto $(1,1)$ ao ponto $(3,3).$


  1. Sim.

  2. $0.$


3089   

Mostre que se \(f_x(x,y)=0\) e \(f_y(x,y)=0\) em toda uma região circular, então \(f(x,y)\) é constante nessa região.


2630   

Utilizando o Teorema de Stokes, transforme a integral $\displaystyle\iint_{ S}\mbox{rot}{\bf F}\cdot{\bf n}dS$ numa integral de linha e calcule.

  • ${\bf F}(x,y,z) = -y^2{\bf i} + x^2{\bf j} + z^2{\bf k}$, $S$ a superfície $x^2 + \dfrac{y^2}{4} + z^2 = 2$, $z \geq 1$, sendo ${\bf n}$ a normal que aponta para cima.


 $0$.


2073   

Seja $R$ o retângulo $1\leq x\leq 2$, $0\leq y\leq 1$. Calcule $\iint\limits_{ R} f(x,y)\,dxdy$, sendo $f(x,y)$ igual a

  1.  $1$

  2.  $x\cos(xy)$


  1. $1.$

  2. $\cos(1) - \cos(2).$


2216   

Determine o rotacional e o divergente do campo vetorial $\mathbf{F}(x,y,z) = \dfrac{1}{\sqrt{x^2+y^2+z^2}}(x\mathbf{i} + y\mathbf{j} + z\mathbf{k})$.


$\text{rot } \mathbf{F} = \bf{0}.$ $\text{div } \mathbf{F} = \dfrac{2}{\sqrt{x^{2} + y^{2} + z^{2}}}.$


2309   

 Calcule $D_{\bf{u}}f(x_0,y_0)$, sendo dados

$f(x,y) = \arctan{\dfrac{x}{y}}$, $(x_0,y_0) = (3,3)$ e $\bf{u} = \left(\frac{1}{\sqrt{2}},\frac{1}{\sqrt{2}}\right)$.


 $\displaystyle D_{\bf{\left(\frac{1}{\sqrt{2}},\frac{1}{\sqrt{2}} \right)}}f(3,3) = 0.$


2982   

Utilize a transformação dada para calcular a integral. $\displaystyle\iint\limits_{R} x^2 \, dA$, em que $R$ é a região limitada pela elipse $9x^2 + 4y^2 = 36$; $x = 2u$, $y = 3v$.


$6\pi.$


2211   

Ao calcular, por integração dupla, o volume $V$ do sólido situado abaixo do parabolóide $z=x^{2}+y^{2}$ e limitado inferiormente por uma certa região $D$ no plano $xy$, chegou-se à seguinte expressão: $$V=\int_{0}^{1}\!\!\int_{0}^{y}(x^{2}+y^{2})\,dx dy+\int_{1}^{2}\int_{0}^{2-y}(x^{2}+y^{2})\,dx dy.$$

  1.  Esboce a região $D.$

  2.  Expresse $V$ numa única integral dupla iterada.

  3.  Efetue a integração para calcular $V.$


  1. ...

  2.  $\displaystyle \int_{0}^{1} \int_{x}^{2 - x} x^{2} + y^{2}\;dy\; dx$

  3.  $\dfrac{4}{3}.$


2657   

Determine as derivadas parciais de $f(x,y)=e^{-x^{2}-y^{2}}$.


$\displaystyle \frac{\partial f}{\partial x} = -2xe^{-x^{2} - y^{2}}\;\;\;\;\;\;\text{e}\;\;\;\;\; \frac{\partial f}{\partial y} = -2ye^{-x^{2} - y^{2}}.$


2964   

Dentre as coordenadas cilíndricas ou esféricas, utilize a que lhe parecer mais apropriada para determinar o volume e o centroide do sólido $E$ que está acima do cone $z=\sqrt{x^{2}+y^{2}}$ e abaixo da esfera $x^{2}+y^{2}+z^{2}=1.$


Volume: $\dfrac{\pi(2 - \sqrt{2})}{3};$ centróide: $\left(0,0, \dfrac{3}{8(2 - \sqrt{2})} \right).$


2629   

Utilizando o Teorema de Stokes, transforme a integral $\displaystyle\iint_{ S}\mbox{rot}{\bf F}\cdot{\bf n}dS$ numa integral de linha e calcule.

  • ${\bf F}(x,y,z) = -y{\bf i} + x{\bf j} + x^2{\bf k}$, $S$ a superfície $x^2+y^2+z^2 = 4$, $\sqrt{2} \leq z \leq \sqrt{3}$ e $y \geq 0$, sendo ${\bf n}$ a normal apontando para cima.


$\pi$.


2088   

Calcule $\displaystyle\int_{C}\dfrac{-y}{x^{2}+y^{2}}\,dx+\dfrac{x}{x^{2}+y^{2}}\,dy$ onde $C:[0,1]\rightarrow \mathbb{R}^{2}$ é uma curva de classe $C^{1}$ por partes, com imagem contida no semiplano $y>0$, tal que $C(0)=(1,1)$ e $C(1)=(-2,3).$


$\pi.$


2184   

No item abaixo: 

  1. expresse $\partial w/\partial u$ e $\partial w/ \partial v$ como funções de $u$ e $v$, usando a Regra da Cadeia e também expressando $w$ diretamente em termos e $u$ e $v$ antes de diferenciar; 
  2. calcule $\partial w/\partial u$ e $\partial w/ \partial v$ no ponto dado $(u,v)$.

$w=xy+yz+xz$,  $x=u+v$, $y=u-v$,  $z=uv$;  $(u,v)=(1/2,1).$


  1. $\displaystyle w(u,v) = u^{2} - v^{2} + 2u^{2}v,$$\displaystyle \frac{\partial w}{\partial u}(u,v) = 2u + 4uv$ e $\displaystyle \frac{\partial w}{\partial v}(u,v) = -2v + 2u^{2}.$
  2. $\displaystyle \frac{\partial w}{\partial u}(-2,0) = 3$ e $\displaystyle \frac{\partial w}{\partial v}(-2,0) = -\frac{3}{2}.$


2849   

Passe para coordenadas polares e calcule: $\displaystyle\iint\limits_{R}xy\,dx dy$, onde $R$ é o círculo $x^{2}+y^{2}-2y\leq 0$, $x\geq 0.$


$\displaystyle \frac{2}{3}.$


2268   

Determine a área da superfície dada pela parte de baixo da esfera $x^{2}+y^{2}+z^{2}=2$ cortada pelo cone $z=\sqrt{x^{2}+y^{2}}.$



Sejam

$$\left \{\begin{array}{cc}x=r\,\sin \phi\,\cos \theta\\y=r\,\sin \phi\,\sin \theta\\z=r\,\cos \phi\\\end{array}\right. \Rightarrow r=\sqrt{x^{2}+y^{2}+z^{2}}=\sqrt{2},\, \mbox{na\,esfera}.$$

Temos que

$$x^{2}+y^{2}+z^{2}=2    \mbox{e}\,\,\,\, z=\sqrt{x^{2}+y^{2}}\Rightarrow z^{2}+z^{2}=2\Rightarrow z^{2}=1\Rightarrow z=1\,(\mbox{pois}\, z\geq 0).$$

Logo, $\phi=\frac{\pi}{4}.$ Para a parte inferior da esfera cortado pelo cone, temos que $\phi=\pi.$

Então,

$$r(\phi,\theta)=(\sqrt{2}\,\sin \phi,\,\cos\theta)\,{\bf i}+(\sqrt{2}\,\sin \phi\,\sin \theta)\,{\bf j}+(\sqrt{2}\,\cos \phi)\,{\bf k},$$

$$\frac{\pi}{4}\leq \phi\leq \pi\,\,\,\, \mbox{e}\,\,\,\, 0\leq \theta \leq 2\pi.$$

Isso implica que

$$r_{\phi}(\phi,\theta)=(\sqrt{2}\,\cos \phi,\,\cos\theta)\,{\bf i}+(\sqrt{2}\,\cos \phi\,\sin \theta)\,{\bf j}-(\sqrt{2}\,\sin \phi)\,{\bf k}$$

e

$$r_{\theta}(\phi,\theta)=(-\sqrt{2}\,\sin \phi,\,\sin\theta)\,{\bf i}+(\sqrt{2}\,\sin \phi\,\cos \theta)\,{\bf j}+0\,{\bf k}$$

Logo,

$$\begin{array}{rcl}r_{\phi}\times r_{\theta}&=&\left|\begin{array}{ccc}{\bf i}&{\bf j}&{\bf k}\\\sqrt{2}\,\cos \phi\,\cos \theta & \sqrt{2}\,\cos \phi\,\sin \theta& -\sqrt{2}\,\sin \phi\\-\sqrt{2}\,\sin \phi\,\sin \theta & \sqrt{2}\,\sin \phi\,\cos \theta & 0\end{array}\right|\\&=&(2\,\sin^{2}\phi\,\cos \theta)\,{\bf i}+(2\sin^{2}\phi\,\sin \theta)\,{\bf j}+(2\,\sin \phi \,\cos \phi)\,{\bf k}.\\\end{array}$$

Isso resulta que

$$\begin{array}{rcl}|r_{\phi}\times r_{\theta}|&=&\sqrt{4\sin^{2}\phi\,\cos^{2}\theta+4\,\sin^{4}\,\sin^{2}\theta+4\sin^{2}\phi\,\cos^{2}\phi}\\&=&\sqrt{4\,\sin^{2}\phi}=2|\sin\phi|=2\sin \phi   \bigg(\mbox{pois},\, \frac{\pi}{4}\leq \phi \leq \pi\bigg).\end{array}$$

Assim,

$$A=\iint\limits_{ D}|r_{\phi}\times r_{\theta}|\,dA=\int_{\frac{\pi}{4}}^{\pi}\int_{0}^{2\pi}2\sin \phi\, d\theta d \phi=2\int_{\frac{\pi}{4}}^{\pi}\sin \phi\,d\phi \cdot \int_{0}^{2\pi}d\theta$$

$$=2\cdot (-\cos \phi)\bigg|_{\frac{\pi}{4}}^{\pi}\cdot \theta\bigg|_{0}^{2\pi}=2\cdot \bigg(1-\frac{\sqrt{2}}{2}\bigg)\cdot 2\pi=4\pi\bigg(1-\frac{\sqrt{2}}{2}\bigg)=\pi(4-2\sqrt{2})$$


1958   

Calcule a integral de linha $\displaystyle\int_{C}{\bf F}\cdot d{\bf r}$, onde ${\bf F}(x,y,z)=(yz,xz,xy+2y)$ e $C$ é o segmento de reta que liga o ponto $(1,0,1)$ ao ponto $(-2,2,2).$


$-6.$


2867   

Estude com relação a máximos e mínimos a função dada com as restrições dadas.

$f(x,y) = x^2 - 2y^2$ e $x^2 + y^2 - 2x = 0.$


Ponto de máximo: $\displaystyle \left( 2,0 \right)$; pontos de mínimo: $\displaystyle \left( \frac{2}{3}, \frac{2\sqrt{2}}{3} \right)$ e $\displaystyle \left( \frac{2}{3}, \frac{-2\sqrt{2}}{3} \right)$.


2984   

Utilize a transformação dada para calcular a integral. $\displaystyle\iint\limits_{R} xy \, dA$, em que $R$ é a região do primeiro quadrante limitada pelas retas $y = x$ e $y = 3x$ e pelas hipérboles $xy = 1$, $xy = 3$; $x = \dfrac{u}{v}$, $y = v$.


$2 \ln 3.$


2728   

Quatro números positivos, cada um menor que $50$, são arredondados até a primeira casa decimal e depois multiplicados. Utilize os diferenciais para estimar o máximo erro possível no cálculo do produto que pode resultar do arredondamento.


Se $x,y,z,w$ são os quatro números e $p(x,y,z,w) = xyzw,$ temos $\Delta p \leq 25000.$


3069   

Calcule a integral de linha $\displaystyle\int_{C}{\bf F}\cdot d{\bf r}$, onde ${\bf F}(x,y)=(x+y^{2})\,{\bf j}$ e $C$ é a curva da figura abaixo.

A expressão 1 cossec x e o mesmo que


$4.$


2427   

Calcule a integral tripla.

  1.  $\displaystyle\iiint\limits_{  E} x^2 e^y\,dV$, onde $E$ é delimitado pelo cilindro parabólico $z=1-y^{2}$ e pelos planos $z=0$, $x=1$ e $x=-1.$

  2.  $\displaystyle\iiint\limits_{  E}x\,dV$, onde $E$ é limitado pelo paraboloide $x=4y^{2}+4z^{2}$ e  pelo plano $x=4.$


  1.  $\dfrac{8}{3e}.$

  2.  $\dfrac{16\pi}{3}.$


2113   

Se $u=f(x,y)$, onde $x=e^{s}\cos{t}$ e $y=e^{s}\sin{t}$, mostre que
$$\bigg(\dfrac{\partial u}{\partial x}\bigg)^{2}+ \bigg(\dfrac{\partial u}{\partial y}\bigg)^{2}=
e^{-2s}\bigg[ \bigg(\dfrac{\partial u}{\partial s}\bigg)^{2}+\bigg(\dfrac{\partial u}{\partial t}\bigg)^{2}\bigg].$$



Note que $\displaystyle \frac{\partial u}{\partial s} = e^{s} \cos(t) \frac{\partial u}{\partial x}  + e^{s} \sin(t) \frac{\partial u}{\partial y} $e
$\displaystyle \frac{\partial u}{\partial t} = -e^{s} \sin(t) \frac{\partial u}{\partial x}  + e^{s} \cos(t) \frac{\partial u}{\partial y} .$


1994   

Sejam ${\bf u}(t)=t{\bf i}+{\bf j}+e^{t}{\bf k}$ e ${\bf v}(t)={\bf i}+{\bf j}+{\bf k}.$ Calcule

  1. $\displaystyle\int_{0}^{1}({\bf u}(t)\times{\bf v}(t))\mathrm{d}t$
  2. $\displaystyle\int_{0}^{1}({\bf u}(t)\cdot {\bf v}(t))\mathrm{d}t$


2902   

Determine os valores máximo e mínimo absolutos de

$$f(x,y) = x^2 + 2y^2 - x$$

no conjunto $D = \{(x,y) \in \mathbb{R}^2: x^2 + y^2 \leq 1 \}$.


Valor máximo: $\dfrac{9}{4};$ valor mínimo: $-\dfrac{1}{4}.$


2883   

Determine a curva de nível de $f(x,y) = x^2 + 16y^2$ que seja tangente à curva $xy = 1$, $x>0$ e $y>0$. Qual o ponto de tangência?


$x^{2} + 16 y^{2} = 8;$ o ponto de tangência é $\displaystyle \left( 2, \frac{1}{2} \right).$


2365   

Existe uma direção $\bf{u}$ na qual a taxa de variação de $f(x,y) = x^2 - 3xy + 4y^2$ em $P = (1,2)$ é igual a 14? Justifique sua resposta.


 Não, já que $|\nabla f(1,2)| = \sqrt{185} < 14.$


2796   

Determine os valores máximos e mínimos locais e pontos de sela da função $f(x,y)=\sqrt[3]{x^{2}+2xy+4y^{2}-6x-12y}$.


Ponto de mínimo: $\displaystyle \left( 2,1\right).$


2066   

Dados ${\bf F}(x,y,z)=y^{2}\,\cos z\,{\bf i}+2xy\,\cos z\,{\bf j}-xy^{2}\,\sin z\,{\bf k}$, $C: {\bf r}(t)=t^{2}\,{\bf i}+\sin t\,{\bf j}+t\,{\bf k}$, $0\leq t\leq \pi.$

  1. Determine uma função $f$ tal que ${\bf F}=\nabla f$.

  2. Use o resultado anterior para calcular $\int_{C}{\bf F}\cdot d{\bf r}$ sobre a curva $C$ dada.


  1. $f(x,y,z) = xy^{2} \cos(z);$

  2. $0.$


2355   

Calcule o volume do conjunto dado.

  1.  $x+y+z\leq 1$, $x\geq 0$, $y\geq 0$ e $z\geq 0.$

  2.  $x\leq y\leq 1$, $x\geq 0$, $z\geq 0$ e $z^{2}+x^{4}+x^{2}y^{2}\leq 2x^{2}.$


  1.  $\dfrac{1}{6}.$

  2.  $\dfrac{\pi(1 - \sqrt{2})}{8} + \dfrac{1}{3}.$


1937   

Calcule a integral de linha, onde $C$ é a curva dada.
$\displaystyle\int_{C}xy^{3}\,ds$,   $C:\,x=4\,\sin t,\, y=4\,\cos t,\, z=3t,\, 0\leq t\leq \pi/2.$


$320.$


3150   

Seja \(\mathbf{F}(x,y)= (ye^{xy}-1)\mathbf{i} + xe^{xy}\mathbf{j}.\)

  1.  Mostre que \(\mathbf{F}\) é um campo vetorial conservativo.

  2.  Calcule uma função potencial de \(\mathbf{F}\).

  3.  Calcule o trabalho realizado pelo campo vetorial sobre uma partícula que se move ao longo da curva representada pelas seguintes equações paramétricas \begin{align*} x  & = t+ \arcsin(\sin t) \\ y & = \dfrac{2}{\pi}\arcsin(\sin t), \ \left(0\leq t\leq 8\pi\right).  \end{align*}


3123   

Se \(x=x(u,v,w)\), \(y=y(u,v,w)\) e \(z=z(u,v,w)\) for uma transformação injetora, então \(u=u(x,y,z)\), \(v=v(x,y,z)\) e \(w=w(x,y,z)\). Supondo a diferenciabilidade das funções, mostre que \[\dfrac{\partial(x,y,z)}{\partial(u,v,w)}\cdot\dfrac{\partial(u,v,w)}{\partial(x,y,z)} = 1.\] Use este resultado para mostrar que o volume \(V\) do paralelepípedo oblíquo limitado pelos planos \(x+y+2z=\pm 3\), \(x-2y+z=\pm 2\), \(4x+y+z=\pm 6\) é dado por \(V=16\).


2818   

Determine os valores máximo e mínimo absolutos de $f$ no conjunto $D.$

$f(x,y)=3x-y$ no conjunto $D$ de todas $(x,y)$ tais que $x\geq 0$, $y\geq 0$, $y-x\leq 3$, $x+y\leq 4$ e $3x+y\leq 6.$


Valor máximo: $6;$ valor mínimo: $-3.$


3058   

Esboce o campo vetorial ${\bf F}(x,y)=(x-y)\textbf{i} + x \textbf{j}$, desenhando um diagrama.


2239   

Use o Teorema de Green na forma $\oint_{C} \mathbf{F} \cdot \mathbf{n} \, ds = \iint\limits_{ D} \text{div }{\mathbf{F}(x,y)} \, dA$ para demonstrar a primeira identidade de Green:

$$\iint\limits_{ D} f\nabla^2g \, dA = \oint_{C}f(\nabla{g}) \cdot \mathbf{n} \, ds - \iint\limits_{ D}\nabla{f} \cdot \nabla{g} \, dA,$$

em que $D$ e $C$ satisfazem as hipóteses do Teorema de Green e as derivadas parciais apropriadas de $f$ e $g$ existem e são contínuas. (A quantidade $ \nabla{g} \cdot \mathbf{n} = D_{\mathbf{n}}g$ aparece na integral de linha. Essa é a derivada direcional na direção do vetor normal $\mathbf{n}$ e é chamada derivada normal de $g$.)


Note que $\oint_{C} f(\nabla{g}) \cdot \mathbf{n} \, ds = \iint\limits_{ D} \text{div }(f \nabla g) \, dA = \iint\limits_{ D} f\text{div }(\nabla g) + \nabla{f} \cdot \nabla{g} \, dA.$


2695   

Calcule todas as derivadas parciais de $2^{\underline{a}}$ ordem de $f(x,y)=x^{3}y^{2}$.


$\displaystyle \frac{\partial^{2} f}{\partial x^{2}}= 2xy^{2},\;\;\;\;\; \frac{\partial^{2} f}{\partial y^{2}}= 2x^{3}\;\;\;\;\;\text{e}\;\;\;\;\; \frac{\partial^{2} f}{\partial x\partial y}= \frac{\partial^{2} f}{\partial y\partial x}= 6x^{2}y.$


2510   

Ache $\displaystyle\iint \limits_{ S}{\bf F}\cdot {\bf n} dS$ se ${\bf n}$ é uma normal unitária superior de $S.$

  • ${\bf F}=x{\bf i}-y{\bf j}$; $S$ é a parte no primeiro octante da esfera $x^{2}+y^{2}+z^{2}=a^{2}.$


$0.$


1977   

Mostre que a curva com equações paramétricas $x = t^2$, $y = 1 - 3t$, $z = 1 + t^3$ passa pelos pontos $(1,4,0)$ e $(9,-8,28)$, mas não passa pelo ponto $(4,7,-6).$


2393   

Determine a equação da reta tangente à curva de nível dada, no ponto dado.

$x^2 + xy + y^2 - 3y = 1$, em $(1,2)$.


$y - 2 = -2(x - 1).$


2560   

Determine o limite, se existir, ou mostre que o limite não existe.

$\displaystyle \lim_{(x,y) \to (0,0)} \dfrac{x^2 + y^2}{\sqrt{x^2 + y^2 + 1} - 1}$.


$2.$


2392   

Se $g(x,y) = x^2 + y^2 - 4x$, encontre o vetor gradiente $\nabla g(1,2)$ e use-o para encontrar a reta tangente à curva de nível $g(x,y) = 1$ no ponto $(1,2)$. Esboce a curva de nível, a reta tangente e o vetor gradiente.


 $\nabla g(1,2) = (1,2) = (-2,4);$ reta tangente à curva de nível $g(x,y) = 1$ em $(1,2)$: $-x + 2y = 3.$

A expressão 1 cossec x e o mesmo que


3061   

Esboce o campo vetorial $\textbf{F}= y\textbf{i} + \dfrac{1}{2}\textbf{j}$, desenhando um diagrama.


3009   

Calcule o centro de massa da região $D$ dada.

  1.  $D$ é o conjunto de todos $(x,y)$ tais que $x^3 \leq y \leq x$ e a densidade é constante e igual a 1.

  2.  $D$ é o conjunto de todos $(x,y)$ tais que $x \leq y \leq x + 1$, $0 \leq x \leq 1$, e a densidade é o produto das coordenadas do ponto.

  3.  $D$ é o conjunto de todos $(x,y)$ tais que $1 \leq x^2 + y^2 \leq 4$, $y \geq 0$, e a densidade é proporcional à distância do ponto à origem.


  1.  $\displaystyle \left(0,0\right).$

  2.  $\displaystyle \left(\frac{5}{7},\frac{9}{7}\right).$

  3.  $\displaystyle \left(0, \frac{45}{14\pi} \right).$


2135   

Mostre que cada a equação a seguir define implicitamente pelo menos uma função diferenciável $z=z(x,y)$. 
Expresse $\partial z /\partial x$ e $\partial z/\partial y$ em termos de $x$, $y$ e $z.$
$e^{x+y+z}+xyz=1$


 $\displaystyle \frac{\partial z}{\partial x} = - \frac{e^{x + y + z} + yz}{e^{x + y + z} + xy}$ e $\displaystyle \frac{\partial z}{\partial y} = - \frac{e^{x + y + z} + xz}{e^{x + y + z} + xy}.$


2495   

Dada a função $f(x,y)=\dfrac{y}{x^{2}}$.

  1. Encontre o domínio da função.

  2. Encontre a imagem da função.

  3. Descreva as curvas de nível da função.


  1. $D_{f} = \left\lbrace (x,y);\; (x,y) \neq (0,y) \right\rbrace$.

  2. $Im(f) =\mathbb{R}.$

  3. As curvas de nível são as parábolas $y = C x^{2}$ sem a origem se $C \neq 0$ e o eixo $x$ se $C \neq 0.$


2637   

As seguintes superfícies, rotuladas $a$, $b$ e $c$ de cima para baixo, são gráficos de uma função $f$ e de suas derivadas parciais $f_{x}$ e $f_{y}$. Identifique cada superfície e dê razões para sua escolha.

A expressão 1 cossec x e o mesmo que


a) $f_{y},$ b) $f_{x},$ c) $f$.


2911   

Ao calcular por integração dupla o volume $V$ do sólido situado abaixo do gráfico de $f(x,y)=e^{x^{2}+y^{2}}$ e limitado inferiormente por uma certa região $D$ no plano $xy$, chegou-se à seguinte expressão: $$V=\int_{0}^{2}  \int_{0}^{\sqrt{4-x^{2}}}e^{x^{2}+y^{2}}\,dy dx-\int_{0}^{1}  \int_{0}^{\sqrt{1-x^{2}}}e^{x^{2}+y^{2}}\,dy dx.$$

  1.  Esboce a região $D.$

  2.  Expresse $V$ numa única integral dupla em coordenadas polares.

  3.  Efetue a integração para calcular $V.$


  1.  $D = \left\lbrace (x,y); 1 \leq x^{2} + y^{2} \leq 2, x \geq 0, y \geq 0 \right\rbrace.$

  2.  $\displaystyle \int_{0}^{\frac{\pi}{2}} \int_{1}^{2} re^{r^2}  dr d\theta.$

  3.  $\dfrac{\pi}{4}(e^4 - 1).$


2225   

Determine se o campo vetorial $\mathbf{F}(x,y,z) = y^2z^3\mathbf{i} + 2xyz^3\mathbf{j} + 3xy^2z^2\mathbf{k}$ é conservativo ou não. Se for conservativo, determine uma função $f$ tal que $\mathbf{F} = \nabla{f}$.


$\mathbf{F}$ é conservativo. $f(x,y,z) = xy^2 z^3.$


2414   

Inverta a ordem de integração, integrando primeiro em $y$ e depois em $x$ para calcular a integral:

  1.  $\displaystyle\int_{0}^{1}\!\!\int_{\sqrt{y}}^{1}\sqrt{x^{3}+1}\,dx dy$

  2.  $\displaystyle\int_{0}^{1}\!\!\int_{\sqrt{y}}\sin{x^{3}}\,dx dy$


  1.  $\dfrac{2(2\sqrt{2} - 1)}{9}.$

  2.  $\dfrac{2}{3} \sin^{2}\left(\dfrac{1}{2} \right).$


2148   

Use o Teorema do Divergente para calcular o fluxo de ${\bf F}$ através de $S,$ onde ${\bf F}(x,y,z)=3xy^{2}\,{\bf i}+xe^{z}\,{\bf j}+z^{3}\,{\bf k}$, $S$ é a superfície do sólido delimitado pelo cilindro $y^{2}+z^{2}=1$ e pelos planos $x=-1$ e $x=2.$


3088   

Mostre que se \(f\), \(f_x\) e \(f_y\) são contínuas numa região circular contendo os pontos \(A=(x_0,y_0)\) e \(B=(x_1,y_1)\), então existe um ponto \((x^\ast,y^\ast)\) no segmento que une \(A\) e \(B\) tal que \[ f(x_1,y_1)-f(x_0,y_0) = f_x(x^\ast,y^\ast)(x_1-x_0)+f_y(x^\ast,y^\ast)(y_1-y_0). \] Este resultado é a versão bidimensional do Teorema do Valor Médio. [Sugestão: expresse o segmento de reta  unindo \(A\) e \(B\) na forma paramétrica e use o Teorema do Valor Médio para funções de uma variável.]


2023   

Calcule a integral dupla, identificando-a antes com o volume de um sólido.

  1.   $\displaystyle\iint\limits_{R} 3 \, dA, \quad R = \{(x,y) \in \mathbb{R}^2: -2 \leq x \leq 2, \ 1 \leq y \leq 6\}.$

  2.   $\displaystyle\iint\limits_{R} (4 - 2y) \, dA, \quad R = [0,1] \times [0,1].$


  1.  $60.$

  2.  $3.$


2565   

Determine o maior conjunto no qual a função $f(x,y) = \begin{cases}\dfrac{xy}{x^2 + xy + y^2}, & \quad \text{se } (x,y) \neq (0,0), \\0, & \quad \text{se } (x,y) = (0,0)\end{cases}$ é contínua.


$\left\lbrace (x,y);\; (x,y) \neq (0,0) \right\rbrace.$


2253   

Demonstre a identidade abaixo, supondo que $S$ e $E$ satisfaçam as condições do Teorema do Divergente e que as funções escalares e as componentes dos campos vetoriais tenham derivadas parciais de segunda ordem contínuas.

  1. $V(E)=\dfrac{1}{3}\displaystyle\iint\limits_{S}{\bf F}\cdot dS$, onde ${\bf F}(x,y,z)=x\,{\bf i}+y\,{\bf j}+z\,{\bf k}.$



Dica: Note que $\displaystyle\iiint\limits_{E}{\mbox{div} {\bf F}}\, dV = \iiint \limits_{E}{3}\,dV$.


2632   

Determine as derivadas parciais de primeira ordem da função $f(r,s)=r\ln(r^{2}+s^{2})$.



Sendo $f(r,s)=r\cdot \ln(r^{2}+s^{2})$, temos que as derivadas parciais em relação a $r$ e $s$, respectivamente, são:

$\bullet f_{r}(r,s)=1\cdot \ln(r^{2}+s^{2})+r\cdot \dfrac{1}{r^{2}+s^{2}}\cdot 2r=\ln(r^{2}+s^{2})+\dfrac{2r^{2}}{r^{2}+s^{2}}.$

$\bullet f_{s}(r,s)=0\cdot \ln(r^{2}+s^{2})+r\cdot \dfrac{1}{r^{2}+s^{2}}\cdot 2s=\dfrac{2rs}{r^{2}+s^{2}}.$


2331   

Determine a taxa de variação máxima de $f$ no ponto dado e a direção em que isso ocorre.

$f(x,y) = \sin{xy},  (1,0).$


$1.$


3103   

Use uma integral dupla para calcular a área da região \(R\) entre a parábola \(y=\dfrac{1}{2}x^2\) e a reta \(y = 2x\).



Denotando por \(A(R)\) a área de \(R\), teremos que \begin{align*} A(R) & = \iint_R\,dA = \int_0^4\int_{x^2/2}^{2x}\,dydx = \int_0^4\left[y\right]_{y=x^2/2}^{2x}\,dx \\  & = \int_0^4\left(2x-\dfrac{1}{2}x^2\right)\,dx = \left[x^2-\dfrac{x^3}{6}\right]_0^4= \dfrac{16}{3}. \end{align*} De outra forma, fixando primeiro a variável \(y\), teríamos \begin{align*} A(R) & = \iint_R\,dA = \int_0^8\int_{y/2}^{\sqrt{2y}}\,dxdy = \int_0^8\left[x\right]_{x=y/2}^{\sqrt{2y}}\,dy \\  & = \int_0^8\left(2y-\dfrac{1}{2}y\right)\,dy = \left[\dfrac{2\sqrt{2}}{3}y^{3/2}-\dfrac{y^2}{4}\right]_0^8= \dfrac{16}{3}. \end{align*}


2308   

 Calcule $D_{\bf{u}}f(x_0,y_0)$, sendo dados

$f(x,y) = e^{x^2-y^2}$, $(x_0,y_0) = (1,1)$ e $\bf{u}$ o versor de $(3,4)$.


 $\displaystyle D_{\bf{(3,4)}}f(1,1) = -\frac{2}{5}.$ 


2584   

Considere a função

$$f(x,y) = \begin{cases}x + y, & \quad \text{se } xy = 0, \\k, & \quad \text{caso contrário},\end{cases}$$

em que $k$ é um número real. É possível escolher $k$ de modo que $f$ seja contínua em $(0,0)$? Em caso afirmativo, qual deve ser o valor de $k$?


$k = 0.$


2486   

Esboce o gráfico da função $f(x,y)=10-4x-5y$.


$z = 10 - 4x - 5y.$

A expressão 1 cossec x e o mesmo que


2587   

Calcule a massa do cilindro $x^{2}+y^{2}\leq 4$ e $0\leq z \leq 2$, sabendo que a densidade no ponto $(x,y,z)$ é o dobro da distância do ponto ao plano $z=0.$


$16\pi.$


2413   

Determine o volume do sólido que está abaixo do paraboloide elíptico $x^{2}/4+y^{2}/9+z=1$ e acima do retângulo $R=[-1,1]\times [-2,2].$


$\dfrac{166}{27}.$


2003   

Sejam $A=(3,0)$, $B=(1,1)$ e $C=(0,3)$ pontos de $\mathbb{R}^{2}$ e $C$ a trajetória que vai em linha reta de $A$ até $B$ e em seguida de $B$ até $C$. Determine o trabalho ao longo de $C$ do campo de forças ${\bf F}:\mathbb{R}^{2}\rightarrow \mathbb{R}^{2}$, sendo

$${\bf F}(x,y)=\bigg(-\frac{y}{x^{2}+y^{2}},\frac{x}{x^{2}+y^{2}}\bigg).$$


$\displaystyle 2\arctan(2) + \arctan\left(\frac{1}{2} \right) - \arctan\left(\frac{1}{3} \right).$


2283   

Determine se os pontos $P(7,10,4)$ e $Q(5,22,5)$ estão na superfície ${\bf r}(u,v)=(2u+3v,1+5u-v,2+u+v)$.


$P$ não está na superfície; $Q$ está na superfície.


3128   

Sejam \(\mathbf{r}=x\mathbf{i}+y\mathbf{j}+z\mathbf{k}\), \(r=\|\mathbf{r}\|\), \(f\) uma função diferenciável de uma variável e \(\mathbf{F}(\mathbf{r})=f(r)\mathbf{r}\).

  1.  Mostre que \[\nabla f(r) = \dfrac{f'(r)}{r}\mathbf{r}.\]

  2.  Use o resultado anterior para mostrar que \(\displaystyle \mathbf{F}=3f(r)+rf'(r). \)


2504   

Dois mapas de contorno são mostrados na figura. Um é de uma função $f$ cujo gráfico é um cone. O outro é de uma função $g$ cujo gráfico é um paraboloide. Qual é qual? Por quê?

A expressão 1 cossec x e o mesmo que


O da esquerda corresponde ao cone e o da direita ao paraboloide.


2255   

Demonstre a identidade abaixo, supondo que $S$ e $E$ satisfaçam as condições do Teorema do Divergente e que as funções escalares e as componentes dos campos vetoriais tenham derivadas parciais de segunda ordem contínuas.

  1. $\displaystyle\iint\limits_{S}(f\nabla g)\cdot {\bf n}\,dS=\displaystyle\iiint\limits_{E}(f\nabla^{2}g+\nabla f+\nabla g)\,dV.$



Note que $\displaystyle\iint\limits_{S}(f\nabla g)\cdot {\bf n}\,dS=\displaystyle\iiint\limits_{E} \mbox{div} (f\nabla g)\,dV.$


2650   

Verifique que a função $z=\ln(e^{x}+e^{y})$ é uma solução das equações diferenciais

$$\frac{\mathrm{\partial}z}{\mathrm{\partial}x} + \frac{\mathrm{\partial}z}{\mathrm{\partial}y}=1\;\;\;\;\;\;\; e\;\;\;\;\;\;\; \frac{\mathrm{\partial}^{2}z}{\mathrm{\partial}^{2}x}+\frac{\mathrm{\partial}^{2}z}{\mathrm{\partial}^{2}y}-\bigg(\frac{\mathrm{\partial}^{2}z}{\mathrm{\partial}x\mathrm{\partial}y}\bigg)^{2}=0.$$


$\begin{aligned}[t]\frac{\partial z }{\partial x} &= \frac{e^{x}}{e^{x} + e^{y}},\;\;\; \frac{\partial z }{\partial y} = \frac{e^{y}}{e^{x} + e^{y}},\\\frac{\partial^{2} z }{\partial x^{2}} &= \frac{\partial^{2} z }{\partial y^{2}} = \frac{e^{x + y}}{(e^{x} + e^{y})^{2}},\;\;\; \frac{\partial^{2} z }{\partial x \partial y} = -\frac{e^{x + y}}{(e^{x} + e^{y})^{2}}.\end{aligned}$


2301   

Identifique e faça um esboço da imagem da superfície parametrizada dada por ${\bf r}(u,v)=(u,v,1-u-v)$, $u\geq 0$, $v\geq 0$ e $u+v\leq 1.$


Região triangular do plano $x + y + z = 1:$ $0 \leq x \leq 1, $ $0 \leq y \leq 1,$ $0 \leq z \leq 1.$


2221   

Seja $f$ um campo escalar e $\mathbf{F}$ um campo vetorial. Diga se cada expressão tem significado. Em caso negativo, explique por quê. Em caso afirmativo, diga se é um campo vetorial ou escalar.  

  1. $\text{grad }{f}$;

  2. $\text{rot }{(\text{grad }{f})}$;

  3. $\text{grad }{(\text{div }{\mathbf{F}})}$.


  1. $\text{grad }{f}$ é um campo gradiente.

  2. $\text{rot }{(\text{grad }{f})}$ é um campo vetorial.

  3. $\text{grad }{(\text{div }{\mathbf{F}})}$ é um campo vetorial.


1935   

Calcule a integral de linha, onde $C$ é a curva dada.

$\displaystyle\int_{C}xy\,dx+(x-y)\,dy$,   $C$ consiste nos segmentos de reta de $(0,0)$ a $(2,0)$ e de $(2,0)$ a $(3,2).$


$\displaystyle \frac{17}{3}.$


1983   

Nos itens abaixo: 

  • Esboce o gráfico da curva plana com a equação vetorial dada; 

  • Determine ${\bf r}'(t)$; 

  • Esboce o vetor posição ${\bf r}(t)$ e o vetor tangente ${\bf r}'(t)$ para o valor dado de  $t.$

  1. ${\bf r}(t)=(t-2,t^{2}+1)$ e $t=-1.$

  2. ${\bf r}(t)=\sin(t){\bf i}+2\; \cos(t){\bf j}$ e $t=\pi/4.$

  3. ${\bf r}(t)=(1+\cos{t}){\bf i}+(2+\sin{t}){\bf j}$ e $t=\pi/6.$


1970   

Se ${\bf r}(t)\neq {\bf 0}$, mostre que 

$$\frac{\mathrm{d}}{\mathrm{d}t}|{\bf r}(t)|=\frac{1}{|{\bf r}(t)|}{\bf r}(t)\cdot {\bf r}'(t).$$ 
(Sugestão: $|{\bf r}(t)|^{2}={\bf r}(t)\cdot {\bf r}(t)$).



Sabemos que $|{\bf r}(t)|^{2}={\bf r}(t)\cdot {\bf r}(t)$ ou $|{\bf r}(t)|= ({\bf r}(t)\cdot {\bf r}(t))^{1/2}.$ Então

\begin{eqnarray*}
\frac{d}{dt}|{\bf r}(t)|&=&\frac{d}{dt}[({\bf r}(t)\cdot {\bf r}(t))^{1/2}]\\
&=&\frac{1}{2}[{\bf r}(t)\cdot {\bf r}(t)]^{-1/2}[{\bf r}'(t)\cdot {\bf r}(t)+{\bf r}(t)\cdot {\bf r}'(t)]\\
&=& \frac{1}{2}[{\bf r}(t)\cdot {\bf r}(t)]^{-1/2}[2\,{\bf r}(t)\cdot {\bf r}'(t)]\\
&=&\frac{1}{2}\frac{1}{[{\bf r}(t)\cdot {\bf r}(t)]}[2\,{\bf r}(t)\cdot {\bf r}'(t)]\\
&=&\frac{1}{|{\bf r}(t)|}\,{\bf r}(t)\cdot {\bf r}'(t).
\end{eqnarray*}


2512   

Ache $\displaystyle\iint \limits_{ S}{\bf F}\cdot {\bf n} dS$ se ${\bf n}$ é uma normal unitária superior de $S.$

${\bf F}=x{\bf i}+y{\bf j}+z{\bf k}$; $S$ é a parte do plano $3x+2y+z=12$ intersectada pelos planos $x=0$,$y=0$, $x=1$ e $y=2.$


$24.$


2206   

Se $z=f(x,y)$ com $x=u+v$ e $y=u-v$, demonstre que 
$$\frac{\partial z}{\partial u}+\frac{\partial z}{\partial v}=2 \frac{\partial f}{\partial x}.$$



Note que $\displaystyle \frac{\partial z}{\partial u} = \frac{\partial f}{\partial x} + \frac{\partial f}{\partial y}$ e $\displaystyle \frac{\partial z}{\partial v} = \frac{\partial f}{\partial x} - \frac{\partial f}{\partial y}.$


2599   

Calcule as seguintes integrais triplas.

  1.  $\displaystyle\iiint\limits_{  E}  \sqrt{x^2 + y^2} \, dV$, em que $E$ é a região que está dentro do cilindro   $x^2 + y^2 = 16$ e entre os planos $z = -5$ e $z = 4$.

  2.  $\displaystyle\iiint\limits_{  E}  y \, dV$, em que $E$ é o sólido que está entre os cilindros $x^2 + y^2 = 1$ e $x^2 + y^2 = 4$, acima do plano $xy$ e abaixo do plano $z = x + 2$.

  3.  $\displaystyle\iiint\limits_{  E}  x \, dV$, em que $E$ está delimitidado pelos planos $z = 0$ e $z = x + y + 5$ e pelos cilindros $x^2 + y^2 = 4$ e $x^2 + y^2 = 9$.


  1.  $384\pi$.

  2. $0$.

  3. $\dfrac{65\pi}{4}$.


2178   

Seja $D$ a região limitada por um caminho fechado e simples $C$ no plano $xy$. Utilize o Teorema de Green para demonstrar que as coordenadas do centroide $(\bar{x},\bar{y})$ de $D$ são

$$\bar{x} = \dfrac{1}{2A}\oint_{C}x^2 \, dy \quad \quad\quad\quad \bar{y} = -\dfrac{1}{2A}\oint_{C}y^2 \, dx,$$

em que $A$ é a área de $D$.


$\dfrac{1}{2A}\oint_{C}x^2 \, dy = \dfrac{1}{2A} \iint_{D} 2x \, dA = \bar{x}$ e $-\dfrac{1}{2A}\oint_{C}y^2 \, dx = -\dfrac{1}{2A}\iint_{D} (-2y) \, dA = \bar{y}$


2433   

Calcule a integral tripla.

  1.  $\displaystyle\iiint\limits_{  E}2z\;dx dy dz$, onde $E$ é o conjunto $x^{2}+y^{2}\leq 1$, $x^{2}+y^{2}+z^{2}\leq 4$ e $z\geq 0.$

  2.  $\displaystyle\iiint\limits_{  E}x\;dx dy dz$, onde $E$ é o conjunto $x^{2}-y^{2}\leq z \leq 1-2y^{2}.$


  1.  $\dfrac{7\pi}{2}.$

  2.  $0.$


2747   

Explique por que a função é diferenciável no ponto dado. $f(x,y) = x\sqrt{y}, \quad (1,4)$.


As derivadas $f_{x}$ e $f_{y}$ de cada $f$ existem e são contínuas no ponto dado, logo $f$ é diferenciável.


2454   

Calcule as integrais mudando a ordem de integração de maneira apropriada.

  1.  $\displaystyle\int_{0}^{4}\int_{0}^{1}\int_{2y}^{2}\dfrac{4\;\cos(x^{2})}{2\sqrt{z}}\;dx dy dz$

  2.    $\displaystyle\int_{0}^{1}\int_{0}^{1}\int_{x^{2}}^{1}12xze^{zy^{2}}\;dy dx dz$

  3.  $\displaystyle\int_{0}^{1}\int_{\sqrt[3]{z}}^{1}\int_{0}^{\ln 3}\dfrac{\pi e^{2x}\;\sin(\pi y^{2})}{y^{2}}\;dx dy dz$


  1.  $2 \sin(4).$

  2.  $3e - 6.$

  3.  $4.$


2977   

Determine o jacobiano da transformação dada por: $x = uv, \quad y = vw, \quad z = uw$.


$2uvw.$


1968   

Calcule $\int_{C}(x+y+z)\,dx+(x-2y+3z)\,dy+(2x+y-z)\,dz$, onde $C$ é a curva de $(0,0,0)$ a $(2,3,4)$ se

  1. $C$ consiste em três segmentos de reta, o primeiro paralelo ao eixo $x$, o segundo paralelo ao eixo $y$ e o terceiro paralelo ao eixo $z$.

  2. $C$ consite em três segmentos de reta, o primeiro paralelo ao eixo $z$, o segundo ao eixo $x$ e o terceiro paralelo ao eixo $y.$

  3. $C$ é um segmento retilíneo.


  1. $19.$

  2. $35.$

  3. $27.$


2946   

Calcule utilizando coordenadas esféricas. $\displaystyle\iiint\limits_{B}x\,dxdydz$, onde $B$ é o conjunto $\dfrac{x^{2}}{4}+\dfrac{y^{2}}{9}+z^{2}\leq 1$ e $x\geq 0.$


$3\pi.$


2960   

Usando coordenadas esféricas, determine o volume e o centroide do sólido que está acima do cone $\phi=\pi/3$ e abaixo da esfera $\rho=4\cos{\phi}.$


Volume: $10\pi;$ centróide: $(0,0,2,1).$


3100   

Mostre (verifique) que as integrais abaixo podem ser calculadas como:

  1.  \[ \int_1^5\int_2^{y/2}6x^2y\,dxdy = \int_1^5\left(\dfrac{1}{4}y^4-16y\right)\,dy \]

  2.  \[ \int_1^5\int_2^{x/2}6x^2y\,dydx = \int_1^5\left(\dfrac{3}{4}x^4-12x^2\right)\,dx \]


2424   

Use o Teorema de Stokes para calcular $\displaystyle\int_C {\bf F} \cdot d{\bf r}$, com ${\bf F} (x,y,z) = yz{\bf i} + 2xz{ \bf j} + e^{xy} {\bf k} $ e $C$ é a circunferência $x^2+y^2 = 16$, $z=5$, orientada no sentido anti-horário quando vista de cima.


2232   

Seja $\mathbf{r} = x\mathbf{i} + y\mathbf{j} + z\mathbf{k}$ e $r=|\mathbf{r}|$. Verifique a identidade $\nabla \cdot (r\mathbf{r}) = 4r$.


$\nabla \cdot (r\mathbf{r}) = \left( \dfrac{x^{2}}{\sqrt{x^{2} + y^{2} + z^{2}}} + \sqrt{x^{2} + y^{2} + z^{2}} \right) + \left( \dfrac{x^{2}}{\sqrt{y^{2} + y^{2} + z^{2}}} + \sqrt{y^{2} + y^{2} + z^{2}} \right)\\ + \left( \dfrac{z^{2}}{\sqrt{x^{2} + y^{2} + z^{2}}} + \sqrt{x^{2} + y^{2} + z^{2}} \right)$ (Note que: $r = \sqrt{x^{2} + y^{2} + z^{2}}.$)


2238   

Mostre que $f(x,y) = \ln{(x^2+y^2)}$ satisfaz a equação de Laplace $\nabla^2f = 0$, exceto no ponto $(0,0).$


Note que se $(x,y) \neq (0,0),$  $\nabla^2f = \dfrac{\partial}{\partial x} \left[ \dfrac{2x}{x^{2} + y^{2}} \right] + \dfrac{\partial}{\partial y} \left[ \dfrac{2y}{x^{2} + y^{2}} \right].$


2846   

Passe para coordenadas polares e calcule: $\displaystyle\iint\limits_{D}e^{-x^{2}-y^{2}}\,dA$, onde $D$ é a região delimitada pelo semicírculo $x=\sqrt{4-y^{2}}$ e o eixo $y.$


$\displaystyle \frac{\pi}{2} (1 - e^{-4}).$


2327   

Calcule a área da superfície dada por: ${\bf r}(u,v)=(\cos u,v,\sin u)$ e $u^{2}+4v^{2}\leq 1.$. (Sugerimos ao leitor desenhar a imagem da superfície.)


$\dfrac{\pi}{2}.$


2763   

Determine o maior conjunto de pontos em que a função $f(x,y) = \begin{cases}\dfrac{xy^3}{x^2 + y^2}, & \quad \text{se } (x,y) \neq (0,0),\\0, & \quad \text{se } (x,y) = 0\end{cases}$ é diferenciável. Justifique.


$\mathbb{R}^{2}$.


2649   

Verifique que a função $u=1/\sqrt{x^{2}+y^{2}+z^{2}}$ é uma solução da equação de Laplace tridimensional $u_{xx}+u_{yy}+u_{zz}=0.$


$\displaystyle u_{xx} = \frac{2x^{2} - y^{2} - z^{2}}{(x^{2} + y^{2} + z^{2})^{5/2}},\;\;\; u_{yy} = \frac{2y^{2} - x^{2} - z^{2}}{(x^{2} + y^{2} + z^{2})^{5/2}}\;\;\;\text{e}\;\;\;u_{zz} = \frac{2z^{2} - x^{2} - y^{2}}{(x^{2} + y^{2} + z^{2})^{5/2}}$.


2905   

Utilize coordenadas polares para determinar o volume do sólido dado: uma esfera de raio $a.$


$\displaystyle \frac{4\pi}{3}a^3.$


2653   

Seja

$$f(x,y)=\begin{cases}\dfrac{x^{3}y-xy^{3}}{x^{2}+y^{2}}, & \quad \text{se } (x,y)\neq (0,0),\\0, & \quad \text{se } (x,y)=(0,0).\\\end{cases}$$

  1. Use um computador para traçar o gráfico de $f$.

  2. Determine $f_{x}(x,y)$ e $f_{y}(x,y)$ quando $(x,y)\neq (0,0).$

  3. Determine $f_{x}(0,0)$ e $f_{y}(0,0)$ use a definição das derivadas parciais como limite.

  4. Mostre que $f_{xy}(0,0)=-1$ e $f_{yx}(0,0)=1$

  5. O resultado da parte (d) contradiz o Teorema de Clairaut? Use o gráfico de $f_{xy}$ e $f_{yx}$ para ilustrar sua resposta.


  1. Gráfico de $f$:
    A expressão 1 cossec x e o mesmo que
  2. $\displaystyle f_{x} = \frac{x^{4}y + 4x^{2}y^{3} - y^{5}}{(x^{2} + y^{2})^{2}}\;\;\text{e}\;\;f_{y} = \frac{x^{5} - 4x^{3}y^{2} - xy^{4} }{(x^{2} + y^{2})^{2}}$ quando $(x,y)\neq (0,0).$
  3. $f_{x}(0,0) = f_{y}(0,0) = 0$.
  4. Use $\displaystyle f_{xy}(0,0)= \lim_{h \to 0} \frac{f_{x}(0,h) - f_{x}(0,0)}{h}\;\;\text{e}\;\;f_{yx}(0,0)= \lim_{h \to 0} \frac{f_{y}(h,0) - f_{y}(0,0)}{h}$.
  5. Para $(x,y) \neq (0,0),$ $\displaystyle f_{xy} = {x^{6} + 9x^{4}y^{2} - 9x^{2}y^{4} - y^{6}}{(x^{2} + y^{2})^{3}}.$ Como $f_{xy}$ não é contínua na origem, não há uma contradição com o Teorema de Clairaut. Os gráficos de $f_{xy}$ e $f_{yx}$ são idênticos, exceto na origem:
    A expressão 1 cossec x e o mesmo que


2012   

  1. Mostre que um campo de força constante realiza trabalho nulo sobre um partícula que dá uma única volta completa uniformemente na circunferência $x^{2}+y^{2}=1.$

  2. Isso também é verdadeiro para um campo de força ${\bf F}({\bf x})=k{\bf x}$, onde $k$ é uma constante e $\textbf{x}=x{\bf i}+y{\bf j}$?


  1. Dica: tome a parametrização do círculo $C$ dada por $x = cos(t)$ e $y = \sin(t),$ com $t \in [0,2\pi]$ e considere um campo constante arbitrário ${\bf F} = (a,b).$ Segue que $W = \int_{C} F\cdot d\textbf{r} = 0.$

  2. Sim. Realize o mesmo cálculo com ${\bf F}(x,y) = (k x, ky).$


2518   

Dada a expressão $g(x,y)=2f(x,y)$, escreva como o gráfico de $g$ é obtido a partir do gráfico de $f.$


Gráfico de $f$ esticado verticalmente ao dobro.


2924   

Marque o ponto cujas coordenadas esféricas é $(1,0,0)$ e encontre as coordenadas retangulares do ponto.


$(0,0,1).$

A expressão 1 cossec x e o mesmo que


2784   

Calcule a integral dupla usando coordenadas polares: $\displaystyle\iint\limits_{R}\sqrt{x^{2}+y^{2}}\,dA$, onde $R$ é limitado pelo triângulo de vértices $(0,0)$, $(3,0)$ e $(3,3).$



$\displaystyle \frac{9}{2} (\sqrt{2} + \ln(\sqrt{2} + 1)).$


1975   

Mostre que a curva com equações paramétricas $x = \sin{t}, \ y = \cos{t}, \ z = \sin^2t$ é a curva de intersecção das superfícies $z = x^2$ e $x^2 + y^2 = 1$. Use esse fato para esboçar a curva.


2447   

Calcule $\displaystyle\iint\limits_{S}g(x,y,z)dS,$ sendo $g(x,y,z)=x^{2}$ e $S$ o hemisfério superior de $x^{2}+y^{2}+z^{2}=a^{2}.$


$\dfrac{2\pi a^4}{3}.$


3032   

Considere a integral

$$\int_{0}^{1}\!\!\int_{x^{2}}^{1}x^{3}\sin{y^{3}}\,dy dx.$$

  1. Desenhe a região de integração.

  2. Calcule o valor da integral.


  1. A expressão 1 cossec x e o mesmo que

  2. $\dfrac{1 - \cos(1)}{12}$.


2360   

Encontre a área da superfície $z=1+3x+3y^{2}$ que está acima do triângulo com vértices $(0,0)$, $(0,1)$ e $(2,1).$


$\dfrac{1}{54}\left(46\sqrt{46} - 10\sqrt{10} \right).$


2709   

Determine a aproximação linear da função $f(x,y,z) = \sqrt{x^2 + y^2 + z^2}$ em $(3,2,6)$ e use-a para aproximar o número $\sqrt{(3,02)^2 + (1,97)^2 + (5,99)^2}$.


Vamos determinar a aproximação  linear da função $f$ em $(3,2,6)$. Primeiramente, calculamos as derivadas parcias $f_{x}$, $f_{y}$ e $f_{z}$, para todo $(x,y,z).$
$\bullet f_{x}(x,y,z)=\dfrac{1}{2}(x^{2}+y^{2}+z^{2})^{-1/2}\cdot 2x=\dfrac{x}{\sqrt{x^{2}+y^{2}+z^{2}}}.$
$\bullet f_{y}(x,y,z)=\dfrac{1}{2}(x^{2}+y^{2}+z^{2})^{-1/2}\cdot 2y=\dfrac{y}{\sqrt{x^{2}+y^{2}+z^{2}}}.$
$\bullet f_{z}(x,y,z)=\dfrac{1}{2}(x^{2}+y^{2}+z^{2})^{-1/2}\cdot 2z=\dfrac{z}{\sqrt{x^{2}+y^{2}+z^{2}}}.$
Agora, calculamos as derivadas parciais de $f$ no ponto $(3,2,6)$, então
$\bullet f_{x}(3,2,6)=\dfrac{3}{\sqrt{3^{2}+2^{2}+6^{2}}}=\dfrac{3}{7}.$
$\bullet f_{x}(3,2,6)=\dfrac{2}{\sqrt{3^{2}+2^{2}+6^{2}}}=\dfrac{2}{7}.$
$\bullet f_{x}(3,2,6)=\dfrac{6}{\sqrt{3^{2}+2^{2}+6^{2}}}=\dfrac{6}{7}.$
Assim, a aproximação linear da função $f$ em $(3,2,6)$ é
\begin{array}{rcl}f(x,y,z)&\approx & f(3,2,6)+f_{x}(3,2,6)(x-3)+f_{y}(3,2,6)(y-2)+f_{z}(3,2,6)(z-6)\\&=&7+\dfrac{3}{7}(x-3)+\frac{2}{7}(y-2)+\frac{6}{7}(z-6)\\&=&\dfrac{3}{7}x+\frac{2}{7}y+\frac{6}{7}z+\bigg(7-\dfrac{9}{7}-\dfrac{4}{7}-\dfrac{36}{7}\bigg)\\&=&\dfrac{3}{7}x+\frac{2}{7}y+\frac{6}{7}z.\end{array}
Agora, vamos aproximar o número $\sqrt{(3,02)^2 + (1,97)^2 + (5,99)^2}.$ Assim,
\begin{array}{rcl}\sqrt{(3,02)^2 + (1,97)^2 + (5,99)^2}&=&f(3,02\,,\,1,97\,,\,5,99)\\&\approx& \frac{3}{7}(3,02)+\frac{2}{7}(1,97)+\frac{6}{7}(5,99)\\&\approx& 6,9914.\end{array}


2506   

Faça o mapa de contorno da função $f(x,y)=(y-2x)^{2}$  mostrando várias de suas curvas de nível.


$y=2x\pm \sqrt{C}, C \geq0.$

A expressão 1 cossec x e o mesmo que


2246   

Calcule $\int_{C}\mathbf{F} \cdot \mathbf{n} \, ds$ ($\mathbf{n}$ é unitário), onde $\mathbf{F}(x,y) = x\mathbf{i} + y\mathbf{j}$, $C$ dada por $\mathbf{r}(t) = (t,t^2)$, $0 \leq t \leq 1$ e $\mathbf{n}$ a normal com componente $y < 0$.


$\dfrac{1}{3}$.


2525   

Esboce o gráfico da função $f(x,y)=g(\sqrt{x^{2}+y^{2}})$. Em geral, se $g$ é uma função de uma variável, como saber o gráfico de $f(x,y)=g(\sqrt{x^{2}+y^{2}})$ a partir do gráfico de $g$?


O gráfico de $f(x,y) = g(\sqrt{x^{2} + y^{2}})$ pode ser obtido rotacionando o gráfico de $g$ no plano $xz$ ao redor do eixo $z.$

A expressão 1 cossec x e o mesmo que


2874   

Utilize os multiplicadores de Lagrange para determinar os valores máximo e mínimo da função sujeita à(s) restrição(ões) dada(s).

$f(x,y,z) = x^4 + y^4 + z^4; \quad x^2 + y^2 + z^2 = 1.$


Valor máximo: $1;$ valor mínimo: $\dfrac{1}{3}.$


2700   

Verifique que $\dfrac{\partial ^{2}f}{\partial x^{2}}+\dfrac{\partial ^{2}f}{\partial y^{2}}=0$, onde $f(x,y)=\ln(x^{2}+y^{2}).$


$\displaystyle \frac{\partial^{2} f}{\partial x^{2}}= \frac{2 y^{2} - 2 x^{2}}{(x^{2} + y^{2})^{2}}\;\;\;\;\;\text{e}\;\;\;\;\; \frac{\partial^{2} f}{\partial y^{2}}= \frac{2 x^{2} - 2 y^{2}}{(x^{2} + y^{2})^{2}}.$


2715   

Determine uma equação do plano tangente à superfície no ponto especificado.

$z = y \ \mbox{cos}(x-y), \quad (2,2,2)$.


$z = y$.


1952   

Calcule a integral de linha $\int_{C}{\bf F}\cdot d{\bf r}$, onde $C$ é dada pela função vetorial ${\bf r}(t).$

${\bf F}(x,y,z)=x\,{\bf i}+y\,{\bf j}+z\,{\bf k}$, ${\bf r}(t)=(\cos t,\sin t,t)$, $0\leq t\leq 2\pi.$


$2\pi^{2}.$


2026   

Use a Regra da Cadeia para determinar $\mathrm{d}z/\mathrm{d} t$ ou $\mathrm{d}w/ \mathrm{d}t.$

$z=x^{2}y+xy^{2}$, $x=2+t^{2}$, $y=1-t^{3}$.


$\displaystyle \frac{dz}{dt} = 4(2xy + y^{2} )^{3} - 3 (x^{2} + 2xy)t^{2}.$


3070   

Experiências mostram que uma corrente contínua $I$ em um fio comprido produz um campo magnético ${\bf B}$ que é tangente a qualquer círculo em um plano perpendicular ao fio cujo centro seja o eixo do fio (como na figura). A Lei de Ampère relaciona a corrente elétrica ao campo magnético criado e afirma que

$$\int_{C}{\bf B}\cdot d{\bf r}=\mu_{0}I,$$
onde $I$ é a corrente total que passa por qualquer superfície limitada por uma curva fechada $C$ e $\mu_{0}$ é uma constante, chamada permeabilidade no vácuo. Tomando $C$ como um círculo de raio $r$, mostre que o módulo $B=|{\bf B}|$ do campo magnético a uma distância $r$ do centro do fio é dado por 
$$B=\frac{\mu_{0}I}{2\pi r}.$$

A expressão 1 cossec x e o mesmo que


Note que $\textbf{B}$ é tangente a qualquer círculo que está no plano perpendicular ao fio. Logo, $\textbf{B} = |\textbf{B}| \textbf{T},$ onde $\textbf{T}$ é a tangente unitária ao círculo $\textbf{C}$ parametrizado por $x = r \cos(\theta),$ $y = r \sin(\theta).$ Daí,
$\textbf{B} = |\textbf{B}| \left(-\sin(\theta),\cos(\theta) \right)$ e
$$\int_{C} \textbf{B} \cdot d\textbf{r} = \int_{0}^{2\pi} |\textbf{B}| \left( -\sin(\theta), \cos(\theta)\right)\cdot (\left(-r \sin(\theta), r\cos(\theta) \right)  d\theta = 2\pi r |\textbf{B}|.$$


2532   

Seja $f(x,y)=e^{xy}$ uma função de duas variáveis.

  1. Determine o domínio e a imagem de $f.$

  2. Esboce as curvas de nível de $f.$


  1. $D_{f} = \mathbb{R}^{2}$ e $Im(f) = \left\lbrace z \in \mathbb{R};\; z > 0 \right\rbrace.$

  2. $xy = C.$

A expressão 1 cossec x e o mesmo que


2502   

Encontre uma equação para a curva de nível da função $f(x,y)=\sqrt{x^{2}-1}$ que passa pelo ponto $(1,0)$.


$x = 1$ ou $x = -1.$


1928   

Encontre um campo de vetores $\textbf{G} = P(x,y)\textbf{i} + Q(x,y)\textbf{j}$ no plano $xy$ com a propriedade de que, em qualquer ponto $(a,b) \neq (0,0)$, $\textbf{G}$ é um vetor de magnitude $\sqrt{x^2+y^2}$ tangente à circunferência $x^2+y^2=a^2+b^2$ e aponta no sentido horário. (O campo é indefinido em (0,0).)


$\displaystyle \textbf{G} = \frac{y \textbf{i} - x \textbf{j}}{\sqrt{x^{2} + y^{2}}}.$


2016   

Encontre o volume do sólido delimitado pelo parabolóide $z=2+x^{2}+(y-2)^{2}$ e pelos planos $z=1$, $x=1$, $x=-1$, $y=0$ e $y=4.$



Observe que o sólido $E$ está abaixo da superfície $z = 2+x^2+(y-2)^2$ e acima do retângulo $[-1,1]\times [0,4]$ em $z=1$ (ver figura abaixo). 

A expressão 1 cossec x e o mesmo que

Algebricamente, $$E = \{(x,y,z) \in\mathbb{R}^3: -1 \leq x \leq 1, 0 \leq y \leq 4 \mbox{ e } 1 \leq z \leq 2 + x^2 + (y-2)^2\}.$$ Logo, o volume é dado por $$V = \iint\limits_{R}(2+x^2+(y-2)^2)\,dA - \iint\limits_{ R}\,dA,$$ em que $R = \{(x,y) \in \mathbb{R}^2; -1 \leq x \leq 1 \mbox{ e } 0 \leq y \leq 4 \}$. Assim, \begin{eqnarray*} V & = & \displaystyle\int_{-1}^{1}\int_{0}^{4}(x^2+y^2-4y+5)\,dy dx \\   & = & \displaystyle\int_{-1}^{1} \left.\left(x^2y+\frac{y^3}{3}-2y^2+5y \right|_{y=0}^{y=4} \right) \,dx \\     & = & \displaystyle\int_{-1}^{1} \left(4x^2+\frac{28}{3}\right) \,dx \\     & = & \left.\frac{4x^3}{3}+\frac{28x}{3} \right|_{x=-1}^{x=1} = \frac{64}{3}. \end{eqnarray*} Observe que, pelo Teorema de Fubini, podemos optar por calcular a integral $$\int_{0}^{4}\!\int_{-1}^{1}(x^2+y^2-4y+5)\,dy dx,$$ obtendo o mesmo resultado.


2767   

Encontre o valor de $\partial z/\partial x$ no ponto $(1,1,1)$ sabendo que a equação

$$xy+z^{3}x-2yz=0$$

define $z$ como uma função de duas variáveis independentes $x$ e $y$ e que a derivada parcial existe.


$\displaystyle \frac{\partial z}{\partial x} (1,1,1) = -2$.


2294   

Determine uma representação paramétrica para a superfície descrita a seguir. O paraboloide $z=9-x^{2}-y^{2}$, $z\geq 0.$


$x = r \cos(\theta),$ $y = r \sin(\theta),$ $z = 9 - r^2,$ onde $0 \leq r \leq 3$ e $0\leq \theta \leq 2\pi.$


2870   

Utilize os multiplicadores de Lagrange para determinar os valores máximo e mínimo da função sujeita à(s) restrição(ões) dada(s).

$f(x,y) = x^2 + y^2; \quad xy = 1.$


Não há valor máximo; valor mínimo: $2.$


2533   

Escreva seis integrais triplas iteradas diferentes para o volume do sólido retangular no  primeiro octante limitado pelos planos coordenados e pelos planos $x=1$, $y=2$ e $z=3$. Calcule uma das integrais.


$$\begin{split} 6 &= \int_{0}^{1}\int_{0}^{2}\int_{0}^{3} dz dy dx = \int_{0}^{2}\int_{0}^{1}\int_{0}^{3} dz dx dy = \int_{0}^{3}\int_{0}^{2}\int_{0}^{1} dx dy dz\\ &= \int_{0}^{2}\int_{0}^{3}\int_{0}^{1} dx dz dy = \int_{0}^{3}\int_{0}^{1}\int_{0}^{2} dy dx dz = \int_{0}^{1}\int_{0}^{3}\int_{0}^{2} dy dx dx. \end{split} $$


2858   

  1. Determine os pontos críticos da função

    $$f(x,y)=-(x^{2}-1)^{2}-(x^{2}y-x-1)^{2}.$$

  2. Calcule os valores assumidos por $f$ nos pontos críticos. É possível classificar os pontos críticos sem utilizar o críterio da derivada segunda? Se for possível, classifique-os e justifique a resposta.


  1. $(1,2)$ e $(-1,0).$

  2. $f(1,2) = f(-1,0) = 0.$ Note que $f(x,y) \leq 0,$ o que implica que $(1,2)$ e $(-1,0)$ são pontos de máximo.


2387   

Determine as equações do plano tangente e da reta normal à superfície dada, no ponto dado.

$2xyz = 3$, em $\left(\dfrac{1}{2},1,3\right)$.


Plano tangente: $6x + 3y + z = 9$,
Reta normal: $(x,y,z) = \left(\frac{1}{2},1,3\right) + \lambda (6,3,1),$ $\lambda \in \mathbb{R}.$


2751   

Considere a função

$$f(x,y) = \begin{cases}\dfrac{xy}{x^2 + y^2}, & \quad \text{se } (x,y) \neq (0,0),\\0, & \quad \text{se } (x,y) = (0,0).\\\end{cases}$$

Mostre que $f_x(0,0)$ e $f_y(0,0)$ existem, mas $f$ não é diferenciável em $(0,0)$.


$f_{x}(0,0) = f_{y}(0,0) = 0,$ mas $\lim_{(x,y) \to (0,0)} f(x,y)$ não existe, logo $f$ é discontínua em $(0,0)$ e portanto não é diferenciável neste ponto.


3076   

Calcule \(\displaystyle \lim_{(x,y)\to (-1,2)} \dfrac{xy}{x^2+y^2}\).



Como a função \(\displaystyle f(x,y)=\dfrac{xy}{x^2+y^2}\) é contínua no ponto \((-1,2)\) (de acumulação), basta avaliá-la neste mesmo ponto. Ou seja, \[ \lim_{(x,y)\to (-1,2)}\dfrac{xy}{x^2+y^2} = \dfrac{(-1)2}{(-1)^2+2^2} = -\dfrac{2}{5}. \]


2119   

Calcule $\mathrm{d} z/\mathrm{d} t$ por dois processos:

  1. substituindo as expressões para $x(t)$ e $y(t)$ em $z$ e depois derivando diretamente com relação a $t$
  2. aplicando a Regra da Cadeia: $\frac{dz}{dt}=\frac{\partial z}{\partial x}\frac{dx}{dt}+\frac{\partial z}{\partial y }\frac{dy}{dt}$.

$z=\ln(1+x^{2}+y^{2})$, $x=\sin{3t}$ e $y=\cos{3t}.$


$\displaystyle \frac{dz}{dt} (t) = 0.$


2890   

Passe para coordenadas polares e calcule: $\displaystyle\int_{0}^{a} \int_{0}^{\sqrt{a^{2}-x^{2}}}\,dy dx$


$\dfrac{\pi a^2}{4}.$


3080   

Mostre que os limites não existem, considerando que \((x,y)\rightarrow (0,0) \) ao longo dos eixos coordenados.

  1.  \[ \lim_{(x,y)\to(0,0)}\dfrac{x-y}{x^2+y^2} \]

  2.  \[ \lim_{(x,y)\to(0,0)}\dfrac{\cos(xy)}{x^2+y^2} \]


2515   

Encontre uma equação para a superfície de nível da função $f(x,y,z)=\sqrt{x-y}-\ln z$ que passa pelo ponto $(3,-1,1)$.


$\sqrt{x - y} - \ln(z) = 2.$


2326   

Calcule a área da superfície dada por: ${\bf r}(u,v)=\bigg(u,v,\dfrac{1}{2}u^{2}\bigg)$,$0\leq v\leq u$ e $u\leq 2.$. (Sugerimos ao leitor desenhar a imagem da superfície.)


$\dfrac{1}{3}\left(5\sqrt{5} - 1  \right).$


2083   

Calcule $\displaystyle\int_{(1,1)}^{(2,2)} y\,dx+x\,dy$.


$3.$


2156   

Demonstre que se $R$ é uma região no plano limitada por uma curva $C$ simples, fechada e suave por partes, então a área de $R$, denotada por $A(R)$, pode ser dada por

$$\oint_{C}x\, dy,$$

em que a curva está orientada no sentido positivo.



Temos que

$$A(R) = \iint\limits_{R} 1 \, dA.$$

A fim de utilizar o Teorema de Green, devemos encontrar funções $P$ e $Q$    que tenham derivadas de primeira ordem contínuas em um aberto que contenha a curva $C$ e o interior de $C$ e que satisfaçam a relação $\frac{\partial Q}{\partial x} - \frac{\partial P}{\partial y} = 1$. Observe que a curva $C$ já satisfaz as hipóteses desse teorema e $C$ é a fronteira de $R$. Um exemplo de funções $P$ e $Q$ é $P(x,y) = 0$ e $Q(x,y) = x$. Portanto, pelo Teorema de Green,

$$\iint\limits_{R} 1 \, dA = \oint_{C}0 \, dx + x\, dy = \oint_{C}x\, dy.$$


2597   

Seja $D$ a região limitada abaixo pelo plano $z=0$, acima pela esfera   $x^2+y^2+z^2=4$ e dos lados pelo cilindo $x^2+y^2=1$. Monte as integrais triplas em coordenadas cilíndricas que dão o volume de $D$ usando as ordens de integração a seguir.

  1.  $dzdrd\theta$

  2.  $drdzd\theta$

  3.  $d\theta dzdr$


  1.  $\displaystyle \int_{0}^{2\pi} \int_{0}^{1} \int_{0}^{\sqrt{4 - r^2}} r dz dr d\theta.$

  2.  $\displaystyle \int_{0}^{2\pi} \int_{0}^{\sqrt{3}} \int_{0}^{1} r  drdzd\theta +  \int_{0}^{2\pi} \int_{\sqrt{3}}^{2} \int_{0}^{\sqrt{4 - z^2}} r  drdzd\theta.$

  3.  $\displaystyle \int_{0}^{1} \int_{0}^{\sqrt{4 - r^2}} \int_{0}^{2\pi} r  d\theta dzdr.$


2596   

 Uma casca cilíndrica tem $20$ cm de comprimento, com raio interno de 6 cm e raio externo de $7$ cm. Escreva desigualdades que descrevam a casca em um sistema de coordenadas adequado. Explique como você posicionou o sistema de coordenadas em relação à casca.


$6 \leq r \leq 7,$ $0 \leq \theta \leq 2\pi,$ $0 \leq z \leq 20.$


2219   

Seja $f$ um campo escalar e $\mathbf{F}$ um campo vetorial. Diga se cada expressão tem significado. Em caso negativo, explique por quê. Em caso afirmativo, diga se é um campo vetorial ou escalar. 

  1. $\text{rot }{f}$;

  2. $\text{div }{\mathbf{F}}$;

  3. $\text{grad }{\mathbf{F}}$.


  1. $\text{rot }{f}$ não tem significado, pois $f$ é um campo escalar.

  2. $\text{div }{\mathbf{F}}$ é um campo escalar.

  3. $\text{grad }{\mathbf{F}}$ não tem sifnificado, pois $\bf{F}$ não é um campo escalar.


2904   

Utilize coordenadas polares para determinar o volume do sólido dado:

dentro da esfera $x^2+y^2+z^2=16$ e fora do cilindro $x^{2}+y^{2}=4.$


$\displaystyle 32\sqrt{3}\pi.$


3042   

  1. Determine a representação paramétrica do toro obtido girando em torno do eixo $z$ o círculo do plano $xz$ com centro em $(b,0,0)$ e raio $a < b.$ [Sugestão: tome como parâmetros os ângulos $\theta$ e $\alpha$ mostrados na figura.]

  2. Use a representação paramétrica do item anterior para achar a área do toro.

A expressão 1 cossec x e o mesmo que


  1. $x = b\cos(\theta) + a\cos(\alpha)\cos(\theta),$ $y = b\sin(\theta) + a\cos(\alpha)\sin(\theta),$ $z = a\sin(\alpha),$ onde $0 \leq \alpha \leq 2\pi,$ $0 \leq \theta \leq 2\pi.$

  2. $4\pi^2 ab.$


2547   

Um fluido tem densidade $870kg/m^{3}$ e escoa com velocidade $v=z{\bf i}+y^{2}{\bf j}+x^{2}{\bf k},$ onde $x$, $y$ e $z$ são medidos em metros e as componentes de $v$ em metros por segundo. Encontre a vazão para fora do cilindro $x^{2}+y^{2}=4$, $0\leq z\leq 1.$


$0$ kg/s.


2810   

Determine os valores máximos e mínimos locais e pontos de sela da função $f(x,y)=xy+2x-\ln(x^{2}y)$.


Ponto de mínimo: $\displaystyle \left(\frac{1}{2},2\right).$


2765   

Use a derivação implicíta para determinar $\partial z/\partial x$ e $\partial z/\partial y$ na expressão $x-z=\arctan(yz)$.


$\displaystyle \frac{\partial z}{\partial x} = \frac{1 + y^{2}z^{2}}{1 + y + y^{2}z^{2}}$

$\displaystyle \frac{\partial z}{\partial y} = -\frac{z}{1 + y + y^{2}z^{2}}$.


2995   

Calcule a integral, efetuando uma mudança de variáveis apropriada. $\displaystyle\iint\limits_{R}\dfrac{1}{\sqrt{xy}}\, dA$, em que $R$ é a região limitada pela curva $x+y = 1$ e pelos eixos coordenados.


$\pi.$


2299   

Identifique e faça um esboço da imagem da superfície parametrizada dada por ${\bf r}(u,v)=(u,v,u^{2}+v^{2})$, $(u,v)\in \mathbb{R}^{2}.$.


Paraboloide de rotação $z = x^2 + y^2.$


2583   

  1. Defina continuidade de uma função de duas variáveis $f(x,y)$ em um ponto $(x_0, y_0)$ de seu domínio.

  2. Dada a função

    $$f(x,y) = \begin{cases} \dfrac{x^2\sqrt{y}}{x^2 + y^2}, & \quad \text{se } (x,y) \neq (0,0), \\L, & \quad \text{se } (x,y) = (0,0),\end{cases}$$

    é possível encontrar $L$ de maneira que $f$ seja contínua em $(0,0)$?


  1. $f(x,y)$ é contínua em $(x_{0},y_{0}) \in D_{f}$ se

    $$\lim_{(x,y) \to (x_{0},y_{0})} f(x,y) = f(x_{0},y_{0}).$$

  2. $L = 0.$


2850   

Passe para coordenadas polares e calcule: $\displaystyle\iint\limits_{R}\,dx dy$, onde $R$ é a região, no plano $xy$, limitada pela curva (dada em coordenadas polares) $\rho=\cos(2\theta)$, $\dfrac{\pi}{8}\leq \theta \leq \dfrac{\pi}{4}.$


 $\displaystyle \frac{3\pi + 2}{32}.$


2786   

Suponha que $(0,2)$ seja um ponto crítico de uma função $g$ com derivadas de segunda ordem contínuas. Em cada caso, o que se pode dizer sobre $g$?

  1. $g_{xx}(0,2)=-1, \quad g_{xy}(0,2)=6, \quad g_{yy}(0,2)=1.$

  2. $g_{xx}(0,2)=-1, \quad g_{xy}(0,2)=2, \quad g_{yy}(0,2)=-8.$

  3. $g_{xx}(0,2)=4, \quad g_{xy}(0,2)=6, \quad g_{yy}(0,2)=9.$


  1. $g$ possui um ponto de sela em $(0,2).$

  2. $g$ possui um ponto de máximo local em $(0,2).$

  3. Não se pode afirmar algo sobre $g$ pelo Teste da Segunda Derivada.



Para fazer essa análise sobre $g$ iremos utilizar o Teste da Segunda Derivada.

Temos que

  1. \[ D=g_{xx}(0,2)g_{yy}(0,2)-g_{xy} ^2(0,2)=-1\cdot 1-6^2=-1-36=-37<0. \] Logo, pelo Teste da Segunda Derivada, segue que $(0,2)$ é um ponto de sela de $g$.
  2. Temos que \[ D=g_{xx}(0,2)g_{yy}(0,2)-g_{xy} ^2(0,2)=(-1)\cdot (-8)-2^2=8-4=4>0. \] Como $D>0$ e $g_{xx}(0,2)<0$, pelo Teste da Segunda Derivada, segue que $(0,2)$ é um ponto de máximo de $g$.
  3. Temos que \[ D=g_{xx}(0,2)g_{yy}(0,2)-g_{xy} ^2(0,2)=4\cdot 9-6^2=36-36=0. \] Como $D=0$ o Teste da Segunda Derivada não nos fornece nenhuma informação sobre $g$.


2833   

Uma caixa de papelão sem tampa deve ter um volume de $32000\;cm^{3}$. Determine as dimensões que minimizem a quantidade de papelão utilizado. 


$40$cm $\times$ $40$cm $\times$ $20$cm.


2943   

Calcule utilizando coordenadas esféricas. $\displaystyle\iiint\limits_{E}z\,dV$, onde $E$ está entre as esferas $x^{2}+y^{2}+z^{2}=1$ e $x^{2}+y^{2}+z^{2}=4$, no primeiro octante.


$\dfrac{15\pi}{16}.$


2473   

Determine e faça o esboço do domínio da função $f(x,y)=\sqrt{x+y}$.


$\left\lbrace (x,y);\; y \geq -x \right\rbrace$

A expressão 1 cossec x e o mesmo que


2733   

Determine as equações do plano tangente e da reta normal ao gráfico da função dada, no ponto dado. $f(x,y) = \arctan{(x - 2y)}$ em $\left(2, \dfrac{1}{2},f\left(2,\dfrac{1}{2}\right)\right)$.


Plano tangente: $4z = 2x - 4y + (\pi - 2)$

Reta normal: $(x,y,z) = \left(2,\frac{1}{2},\frac{\pi}{4} \right) + \lambda \left(\frac{1}{2},-1,-1 \right)$.


2039   

Utilize um diagrama em árvore para escrever a Regra da Cadeia para o caso dado. Suponha que todas as funções sejam diferenciáveis.

$t=f(u,v,w)$, onde $u=u(p,q,r,s)$, $v=v(p,q,r,s)$, $w=w(p,q,r,s)$.


$\displaystyle \frac{\partial t}{\partial p} = \frac{\partial t}{\partial u}\frac{\partial u}{\partial p} + \frac{\partial t}{\partial v}\frac{\partial v}{\partial p} + \frac{\partial t}{\partial w}\frac{\partial w}{\partial p},$ $\displaystyle \frac{\partial t}{\partial q} = \frac{\partial t}{\partial u}\frac{\partial u}{\partial q} + \frac{\partial t}{\partial v}\frac{\partial v}{\partial q} + \frac{\partial t}{\partial w}\frac{\partial w}{\partial q},$

$\displaystyle \frac{\partial t}{\partial r} = \frac{\partial t}{\partial u}\frac{\partial u}{\partial r} + \frac{\partial t}{\partial v}\frac{\partial v}{\partial r} + \frac{\partial t}{\partial w}\frac{\partial w}{\partial r}$ e $\displaystyle \frac{\partial t}{\partial s} = \frac{\partial t}{\partial u}\frac{\partial u}{\partial s} + \frac{\partial t}{\partial v}\frac{\partial v}{\partial s} + \frac{\partial t}{\partial w}\frac{\partial w}{\partial s}.$


2160   

Use o Teorema de Green para calcular a integral de linha ao longo da curva dada com orientação positiva. $\displaystyle\int_{C}e^y \, dx + 2xe^y \, dy$, $C$ é o quadrado de lados $x=0$, $x=1$, $y=0$ e $y=1$.


$e - 1.$


3154   

A fórmula de Taylor de primeira ordem para $f(\vec{a} + \vec{v})$ pode ser escrita como $ f(\vec{a}) + \nabla f(\vec{a}) \cdot \vec{v}$, já desconsiderando o termo de erro. Calcule-a para $f(x,y) = x^2 + y^2$, $\vec{a} = (1,0)$ e $\vec{v} = (2,1)$. Calcule também o erro cometido, dizendo se é um erro pequeno ou grande e por quê.


1982   

  1.  Faça um esboço grande da curva descrita pela função vetorial ${\bf r}(t)=(t^{2},t)$, $0\leq t\leq 2$, e desenhe os vetores ${\bf r}(1)$, ${\bf r}(1,1)$ e ${\bf r}(1,1)-{\bf r}(1)$.

  2. Desenhe o vetor ${\bf r}(1)$ começando em $(1,1)$ e compare com o vetor $$\frac{{\bf r}(1,1)-{\bf r}(1)}{0,1}.$$


2919   

Utilize a integral dupla para determinar a área da região: no interior do círculo $x^{2}+(y-1)^{2}=1$ e fora do círculo $x^{2}+y^{2}=1.$


$\displaystyle \frac{\pi}{3} + \frac{\sqrt{3}}{2}.$


2310   

 Calcule $D_{\bf{u}}f(x_0,y_0)$, sendo dados

$f(x,y) = xy$, $(x_0,y_0) = (1,1)$ e $\bf{u}$ o versor de $\bf{i} + \bf{j}$.


 $\displaystyle D_{\bf{(1,1)}}f(1,1) = \sqrt{2}.$ 


2045   

Utilize a Equação
$$ \dfrac{dy}{dx}=-\dfrac{\dfrac{\partial F}{\partial x}}{\dfrac{\partial F}{\partial y}}=-\dfrac{F_x}{F_y}$$
para determinar $\mathrm{d}y/\mathrm{d}x$.
$\cos(x-y)=xe^{y}$


$\displaystyle \frac{dy}{dx} = \frac{\sin(x - y) + e^{y} }{\sin(x - y) -x e^{y}} .$


2182   

Nos item abaixo: 

  1. expresse $\mathrm{d} w/\mathrm{d} t$ como uma função de $t$, usando a Regra da Cadeia, expressando $w$ em termos de $t$ e diferenciando em relação a $t$;
  2. calcule $\mathrm{d} w/\mathrm{d} t$ no valor dado de $t$.

$w=x^{2}+y^{2}$,  $x=\cos{t}$,  $y=\sin{t}$;  $t=\pi.$


  1. $\displaystyle \frac{dw}{dt}(t) = 0.$
  2. $\displaystyle \frac{dw}{dt}(\pi) = 0.$


2441   

Calcule a integral de superfície $\displaystyle\iint\limits_{S}z dS$, onde $S$ é a superfície $x=y+2z^{2}$, $0 \leq y\leq 1$, $0 \leq z \leq 1.$


$\dfrac{13\sqrt{2}}{12}.$


2689   

Passe para coordenadas polares e calcule.     

  1.   $\displaystyle\int_{0}^{1} \int_{1-\sqrt{1-x^{2}}}^{1+\sqrt{1-x^{2}}}xy\,dy dx$

  2.  $\displaystyle\int_{-a}^{a}\!\int_{-\sqrt{a^{2}-x^{2}}}^{\sqrt{a^{2}-x^{2}}}\,dy dx$          



  1. Temos que a região de integração é $$R=\{(x,y)\in\mathbb{R}^2|\, 0\leq x \leq 1\,\mbox{e}\, 1-\sqrt{1-x^{2}}\leq y \leq 1+\sqrt{1-x^{2}}\}.$$

    A expressão 1 cossec x e o mesmo que

    Passando para coordenadas polares temos que: $$\left\{ \begin{array}{cc} x=r\,\cos\theta \\ y=r\,\sin\theta \\ dy\,dx=r\,dr\,d\theta \\ \end{array} \right.$$ Agora, \begin{eqnarray*}  x^{2}+y^{2}=2y&\Rightarrow & r^{2}\,\cos^2 \theta+r^{2}\,\sin^{2}\theta=2r\,\sin\theta\\  &\Rightarrow & r^{2}=2r \,\sin\theta\\  &\Rightarrow & r(r-2\sin\theta )=0 \\  &\Rightarrow& r=0 \mbox{ou}  r=2\sin\theta.\end{eqnarray*} Logo, $\displaystyle 0\leq r \leq 2\,\sin\theta$ e $\displaystyle 0\leq\theta \leq\dfrac{\pi}{2}.$  Então, $$\int_{0}^{1}\int_{1-\sqrt{1-x^2}}^{1+\sqrt{1-x^2}}xy\,dy\,dx  =\int_{0}^{\frac{\pi}{2}}\int_{0}^{2\,\sin\theta}(r\,\cos\theta)(r\,\sin\theta)r\,dr d\theta $$
    $$ =\int_{0}^{\frac{\pi}{2}}\int_{0}^{2\,\sin\theta}r^3\,\sin\theta\, \cos\theta\,dr d\theta =\int_{0}^{\dfrac{\pi}{2}}\bigg[\frac{r^{4}} {4}\sin\theta\,\cos\theta\bigg]\bigg|_{0}^{2\,\sin\theta}\,d\theta $$  $$ =\int_0^{\frac{\pi}{2}}\frac{(2\,\sin\theta)^4}{4}\,\sin\theta\,\cos\theta\,d\theta  =4\int_{0}^\frac{\pi}{2}\sin^5\theta\,\cos\theta\, d\theta.$$ Tomando, $u=\sin\theta \Rightarrow du=\cos\theta\, d\theta$ e sendo $\theta =0 \Rightarrow u=0$ e $\theta=\frac{\pi}{2}\Rightarrow u=1.$ Assim,  $$\int_{0}^{1}\int_{1-\sqrt{1-x^{2}}}^{1+\sqrt{1-x^{2}}}xy\,dy dx=4\int_{0}^{1}u^{5}\,du$$  $$=4\cdot \frac{u^{6}}{6}\bigg|_{0}^{1}=\frac{2}{3}.$$

  2. Temos que a região de integração é $$R=\{(x,y)\in \mathbb{R}|\, -a\leq x \leq a,\, -\sqrt{a^{2}-x^{2}}\leq y \leq \sqrt{a^{2}-x^{2}}\}.$$       

    A expressão 1 cossec x e o mesmo que

    Passando para coordenadas polares temos que  $$\left\{ \begin{array}{cc}  x=r\,\cos\theta \\  y=r\,\sin\theta\\  dy\,dx=r\,dr\,d\theta\\ \end{array} \right. $$ Como $x^{2}+y^{2}=a^{2}\Rightarrow r^{2}\,\cos^{2}\theta+r^{2}\,\sin{2}\theta=a^{2}\Rightarrow  r^{2}=a^{2}\Rightarrow r=\pm a.$ Como o raio  deve ser sempre maior ou igual a zero, logo  $$0\leq r\leq a  \mbox{e}  0\leq \theta \leq 2\pi.$$  Então,  $$\int_{-a}^{a}\int_{-\sqrt{a^{2}-x^{2}}}^{\sqrt{a^{2}-x^{2}}}dy\,dx=\int_{0}^{2\pi}\int_{0}^{a}r\,dr\,d \theta=\int_{0}^{2\pi}d\theta \cdot \int_{0}^{a}r\,dr$$   $$=\theta\bigg |_{0}^{2\pi}\cdot \frac{r^{2}}{2}\bigg |_{0}^{a}=(2\pi)\cdot \bigg(\frac{a^{2}}{2}\bigg)=a^{2}\pi.$$


2677   

Calcule as derivadas parciais de $s = f(x,y,z,w)$ dada por $s = xw \ln{(x^2 + y^2 + z^2 + w^2)}$.


$\begin{aligned}[t]\frac{\partial s}{\partial x} &= w \left( \frac{2x^{2}}{x^{2} + y^{2} + z^{2} + w^{2}} + \ln (x^{2} + y^{2} + z^{2} + w^{2})\right),\\\frac{\partial s}{\partial y} &= \frac{2xyw}{x^{2} + y^{2} + z^{2} + w^{2}},\;\;\;\; \frac{\partial s}{\partial z} = w \frac{2xzw}{x^{2} + y^{2} + z^{2} + w^{2}}\;\;\;\;\;\text{e}\\\frac{\partial s}{\partial w} &= x \left( \frac{2w^{2}}{x^{2} + y^{2} + z^{2} + w^{2}} + \ln (x^{2} + y^{2} + z^{2} + w^{2})\right).\end{aligned}$


2901   

Determine os valores de máximo e mínimo de $f(x,y,z) = x^2 - yz$ em pontos da esfera $x^2 + y^2 + z^2 = 1$.


Valor máximo: $1;$ valor mínimo: $\displaystyle -\frac{1}{2}.$


3151   

Enuncie o Teorema da Divergência e o Teorema de Stokes, incluindo todas as hipóteses envolvidas.


2261   

Calcule $\displaystyle\iint\limits_{B} y\,dx dy$, onde $B$ é o conjunto dado.

  1. $B$ é o triângulo de vértices $(0,0)$, $(1,0)$ e $(1,1)$.

  2.  $B=\{(x,y)\in \mathbb{R}^{2}|\;-1\leq x\leq 1,\;0\leq y\leq x+2\}.$

  3.  $B$ é o conjunto de todos $(x,y)$ tais que $x^{2}+4y^{2}\leq 1.$

  4.  $B$ é o triângulo de vértices $(0,0)$, $(1,0)$ e $(2,1).$


  1.  $\dfrac{1}{6}$.

  2.  $\dfrac{13}{3}$.

  3.  $0$.

  4.  $\dfrac{1}{6}$.


2778   

Esboce a região cuja área é dada pela integral e calcule-a:  $\displaystyle\int_{0}^{\pi/2}  \int_{0}^{4\cos{\theta}}   r \, dr d\theta$


$2\pi;$ região de integração:

A expressão 1 cossec x e o mesmo que


2050   

Determine se o conjunto $\{(x,y)|\,x^{2}+y^{2}\leq 1\,$ ou\, $4\leq x^{2}+y^{2}\leq 9\}$ é ou não:

  1. aberto;

  2. conexo; e

  3. simplesmente conexo.



Temos que o conjunto $D=\{(x,y)|\,x^{2}+y^{2}\leq 1\,$ ou\, $4\leq x^{2}+y^{2}\leq 9\}$ consiste dos pontos que estão sobre ou dentro do círculo $x^{2}+y^{2}\leq 1$ juntamente com os pontos que estão em ou entre os círculos $x^{2}+y^{2}=4$ e $x^{2}+y^{2}=9$.

  1. $D$ não é aberto pois qualquer disco centrado em $(0,1)$ contém pontos que não estão em $D$.

  2. $D$ não é conexo pois não existe um caminho em $D$ conectando, por exemplo, os pontos $(1,0)$ e $(2,0)$.

  3. $D$ não é simplesmente conexo porque possui um buraco. Com efeito, a região delimitada pela curva simples e fechada $x^2+y^2=(5/2)^2$, contém pontos que não pertencem a $D$, por exemplo, o ponto $(0,3/2)$.


2484   

Use a integral tripla para determinar o volume do sólido dado.

  1.  $x^{2}\leq z \leq 1-y$ e $y\geq 0.$

  2.  $x^{2}+2y^{2}\leq z\leq 2a^{2}-x^{2}$ $(a>0).$

  3.  $x^{2}+y^{2}+(z-1)^{2}\leq 1$ e $z\geq x^{2}+y^{2}.$

  4.  $4x^{2}+9y^{2}+z^{2}\leq 4$ e $4x^{2}+9y^{2}\leq 1.$


  1.  $\dfrac{4}{15}.$

  2.  $\pi a^4.$

  3.  $\dfrac{71\pi}{54}.$

  4.  $\dfrac{7\pi}{12}.$


2794   

Determine os valores máximos e mínimos locais e pontos de sela da função $f(x,y)=x^{3}-3x^{2}+27y$.


Pontos de sela: $\displaystyle \left(3,\frac{3}{2}\right)$ e $\displaystyle \left(-3,-\frac{3}{2}\right).$


3143   

Considere o campo vetorial \(\mathbf{F}(x,y,z)=x^2\mathbf{i} + y^2\mathbf{j}+z^2\mathbf{k}\) e a superfície \(\sigma\) descrita como sendo a porção do cone \(z=\sqrt{x^2+y^2}\) abaixo do plano \(z=1\) e tendo orientação para cima. Verifique o Teorema de Stokes calculando, separadamente, a integral de linha e a integral dupla e, em seguida, comparando os valores.


2129   

Considere a função $F(x,y)=f\bigg(\dfrac{x}{y},\dfrac{y}{x}\bigg)$. Mostre que

$$x\dfrac{\partial F}{\partial x}+y\dfrac{\partial F}{\partial y}=0.$$



Note que$\displaystyle \frac{\partial F}{\partial x} = \frac{1}{y}\frac{\partial f}{\partial x}\left(\frac{x}{y}, \frac{y}{x} \right) - \frac{y}{x^{2}} \frac{\partial f}{\partial y}\left(\frac{x}{y}, \frac{y}{x} \right)$ e $\displaystyle \frac{\partial F}{\partial y} = -\frac{x}{y^{2}} \frac{\partial f}{\partial x}\left(\frac{x}{y}, \frac{y}{x} \right) + \frac{1}{x} \frac{\partial f}{\partial y}\left(\frac{x}{y}, \frac{y}{x} \right).$ 


2240   

A primeira identidade de Green é dada por:

$$\iint\limits_{ D} f\nabla^2g \, dA = \oint_{C}f(\nabla{g}) \cdot \mathbf{n} \, ds - \iint\limits_{ D}\nabla{f} \cdot \nabla{g} \, dA,$$

em que $D$ e $C$ satisfazem as hipóteses do Teorema de Green e as derivadas parciais apropriadas de $f$ e $g$ existem e são contínuas. (A quantidade $ \nabla{g} \cdot \mathbf{n} = D_{\mathbf{n}}g$ aparece na integral de linha. Essa é a derivada direcional na direção do vetor normal $\mathbf{n}$ e é chamada derivada normal de $g$.) Use-a para demonstrar a segunda identidade de Green:

$$\iint\limits_{ D} (f\nabla^2g - g\nabla^2f)\, dA = \oint_{C}(f\nabla{g} - g\nabla{f}) \cdot \mathbf{n} \, ds,$$

em que $D$ e $C$ satisfazem as hipóteses do Teorema de Green e as derivadas parciais apropriadas de $f$ e $g$ existem e são contínuas.


Note que pela primeira identidade de Green,

$$\iint\limits_{ D} (f\nabla^2g - g\nabla^2f)\, dA = \oint_{C}(f\nabla{g} \cdot \mathbf{n} - g\nabla{f} \cdot \mathbf{n}) \, ds, + \iint\limits_{ D} (\nabla f \cdot \nabla g - \nabla g \cdot \nabla f)\, dA.$$


2004   

Um arame fino é entortado no formato da semicircunferência $x^{2}+y^{2}=4$, $x\geq 0$. Se a densidade linear for uma constante $k$, determine a massa e o centro de massa do arame.


Massa: $k2\pi;$ centro de massa: $\displaystyle \left( \frac{4}{\pi},0 \right).$


2647   

Determine as derivadas parciais indicadas. $w=\dfrac{x}{y+2z}$; \;\;\;\;$\dfrac{\partial^{3}w}{\partial z\partial y \partial x}$, \;\;\;\;$\dfrac{\partial^{3}w}{\partial x^{2}\partial y}$.


$\displaystyle \frac{\partial^{3}w}{\partial z\partial y \partial x} = \frac{4}{(y + 2z)^{3}}\;\;\;\text{e} \;\;\;\; \frac{\partial^{3}w}{\partial x^{2}\partial y} = 0$.


3094   

Cada integral iterada abaixo representa o volume de um sólido. Faça um esboço do sólido. (Não é necessário calcular o volume.)

  1.  \(\displaystyle \int_0^5\int_1^2 4\, dxdy\)

  2.  \(\displaystyle \int_0^3\int_0^4\sqrt{25-x^2-y^2}\,dydx\)


2973   

Determine o jacobiano da transformação dada por: $x = 5u - v, \quad y = u + 3v$.


$16.$


2369   

A superfície de um lago é representada por uma região $D$ no plano $xy$, tal que a profundidade (em pés) sob o ponto correspondente a $(x,y)$ é dada por
$$f(x,y) = 300 - 2x^2 - 3y^2.$$
Se um nadador está no ponto $(4,9)$, em que direção deve nadar para que a profundidade sob ele decresça mais rapidamente?


 Na direção dada pelo vetor $(16,54).$


2745   

Mostre que o plano tangente ao parabolóide $z = x^2 + y^2$ no ponto $(1,2,5)$ intercepta o plano $xy$ na reta

$$\begin{cases}2x + 4y - 5 = 0 \\z = 0\end{cases}.$$


Note que o plano tangente no ponto $(1,2,5)$ é $z = 2x + 4y - 5$.


2455   

Encontre a constante $a$ tal que $$\int_{0}^{1}\int_{0}^{4-a-x^{2}}\int_{a}^{4-x^{2}-y}\;dz dy dx=\frac{4}{15}.$$


$\dfrac{13}{3}$ ou $3.$


2436   

Calcule a integral tripla.

  1. $\displaystyle\iiint\limits_{  E}\cos{z} \; dx dy dz$, onde $E$ é o conjunto $0\leq x \leq \dfrac{\pi}{2}$, $0\leq y \leq \dfrac{\pi}{2}$ e $x-y\leq z \leq x+y.$

  2. $\displaystyle\iiint\limits_{  E}(y-x)\;dx dy dz$, onde $E$ é o conjunto $4\leq x+y\leq 8$, $\dfrac{1}{x}\leq y\leq \dfrac{2}{x}$,  $y> x$ e $0\leq z \leq \dfrac{\sqrt[3]{xy}}{\sqrt{x+y}}.$


  1.  $2.$

  2.  $3 - 6\sqrt[3]{2} - 2\sqrt{2} + 6 \sqrt[6]{2^5}.$


2289   

Determine uma representação paramétrica para a superfície descrita a seguir. A parte do paraboloide elíptico $x+y^{2}+2z^{2}=4$ que está em frente ao plano $x=0.$


$y = u,$ $z = v,$ $x = 4 - u^2 - 2v^2,$ onde $u^{2} + 2v^2 \leq 4.$


2316   

Determine uma equação do plano tangente à superfície parametrizada dada no ponto especificado. $x=u+v$, $y=3u^{2}$, $z=u-v$; $(2,3,0).$


$3x - y + 3z = 3.$


2704   

Considere a função

$$f(x,y)=\begin{cases}x+y, & \quad \text{se } xy=0,\\\kappa, & \quad \text{caso contrário},\\\end{cases}$$

em que $\kappa$ é um número real. Determine as derivadas parciais de primeira ordem de $f$ em $(0,0).$


$\displaystyle \frac{\partial f}{\partial x} (0,0) = \frac{\partial f}{\partial y} (0,0) = 1$.


2801   

Determine os valores máximos e mínimos locais e pontos de sela da função $f(x,y)=x^{4}+y^{4}-2x^{2}-2y^{2}$.


Pontos de mínimo: $(-1,1)$ e $(-1,-1);$ ponto de máximo: $(0,0);$ pontos de sela: $(0,1), (0,-1), (1,0)$ e $(-1,0).$


2203   

Quando o tamanho das moléculas e suas forças de atração são levadas em conta, a pressão $P$, o volume $V$ e a temperatura $T$ 
de um mol de gás confinado estão relacionados pela {\it equação de van der Waals}
$$\bigg(P+\frac{a}{V^{2}}\bigg)(V-b)=kT,$$
em que $a$, $b$ e $k$ são constantes positivas. Se $t$ é o tempo, estabeleça uma fórmula para $\mathrm{d}T/ \mathrm{d}t$ em termos de $\mathrm{d}P/\mathrm{d} t$, 
$\mathrm{d} V/\mathrm{d}t$, $P$ e $V$.


$\displaystyle \frac{dT}{dt} = \frac{1}{k} \left( \left(\frac{dP}{dt} - \frac{2a}{V^{3}} \frac{dV}{dt}\right)(V - b) + \left( P + \frac{a}{V^{2}} \right) \frac{dV}{dt} \right).$


2550   

Suponha que $\displaystyle \lim_{(x,y) \to (3,1)}f(x,y) = 6$. O que podemos dizer do valor de $f(3,1)$? E se a função $f$ for contínua?


Nada se pode afirmar. Se $f$ for contínua em $(x_{0},y_{0}),$ $f(3,1) = 6.$


1984   

Determine a derivada da função vetorial.

  1. ${\bf r}(t)=(\tan (t), \sec (t), 1/t^{2})$
  2. ${\bf r}(t)=\sin^{-1}(t){\bf i}+\sqrt{1-t^{2}}{\bf j}+{\bf k}$


2601   

Seja $E$ a região limitada pelos paraboloides $z = x^2 + y^2$ e $z = 36 - 3x^2 - 3y^2$.

  1.  Ache o volume da região $E$.

  2.  Encontre o centroide de $E$ (centro de massa no caso em que a densidade é constante).


  1.  $162\pi.$

  2.  $(0,0,15)$.


3071   

Seja ${\bf F}(x,y)=(e^{x}\,\cos y+y, x-e^{x}\,\sin y)$. Calcule $\int_{C}{\bf F}\cdot d{\bf r}$, onde $C$ é o arco de circunferência que une o ponto $(-\sqrt{2}/2, \sqrt{2}/2)$ ao ponto $(1,0)$. Veja a figura abaixo.

A expressão 1 cossec x e o mesmo que



Notemos que ${\bf F}$ é um campo vetorial conservativo, pois: ${\bf F}$ é definido em todo $\mathbb{R}^{2}$; $P(x,y)=e^{x}\,\cos y+y$ e $Q(x,y)=x-e^{x}\,\sin y$ possuem derivadas parciais de primeira ordem contínuas;  $\dfrac{\partial P}{\partial y}(x,y)=1-e^{x}\,\sin y=\dfrac{\partial Q}{\partial x}(x,y).$

Sendo $F$ conservativo, existe $f$ tal que $\nabla f={\bf F}.$ Vamos encontrar $f$. Temos que 

$$f_{x}(x,y)=P(x,y)     \mbox{ e }      f_{y}(x,y)=Q(x,y).$$

Então,

$$\label{(2)}f_{x}(x,y)=e^{x}\,\cos y+y\Rightarrow f(x,y)=e^{x}\,\cos y+y+g(y).$$

Logo,  temos que

$$f_{y}(x,y)=-e^{x}\,\sin y+x+g'(y).$$

Como $f_{y}(x,y)=Q(x,y)$, obtemos que

$$-e^{x}\,\sin y+x+g'(y)=x-e^{x}\,\sin y\Rightarrow g'(y)=0\Rightarrow g(y)=C.$$

Assim, tomando $C=0$ segue que

$$f(x,y)=e^{x}\,\cos y+xy.$$

Do resultado acima e pelo Teorema Fundamental da Integral de Linha, temos que

$$\int_{C}{\bf F}\cdot d{\bf r}=f(1,0)-f\bigg(-\frac{\sqrt{2}}{2},\frac{\sqrt{2}}{2}\bigg)=e-e^{-\frac{\sqrt{2}}{2}}\,\cdot\cos\bigg(\frac{\sqrt{2}}{2}\bigg)+\frac{1}{2}.$$


3140   

Prove a seguinte identidade \[ \iint\limits_\sigma\nabla f\cdot\mathbf{n}\,dS = \iiint\limits_G\Delta f\,dV, \] supondo que \(\sigma\) e \(G\) satisfaçam as hipóteses do Teorema da Diverência e que \(f(x,y,z)\) cumpra os requisitos de diferenciabilidade necessários. Acima, \(\displaystyle \Delta f= \dfrac{\partial^2f}{\partial x^2}+\dfrac{\partial^2f}{\partial y^2}+\dfrac{\partial^2f}{\partial z^2}\) é denominado Laplaciano de \(f\).


2069   

Utilize as Equações 

$\dfrac{\partial z}{\partial x}=-\dfrac{\dfrac{\partial F}{\partial x}}{\dfrac{\partial F}{\partial z}}$ e $\dfrac{\partial z}{\partial y}=-\dfrac{\dfrac{\partial F}{\partial y}}{\dfrac{\partial F}{\partial z}}$

para determinar $\partial z/\partial x$ e $\partial z/\partial y$.

$xyz=\cos(x+y+z)$


$\displaystyle \frac{dz}{dx} = \frac{yz + \sin(x + y + z)}{xy + \sin(x + y + z)}$ e $\displaystyle \frac{dz}{dy} = \frac{xz + \sin(x + y + z)}{xy + \sin(x + y + z)}.$


2787   

Utilize as curvas de nível da figura para predizer a localização dos pontos críticos de $f(x,y)=4+x^{3}+y^{3}-3xy$ e se $f$ tem um ponto de sela ou um máximo ou mínimo local em cada um desses pontos. Explique seu raciocínio. Em seguida, empregue o Teste da Segunda Derivada para confirmar suas predições.

A expressão 1 cossec x e o mesmo que


$f$ possui um ponto de sela em $(0,0)$ e um mínimo local em $(1,1).$


2963   

O centróide de uma região $E$ é dado por

$$\overline{x}=\frac{1}{vol(E)}\int_{E}x\,dV,\;\;\;\; \overline{y}=\frac{1}{vol(E)}\int_{E}y\,dV\;\; \text{e}\;\; \overline{z}=\frac{1}{vol(E)}\int_{E}z\,dV.$$

Calcule o centróide da região dada em coordenadas esféricas por $0\leq \rho \leq 1$, $0\leq\phi \leq \pi/3$ e $0\leq \theta \leq 2\pi$ (observe que, devido à simetria da região, $\overline{x}$ e $\overline{y}$ se anulam, bastando calcular a terceira coordenada).


$\overline{z} = \dfrac{9}{16}.$


2681   

Encontre $\partial f/\partial x$ e $\partial f/\partial y$ para $f(x,y)=(xy-1)^{2}$.


$\displaystyle \frac{\partial f}{\partial x} = 2y(xy - 1)\;\;\;\;\text{e}\;\;\;\; \frac{\partial f}{\partial y} = 2x (xy - 1)$.


2252   

Demonstre a identidade abaixo, supondo que $S$ e $E$ satisfaçam as condições do Teorema do Divergente e que as funções escalares e as componentes dos campos vetoriais tenham derivadas parciais de segunda ordem contínuas.

  1. $\displaystyle\iint\limits_{S}{\bf a}\cdot {\bf n}\,dS=0$, onde ${\bf a}$ é um vetor constante.



Dica: Note que $\mbox{div} {\bf a} = 0.$


2051   

Determine se ${\bf F}(x,y)=y\,{\bf i}+x\,{\bf j}$ é ou não um campo vetorial conservativo. Se for, determine uma função $f$ tal que ${\bf F}=\nabla f.$


Sim. $f(x,y) = xy + K.$


2684   

Encontre $\partial f/\partial x$ e $\partial f/\partial y$ para $f(x,y)=e^{xy}\ln{y}$.


$\displaystyle \frac{\partial f}{\partial x} = ye^{xy}\ln y\;\;\;\;\text{e}\;\;\;\; \frac{\partial f}{\partial y} = x e^{xy} \ln y + \frac{e^{xy}}{y}$.


2730   

Determine as equações do plano tangente e da reta normal ao gráfico da função dada, no ponto dado. $f(x,y) = x^2 + y^2$ em $(0,1,f(0,1))$.


Plano tangente: $z = 2y - 1$

Reta normal: $(x,y,z) = \left(0,1,1 \right) + \lambda \left(0,2,-1 \right)$.


3065   

As linhas de escoamento (ou linhas de corrente) de um campo vetorial são as trajetórias seguidas por uma partícula cujo campo de velocidade é um campo vetorial dado. Assim, os vetores do campo vetorial são tangentes a suas linhas de escoamento.

Use um esboço do campo vetorial $\textbf{F}(x,y) = x\textbf{i} - y\textbf{j}$ para desenhar algumas linhas de escoamento. Desses seus esboços é possível descobrir qual é a equação das linhas de escoamento?
Se as equações paramétricas de uma linha de escoamento são $x=x(t)$ e $y=y(t)$, explique por que essas funções satisfazem as equações diferenciais $dx/dt = x$ e $dy/dt = -y$. Resolva então as equações de forma a obter uma equação da linha de escoamento que passe pelo ponto $(1,1)$.


2198   

Encontre $\partial w/ \partial r$ quando $r=1$, $s=-1$ se $w=(x+y+z)^{2}$, $x=r-s$, $y=\cos(r+s)$, $z=\sin(r+s).$


$\displaystyle \frac{\partial w}{\partial r}(x(1,-1),y(1,-1),z(-1,1)) = 12.$


2937   

Determine a massa e o centro de massa da lâmina que ocupa a região $\displaystyle D = \{(x,y) \in \mathbb{R}^2: 0 \leq x \leq 2, \ -1 \leq y \leq 1\}$ e tem função densidade $\rho(x,y) = xy^2.$


Massa: $\dfrac{4}{3};$ centro de massa: $\displaystyle \left(\frac{4}{3},0 \right).$


2104   

Seja ${\bf F}(x,y)=\dfrac{-y\,{\bf i}+x\,{\bf j}}{x^{2}+y^{2}}.$

  1. Mostre que $\dfrac{\partial P}{\partial y}=\dfrac{\partial Q}{\partial x}.$

  2. Mostre que $\int_{C}{\bf F}\cdot d{\bf r}$ não é independente do caminho. [Sugestão: calcule $\int_{C_{1}}{\bf F}\cdot d{\bf r}$ e $\int_{C_{2}}{\bf F}\cdot d{\bf r}$, onde $C_{1}$ e $C_{2}$ são as metades superior e inferior do círculo $x^{2}+y^{2}=1$ de $(1,0)$ a $(-1,0)$.] Isso contraria o Teorema 6 (Seção 16.3 do Livro do James Stewart)?


  1. $\dfrac{\partial P}{\partial y}= \dfrac{y^{2} - x^{2}}{(x^{2} + y^{2})^{2}} = \dfrac{\partial Q}{\partial x}.$

  2. Tome $C_{1}$ a curva parametrizada por $\mathbf{r_{1}}(t) = (\cos(t), \sin(t)),$ $0 \leq t \leq \pi$ e $C_{2}$ a curva parametrizada por $\mathbf{r_{2}}(t) = (\cos(t), \sin(t)),$ de $t = 2\pi$ a $t = \pi.$ Segue que $\int_{C_{1}} \mathbf{F} \cdot d \mathbf{r}  = \pi \neq -\pi = \int_{C_{2}} \mathbf{F} \cdot d \mathbf{r}.$ Como o domínio de $\mathbf{F}$ é $\mathbb{R}^{2} \setminus \left\lbrace (0,0) \right\rbrace$ que não é simplesmente conexo, o resultado não contradiz o Teorema 6.


1938   

Calcule a integral de linha, onde $C$ é a curva dada.

$\displaystyle\int_{C}xe^{yz}\,ds$,   $C$ é o segmento de reta de $(0,0,0)$ a $(1,2,3).$


$\dfrac{\sqrt{14}}{12}\left(e^{6} - 1 \right).$


2134   

Mostre que cada a equação a seguir define implicitamente pelo  menos uma função diferenciável $y=y(x).$ 
$y^{4}+x^{2}y^{2}+x^{4}=3$
 


$\displaystyle \frac{d y}{d x} = - \frac{2xy^{2} + 4x^{3}}{4y^{3} + 2x^{2}y}.$


3049   

 Calcule a integral tripla $\int\int\int\limits_{T}x^{2}dV$, 

onde $T$ é o tetraedro sólido com vértices $(0,0,0)$, $(1,0,0)$, $(0,1,0)$ e $(0,0,1).$



Para resolvermos a integral tripla, vamos desenhar dois diagramas: um da região sólida $T$ (Figura 1) e o outro a sua projeção $D$ no plano $xy$ (Figura 2). 

A expressão 1 cossec x e o mesmo que

A fronteira inferior do tetraedro $T$ é o plano $z=0$ e a superior é o plano $x+y+z=1$ (ou $z=1-x-y$). 

Notemos que os planos $x+y+z=1$ e $z=0$ se interceptam na reta $x+y=1$ (ou $y=1-x$) no plano $xy.$ 

Logo a projeção de $T$ é a região triangular da Figura 2 e temos 

$$T=\{(x,y,z)|\,0\leq x \leq 1,\, 0\leq y \leq 1-x,\, 0\leq z \leq 1-x-y\}.$$

Assim, 

$$\int\int\int\limits_{T}x^{2}\,dV=\int_{0}^{1}\int_{0}^{1-x}\int_{0}^{1-x-z}x^{2}\,dz\,dy\,dx=\int_{0}^{1}\int_{0}^{1-x}x^{2}z\bigg|_{0}^{1-x-y}\,dy\,dx$$

$$=\int_{0}^{1}\int_{0}^{1-x}x^{2}(1-x-y)\,dy\,dx=\int_{0}^{1}\int_{0}^{1-x}(x^{2}-x^{3}-x^{2}y)\,dy\,dx$$

$$=\int_{0}^{1}\bigg(x^{2}y-x^{3}y-x^{2}\frac{y^{2}}{2}\bigg)\bigg|_{0}^{1-x}\,dx=\int_{0}^{1}\bigg(x^{2}(1-x)-x^{3}(1-x)-\frac{x^{2}}{2}(1-x)^{2}\bigg)dx$$

$$=\int_{0}^{1}\bigg(\frac{x^{2}}{2}-x^{3}+\frac{x^{4}}{2}\bigg)\,dx =\bigg[\frac{1}{2}\cdot\frac{x^{3}}{3}-\frac{x^{4}}{4}+\frac{1}{2}\cdot\frac{x^{5}}{5}\bigg]\bigg|_{0}^{1}=\frac{1}{60}.$$


2988   

Calcule a integral, efetuando uma mudança de variáveis apropriada. $\displaystyle\iint\limits_{R} \sin{(9x^2 + 4y^2)} \, dA$, em que $R$ é a região do primeiro quadrante limitada pela elipse $9x^2 + 4y^2 = 1$.


$\dfrac{\pi}{24}(1 - \cos(1)).$


2230   

Mostre que qualquer campo vetorial da forma

$$\mathbf{F}(x,y,z) = f(y,z)\mathbf{i} + g(x,z)\mathbf{j} + h(x,y)\mathbf{k}$$

é incompressível.


Note que $\text{div } \mathbf{F} = 0.$


2928   

Esboce o sólido descrito por $\rho \leq 2$, $0\leq \phi \leq \pi/2$ e $0\leq \theta \leq \pi/2.$


3034   

Uma região $R$ é mostrada na figura abaixo. Decida se você deve usar coordenadas polares ou retangulares e escreva $\iint \limits_{ R}f(x,y)\,dA$ como uma integral iterada, onde $f$ é uma função qualquer contínua em $R.$


A expressão 1 cossec x e o mesmo que


$\displaystyle \int_{0}^{\frac{3\pi}{2}} \int_{0}^{4} f(r\cos(\theta),r\sin(\theta)) r  d r d \theta.$


2634   

Considere a função dada por $z=x \sin\bigg(\dfrac{x}{y}\bigg).$ Verifique que

$$x\;\dfrac{\partial z}{\partial x}+y\;\dfrac{\partial z}{\partial y}=z.$$



Primeiramente, vamos calcular $\dfrac{\partial z}{\partial x}$ e $\dfrac{\partial z}{\partial }.$ Assim,\\

$\bullet $ $\dfrac{\partial z}{\partial x}=$ $\dfrac{\partial}{\partial x}\bigg[x\cdot \sin\bigg(\dfrac{x}{y}\bigg)\bigg]=

1\cdot \sin\bigg(\dfrac{x}{y}\bigg)+x\cdot \cos \bigg(\dfrac{x}{y}\bigg)\cdot \dfrac{1}{y}$

$$=\sin\bigg(\frac{x}{y}\bigg)+\frac{x}{y}\cdot \cos\bigg(\frac{x}{y}\bigg)$$

$\bullet $ $\dfrac{\partial z}{\partial y}=$ $\dfrac{\partial}{\partial y}\bigg[x\cdot \sin\bigg(\dfrac{x}{y}\bigg)\bigg]=

0\cdot \sin\bigg(\dfrac{x}{y}\bigg)+x\cdot \cos \bigg(\dfrac{x}{y}\bigg)\cdot \bigg(-\dfrac{x}{y^{2}}\bigg)$

$$=-\frac{x^{2}}{y^{2}}\cdot \cos\bigg(\frac{x}{y}\bigg).$$

Então,

$$x\cdot \frac{\partial z}{\partial x}+y\frac{\partial z}{\partial y}=x\cdot \bigg[\sin\bigg(\frac{x}{y}\bigg)+\frac{x}{y}\cdot \cos\bigg(\frac{x}{y}\bigg)\bigg] +

y\cdot\bigg[ -\frac{x^{2}}{y^{2}}\cdot \cos\bigg(\frac{x}{y}\bigg)\bigg]$$

$$=x\cdot \sin\bigg(\frac{x}{y}\bigg)+\frac{x^{2}}{y}\cos\bigg(\frac{x}{y}\bigg)-\frac{x^{2}}{y}\cdot \cos\bigg(\frac{x}{y}\bigg)$$

$$x\cdot \sin\bigg(\frac{x}{y}\bigg)=z.$$


2158   

Calcule a integral de linha $\displaystyle\oint_{C} xy \, dx + x^2 \, dy$, $C$ é o retângulo com vértices $(0,0)$, $(3,0)$, $(3,1)$ e $(0,1)$ por dois métodos:

  1. diretamente; e

  2. utilizando o Teorema de Green.


$\dfrac{9}{2}.$


3014   

  1.  Verifique que $$f(x,y) = \begin{cases} 4xy, & \quad \text{se } 0 \leq x \leq 1, \ 0 \leq y \leq 1,\\ 0, & \quad \text{caso contrário}, \end{cases}$$ é uma função densidade conjunta.

  2.  Se $X$ e $Y$ são variáveis aleatórias cuja função densidade conjunta é a função $f$ do item anterior, determine: (i) $P(X \geq \frac{1}{2})$,          (ii) $P(X \geq \frac{1}{2}, Y \leq \frac{1}{2})$.

  3.  Determine os valores esperados de $X$ e $Y$.


  1.  Note que $$\iint_\limits{\mathbb{R}^2} f(x,y)\,dA = \int_{0}^{1} \int_{0}^{1} 4xy\, dydx = 1. $$

  2.  (i) $\dfrac{3}{4}.$               (ii) $\dfrac{3}{16}.$

  3.  $\dfrac{3}{16}.$


2278   

Calcule $\nabla f(x,y)$.

$f(x,y) = \arctan{\dfrac{x}{y}}$


 $\displaystyle \nabla f(x,y) = \left(\frac{y }{x^{2} + y^{2}}, -\frac{x}{x^{2} + y^{2}} \right).$


2910   

Calcule a integral iterada $\int_{-3}^{3} \int_{0}^{\sqrt{9-x^2}}\sin(x^{2}+y^{2})\,dy dx$, convertendo-a antes para coordenadas polares.


$\displaystyle \frac{\pi}{2}(1 - \cos(9)).$


2080   

A temperatura em um ponto $(x,y)$ é $T(x,y)$, medida em graus Celsius. Um inseto rasteja de modo que sua posição depois de 

$t$ segundos seja dada por $x=\sqrt{1+t}$ e $y=2+\dfrac{1}{3}t$, onde $x$ e $y$ são medidas em centímetros. A função temperatura satisfaz 

$T_{x}(2,3)=4$ e $T_{y}(2,3)=3$. Quão rápido a temperatura aumenta no caminho do inseto depois de três segundos?


A temperatura aumenta a uma taxa de $2º$C/s.


1961   

Calcule a integral de linha $\displaystyle\int_{C}(x-y)\,dx+e^{x+y}\, dy$, onde $C$ é a fronteira do triângulo de vértices $(0,0)$, $(0,1)$ e $(1,2)$, orientada no sentido anti-horário.


$\displaystyle \frac{e^{3}}{6} - \frac{e}{2} + \frac{5}{6}.$


2127   

Seja $z=f(u+2v,u^{2}-v)$. Expresse $\partial z/\partial u$ e $\partial z/\partial v$ em termos das 

derivadas parciais de $f$. 


$\displaystyle \frac{\partial z}{\partial u}(u,v) = \frac{\partial f}{\partial x}(u + 2v,u^{2} - v) + 2u \frac{\partial f}{\partial y}(u + 2v,u^{2} - v)$ e\\ $\displaystyle \frac{\partial z}{\partial v}(u,v) = 2 \frac{\partial f}{\partial x}(u + 2v,u^{2} - v) - \frac{\partial f}{\partial y}(u + 2v,u^{2} - v).$


2724   

Se $z = x^2 - xy + 3y^2$ e $(x,y)$ varia de $(3;-1)$ a $(2,96;-0,95)$, compare os valores de $\Delta z$ e $dz$.


$\Delta z = -0.7189$ e $dz = -0.73$.


2573   

Calcule $\displaystyle \lim_{(x,y) \to (0,0)} \dfrac{x + y}{x - y}$, caso exista.


Não existe.


3015   

  1.  Uma luminária tem duas lâmpadas de um tipo com tempo de vida médio de 1.000 horas. Supondo que possamos modelar a probabilidade de falha dessas lâmpadas por uma função densidade exponencial com média $\mu = 1.000$, determine a probabilidade de que ambas as lâmpadas venham a falhar dentro de um período de 1.000 horas.

  2.  Outra luminária tem somente uma lâmpada do mesmo tipo das do item anterior. Se a lâmpada queima e é trocada por outra to mesmo tipo, determine a probabilidade de que as duas venham a falhar dentro de 1.000 horas.


  1. $(e^{-1} - 1)^2.$

  2. $1 - 2e^{-1}.$


2544   

Encontre o fluxo exterior do campo ${\bf F}=2xy{\bf i}+2yz{\bf j}+2xz{\bf k}$ ao longo da superfície do cubo cortado do primeiro octante pelos planos $x=a$, $y=a$ e $z=a.$


$3\pi a^4.$


2070   

Utilize as Equações 

$\dfrac{\partial z}{\partial x}=-\dfrac{\dfrac{\partial F}{\partial x}}{\dfrac{\partial F}{\partial z}}$ e $\dfrac{\partial z}{\partial y}=-\dfrac{\dfrac{\partial F}{\partial y}}{\dfrac{\partial F}{\partial z}}$

para determinar $\partial z/\partial x$ e $\partial z/\partial y$.

$yz=\ln(x+z)$


$\displaystyle \frac{dz}{dx} = \frac{1}{y(x+z)-1}$ e $\displaystyle \frac{dz}{dy} = \frac{z(x+z)}{y(x+z)-1}.$


2672   

Determine $\dfrac{ \partial f}{\partial x}$ e $\dfrac{\partial f}{\partial y}$, sendo $f(x,y)= \begin{cases}\dfrac{x+y^{4}}{x^{2}+y^{2}}, & \quad \text{se } (x,y)\neq (0,0),\\0, & \quad \text{se } (x,y)=(0,0).\\\end{cases}$


$\begin{aligned}[t]\frac{\partial f}{\partial x} &= \begin{cases}\dfrac{y^{2} - x^{2} - 2xy^{4}}{(x^{2}+y^{2})^{2}}, & \quad \text{se } (x,y)\neq (0,0),\\\text{não existe} & \quad \text{se } (x,y)=(0,0)\\\end{cases} \;\;\;\; \text{e}\\\frac{\partial f}{\partial y} &= \begin{cases}\dfrac{4x^{2}y^{3} + 2y^{5} - 2xy}{x^{2}+y^{2}}, & \quad \text{se } (x,y)\neq (0,0),\\0, & \quad \text{se } (x,y)=(0,0).\\\end{cases}\end{aligned}$


3047   

O campo vetorial $\mathbf{F}$ é mostrado no plano $xy$ e é o mesmo em todos os planos horizontais (em outras palavras, $\mathbf{F}$ é independente de $z$ e sua componente $z$ é 0).

  1. O $\text{div }{\mathbf{F}}$ será positivo, negativo ou nulo? Justifique.

  2. Determine se o $\text{rot }{\mathbf{F}} = 0$. Se não, em que direção rot $\mathbf{F}$ aponta?

A expressão 1 cossec x e o mesmo que


  1. Positivo.

  2. $\text{rot } \bf{F} = \bf{0}.$


2703   

Considere a função

$$f(x,y)=\log(9-x^{2}-9y^{2}).$$

  1. Esboce no plano $xy$ o domínio de $f.$

  2. Calcule as derivadas parciais $f_{x}$ e $f_{y}.$


  1. $D_{f} = \left\lbrace (x,y) \in \mathbb{R}^{2};\; x^{2} -9y^{2} < 9 \right\rbrace$.
    A expressão 1 cossec x e o mesmo que

  2. $\displaystyle f_{x} = \frac{-2x}{9 - x^{2} - 9y^{2}}  \;\;\;\text{e}\;\;\;f_{y} = \frac{-18y}{9 - x^{2} - 9y^{2}}$.


2487   

Esboce o gráfico da função $f(x,y)=\cos{x}$.


$z = \cos(x)$

A expressão 1 cossec x e o mesmo que


2559   

Determine o limite, se existir, ou mostre que o limite não existe.

$\displaystyle \lim_{(x,y) \to (0,0)} \dfrac{x^2 \ \mbox{sen}^2y}{x^2 + 2y^2}$.


$0.$


2508   

Faça o mapa de contorno da função $f(x,y)=ye^{x}$ mostrando várias de suas curvas de nível.


$y = Ce^{-x}.$

A expressão 1 cossec x e o mesmo que


2202   

Os lados iguais e o ângulo correspondente de um triângulo isósceles estão aumentando à razão de $3cm/h$ e $2^{\circ}/h$, respectivamente. Ache a taxa à qual a área do triângulo está aumentando no instante em que o comprimento de cada um dos 

lados iguais é de $6$ metros e o ângulo correspondente é $60^{\circ}.$


$\approx 181559$ cm$^{2}/$h.


2673   

Calcule as derivadas parciais de $f(x,y,z) = xe^{x - y - z}$.


$\displaystyle \frac{\partial f}{\partial x} = (1 + x)e^{x - y - z},\;\;\;\; \frac{\partial f}{\partial y} = -x e^{x - y - z}\;\;\;\;\;\text{e}\;\;\;\;\;\frac{\partial f}{\partial z} = -x e^{x - y - z}.$


2876   

Utilize os multiplicadores de Lagrange para determinar os valores máximo e mínimo da função sujeita à(s) restrição(ões) dada(s).

$f(x,y,z) = yz + xy; \quad xy = 1, \quad y^2 + z^2 = 1.$


Valor máximo: $\dfrac{3}{2};$ valor mínimo: $\dfrac{1}{2}.$


2296   

Determine uma representação paramétrica para a superfície descrita a seguir. A porção da esfera $x^{2}+y^{2}+z^{2}=3$ entre os planos $z=\sqrt{3}/2$ e $z=-\sqrt{3}/2.$


$x = \sqrt{3}\sin(\phi)\cos(\theta),$ $y = \sqrt{3}\sin(\phi)\sin(\theta),$ $z = \sqrt{3}\cos(\phi),$ onde $\dfrac{\pi}{3} \leq \phi \leq \dfrac{2\pi}{3}$ e $0 \leq \theta \leq 2\pi.$


2808   

Determine os valores máximos e mínimos locais e pontos de sela da função $f(x,y)=\frac{1}{x^{2}}+\frac{1}{y}+xy$, $x>0$ e $y>0$.


Ponto de mínimo: $\displaystyle \left( 2^{2/5}, 2^{-1/5} \right).$


2059   

Determine se ${\bf F}(x,y,z)=x\,{\bf i}+y\,{\bf j}+z\,{\bf k}$ é ou não um campo vetorial conservativo. Se for, determine uma função $f$ tal que ${\bf F}=\nabla f.$


Sim. $f(x,y,z) = \dfrac{x^{2} + y^{2} + z^{2}}{2} + K.$


2820   

Determine os valores máximo e mínimo absolutos de $f$ no conjunto $D.$

$f(x,y)=x^{2}+3xy-3x$ em $D=\{(x,y)\in \mathbb{R}^2: x\geq 0,\;y\geq 0\; \text{e} \;x+y\leq 1\}.$


Valor máximo: $\displaystyle 0;$ valor mínimo: $-2.$


2538   

Determine o sólido $E$ para o qual a integral $$ \iiint\limits_{  E}(1-x^{2}-2y^{2}-3z^{2})\,dV$$ é máxima.


$E = \left\{ (x,y,z);  x^2 + 2y^2 + 3z^2 \leq 1 \right\}.$


2058   

Determine se ${\bf F}(x,y,z)=\dfrac{x}{(x^{2}+y^{2}+z^{2})^{2}}\,{\bf i}+\dfrac{y}{(x^{2}+y^{2}+z^{2})^{2}}\,{\bf j}+\dfrac{z}{(x^{2}+y^{2}+z^{2})^{2}}\,{\bf k}$ é ou não um campo vetorial conservativo. Se for, determine uma função $f$ tal que ${\bf F}=\nabla f.$


Sim. $f(x,y,z) = -\dfrac{1}{2(x^2 + y^{2} +z^{2})} + K.$


2941   

Calcule utilizando coordenadas esféricas. $\displaystyle\iiint\limits_{B}(x^{2}+y^{2}+z^{2})^{2}\,dV$, onde $B$ é a bola com centro na origem e raio $5.$


$\dfrac{312500\pi}{7}.$


2731   

Determine as equações do plano tangente e da reta normal ao gráfico da função dada, no ponto dado. $f(x,y) = 3x^3y - xy$ em $(1,-1,f(1,-1))$.


Plano tangente: $z = -8x + 2y + 8$

Reta normal: $(x,y,z) = \left(1,-1,-2 \right) + \lambda \left(-8,2,-1 \right)$.


2511   

Ache $\displaystyle\iint \limits_{ S}{\bf F}\cdot {\bf n} dS$ se ${\bf n}$ é uma normal unitária superior de $S.$


${\bf F}=2{\bf i}+5{\bf j}+3{\bf k}$; $S$ é a parte do cone $z=(x^{2}+y^{2})^{1/2}$ interior ao cilindro $x^{2}+y^{2}=1.$


2426   

Calcule a integral tripla.

  1.  $\displaystyle\iiint\limits_{  E}2x\,dV$, onde $E=\{(x,y,z)|\;0\leq y\leq 2,\,0\leq x\leq \sqrt{4-y^{2}},\;\\ 0\leq z\leq y\}.$

  2.  $\displaystyle\iiint\limits_{  E}6xy\,dV$, onde $E$ está abaixo do plano $z=1+x+y$ e acima da região do plano $xy$ limitada pelas curvas $y=\sqrt{x}$, $y=0$ e $x=1.$


  1.  $4.$

  2.  $\dfrac{65}{28}.$


2791   

Determine os valores máximos e mínimos locais e pontos de sela da função $f(x,y)=e^{4y-x^{2}-y^{2}}$.


Ponto de máximo: $\displaystyle \left(0,2 \right).$


2782   

Calcule a integral dupla usando coordenadas polares: $\displaystyle\iint\limits_{R}(x^{2}+y^{2})^{3/2}\,dA$, onde $R$ é limitado pelo círculo $x^{2}+y^{2}=4.$


$\displaystyle \frac{64\pi}{5}.$


2635   

A temperatura $T$ de uma localidade do Hemisfério Norte depende da longitude $x$, da latitude $y$ e do tempo $t$, de modo que podemos escrever $T=f(x,y,t)$. Vamos medir o tempo em horas a partir do início de Janeiro.

  1. Qual é o significado das derivadas parciais $\partial T/\partial x$, $\partial T/\partial y$ e $\partial T/\partial t$?

  2. Honolulu (você sabe onde fica?) tem longitude de $158^{\circ}W$ e latitude de $21^{\circ}N$. Suponha que às 9 horas em $1^{\circ}$ de Janeiro esteja ventando para nordeste uma brisa quente, de forma que a oeste e a sul o ar esteja quente e a norte e leste o ar esteja mais frio. Você esperaria que $f_{x}(158,21,9)$, $f_{y}(158,21,9)$ e $f_{t}(128,21,9)$ fossem positivas ou negativas? Explique.


  1. $\partial T/\partial x$ é a taxa de variação da temperatura quando a longitude muda, mas a latitude e o tempo são constantes;
    $\partial T/\partial y$ é a taxa de variação da temperatura quando a latitude muda, mas a longitude e o tempo são constantes;
    $\partial T/\partial t$ é a taxa de variação da temperatura quando o tempo muda, mas a longitude e a latitude são constantes.
  2. $f_{x}(158,21,9) > 0,$ $f_{y}(158,21,9) < 0$ e $f_{t}(158,21,9) > 0.$


2959   

Usando coordenadas esféricas, determine o volume do sólido que está acima do cone $\phi=\pi/3$ e abaixo da esfera $\rho=4\cos{\phi}.$


$10\pi.$


2936   

Uma carga elétrica é distribuída sobre o retângulo $1 \leq x \leq 3$, $0 \leq y \leq 2$, de modo que a densidade de carga em $(x,y)$ é $\sigma(x,y) = 2xy + y^2$ (medida em coulombs por metro quadrado). Determine a carga total no retângulo.


$\displaystyle \frac{64}{3}$ Coulombs.


2306   

Seja $f$ uma função de três variáveis independentes $x,y$ e $z$. Mostre que $D_{\bf{i}}f = f_x$, $D_{\bf{j}}f = f_y$ e $D_{\bf{k}}f = f_z$.


Lembre que $\bf{i} = (1,0,0),$ $\bf{j} = (0,1,0),$ $\bf{k} = (0,0,1)$ e $D_{\bf{u}}f = \nabla f \cdot \bf{u}.$


3118   

Use coordenadas esféricas para encontrar o volume do sólido: limitado acima pela esfera \(\rho=4\) e abaixo pelo cone \(\phi=\pi/3\).


\(\dfrac{64\pi}{3}\)


2678   

Seja $f(x,y,z) = \dfrac{x}{x^2 + y^2 + z^2}$.

Verifique que

$$x\dfrac{\partial f}{\partial x} + y\dfrac{\partial f}{\partial y} + z\dfrac{\partial f}{\partial z} = -f.$$


$\displaystyle \frac{\partial f}{\partial x} = \frac{-x^{2} + y^{2} + z^{2}}{(x^{2} + y^{2} + z^{2})^{2}},\;\;\;\; \frac{\partial f}{\partial y} = \frac{-2xy}{(x^{2} + y^{2} + z^{2})^{2}} \;\;\;\;\;\text{e}\;\;\;\;\;\frac{\partial f}{\partial z} = \frac{-2xz}{(x^{2} + y^{2} + z^{2})^{2}}.$


1987   

Determine as equações paramétricas para a reta tangente $\grave{a}$ curva dada pelas equações paramétricas 
$x=e^{-t}\; \cos{t}$, $y=e^{-t}\; \sin{t}$, $z=e^{-t}$ no ponto $(1,0,1)$.


2382   

Calcule a integral iterada.

  1. $\displaystyle\int_{0}^{\pi/2} \int_{0}^{\pi/2}\sin{x}\cos{y} \, dy dx$

  2. $\displaystyle\int_{0}^{2}\!\!\int_{0}^{1}(2x+y)^{8}\,dx dy$


  1. $1.$

  2. $\dfrac{4^{10} - 2^{11}}{180}.$


2603   

Determine o volume do sólido limitado pelo cilindro $x^2 + y^2 = 4$ e pelos planos $z = 0$ e $y + z = 3$.


$12\pi.$


2282   

Defina gradiente de uma função de três variáveis. Calcule $\nabla f(x,y,z)$.

$f(x,y,z) = z \arctan{\dfrac{x}{y}}$


$\displaystyle \nabla f(x,y,z) = \left(\frac{yz}{x^{2} + y^{2}},-\frac{xz}{x^{2} + y^{2}},\arctan\left(\frac{x}{y}\right) \right).$


3097   

Suponha que a temperatura, em graus Celsius, num ponto \((x,y)\)  de uma chapa metálica plana seja \( T(x,y)=10-8x^2-2y^2 \), onde \(x\) e \(y\) são medidos em metros. Calcule a temperatura média da porção retangular da chapa dada por \(0\leq x\leq 1\) e \(0\leq y\leq 2\).


\(\dfrac{14}{3}\) \({}^\circ\)C


2892   

Passe para coordenadas polares e calcule: $\displaystyle\int_{0}^{\ln 2}  \int_{0}^{\sqrt{(\ln 2)^{2}-y^{2}}}e^{\sqrt{x^{2}+y^{2}}}\,dx dy$


$\displaystyle \frac{\pi(2\ln(2) - 1)}{2}.$


2655   

Determine as derivadas parciais de $z=\cos(xy)$.


$\displaystyle \frac{\partial z}{\partial x} = -y\sin(xy)\;\;\;\;\;\;\text{e}\;\;\;\;\; \frac{\partial z}{\partial y} = -x\sin(xy).$


2516   

Encontre uma equação para a superfície de nível da função $f(x,y)=\ln (x^{2}+y^{2}+z^{2})$ que passa pelo ponto $(-1,2,1)$.


$x^{2} + y + z^{2} = 6.$


2930   

Calcule a integral em coordenadas esféricas. $\displaystyle\int_{0}^{\pi}\int_{0}^{\pi}\int_{0}^{2\,\sin{\phi}}\rho^{2}\sin{\phi}\,d\rho d\phi d\theta$.


$\pi^2.$


2419   

Encontre o volume do sólido no primeiro octante limitado pelo cilindro $z=16-x^{2}$ e pelo plano $y=5.$


 $\dfrac{640}{3}.$


2895   

Utilize coordenadas polares para determinar o volume do sólido dado: abaixo do cone $z=\sqrt{x^{2}+y^{2}}$ e acima do disco $x^{2}+y^{2}\leq 4.$


$\displaystyle \frac{16\pi}{3}.$


1924   

Calcule a integral de linha, onde $C$ é a curva dada. $\displaystyle\int_{C}x\,dx-y\,dy$, $C$ é o segmento de extremidades $(1,1)$ e $(2,3)$, percorrido no sentido de $(1,1)$ para $(2,3).$



Uma representação paramétrica para o segmento de reta $C$ é

$$\begin{array}{lr}x=1+t \\y=1+2t\\\end{array}\;\;\;\; 0\leq t \leq 1.$$

Logo,

$$\begin{array}{lr}dx=dt \\dy=2\,dt\\\end{array}$$


Assim,

$$\int_{C}x\,dx-y\,dy=\int_{0}^{1}(1+t)\cdot (dt)+(1+2t)\cdot(2\,dt)=\int_{0}^{1}(1+t+2+4t)\,dt$$

$$=\int_{0}^{1}(3+5t)\,dt=\bigg(3t+\frac{5}{2}t^{2}\bigg)\bigg|_{0}^{1}=3+\frac{5}{2}=\frac{11}{2}.$$


2353   

Calcule o volume do conjunto dado.

  1.  $x^{2}+4y^{2}\leq 4$ e $x+y\leq z\leq x+y+1.$

  2.  $x\geq 0$, $x\leq y\leq 1$ e $0\leq z\leq e^{y^{2}}.$


  1.  $2\pi.$

  2.  $\dfrac{e - 1}{2}.$


2033   

Utilize a Regra da Cadeia para determinar $\mathrm{\partial}z/\mathrm{\partial} s$ e $\mathrm{\partial}z/ \mathrm{\partial}t.$

$z=e^{x+2y}$, $x=s/t$, $y=t/s$.


$\displaystyle \frac{\partial z}{\partial s} = e^{x + st}\left(\frac{1}{t} - \frac{2t}{s^{2}} \right) $ e $\displaystyle \frac{\partial z}{\partial t} = e^{x + st}\left(\frac{2}{s} - \frac{s}{t^{2}} \right) $.


2551   

Se $f(x_0,y_0) = 3$, o que podemos dizer sobre

$$\displaystyle \lim_{(x,y) \to (x_0,y_0)}f(x,y)$$

se $f$ for contínua em $(x_0,y_0)$? E se $f$ não for contínua em $(x_0,y_0)$? Justifique sua resposta.


Se $f$ for contínua em $(x_{0},y_{0}),$ então o limite é igual a $f(x_{0},y_{0}) = 3.$ Se não for contínua em $(x_{0},y_{0}),$ então o limite pode ter qualquer valor diferente de $3.$


3144   

Encontre o trabalho realizado pelo campo de forças \[ \mathbf{F}(x,y)= y^2\mathbf{i} + xy\mathbf{j} \] para mover uma partícula de \((0,0)\) até \((1,1)\) ao longo da parábola \(y=x^2\).


3030   

Esboce a região de integração e mude a ordem de integração. $\displaystyle\int_{0}^{1}\!\!\int_{\arctan{x}}^{\pi/4}\!f(x,y)\,dy dx$.


A expressão 1 cossec x e o mesmo que


1988   

As curvas ${\bf r}_{1}(t)=(t,t^{2},t^{3})$ e ${\bf r}_{2}(t)=(\sin{t},\sin{2t},t)$ se interceptam na origem. Determine o ângulo 
de intersecção destas com precisão de um grau.


2281   

Defina gradiente de uma função de três variáveis. Calcule $\nabla f(x,y,z)$.

$f(x,y,z) = (x^2 + y^2 + 1)^{z^2}$


$\displaystyle \nabla f(x,y,z) = (x^{2} + y^{2} + 1)^{z^{2}-1}\left(2xz^{2},2yz^{2},2z(x^{2} + y^{2} + 1)\ln(x^{2} + y^{2} + 1)\right).$


2398   

Determine uma reta que seja tangente à curva $x^2 + xy + y^2 = 7$ e paralela à reta $4x + 5y = 17$.


 $\displaystyle y - 2 = -\frac{4}{5} (x - 1)$ ou $\displaystyle y + 2 = -\frac{4}{5} (x + 1).$


2926   

Identifique a superfície cuja equação é $\rho=\sin{\theta}\sin{\phi}.$


Esfera de raio $\dfrac{1}{2}$ centrada no ponto $\left(0,\dfrac{1}{2},0\right).$


2273   

No item abaixo :

  1.  determine o gradiente de $f$; 
  2.  calcule o gradiente no ponto $P$; e 
  3.  determine a taxa de variação de $f$ em $P$ na direção do vetor $\bf{u}$.

$f(x,y,z) = xe^{2yz},  P = (1,-3),  \bf{u} = \left( \frac{2}{3}, -\frac{2}{3}, \frac{1}{3} \right)$.


  1. $\nabla f(x,y,z) = (e^{yz}, 2xze^{2yz}, 2xye^{2yz}).$
  2. $\nabla f(3,0,2) = (1,12,0).$
  3. $\displaystyle -\frac{22}{3}.$


1946   

Calcule a integral de linha, onde $C$ é a curva dada.

$\displaystyle\int_{C}2\,dx-dy$, $C$ tem por imagem $x^{2}+y^{2}=4$, $x\geq 0$ e $y\geq 0$; sentido de percurso é de $(2,0)$ para $(0,2).$


$\displaystyle -6.$


2227   

Determine se o campo vetorial $\mathbf{F}(x,y,z) = ye^{-x}\mathbf{i} + e^{-x}\mathbf{j} + 2z\mathbf{k}$ é conservativo ou não. Se for conservativo, determine uma função $f$ tal que $\mathbf{F} = \nabla{f}$.


$\mathbf{F}$ não é conservativo.


2819   

Determine os valores máximo e mínimo absolutos de $f$ no conjunto $D.$

$f(x,y)=3x-y$ em $D=\{(x,y)\in \mathbb{R}^2: x^{2}+y^{2}\leq 1\}.$


Valor máximo: $\displaystyle \frac{8\sqrt{10}}{10};$ valor mínimo: $-\sqrt{10}.$


2008   

A força em um ponto $(x,y,z)$ em três dimensões é dada por ${\bf F}(x,y,z)=y\,{\bf i}+z\,{\bf j}+x\,{\bf k}$. Ache o trabalho realizado por ${\bf F}(x,y,z)$ ao longo da cúbica reversa $x=t$, $y=t^{2}$, $z=t^{3}$ de $(0,0,0)$ a $(2,4,8).$


$\dfrac{412}{15}.$


2793   

Determine os valores máximos e mínimos locais e pontos de sela da função $f(x,y)=x^{2}+y^{2}+x^{2}y+4$.


Pontos de mínimo: $(1,1)$ e $(-1,-1);$ ponto de sela: $(0,0).$


2056   

Determine se ${\bf F}(x,y)=(\ln y+2xy^{3})\,{\bf i}+(3x^{2}y^{2}+x/y)\,{\bf j}.$ é ou não um campo vetorial conservativo. Se for, determine uma função $f$ tal que ${\bf F}=\nabla f.$


Sim. $f(x,y) = x^{2}y + xy^{-2} + K.$


2579   

Determine o conjunto dos pontos de continuidade da função $f(x,y) = \sqrt{6 - 2x^2 - 3y^2}$. Justifique sua resposta.


$\left\lbrace (x,y);\; 2x^{2} + 3y^{2} \leq 6 \right\rbrace.$


2631   

Utilizando o Teorema de Stokes, transforme a integral $\displaystyle\iint_{ S}\mbox{rot}{\bf F}\cdot{\bf n}dS$ numa integral de linha e calcule.

  • ${\bf F}(x,y,z) = y{\bf i} + x{\bf j} + xz{\bf k}$, $S$ a superfície $z = x+y+2$ e $x^2 + \dfrac{y^2}{4} \leq 1$, sendo ${\bf n}$ a normal que aponta para baixo.


$4\pi$.


2864   

Estude com relação a máximos e mínimos a função dada com as restrições dadas.

$f(x,y) = xy$ e $x^2 + 4y^2 = 8.$


Pontos de máximo: $\displaystyle \left(2,1\right)$ e $(-2,-1)$; pontos de mínimo: $\displaystyle \left(-2,1\right)$ e $(2,-1)$.


2317   

Determine uma equação do plano tangente à superfície parametrizada dada no ponto especificado. $x=u^{2}$, $y=v^{2}$, $z=uv$; $u=1$, $v=1.$


$x + y - 2z = 0.$


2401   

 Determine um plano que seja tangente à superfície $x^2 + 3y^2 + 2z^2 = \dfrac{11}{6}$ e paralelo ao plano $x + y + z = 10$.


 $\displaystyle x + y + z = \frac{11}{6}$ ou $\displaystyle x + y + z = -\frac{11}{6}.$


2804   

Determine os valores máximos e mínimos locais e pontos de sela da função $f(x,y)=-x^{2}+y^{2}+2xy+4x-2y$.


Ponto de sela: $\displaystyle \left(\frac{3}{2},-\frac{1}{2}\right).$


2343   

Seja $A=\{(0,y,z)\in \mathbb{R}^{3}| z^{2}+(y-2)^{2}=1\}$; ache a área da superfície gerada pela rotação em torno do eixo $Oz$ do conjunto $A.$


$8\pi^2.$


2595   

Identifique a superfície cuja equação é dada por $z = 4 - r^2$.


$z = 4 - x^2 - y^2,$ o parabolóide circular com vértice $(0,0,4)$.


3085   

Mostre que se \(f\) é diferenciável e \(z=xf(x/y)\), então todos os pontos planos tangentes ao gráfico dessa equação passam pela origem.


2845   

Passe para coordenadas polares e calcule: $\displaystyle\int_{0}^{6}  \int_{0}^{y}x\,dx dy$


$36.$


2822   

Determine os valores máximo e mínimo absolutos de $f$ no conjunto $D.$

$f(x,y)=y^{2}-x^{2}$ em $D=\{(x,y)\in \mathbb{R}^2: x^{2}+y^{2}\leq 4\}.$


Valor máximo: $\displaystyle  4;$ valor mínimo: $-4.$


2852   

Entre todos os pontos do gráfico de $z=10-x^{2}-y^{2}$ que estão acima do plano $x+2y+3z=0$, encontre o ponto mais afastado do plano.


$\displaystyle \left( \frac{1}{6}, \frac{1}{3}, \frac{355}{36} \right).$


2600   

Calcule as seguintes integrais triplas.

  1.  $\displaystyle\iiint\limits_{  E}  x^2 \, dV$, em que $E$ é o sólido que está dentro do cilindro $x^2 + y^2 = 1$, acima do plano $z = 0$ e abaixo do cone $z^2 = 4x^2 + 4y^2$.

  2.  $\displaystyle\iiint\limits_{  E}   xyz \, dV,$ em que $E$ é o sólido limitado pelos paraboloides $z = x^2 + y^2$, $z = 8 - x^2 - y^2$.

  3.  $\displaystyle\int_{-2}^2\int_{-\sqrt{4 - y^2}}^{\sqrt{4 - y^2}}\int_{\sqrt{x^2 + y^2}}^2 xz \, dz dx dy$


  1.  $\dfrac{2\pi}{5}$.

  2.  $0.$

  3.  $0.$


2826   

Suponha que $T(x,y)=4-x^{2}-y^{2}$ represente uma distribuição de temperatura em uma região que pode ser aproximada por um plano. Seja $D=\{(x,y)\in \mathbb{R}^2: x\geq 0,\;y\geq x\;\text{e}\;2y+x\leq 4\}$. Determine o ponto de $D$ de menor temperatura.


$(0,2).$


2521   

Esboce o gráfico da função $f(x,y)=\sqrt{x^{2}+y^{2}}$. Em geral, se $g$ é uma função de uma variável, como saber o gráfico de $f(x,y)=g(\sqrt{x^{2}+y^{2}})$ a partir do gráfico de $g$?


O gráfico de $f(x,y) = g(\sqrt{x^{2} + y^{2}})$ pode ser obtido rotacionando o gráfico de $g$ no plano $xz$ ao redor do eixo $z.$

A expressão 1 cossec x e o mesmo que


2492   

Dada a função $f(x,y)=\sqrt{y-x}$.

  1. Encontre o domínio da função.

  2. Encontre a imagem da função.

  3. Descreva as curvas de nível da função.


  1. $D_{f} = \left\lbrace (x,y);\; x \leq y \right\rbrace$.

  2. $Im(f) = \left\lbrace z \in \mathbb{R};\; z \geq 0 \right\rbrace.$

  3. As curvas de nível são as retas $y - x = C,$ com $C \geq 0.$


2217   

Determine o rotacional e o divergente do campo vetorial $\mathbf{F}(x,y,z) = e^{xy}\sin{z}\mathbf{j} + y\tan^{-1}(x/z)\mathbf{k}$.


$\text{rot } \mathbf{F} = (\arctan(x/z) - e^{xy}\cos(z))\mathbf{i} - \dfrac{yz}{x^{2} + z^{2}} \mathbf{j} + ye^{xy}\sin(z) \mathbf{k}.$ $\text{div } \mathbf{F} = xe^{xy}\sin(z) - \dfrac{xy}{x^{2} + z^{2}}.$


3021   

Esboce a região de integração e calcule a integral $\displaystyle\int_{0}^{\pi}\!\!\int_{0}^{x}x\sin{y}\,dy dx$.


$\dfrac{\pi^{2}}{2} + 2.$

A expressão 1 cossec x e o mesmo que


2771   

Uma carga elétrica é distribuída sobre um disco $x^2 + y^2 \leq 4$ de modo que a densidade de carga em $(x,y)$ é $\sigma(x,y) = x + y + x^2 + y^2$ (medida em coulombs por metro quadrado). Determine a carga total do disco.



Como a carga elétrica é distribuída sobre o disco $x^2 + y^2 \leq 4$, em coordenadas polares temos que $0\leq r \leq 2$ e $0\leq \theta \leq 2\pi.$ Temos que $$Q=\iint\limits_{D}\sigma(x,y)\,dA=\iint\limits_{D}(x+y+x^{2}+y^{2})\,dA$$ $$=\int_{0}^{2\pi}\int_{0}^{2}(r\,\cos \theta+r\,\sin \theta+r^{2})r\,dr\, d \theta=\int_{0}^{2\pi}\int_{0}^{2}(r^{2}\,\cos \theta+r^{2}\,\sin \theta+r^{3})\,dr\, d \theta$$ $$=\int_{0}^{2\pi}\bigg(\frac{r^{3}}{3}\cos \theta+\frac{r^{3}}{3}\sin \theta +\frac{r^{4}}{4}\bigg)\bigg|_{0}^{2}\,d\theta= \int_{0}^{2\pi}\bigg(\frac{8}{3}\cos \theta+\frac{8}{3}\sin \theta+4\bigg)\,d\theta$$ $$=\bigg(\frac{8}{3}\sin\theta-\frac{8}{3}\cos\theta+4\theta\bigg)\bigg|_{0}^{2\pi}=\bigg(-\frac{8}{3}+8\pi\bigg)-\bigg(-\frac{8}{3}\bigg)$$ $$=-\frac{8}{3}+8\pi+\frac{8}{3}=8\pi.$$


3108   

Encontre a área da superfície descrita como sendo a parte do cone \(z^2=4x^2+4y^2\) que está acima da região do primeiro quadrante limitada pela reta \(y=x\) e a parábola \(y=x^2\).


\( \dfrac{\sqrt{5}}{6}\)


2686   

Encontre $f_{x}$, $f_{y}$ e $f_{z}$ para $f(x,y,z)=1+xy^{2}-2z^{2}$.


$\displaystyle f_{x} = 1+ y^{2} ,\;\;\;\; f_{y} = 2xy \;\;\;\;\text{e}\;\;\;\; f_{z} = -4z$.


2122   

Expresse $\partial z/\partial t$ em termos das derivadas parciais de $f$, sendo $z=f(x,y)$ e $x=\sin{3t}$ e $y=\cos{2t}.$


 $\displaystyle \frac{dz}{dt} (t) = 3 \cos(3t) \frac{\partial f}{\partial x}(\sin(3t),\cos(2t)) - 2\sin(2t) \frac{\partial f}{\partial y}(\sin(3t),\cos(2t)).$


2986   

Considere a transformação do plano $xy$ no plano $uv$ dada por $u=x-2y$ e $v=3x-y$.

  1. Inverta a transformação, isto é, obtenha as expressões da transformação do plano $uv$ no plano $xy$.

  2. Represente geometricamente a região $R$ no plano $xy$ obtida como imagem da transformação aplicada à região delimitada por $u=0$, $u=4$, $v=1$, $v=8$.

  3. Utilize a transformação dada para calcular a integral

    $$\iint\limits_{R}\dfrac{x-2y}{3x-y} \, dA.$$


  1. $x = \dfrac{2v - u}{5},$ $y = \dfrac{v - 3u}{5}.$

  2. Região delimitada pelas retas $x = 2y,$ $x = 2y + 4,$ $y = 3x - 1$ e $3x - 8.$

  3. $\dfrac{8 \ln(8)}{5}.$


2348   

Calcule a área da parte da superfície cilíndrica $z^{2}+x^{2}=4$ que se encontra dentro do cilindro $x^{2}+y^{2}\leq 4$ e acima do plano $xy.$


$16.$


3052   

Esboce o sólido descrito pelas desigualdades $0 \leq r \leq 2$, $-\pi/2 \leq \theta \leq \pi/2$ e $0 \leq z \leq 1$.


A expressão 1 cossec x e o mesmo que


2567   

Sabendo que $\left|\sin\frac{1}{x}\right| \leq 1$, podemos dizer algo sobre

$$\displaystyle \lim_{(x,y) \to (0,0)}y\sin\dfrac{1}{x}?$$

Justifique sua resposta.


$\displaystyle \lim_{(x,y) \to (0,0)} y\sin\left(\frac{1}{x} \right) = 0.$


2577   

Determine se a função

$$f(x,y) = \begin{cases}e^{\left( \dfrac{1}{x^2 + y^2 - 1} \right)}, & \quad \text{se } x^2 + y^2 < 1, \\0, & \quad \text{se } x^2 + y^2 \geq 1.\end{cases}$$

é contínua em $\displaystyle{\left( \frac{\sqrt{2}}{2}, \frac{\sqrt{2}}{2} \right)}$. Justifique sua resposta.


$0.$


1951   

Calcule a integral de linha $\int_{C}{\bf F}\cdot d{\bf r}$, onde $C$ é dada pela função vetorial ${\bf r}(t).$

${\bf F}(x,y,z)=\sin{x}\,{\bf i}+\cos{y}\,{\bf j}+xz\,{\bf k}$, ${\bf r}(t)=t^{3}\,{\bf i}-t^{2}\,{\bf j}+t\,{\bf k}$, $0\leq t\leq 1.$


$\dfrac{6}{5} - \cos(1) - \sin(1).$


3056   

 Faça o esboço do sólido cujo volume é dado pela integral e calcule essa integral.

$\displaystyle \int_0^{\pi/2}\int_0^2\!\!\int_0^{9 - r^2} r dz dr d\theta$


A expressão 1 cossec x e o mesmo que


2775   

Use multiplicadores de Lagrange para demonstrar que o triângulo com área máxima, e que tem um perímetro constante $p$, é equilátero.

(Sugestão: Utilize a fórmula de Heron para a área:

$$A = \sqrt{s(s-x)(s-y)(s-z)},$$

em que $s = p/2$ e $x,y$ e $z$ são os comprimentos dos lados.)



Utilizando a fórmula de Heron temos que a área e um triânulo é

$$A=\sqrt{s(s-x)(s-y)(s-z)},$$

com $s=p/2$ e $x,\,y,\,z$ lados do triângulo.

Mas a álgebra fica mais simples se maximizarmos o quadrado da área, isto é,

$$A^{2}=f(x,y,z)=s(s-x)(s-y)(s-z).$$

A restrição é que o triângulo têm perímetro constante $p$, ou seja,

$$g(x,y,z)=x+y+z=p.$$

De acordo com o método dos multiplicadores de Lagrange, resolvemos $\nabla f=\lambda \nabla g$ e $g=p.$ Então

$$\nabla f(x,y,z)=(\,-s(s-y)(s-z),\, -s(s-x)(s-z),\,-s(s-x)(s-y)\,)$$

e

$$\lambda \nabla g(x,y,z)=\lambda (1,1,1)=(\lambda, \lambda, \lambda).$$

Logo temos as seguintes equações

\begin{array}{rcl}-s(s-y)(s-z)&=&\lambda\\-s(s-x)(s-z)&=&\lambda\\-s(s-x)(s-y)&=&\lambda\\x+y+z&=&p\end{array}

Assim, das três primeiras equações, temos que

$$-s(s-y)(s-z)=-s(s-x)(s-z)=-s(s-x)(s-y).$$

Da primeira igualdade obtemos que $s-y=s-x\Rightarrow y=x$ e da segunda igualdade obtemos que $s-z=s-y\Rightarrow z=y$, resultando que $x=y=z.$

Portanto, o triângulo com área máxima e perímetro constante $p$ é um triângulo equilátero.


3059   

Esboce o campo vetorial $\textbf{F}=\dfrac{1}{2}(\textbf{i} + \textbf{j})$, desenhando um diagrama.


2335   

Mostre que uma função diferenciável $f$ decresce mais rapidamente em $\bf{x}$ na direção oposta à do vetor gradiente, ou seja, na direção de $-\nabla f(\bf{x})$.


Se $\bf{u}$ é um versor e $\theta$ é o ângulo entre $\nabla f$ e $\bf{u},$ então
$$
D_{\bf{u} f} = \nabla f \cdot \bf{u} = |\nabla f||\bf u| \cos(\theta) = |\nabla f|\cos(\theta).
$$
O valor mínimo de $\cos(\theta)$ é $-1$ e isto ocorre quando $\theta = \pi.$ Portanto o valor mínimo de $D_{\bf{u}} f$ é $-|\nabla f|$ e ocorre quando $\theta = \pi,$ ou seja, quando $\bf{u}$ tem a direção oposta à de $\nabla f.$


2300   

Identifique e faça um esboço da imagem da superfície parametrizada dada por ${\bf r}(u,v)=(1,u,v)$, $0\leq u\leq 1$, $0\leq v \leq 1.$


Região quadrada do plano $x = 1:$ $0 \leq y \leq 1$ e $0 \leq z \leq 1.$


2014   

Seja ${\bf E}(x,y)=\dfrac{1}{x^{2}+y^{2}}\dfrac{x\,{\bf i}+y\,{\bf j}}{\sqrt{x^{2}+y^{2}}}$ e seja $C$ a curva dada por $x=t$ e $y=1-t^{4}$, $-1\leq t\leq 1.$

  1. Que valor é razoável esperar para $\int_{C}{\bf E}\cdot d{\bf l}$? Por quê? (O ${\bf l}$ desempenha aqui o mesmo papel que ${\bf r}:{\bf l}(t)={\bf r}(t).$)

  2. Calcule $\int_{C}{\bf E}\cdot d{\bf l}.$


$0.$


2980   

Determine a imagem do conjunto $S$ sob a transformação dada. $S$ é a região triangular com vértices $(0,0), (1,1), (0,1)$;$x = u^2$, $y = v$.


A região limitada pela reta $y = 1,$ pelo eixo $y$ e por $y = \sqrt{x}.$


2462   

Calcule a integral de superfície $\displaystyle\iint \limits_{ S}{\bf F}\cdot d{\bf S}$ para o campo vetorial ${\bf F}$ e superfície orientada $S$ dados abaixo. Em outras palavras, determine o fluxo de ${\bf F}$ através de $S$. Para superfícies fechadas, use a orientação positiva (para fora).

  • ${\bf F}(x,y,z)=xze^{y}{\bf i}-xze^{y}{\bf j}+z{\bf k}$ e $S$ é a parte do plano $x+y+z=1$ no primeiro octante, com orientação para baixo.


$-\dfrac{1}{6}.$


2358   

Calcule a área da parte da superfície esférica $x^{2}+y^{2}+z^{2}=1$ que se encontra dentro do cone $z\geq \sqrt{x^{2}+y^{2}}.$


$\pi(2 - \sqrt{2}).$


2383   

Calcule a integral iterada.

  1. $\displaystyle\int_{1}^{4} \int_{1}^{2}\bigg(\dfrac{x}{y}+\dfrac{y}{x}\bigg)\,dy dx$

  2. $\displaystyle\int_{0}^{1} \int_{0}^{3}e^{x+3y}\,dx dy$


  1. $\dfrac{21}{2} \ln(2).$

  2. $\dfrac{(e^{3} - 1)^{2}}{3}.$


2616   

Verifique que o Teorema de Stokes é verdadeiro para o campo vetorial ${\bf F}$ dado e a superfície $S$.

  • ${\bf F}(x,y,z) = y{\bf i} + z{\bf j} + x{\bf k}$, $S$ é o hemisfério $x^2+y^2+z^2=1$, $y \geq 0$, orientado na direção positiva do eixo $y$.


$\displaystyle\int_{C} {\bf F} \cdot d{\bf R} = \displaystyle\iint_{S} \mbox{rot} {\bf F} \cdot d{\bf S} = -\pi$.


2795   

Determine os valores máximos e mínimos locais e pontos de sela da função $f(x,y)=xy-2x-y$.


Ponto de mínimo: $(2,1);$ ponto de sela: $(0,0).$


2798   

Determine os valores máximos e mínimos locais e pontos de sela da função $f(x,y)=x^{4}+xy+y^{2}-6x-5y$.


Ponto de mínimo: $\displaystyle \left( 1,2\right).$


2726   

Utilize as diferenciais para estimar a quantidade de estanho em uma lata cilíndrica fechada com $8$ cm de diâmetro e $12$ cm de altura se a espessura da folha de estanho for de $0,04$ cm.


Para $V = \pi r^{2}h$ o volume da lata de raio $r$ e altura $h,$ temos $\Delta V \approx 16$ cm$^{3}.$


2934   

Uma piscina circular tem diâmetro de 10 metros. A profundidade é constante ao longo das retas de leste a oeste e cresce linearmente de 1 metro na extremidade sul para dois metros na extremidade norte. Encontre o volume de água da piscina.


$1800 \pi$ m$^3.$


2116   

Se $z=f(x,y)$, onde $x=r^{2}+s^{2}$ e $y=2rs$, determine $\partial^{2}z/\partial r\partial s.$ 



$\displaystyle \frac{\partial^{2}z}{\partial r\partial s} = 4rs \frac{\partial^{2}z}{\partial x^{2}} + 4rs \frac{\partial^{2}z}{\partial y^{2}} + (4r^{2} + 4s^{2}) \frac{\partial^{2}z}{\partial x\partial y} +  2 \frac{\partial z}{\partial y}.$ 


3001   

A figura mostra o mapa de contorno de $f$ no quadrado $R = [0,4] \times [0,4]$.

  1. Use a Regra do Ponto Médio com $m = n = 2$ para estimar o valor de $\int\!\!\!\int \limits_{\!\!\!\!\!R} \! f(x,y) \, dA$.

  2. Estime o valor médio de $f$.

A expressão 1 cossec x e o mesmo que


3023   

Esboce a região de integração e calcule a integral $\displaystyle\int_{1}^{\ln 8}\!\!\!\int_{0}^{\ln y}e^{x+y}\,dx dy$.


$8 \ln(8) - 16 + e.$

A expressão 1 cossec x e o mesmo que


2420   

Use a integral tripla para determinar o volume do sólido dado por $x^{2}+y^{2}\leq z\leq \sqrt{4-3x^{2}-3y^{2}}.$



Primeiramente, vamos determinar a projeção no plano $xy$ da interseção de \begin{eqnarray*} z&=&\sqrt{4-3x^{2}-3y^{2}}\\ z&=&x^{2}+y^{2}. \end{eqnarray*} Da primeira equação temos que \begin{eqnarray*} \label{1}z=\sqrt{4-3x^{2}-3y^{2}}\Leftrightarrow z^{2}=4-3x^{2}-3y^{2}\Leftrightarrow z^{2}=4-3(x^{2}+y^{2}). \end{eqnarray*} Substituindo a segunda equação  na primeira, obtemos que $$z^{2}=4-z\Leftrightarrow z^{2}+3z-4=0\Leftrightarrow (z-1)(z-4)=0.$$ Logo, $z=-4$ e $z=1.$ Notemos que $z=-4$ não satisfaz as duas primeiras equações acima, então a projeção $D$ no plano $xy$ é o círculo de raio 1, isto é, $D=\{(x,y)\in \mathbb{R};\;\, x^{2}+y^{2}\leq 1\}.$ Assim, o volume, $V$, do sólido é: $$V=\iint\limits_{D}\bigg[\int_{x^{2}+y^{2}}^{\sqrt{4-3x^{2}-3y^{2}}}1\, dz\bigg]\,dA = \iint\limits_{ D}\sqrt{4-3x^{2}-3y^{2}}-(x^{2}+y^{2})\,dA.$$ Passando para coordenadas polares temos que \begin{eqnarray*}  x=r\cos \theta\\ y=r\sin \theta\\ dA=r\,dr\,d\theta\\ 0\leq r\leq 1\\ 0\leq \theta \leq 2\pi.\\ \end{eqnarray*} Então, $$V=\int_{0}^{2\pi}\int_{0}^{1}(\sqrt{4-3r^{2}}-r^{2})r\,dr\,d \theta=\int_{0}^{2\pi}\int_{0}^{1}(r\sqrt{4-3r^{2}}-r^{3})\,dr\,d\theta$$ $$=\int_{0}^{2\pi}\,d\theta\cdot \bigg[\bigg(\underbrace{\int_{0}^{1}r\sqrt{4-3r^{2}}\,dr}_{\substack{ u=4-3r^{2}\\ du=-6r\,dr}}\bigg)-\bigg(\int_{0}^{1}r^{3}\,dr\bigg)\bigg]$$ $$=\theta\bigg|_{0}^{2\pi}\cdot \bigg[\bigg(\int_{4}^{1}r\cdot u^{1/2}\frac{du}{-6r}\bigg)-\bigg(\frac{r^{4}}{4}\bigg|_{0}^{1}\bigg)\bigg]$$ $$=2\pi\cdot \bigg[\bigg(-\frac{1}{6}\int_{4}^{1}u^{1/2}\,du\bigg)-\frac{1}{4}\bigg]=2\pi \cdot \bigg[\bigg(-\frac{1}{6}\cdot \frac{2}{3}u^{3/2}\bigg|_{4}^{1}\bigg)-\frac{1}{4}\bigg]$$ $$=2\pi \cdot \bigg[-\frac{1}{9}+\frac{1}{9}\cdot 8-\frac{1}{4}\bigg]=2\pi \cdot \frac{19}{36}=\frac{19\pi}{18}.$$


2271   

No item abaixo :

  1.  determine o gradiente de $f$; 
  2.  calcule o gradiente no ponto $P$; e 
  3.  determine a taxa de variação de $f$ em $P$ na direção do vetor $\bf{u}$.

$f(x,y) = 5xy^2 - 4x^3y,  P = (1,2),  \bf{u} = \left( \frac{5}{13},\frac{12}{13} \right)$.


  1. $\nabla f(x,y) = (5y^{2} - 12x^{2}y, 10xy - 4x^{3}).$
  2. $\nabla f(1,2) = (-4, 16).$
  3. $\displaystyle \frac{172}{13}.$


2908   

Utilize coordenadas polares para determinar o volume do sólido dado: delimitado pelo cone $z^2=x^2+y^2$ e pelo cilindro $x^2+y^2=2x.$


$\dfrac{8}{9}.$


2065   

Dados ${\bf F}(x,y,z)=yz\,{\bf i}+xz\,{\bf j}+(xy+2z)\,{\bf k}$, $C$ é o segmento de reta de $(1,0,-2)$ a $(4,6,3).$

  1. Determine uma função $f$ tal que ${\bf F}=\nabla f$.

  2. Use o resultado anterior para calcular $\int_{C}{\bf F}\cdot d{\bf r}$ sobre a curva $C$ dada.


  1. $f(x,y,z) = xyz + z^{2};$

  2. $77.$


2749   

Explique por que a função é diferenciável no ponto dado. $f(x,y) = e^{-xy} \cos{y}, \quad (\pi,0)$.


As derivadas $f_{x}$ e $f_{y}$ de cada $f$ existem e são contínuas no ponto dado, logo $f$ é diferenciável.


2836   

Mostre que $(0,0)$ é um ponto crítico de $f(x,y)=x^{2}+kxy+y^{2}$, não importando o valor da constante $k$.


Note que $f_{x} (0,0) = f_{y} (0,0) = 0.$


1925   

Calcule a integral de linha, onde $C$ é a curva dada. $\displaystyle\int_{C}x^{2}y\sqrt{z}\,dz$, $C:\,x=t^{3},\, y=t,\, z=t^{2},\, 0\leq t\leq 1.$



As equações paramétricas de $C$ são

$$x=t^{3},\, y=t,\, z=t^{2},\, 0\leq t\leq 1.$$

Logo,

$$dx=3t^{2}\,dt,\, dy=dt,\, dz=2t\,dt.$$


Assim,

$$\int_{C}x^{2}y\sqrt{z}\,dz=\int_{0}^{1}((t^{3})^{2}\cdot t \cdot \sqrt{t^{2}})(2t\,dt)=2\int_{0}^{1}t^{9}\,dt$$

$$=2\cdot\frac{t^{10}}{10}\bigg|_{0}^{1}=2\cdot \frac{1}{10}=\frac{1}{5}.$$


2641   

Determine as derivadas parciais de primeira ordem da função $u=te^{w/t}$.


$\displaystyle \frac{\partial u}{\partial t} = e^{w/t} \left( 1 - \frac{w}{t} \right)\;\;\;\text{e}\;\;\; \frac{\partial u}{\partial w} = e^{w/t}$.


2651   

A lei dos gases para uma massa fixa $m$ de um gás ideal à temperatura absoluta $T$, pressão $P$ e o volume $V$ é $PV=mRT$, onde $R$ é a constante do gás. Mostre que

$$\frac{\mathrm{\partial}P}{\mathrm{\partial}V}\frac{\mathrm{\partial}V}{\mathrm{\partial}T}\frac{\mathrm{\partial}T}{\mathrm{\partial}P}=-1.$$


$\displaystyle \frac{\partial P}{\partial V} = -\frac{mRT}{V^{2}},\;\;\;\frac{\partial V}{\partial T} = \frac{mR}{P}\;\;\;\text{e}\;\;\; \frac{\partial T}{\partial P} = \frac{V}{mR}.$


2089   

Calcule $\int_{C}2x\,\cos y\,dx-x^{2}\,\sin y\,dy$ ao longo dos caminhos $C$ a seguir no plano $xy.$

  1. A parabóla $y=(x-1)^{2}$ de $(1,0)$ a $(0,1).$

  2. O segmento de reta de $(-1,\pi)$ a $(1,0).$

  3. O eixo $x$ de $(-1,0)$ a $(1,0).$

  4. O astróide ${\bf r}(t)=(\cos^{3} t)\,{\bf i}+(\sin^{3}t)\,{\bf j}$, $0\leq t\leq 2\pi$, no sentido anti-horário de $(1,0)$ de volta a $(1,0).$


  1. $-1.$

  2. $2.$

  3. $0.$

  4. $0.$


2772   

Determine os valores máximos e mínimos locais e pontos de sela da função $f(x,y)=x^{3}-12xy+8y^{3}$.



Sendo $f(x,y)=x^{3}-12xy+8y^{3}$, vamos inicialmente localizar seus pontos críticos:

$$f_{x}(x,y)=3x^{2}-12y \;\;\;\;\;\;\;\; \mbox{e} \;\;\;\;\;\;\;\; f_{y}(x,y)=-12x+24y^{2}.$$

Igualando essas derivadas parciais a zero, obtemos as equações

$$x^{2}-4y=0 \;\;\;\;\;\;\;\; \mbox{e} \;\;\;\;\;\;\;\; 2y^{2}-x=0.$$

Para resolvê-las, substituímos $x=2y^{2}$ da segunda equação na primeira. Isso resulta em

$$0=y^{4}-y=y(y^{3}-1)$$

e existem duas raízes reais $y=0$ e $y=1.$ Os dois pontos críticos de $f$ são $(0,0)$ e $(2,1).$\\

Agora vamos calcular as segundas derivadas parciais e $D(x,y)$:

$$f_{xx}(x,y)=6x\,\,\,\, f_{xy}(x,y)=-12 \;\;\;\; f_{yy}(x,y)=48y$$

$$\begin{split}D(x,y)&=f_{xx}(x,y)\cdot f_{yy}(x,y)-(f_{xy}(x,y))^{2}\\&=(6x)\cdot (48y)-(-12)^{2}=288xy-144.\end{split}$$

Como $D(0,0)=-144<0$, segue do Teste da Derivada Segunda que $(0,0)$ é um ponto de sela, ou seja, $f$ não tem nem máximo local nem mínimo local em $(0,0).$ Como $D(2,1)=432>0$ e $f_{xx}(2,1)=12>0$, vemos do Teste da Derivada Segunda que $f(2,1)=-8$ é um mínimo local.


2885   

Determine o ponto da parábola $y = x^2$ mais próximo de $(14,1)$.


$(2,4).$


2011   

Um homem pesando $160$ lb carrega uma lata de tinta de $25$ lb por uma escada helicoidal em torno de um silo com raio de $20$ pés. Se o silo tem $90$ pés de altura e o homem dá três voltas completas em torno do silo. Além disso, $9$ lb de tinta vazam da lata de modo contínuo e uniforme durante a subida do homem. Quanto trabalho é realizado?


$16245$ ft-lb.


3134   

Considere o campo vetorial \[\mathbf{F}(x,y,z)=(x-z)\mathbf{i}+(y-x)\mathbf{j}+(z-xy)\mathbf{k}. \]

  1.  Use o Teorema de Stokes para encontrar a circulação em torno do triângulo de vértices \(A=(1,0,0)\), \(B=(0,2,0)\) e \(C=(0,0,1)\), orientado no sentido anti-horário quando visto da origem para o primeiro octante.

  2.  Encontre a densidade de circulação de \(\mathbf{F}\) na origem na direção de \(\mathbf{k}\), ou seja, \(\displaystyle\mathrm{rot\,}\mathbf{F}(\mathbf{0})\cdot\mathbf{k}\).

  3.  Encontre o vetor unitário \(\mathbf{n}\) tal que a densidade de circulação de \(\mathbf{F}\) na origem seja máxima na direção de \(\mathbf{n}\).


  1.  \(\dfrac{3}{2}\)

  2.  \(-1\)

  3.  \(\displaystyle \mathbf{n}= -\dfrac{1}{\sqrt{2}}\mathbf{j} -\dfrac{1}{\sqrt{2}}\mathbf{k} \)


2068   

Utilize as Equações 

$\dfrac{\partial z}{\partial x}=-\dfrac{\dfrac{\partial F}{\partial x}}{\dfrac{\partial F}{\partial z}}$ e $\dfrac{\partial z}{\partial y}=-\dfrac{\dfrac{\partial F}{\partial y}}{\dfrac{\partial F}{\partial z}}$

para determinar $\partial z/\partial x$ e $\partial z/\partial y$.

$x^{2}+y^{2}+z^{2}=3xyz$


$\displaystyle \frac{dz}{dx} = \frac{3yz - 2x}{2z - 3xy}$ e $\displaystyle \frac{dz}{dy} = \frac{3xz - 2y}{2z - 3xy} .$


2859   

Considere a função

$$f(x,y)=-\frac{y^{2}}{2}+3x^{2}-2x^{3}.$$

  1. Determine e classifique os pontos críticos de $f.$

  2. Mostre que a curva de nível $f(x,y)=0$ com $x\geq 0$ é uma curva fechada, isto é, é a fronteira de uma região $R$ limitada do plano $xy.$ Calcule o valor máximo de $f$ nessa região $R$.


  1. Pontos críticos: $(0,0)$ e $(1,0).$ Ponto de máximo: $(1,0);$ ponto de sela: $(0,0).$

  2. $\max \{ f(s); s\in R \} =1$.


2580   

Determine o conjunto dos pontos de continuidade da função $f(x,y) = \mbox{ln} \ \dfrac{x - y}{x^2 + y^2}$. Justifique sua resposta.


$\left\lbrace (x,y);\; x > y \right\rbrace.$


2932   

Calcule a integral em coordenadas esféricas. $\displaystyle\int_{0}^{2\pi}\int_{0}^{\pi}\int_{0}^{(1-\cos{\phi})/2}\rho^{2}\sin{\phi}\,d\rho d\phi d\theta$.


$\dfrac{\pi}{3}.$


2464   

Calcule a integral de superfície $\displaystyle\iint \limits_{ S}{\bf F}\cdot d{\bf S}$ para o campo vetorial ${\bf F}$ e superfície orientada $S$ dados abaixo. Em outras palavras, determine o fluxo de ${\bf F}$ através de $S$. Para superfícies fechadas, use a orientação positiva (para fora).

  • ${\bf F}(x,y,z)=y{\bf j}-z{\bf k}$ e $S$ é formada pelo parabolóide $y=x^{2}+z^{2}$, $0 \leq y \leq 1$ e pelo círculo $x^{2}+z^{2} \leq 1$, $y=1.$


$0.$


2916   

Calcule utilizando coordenadas esféricas. $\displaystyle\iiint\limits_{B} z \,dxdydz$, onde $B$ é o conjunto $1\leq x^{2}+y^{2}+z^{2}\leq 4$ e $z\geq 0.$



Usando coordenadas esféricas, o sólido pode ser descrito por

$$B = \left\{(\rho, \theta, \phi): 1 \leq \rho \leq 2, 0 \leq \theta \leq 2\pi \mbox{ e } 0 \leq \phi \leq \frac{\pi}{2}\right\}.$$

Lembre que o Jacobiano dessa transformação é $\rho^2 \sin{\phi}$. Assim, obtemos

\begin{array}{rcl}\displaystyle\iiint\limits_{B} z \,dxdydz & = & \displaystyle\int_{0}^{2\pi}\int_{0}^{\frac{\pi}{2}}\int_{1}^{2}(\rho \cos{\phi})(\rho^2 \sin{\phi})\,d\rho d\phi d\theta \\  & = & \displaystyle\int_{0}^{2\pi}\int_{0}^{\frac{\pi}{2}}\left.\left(\frac{\rho^4}{4} \frac{\sin{2\phi}}{2}\right|_{\rho=1}^{\rho=2}\right)\, d\phi d\theta \\  & = & \displaystyle\int_{0}^{2\pi}\left.\left(\frac{(16-1)}{8} \frac{(-\cos{2\phi)}}{2}\right|_{\phi=0}^{\rho=\frac{\pi}{2}}\right)\, d\theta \\  & = & \left.-\frac{15}{16}(-1-1) \theta \right|_{\theta=0}^{\theta=2\pi} = \frac{15\pi}{4}.    \end{array}


2231   

Seja $\mathbf{r} = x\mathbf{i} + y\mathbf{j} + z\mathbf{k}$ e $r=|\mathbf{r}|$. Verifique a identidade $\nabla \cdot \mathbf{r} = 3$.


$\nabla \cdot \mathbf{r} = \left(\dfrac{\partial}{\partial x}, \dfrac{\partial}{\partial y}, \dfrac{\partial}{\partial z} \right) \cdot \left(x,y,z \right)$ (Note que: $r = \sqrt{x^{2} + y^{2} + z^{2}}.$)


2394   

Determine a equação da reta tangente à curva de nível dada, no ponto dado.

$e^{2x - y} + 2x + 2y = 4$, em $\left(\dfrac{1}{2},1\right)$.


 $y = -4x + 3.$


2449   

Calcule $\displaystyle\iint\limits_{S}g(x,y,z)dS,$ onde $g(x,y,z)=x+y$ e $S$ é parte do primeiro octante do plano $2x+3y+z=6.$


3087   

Suponha que a equação \(z=f(x,y)\) seja expressa na forma polar \(z=g(r,\theta)\) através da substituição \(x=r\cos\theta\) e \(y=r\sin\theta\).

  1.  Considere \(r\) e \(\theta\) como funções de \(x\) e \(y\) e use derivação implícita para mostrar que \[ \frac{\partial r}{\partial x} = \cos\theta \quad \text{e}\quad\frac{\partial\theta}{\partial x} =-\frac{\sin\theta}{r}.\]

  2.  Considere \(r\) e \(\theta\) como funções de \(x\) e \(y\) e use derivação implícita para mostrar que \[\dfrac{\partial r}{\partial y}=\sin\theta \quad \text{e}\quad \dfrac{\partial\theta}{\partial y}=\dfrac{\cos\theta}{r}.\]

  3.  Use os resultados anteriores para mostrar que \begin{align*} \dfrac{\partial z}{\partial x} & = \dfrac{\partial z}{\partial r}\cos\theta - \dfrac{1}{r}\dfrac{\partial z}{\partial\theta}\sin\theta \\ \dfrac{\partial z}{\partial y} & = \dfrac{\partial z}{\partial r}\sin\theta + \dfrac{1}{r}\dfrac{\partial z}{\partial\theta}\cos\theta\end{align*}

  4.  Use o resultado do item anterior para mostrar que \[ \left(\dfrac{\partial z}{\partial x}\right)^2 + \left(\dfrac{\partial z}{\partial y}\right)^2 = \left(\dfrac{\partial z}{\partial r}\right)^2 +\dfrac{1}{r^2}\left(\dfrac{\partial z}{\partial\theta}\right)^2. \]

  5.  Ainda usando o resultado do terceiro item, mostre que \(z=f(x,y)\) satisfaz a equação de Laplace \[ \dfrac{\partial^2z}{\partial x^2} + \dfrac{\partial^2z}{\partial y^2}= 0, \] se, e somente se, \(z=g(r,\theta)\) satisfaz a equação \[ \dfrac{\partial^2z}{\partial r^2} + \dfrac{1}{r^2}\dfrac{\partial^2z}{\partial\theta^2}+\dfrac{1}{r}\dfrac{\partial z}{\partial r} = 0. \] A última equação acima é chamada de forma polar da equação de Laplace.


2557   

Determine o limite, se existir, ou mostre que o limite não existe.

$\displaystyle \lim_{(x,y) \to (0,0)} \dfrac{xy}{\sqrt{x^2 + y^2}}$.


$0.$


2399   

Existem pontos no hiperboloide $x^2 - y^2 - z^2 = 1$ nos quais o plano tangente é paralelo ao plano $z = x + y$?


Não.


2718   

Explique por que a função é diferenciável no ponto dado. A seguir, encontre a linearização $L(x,y)$ da função naquele ponto. $f(x,y) = e^{-xy} \cos{y}, \quad (\pi,0)$.


As derivadas $f_{x}$ e $f_{y}$ de cada $f$ existem e são contínuas no ponto dado, logo $f$ é diferenciável.

$L(x,y) = 1 - \pi y$.


2470   

Calcule a integral de superfície $\displaystyle\iint \limits_{ S}{\bf F}\cdot d{\bf S}$ para o campo vetorial ${\bf F}$ e superfície orientada $S$ dados abaixo. Em outras palavras, determine o fluxo de ${\bf F}$ através de $S$. Para superfícies fechadas, use a orientação positiva (para fora).

  • ${\bf F}(x,y,z)=x{\bf i}-y{\bf j}+z{\bf k}$ e $S$ é a superfície do sólido delimitado pelos gráficos de $z=x^{2}+y^{2}$ e $z=4.$


$8\pi.$


2656   

Determine as derivadas parciais de $z=\dfrac{x^{3}+y^{2}}{x^{2}+y^{2}}$.


$\displaystyle \frac{\partial z}{\partial x} =  \frac{x^{4} + 3x^{2}y^{2} - 2xy^{2}}{(x^{2} + y^{2})^{2}}\;\;\;\;\;\;\text{e}\;\;\;\;\; \frac{\partial z}{\partial y} = \frac{2x^{2}y(1 - x)}{(x^{2} + y^{2})^{2}}.$


2046   

Determine se ${\bf F}(x,y)=(ye^{x}+\sin y)\,{\bf i}+(e^{x}+x\,\cos y)\,{\bf j}$ é ou não um campo vetorial conservativo. Se for, determine uma função $f$ tal que ${\bf F}=\nabla f.$



Primeiramente, temos que o domínio de ${\bf F}$ é todo o $\mathbb{R}^{2}$, o qual é uma região aberta e simplesmente conexa. Sendo $P(x,y)=ye^{x}+\sin y$ e $Q(x,y)=e^{x}+x\,\cos y$, temos que $P$ e $Q$ possuem derivadas de primeira ordem contínuas. Também temos que

$$\frac{\partial P}{\partial y}=e^{x}+\cos y    \,\,\,  \text{   e   }   \,\,\,   \frac{\partial Q}{\partial x}=e^{x}+\cos y,$$

ou seja,

$$\frac{\partial P}{\partial y}=\frac{\partial Q}{\partial x}.$$

Assim, das condições acima verificadas, temos que ${\bf F}$ é um campo conservativo. Agora, vamos determinar $f$ tal que $\nabla f={\bf F}.$ Isto é, devemos encontrar $f$ tal que

$$f_{x}(x,y)=P(x,y)      \text{   e   }      f_{y}(x,y)=Q(x,y).$$

Como $f_{x}(x,y)=P(x,y)$ temos que

$$f_{x}(x,y)=ye^{x}+\sin y\Rightarrow f(x,y)=ye^{x}+x\,\sin y+g(y)$$

Assim obtemos que

$$f_{y}(x,y)=e^{x}+x\cos y+g'(y)$$

Mas, $f_{y}(x,y)=Q(x,y)$ logo obtemos que

$$e^{x}+x\cos y+g'(y)=e^{x}+x\,\cos y\Rightarrow g'(y)=0\Rightarrow g(y)=C.$$

Portanto,

$$f(x,y)=ye^{x}+x\sin y+C      \text{   e   }      \nabla f={\bf F}.$$


2906   

Utilize coordenadas polares para determinar o volume do sólido dado: acima do cone $z=\sqrt{x^2+y^2}$ e abaixo da esfera $x^2+y^2+z^2=1.$


$\displaystyle \frac{\pi}{3}(2 - \sqrt{2}).$


2320   

Determine uma equação do plano tangente à superfície parametrizada dada no ponto especificado. ${\bf r}(u,v)=(3\sin 2u,6\sin^{2} u, v)$,$0\leq u\leq \pi$, no ponto ${\bf r}(\pi/3,0).$


$x^{2} + (y-3)^{2} = 9.$


2605   

Use o Teorema de Stokes para calcular $\displaystyle\iint\limits_{ S}\mbox{rot}{\bf F}\cdot d{\bf S}.$

  • ${\bf F}(x,y,z) = x^2z^2{\bf i} + y^2z^2{\bf j} + xyz {\bf k}$ e $S$ é a parte do parabolóide $z = x^2+y^2$ que está dentro do cilindro $x^2+y^2=4$, orientado para cima.


$0.$


2477   

Determine e faça o esboço do domínio da função $f(x,y,z)=\sqrt{1-x^{2}-y^{2}-z^{2}}$.


$\left\lbrace (x,y);\; x^{2} + y^{2} + z^{2} \leq 1 \right\rbrace.$

A expressão 1 cossec x e o mesmo que


2475   

Determine e faça o esboço do domínio da função $f(x,y)=\sqrt{1-x^{2}}-\sqrt{1-y^{2}}$.


$\left\lbrace (x,y);\; -1 \leq x \leq 1,\;-1\leq y \leq 1 \right\rbrace.$

A expressão 1 cossec x e o mesmo que


2602   

Calcule, usando integração, o volume do sólido limitados pelas superfícies $z = 1$, $z = 2$ e $z = \sqrt{x^2 + y^2}.$


$\dfrac{7\pi}{6}.$


2354   

Calcule o volume do conjunto dado.

  1.  $x^{2}+y^{2}\leq a^{2}$ e $y^{2}+z^{2}\leq a^{2}$, $a >0.$

  2.  $x^{2}+y^{2}\leq z\leq 1-x^{2}.$


  1.  $\dfrac{16a^{3}}{3}.$

  2.  $\dfrac{\pi}{2\sqrt{2}}.$


2431   

Calcule a integral tripla.

  1.  $\displaystyle\iiint\limits_{  E}(x^{2}+z^{2})\;dx dy dz$, onde $E$ é o cilindro $x^{2}+y^{2}\leq 1$ e $0\leq z \leq 1.$

  2.  $\displaystyle\iiint\limits_{  E}\;dx dy dz$, onde $E$ é o conjunto $x^{2}+y^{2}\leq z \leq 2x+2y-1.$


  1.  $\dfrac{7\pi}{12}.$

  2.  $\dfrac{\pi}{2}.$


1964   

Calcule a integral de linha $\displaystyle\int_{C}x^{2}\,dx+y^{2}\,dy+z^{2}\,dz$, onde $C$ é o segmento de reta que liga o ponto $(1,0,1)$ ao ponto $(-2,2,2).$.


$\displaystyle \frac{2}{3}.$


3106   

Encontre a área da superfície descrita como sendo a parte do cilindro \(y^2+z^2=9\) que está acima do retângulo \(\displaystyle R=\{(x,y)\in\mathbb{R}^2;\ 0\leq x\leq 2,\ -3\leq y\leq 3\}\).


\( 6\,\pi\)


2114   

Se $z=f(x-y)$, mostre que
$$\dfrac{\partial z}{\partial x}+\dfrac{\partial z}{\partial y}=0.$$



Note que se $u = x - y,$ então $\displaystyle \frac{\partial z}{\partial  x} = \frac{dz}{du}$e$\displaystyle \frac{\partial  z}{\partial  y} = -\frac{dz}{du}.$


2293   

Determine uma representação paramétrica para a superfície descrita a seguir. O paraboloide $z=x^{2}+y^{2}$, \, $z\leq 4.$


$x = r \cos(\theta),$ $y = r \sin(\theta),$ $z = r^2,$ onde $0 \leq r \leq 2$ e $0\leq \theta \leq 2\pi.$


2993   

Calcule a integral, efetuando uma mudança de variáveis apropriada. $\displaystyle\iint\limits_{R} x \, dA$, em que $R$ é o conjunto, no plano $xy$, limitado pela cardioide $\rho = 1 - \cos{\theta}$.


$-\dfrac{5\pi}{4}.$


3002   

A integral $\int \!\!\! \int\limits_{\!\!\!\!\!R} \! \sqrt{9 - y^2} \, dA$, em que $R = [0,4] \times [0,2]$, representa o volume de um sólido. Esboce o sólido.


A expressão 1 cossec x e o mesmo que


2716   

Explique por que a função é diferenciável no ponto dado. A seguir, encontre a linearização $L(x,y)$ da função naquele ponto. $f(x,y) = x\sqrt{y}, \quad (1,4)$.


As derivadas $f_{x}$ e $f_{y}$ de cada $f$ existem e são contínuas no ponto dado, logo $f$ é diferenciável.

$L(x,y) = 2x + \frac{1}{4}y - 1$.


2328   

Determine a área da superfície dada pela parte do plano $3x+2y+z=6$ que está no primeiro octante.


$3\sqrt{14}.$


2111   

Utilize simetria para calcular $\iint\limits_{D}(2-3x+4y)\,dA$, onde $D$ é a região limitada pelo quadrado com vértices $(\pm 5,0)$ e $(0,\pm 5).$


$100.$


2942   

Calcule utilizando coordenadas esféricas. $\displaystyle\iiint\limits_{H}(9-x^{2}-y^{2})\,dV$, onde $H$ é o hemisfério sólido $x^{2}+y^{2}+z^{2}\leq 9$ e $z\geq 0.$


$\dfrac{486\pi}{5}.$


1995   

Seja ${\bf F}(t)$ uma força dependendo do tempo $t$, que atua sobre uma partícula entre os instantes $t_{1}$

e $t_{2}$. Supondo ${\bf F}$ integrável em $[t_{1},t_{2}]$, o vetor 

$${\bf I}=\int_{t_{1}}^{t_{2}}{\bf F}(t)\mathrm{d}t$$

denomina-se impulso de ${\bf F}$ no intervalo de tempo $[t_{1},t_{2}]$. Calcule o impulso de ${\bf F}$ no intervalo 

de tempo dado.

  1. ${\bf F}(t)=t{\bf i}+{\bf j}+t^{2}{\bf k}$, $t_{1}=0$ e $t_{2}=2.$
  2. ${\bf F}(t)=\dfrac{1}{t+1}{\bf i}+t^{2}{\bf j}+{\bf k}$, $t_{1}=0$ e $t_{2}=1.$


2095   

Seja ${\bf F}:\Omega\subset \mathbb{R}^{n}\rightarrow \mathbb{R}^{n}$ contínuo no aberto $\Omega$. Prove que uma condição necessária para que ${\bf F}$ seja

conservativo é que $\int_{C}{\bf F}\cdot d{\bf r}=0$ para toda curva $C$ fechada, de classe $C^{1}$ por partes, com imagem contida em $\Omega.$


Se $C$ é uma curva fechada em $\Omega$  parametrizada por $\mathbf{r}(t),$ com $a \leq t \leq b,$ $\mathbf{r}(a) = \mathbf{r}(b)$ e $\mathbf{F} = \nabla f,$ então $\int_{C}{\bf F}\cdot d{\bf r} = f(\mathbf{r}(a)) - f(\mathbf{r}(b)) = 0.$


2825   

Determine $(x,y)$, com $x^{2}+4y^{2}\leq 1$, que maximiza a soma $2x+y.$


$\displaystyle \left( \frac{4\sqrt{17}}{17}, \frac{\sqrt{17}}{34} \right).$


3152   

Mostre que o determinante Jacobiano da mudança de coordenadas cartesianas para esféricas é $-\rho^2 \sin \varphi$.


2685   

Encontre $\partial f/\partial x$ e $\partial f/\partial y$ para $f(x,y)=\cos^{2}(3x-y^2)$.


$\displaystyle \frac{\partial f}{\partial x} = -6\cos (3x - y^{2}) \sin(3x - y^{2}) \;\;\;\;\text{e}\;\;\;\; \frac{\partial f}{\partial y} = 4y \cos (3x - y^{2}) \sin(3x - y^{2})$.


2192   

Calcule $\displaystyle\iint \limits_{S}{\bf u}\cdot {\bf n}\,dS$, sendo $B=\{(x,y,z)\in \mathbb{R}^{3}|\, x^{2}+y^{2}+z^{2}\leq 1\}$ e ${\bf u}=x\,{\bf i}+y\,{\bf j}+z^{2}\,{\bf k}.$


2000   

Calcule o trabalho realizado pela força ${\bf F}(x,y)=xy\,{\bf i}+y^{2}\,{\bf j}$ ao mover uma partícula da origem ao longo da reta $y=x$ até $(1,1)$ e então de volta à origem ao longo da curva $y=x^{2}.$


$\dfrac{1}{12}.$


2362   

Encontre a derivada direcional de $f(x,y) = x^2 + y^2$ na direção do versor tangente da curva

$$\bf{r}(t) = (\cos{t} + t\sin{t})\bf{i} + (\sin{t} - t\cos{t})\bf{j},  t > 0.$$


Versor tangente a $\mathbf{r}(t):$ $\mathbf{u} = \cos(t)\mathbf{i} + (\sin(t))\mathbf{j};$ $D_{\mathbf{u}} f = 2.$


2314   

Determine a derivada direcional da função no ponto dado e na direção do vetor $\bf{v}$.

$f(x,y,z) = \sqrt{xyz},  (3,2,6),  \bf{v} = \left(-1,-2,2\right).$


$-1.$


2213   

Demonstre as identidades, admitindo que as derivadas parciais apropriadas existem e são contínuas. Se $f$ for um campo escalar e $\mathbf{F}$, $\mathbf{G}$ foram campos vetoriais, então $f\mathbf{F}$, $\mathbf{F} \cdot \mathbf{G}$ e $\mathbf{F} \times \mathbf{G}$ serão definidos por

$$\begin{array}{rcl}(f\mathbf{F})(x,y,z) & = & f(x,y,z)\mathbf{F}(x,y,z) \\(\mathbf{F} \cdot \mathbf{G})(x,y,z) & = & \mathbf{F}(x,y,z) \cdot \mathbf{G}(x,y,z) \\(\mathbf{F} \times \mathbf{G})(x,y,z) & = & \mathbf{F}(x,y,z) \times \mathbf{G}(x,y,z).\end{array}$$

  1. $\text{div }{(\mathbf{F}+\mathbf{G})} = \text{div }{\mathbf{F}}+\text{div }{\mathbf{G}}$.

  2. $\text{div }{(f\mathbf{F})} = f\text{div }{\mathbf{F}} + \mathbf{F} \cdot \nabla{f}$.

  3. $\text{div }{(\mathbf{F} \times \mathbf{G})} = \mathbf{G} \cdot \text{rot }{\mathbf{F}} - \mathbf{F}\cdot\text{rot }{\mathbf{G}}$.

  4. $\text{div }{(\nabla{f} \times \nabla{g})} = 0$.



Suponhamos que ${\bf F}=P_{1}\,{\bf i}+Q_{1}\,{\bf j}+R_{1}\,{\bf k}$ e ${\bf G}=P_{2}\,{\bf i}+Q_{2}\,{\bf j}+R_{2}\,{\bf k}.$

  1. Temos que $F+G=(P_{1}+P_{2})\,{\bf i}+(Q_{1}+Q_{2})\,{\bf j}+(R_{1}+R_{2})\,{\bf k}$. Então,

    $$\begin{array}{rcl}\text{div } ({\bf F}+{\bf G})&=&\frac{\partial(P_{1}+P_{2})}{\partial x}+\frac{\partial(Q_{1}+Q_{2})}{\partial y}+\frac{\partial(R_{1}+R_{2})}{\partial z}\\&=&\frac{\partial P_{1}}{\partial x}+\frac{\partial P_{2}}{\partial x}+\frac{\partial Q_{1}}{\partial y}+\frac{\partial Q_{2}}{\partial y}+\frac{\partial R_{1}}{\partial z}+\frac{\partial R_{2}}{\partial z}\\&=&\underbrace{\frac{\partial P_{1}}{\partial x}+\frac{\partial Q_{1}}{\partial y}+\frac{\partial R_{1}}{\partial z}}+\underbrace{\frac{\partial P_{2}}{\partial x}+\frac{\partial Q_{2}}{\partial y}+\frac{\partial R_{2}}{\partial z}}\\&=&          \text{div } {\bf F}           +          \text{div }{\bf G}.\end{array}$$

  2. Temos que $f{\bf F}=(fP_{1})\,{\bf i}+(fQ_{1})\,{\bf j}+(fR_{1})\,{\bf k}.$ Então,

    $$\begin{array}{rcl}\text{div } (f{\bf F})&=&\frac{\partial(fP_{1})}{\partial x}+\frac{\partial(fQ_{1})}{\partial y}+\frac{\partial(fR_{1})}{\partial z}\\&=&\frac{\partial f}{\partial x}\cdot P_{1}+f\cdot\frac{\partial P_{1}}{\partial x}+\frac{\partial f}{\partial y}\cdot Q_{1}+f\cdot\frac{\partial Q_{1}}{\partial y}+\frac{\partial f}{\partial z}\cdot R_{1}+f\cdot \frac{\partial R_{1}}{\partial z}\\&=&f\cdot\bigg(\underbrace{\frac{\partial P_{1}}{\partial x}+\frac{\partial Q_{1}}{\partial y}+\frac{\partial R_{1}}{\partial z}}\bigg)+\underbrace{\frac{\partial f}{\partial x}P_{1}+\frac{\partial f}{\partial y}Q_{1}+\frac{\partial f}{\partial z}R_{1}}\\&=&          f\cdot     \text{div } {\bf F}+              \nabla f\cdot {\bf F}\end{array}$$

  3. Temos que ${\bf F}\times {\bf G}=(Q_{1}R_{2}-Q_{2}R_{1})\,{\bf i}+(P_{2}R_{1}-P_{1}R_{2})\,{\bf j}+(P_{1}Q_{2}-Q_{1}R_{2})\,{\bf k}.$ Então,

    $$\begin{array}{rcl}\text{div } ({\bf F}\times {\bf G})&=&\frac{\partial(Q_{1}R_{2}-Q_{2}R_{1})}{\partial x}+\frac{\partial(P_{2}R_{1}-P_{1}R_{2})}{\partial y}+\frac{\partial(P_{1}Q_{2}-P_{2}Q_{1})}{\partial z}\\&=&\frac{\partial (Q_{1}R_{2})}{\partial x}-\frac{\partial (Q_{2}R_{1})}{\partial x}+\frac{\partial (P_{2}R_{1})}{\partial y}-\frac{\partial (P_{1}R_{2})}{\partial y}+\frac{\partial (P_{1}Q_{2})}{\partial z}-\frac{\partial (Q_{1}R_{2})}{\partial z}\\&=&\frac{\partial Q_{1}}{\partial x}\cdot R_{2}+Q_{1}\cdot \frac{\partial R_{2}}{\partial x}-\frac{\partial Q_{2}}{\partial x}\cdot R_{1}-Q_{2}\cdot \frac{\partial R_{1}}{\partial x}+\frac{\partial P_{2}}{\partial y}\cdot R_{1}+P_{2}\cdot \frac{\partial R_{1}}{\partial y}\\&-&\frac{\partial P_{1}}{\partial y}\cdot R_{2}-P_{1}\cdot \frac{\partial R_{2}}{\partial y}+\frac{\partial P_{1}}{\partial z}\cdot Q_{2}+P_{1}\cdot \frac{\partial Q_{2}}{\partial z}-\frac{\partial P_{2}}{\partial z}-P_{2}\cdot \frac{\partial Q_{1}}{\partial z}\\&=&P_{1}\bigg(\frac{\partial Q_{2}}{\partial z}-\frac{\partial R_{2}}{\partial y}\bigg)+Q_{1}\bigg(\frac{\partial R_{2}}{\partial x}-\frac{\partial P_{2}}{\partial z}\bigg)+R_{1}\bigg(\frac{\partial P_{2}}{\partial y}-\frac{\partial Q_{2}}{\partial x}\bigg)\\&+&P_{2}\bigg(\frac{\partial R_{1}}{\partial y}-\frac{\partial Q_{1}}{\partial z}\bigg)+Q_{2}\bigg(\frac{\partial P_{1}}{\partial z}-\frac{\partial R_{1}}{\partial x}\bigg)+R_{2}\bigg(\frac{\partial Q_{1}}{\partial x}-\frac{\partial P_{1}}{\partial y}\bigg)\\&=&\bigg[-P_{1}\bigg(\frac{\partial R_{2}}{\partial y}-\frac{\partial Q_{2}}{\partial z}\bigg)-Q_{1}\bigg(\frac{\partial P_{2}}{\partial z}-\frac{\partial R_{2}}{\partial x}\bigg)-R_{1}\bigg(\frac{\partial Q_{2}}{\partial x}-\frac{\partial P_{2}}{\partial y}\bigg)\bigg]\\&+&\bigg[P_{2}\bigg(\frac{\partial R_{1}}{\partial y}-\frac{\partial Q_{1}}{\partial z}\bigg)+Q_{2}\bigg(\frac{\partial P_{1}}{\partial z}-\frac{\partial R_{1}}{\partial x}\bigg)+R_{2}\bigg(\frac{\partial Q_{1}}{\partial x}-\frac{\partial P_{1}}{\partial y}\bigg)\bigg]\\&=&-{\bf F}\cdot \text{rot } {\bf G}+{\bf G}\cdot \text{rot } {\bf F}\\&=&{\bf G}\cdot \text{rot } {\bf F}-{\bf F}\cdot \text{rot } {\bf G}.\end{array}$$

  4. Do item anterior temos que

    $$\begin{array}{rcl}\text{div }(\nabla f\times \nabla g)&=&\nabla g \cdot \text{rot } (\nabla f)-\nabla f \cdot \text{rot } (\nabla g).\\\end{array} $$

    Sabemos que, se $f$ é uma função de três variáveis que tem derivadas parciais de segunda ordem contínuas, então $\text{rot } (\nabla f)={\bf 0}.$ Deste resultado, obtemos que

    $$\text{div }(\nabla f\times \nabla g) =\nabla g \cdot {\bf 0}-\nabla f \cdot {\bf 0}=0.$$


2693   

Encontre $f_{x}$, $f_{y}$ e $f_{z}$ para $f(x,y,z)=e^{-xyz}$.


$\displaystyle f_{x} = -yz e^{-xyz},\;\;\;\; f_{y} = -xz e^{-xyz}\;\;\;\;\text{e}\;\;\;\; f_{z} = -xy e^{-xyz}$.


3073   

A figura mostra o campo vetorial ${\bf F}(x,y)=(2xy, x^{2})$ e três curvas que começam em $(1,2)$ e terminam em $(3,2).$

A expressão 1 cossec x e o mesmo que

  1. Explique por que $\int_{c}{\bf F}\cdot d{\bf r}$ tem o mesmo valor para as três curvas.
  2. Qual é esse valor comum?


  1. ${\bf F}$ é conservativo, logo $\int_{C} \bf F \cdot d\bf r$ depende somente dos pontos inicial e final de $C.$
  2. $16.$


2992   

Calcule a integral, efetuando uma mudança de variáveis apropriada. $\displaystyle\iint\limits_{R} \dfrac{\sqrt[3]{y - x}}{1 + y + x} \, dA$, em que $R$ é o triângulo de vértices $(0,0), (1,0)$ e $(0,1)$.


$0$.


2918   

Considere uma pá quadrada de um ventilador com lados de comprimento 2 e com o canto inferior esquerdo colocado na origem. Se a densidade da pá for $\rho(x,y) = 1 + 0,1\cdot x$, é mais difícil girar a pá em torno do eixo $x$ ou do eixo $y$?



Se calcularmos os momentos de inércia sobre $x$ e $y$, poderemos determinar em qual direção será mais difíciel de girar a pá do ventilador. Notemos que a região de integração é o quadrado com lados de comprimento 2 e com o canto inferior esquerdo colocado na origem em ambas as integrais. Então, o momento de inércia sobre o eixo $x$ é dada por: $$I_{x}=\iint\limits_{D}y^{2}\rho(x,y)\,dA=\int_{0}^{2}\int_{0}^{2}y^{2}(1+0,1x)dydx$$ $$=\int_{0}^{2}(1+0,1x)\,dx\cdot \int_{0}^{2}y^{2}\,dy=\bigg(x+0,1\frac{x^{2}}{2}\bigg)\bigg|_{0}^{2}\cdot \bigg(\frac{y^{3}}{3}\bigg)\bigg|_{0}^{2}$$ $$=\bigg[(2+0,2)-0\bigg]\cdot \bigg[\frac{8}{3}\bigg]=\frac{17,6}{3}.$$ Da mesma forma, o momente de inércia sobre o eixo $y$ é dado por: $$I_{y}=\iint\limits{D}x^{2}\rho(x,y)\,dA=\int_{0}^{2}\int_{0}^{2}x^{2}(1+0,1x)dydx$$ $$=\int_{0}^{2}(x^{2}+0,1x^{3})\,dx\cdot \int_{0}^{2}\,dy=\bigg(\frac{x^{3}}{3}+0,1\frac{x^{4}}{4}\bigg)\bigg|_{0}^{2}\cdot \bigg(y\bigg)\bigg|_{0}^{2}$$ $$=\bigg[\bigg(\frac{8}{3}+0,4\bigg)-0\bigg]\cdot \bigg[2-0\bigg]=\frac{18,4}{3}.$$ Como $I_{y}>I_{x}$ é mais difícil girarmos a pá do ventilador em torno do eixo $y.$


3060   

Esboce o campo vetorial $\textbf{F}= \dfrac{y\textbf{i} - x\textbf{j}}{\sqrt{x^2+y^2}}$, desenhando um diagrama.


2037   

Se $z=f(x,y)$, onde $f$ é diferenciável, e $x=g(t)$, $g(3)=2$, $g'(3)=5$, $f_{x}(2,7)=6$, $y=h(t)$, $h(3)=7$, $h'(3)=-4$, $f_{y}(2,7)=-8,$ determine $\mathrm{d}z/ \mathrm{d}t$ quando $t=3.$


$\displaystyle \frac{dz}{dt}(3) = 62.$


2624   

Utilizando o Teorema de Stokes, transforme a integral $\iint_{ S}\mbox{rot}{\bf F}\cdot{\bf n}dS$ numa integral de linha e calcule.

  • ${\bf F}(x,y,z) = y{\bf i} + x^2{\bf j}+z{\bf k}$, $S$ a superfície parametrizada por ${\bf R}(u,v) = (u,v,2u+v+1)$, $u\geq 0$, $u+v\leq 2$, sendo ${\bf n}$ a normal apontando para baixo.


3138   

Use o Teorema da Divergência para encontrar todos os valores positivos \(k\) tais que \[ \mathbf{F}(\mathbf{r}) = \dfrac{\mathbf{r}}{\|\mathbf{r}\|^k} \] satisfaça a condição \(\mathrm{div\,}\mathbf{F}=0\) quando \(\mathbf{r}\neq \mathbf{0}\).


2418   

Calcule a área limitada pelas curvas $x=y^{2}-1$ e $x=2y^{2}-2.$


$\dfrac{4}{3}.$


3028   

Esboce a região de integração e mude a ordem de integração. $\displaystyle\int_{0}^{4}\!\!\int_{0}^{\sqrt{x}} \! f(x,y)\,dy dx$.


A expressão 1 cossec x e o mesmo que


2251   

Seja $S$ a parte do parabolóide $z=2-x^{2}-y^{2}$ que está acima do plano $z=1.$ Calcule o fluxo do campo vetorial ${\bf F}(x,y,z)=\frac{1}{(x^{2}+y^{2}+z^{2})^{3/2}}(x,y,z)$ através de $S.$


2086   

Calcule $\displaystyle\int_{C}(\sin(xy)+xy\,\cos(xy))\,dx+x^{2}\,\cos(xy)\,dy$, onde $C(t)=(t^{2}-1,t^{2}+1)$, $-1\leq t\leq 1.$


$0.$


2811   

Mostre que $f(x,y)=x^{2}+4y^{2}-4xy+2$ tem um número infinito de pontos críticos e que $f_{xx}f_{yy} - (f_{xy})^2 = 0$ em cada um. A seguir, mostre que $f$ tem um mínimo local (e absoluto) em cada ponto crítico.


Note que todos os pontos críticos são da forma $\displaystyle \left(x,\frac{1}{2}x \right)$ e que $f(x,y) = (x - 2y)^{2} + 2 \geq 2,$ com igualdade justamente se $\displaystyle y =  \frac{1}{2}x.$



Para encontrar os pontos críticos de $f$, devemos encontrar os pontos $(a,b)$ do domínio de $f$ tal que $f_x(a,b)=0$ e $f_y(a,b)=0$. Temos que $f_x(x,y)=2x-4y$ e $f_y(x,y)=8y-4x$ se anulam simultaneamente se $x=2y$. Logo, todos os pontos do conjunto $\{(x,y)\in \mathbb{R}^{2} | x=2y\}$ são pontos críticos de $f$, provando que $f$ tem infinitos pontos críticos.
Agora, queremos ver que $f_{xx}f_{yy}-f_{xy} ^{2}=0$ em todos os pontos críticos. Para isso, calculemos as segundas derivadas de $f$ \[ f_{xx}(x,y)=2, f_{xy}(x,y)=-4 \text{ e } f_{yy}(x,y)=8. \] Daí temos que $f_{xx}f_{yy}-f_{xy} ^{2}=2\cdot 8-(-4)^{2}=16-16=0$, como queríamos.
Por fim, queremos ver que esses pontos críticos são pontos de mínimo de $f$, mas como $f_{xx}f_{yy}-f_{xy} ^{2}=0$ o Teste de Derivada Segunda é inconclusivo. Mas, note que $f$ pode ser reescrita como \[ f(x,y)=x^2+4y^2-4xy+2=(x-2y)^2+2. \]

Como $(x-2y)^2\geq 0$, segue que $f(x,y)\geq 2$ para todo $(x,y)\in \mathbb{R}^2$. Em particular, se tomamos um ponto crítico de $f$, isto é, um ponto da forma $(2y,y)$ então $f(2y,y)=2$. Provando que os pontos críticos são pontos de mínimo de $f$.


2218   

Determine o rotacional e o divergente do campo vetorial $\mathbf{F}(x,y,z) = (\ln{x},\ln{(xy)},\ln{(xyz)})$.


$\text{rot } \mathbf{F} = \dfrac{1}{y}\mathbf{i} - \dfrac{1}{x} \mathbf{j} +\dfrac{1}{x} \mathbf{k}.$ $\text{div } \mathbf{F} = \dfrac{1}{x} + \dfrac{1}{y} + \dfrac{1}{z}.$


2190   

Calcule $\displaystyle\iint \limits_{S}{\bf u}\cdot {\bf n}\,dS$, sendo $S$ a fronteira de $B$ com normal exterior ${\bf n}$, sendo $B=\{(x,y,z)\in \mathbb{R}^{3}|\, 0\leq x\leq 1,\,0\leq y\leq x$ e $0\leq z\leq 4\}$ e ${\bf u}=xy\,{\bf i}+yz\,{\bf j}+z^{2}\,{\bf k}.$


3104   

Use coordenadas polares para calcular a integral dupla

\[ \iint_R e^{-(x^2+y^2)}\,dA, \]

onde \(R\) é a região contida no círculo \(x^2+y^2=1\).


\(\displaystyle (1-e^{-1})\pi \)


2702   

Seja $f(x,y)=\dfrac{x^{2}y^{2}}{x^{2}+y^{2}}.$

  1. Calcule as derivadas parciais $\dfrac{\partial f}{\partial x}(x,y)$ e $\dfrac{\partial f}{\partial y}(x,y)$, num ponto  $(x,y)\neq\;(0,0).$

  2. Calcule o limite, se existir.

    $$\lim_{(x,y)\rightarrow (0,0)}\frac{\partial f}{\partial x}(x,y)$$


  1. $\displaystyle \frac{\partial f}{\partial x} = \frac{2xy^{4}}{(x^{2} + y^{2})^{2}} \;\;\;\text{e}\;\;\; \frac{\partial f}{\partial y} = \frac{2x^{4}y}{(x^{2} + y^{2})^{2}}$.

  2. $\displaystyle \lim_{(x,y)\rightarrow (0,0)}\frac{\partial f}{\partial x}(x,y) = 0$.


2613   

Use o Teorema de Stokes para calcular $\displaystyle\int_C {\bf F}\cdot  d{\bf R}$. $C$ é orientada no sentido anti-horário quando vista de cima.

  • ${\bf F}(x,y,z) = (y+z,-z,y)$, $C$ é a curva obtida como interseção do cilindro $x^2+y^2=2y$ com o plano $y = z$.


$\dfrac{4\pi}{3}$.


2259   

Considere a integral $$\int_{0}^{1}\int_{3y}^{3}e^{x^{2}}\,dx dy.$$

  1.  Esboce a região de integração.

  2.  Calcule a integral usando a ordem de integração apropriada.


  1. (...)

  2.  $\dfrac{e^9 - 1}{6}.$


2777   

Esboce a região cuja área é dada pela integral $\displaystyle\int_{\pi}^{2\pi}  \int_{4}^{7}  r\, dr d\theta$ e calcule-a:


$\displaystyle \frac{33\pi}{2};$ região de integração:

A expressão 1 cossec x e o mesmo que


3156   

Seja $S$ uma superfície plana paralela ao plano $xy$. Mostre que a fórmula para o cálculo de áreas de superfícies nesse caso reduz à fórmula de integrais duplas para o cálculo de área de regiões planas.


2057   

Determine se ${\bf F}(x,y,z)=(x-y)\,{\bf i}+(x+y+z)\,{\bf j}+z^{2}\,{\bf k}$ é ou não um campo vetorial conservativo. Se for, determine uma função $f$ tal que ${\bf F}=\nabla f.$


Não.


2245   

Calcule $\int_{C}\mathbf{F} \cdot \mathbf{n} \, ds$ ($\mathbf{n}$ é unitário), onde $\mathbf{F}(x,y) = x^2\mathbf{i}$, $C$ dada por $\mathbf{r}(t) = (2\cos{t},\sin{t})$, $0 \leq t \leq \pi$ e $\mathbf{n}$ a normal com componente $y \geq 0$.


$0$.


2956   

Usando coordenadas esféricas, determine o volume da menor região cortada da esfera sólida $\rho \leq 2$ pelo plano $z=1.$


$\dfrac{5\pi}{3}.$


2868   

Estude com relação a máximos e mínimos a função dada com as restrições dadas.

$f(x,y) = x^3 + y^3 - 3x - 3y$ e $x + 2y = 3.$


Ponto de máximo local: $\displaystyle \left(- \frac{13}{7}, \frac{17}{7} \right)$; ponto de mínimo local:  $\displaystyle \left( 1,1 \right)$.


2800   

Determine os valores máximos e mínimos locais e pontos de sela da função $f(x,y)=x^{5}+y^{5}-5x-5y$.


Ponto de mínimo: $\displaystyle \left( 1,1\right);$ ponto de máximo: $\displaystyle \left( -1,-1\right);$ pontos de sela: $\displaystyle \left(1,-1\right)$ e $\displaystyle \left(-1,1\right).$


2652   

Disseram-lhe que existe uma função $f$ cujas derivadas parciais são \[f_{x}(x,y)=x+4y  \quad \mbox{e} \quad f_{y}(x,y)=3x-y,\] e cujas derivadas parciais de segunda ordem são contínuas. Você deve acreditar nisso?


Não, pois pelo Teorema de Clairaut deveria ser verdade que $f_{xy} = f_{yx},$ mas temos $f_{xy} = 4 \neq 3 = f_{yx}.$


2548   

Calcule

$$\displaystyle \lim_{(x,y) \to (0,0)} \dfrac{\mbox{sen}(x^2 + y^2)}{x^2 + y^2}.$$



Considere $t=x^{2}+y^{2}$.
Assim , se $(x,y)\rightarrow (0,0)$ temos que $t \to 0.$ Portanto,

$$\lim_{(x,y) \to (0,0)} \frac{\sin(x^{2}+y^{2})}{x^{2}+y^{2}}=\lim_{t \to 0}\frac{\sin t}{t}=1.$$


2711   

O elipsoide $4x^{2}+2y^{2}+z^{2}=16$ intercepta o plano $y=2$ em uma elipse. Determine as equações paramétricas da reta tangente à elipse no ponto $(1,2,2).$


$x = 1 + t,$ $y = 2,$ $z = 2 - 2t$.


1956   

Calcule a integral de linha $\int_{C}{\bf F}\cdot d{\bf r}$, onde $C$ é dada pela função vetorial ${\bf r}(t).$

${\bf F}(x,y,z)=x^{2}\,{\bf i}+y^{2}\,{\bf j}+z^{2}\,{\bf k}$, ${\bf r}(t)=(2\cos t,3\sin t,t)$, $0\leq t\leq 2\pi.$


$\dfrac{8\pi^{3}}{3}.$


2953   

Usando coordenadas esféricas, determine o volume da parte da bola $\rho\leq a$ que está entre os cones $\phi=\pi/6$ e $\phi=\pi/3.$


$\displaystyle \left( \sqrt{3} - 1 \right) \dfrac{\pi a^3}{3}.$


3105   

Mostre que

\[ \int_0^{+\infty}\int_0^{+\infty}\dfrac{1}{(1+x^2+y^2)^2}\,dxdy= \dfrac{\pi}{4}.\]


2866   

Estude com relação a máximos e mínimos a função dada com as restrições dadas.

$f(x,y) = x^2 - 2xy + y^2$ e $x^2 + y^2 = 1.$


Pontos de máximo: $\displaystyle \left( \frac{1}{\sqrt{2}}, -\frac{1}{\sqrt{2}} \right)$ e $\displaystyle \left( -\frac{1}{\sqrt{2}}, \frac{1}{\sqrt{2}} \right)$; ponto de mínimo: $\displaystyle \left( \frac{1}{\sqrt{2}}, \frac{1}{\sqrt{2}}\right)$.


2451   

Integre $g(x,y,z)=x+y+z$ sobre a superfície do cubo cortado do primeiro octante pelos planos $x=a$, $y=a$ e $z=a.$


$9a^3.$


2406   

Inverta a ordem de integração.

  1.  $\displaystyle\int_{1}^{e}\bigg[\int_{\ln(x)}^{x}f(x,y)\,dy\bigg]dx.$

  2.  $\displaystyle\int_{0}^{1}\bigg[\int_{y}^{y+3}f(x,y)\,dx\bigg]dy$

  3.  $\displaystyle\int_{-1}^{1}\bigg[\int_{-\sqrt{1-x^{2}}}^{\sqrt{1-x^{2}}}f(x,y)\,dy\bigg]dx$


  1.  $\displaystyle\int_{0}^{1}\bigg[\int_{1}^{e^{y}}f(x,y)\,dx\bigg]dy. + \displaystyle\int_{1}^{e}\bigg[\int_{y}^{1}f(x,y)\,dx\bigg]dy.$

  2.  $\displaystyle\int_{0}^{1}\bigg[\int_{0}^{x}f(x,y)\,dy\bigg]dx + \displaystyle\int_{1}^{3}\bigg[\int_{0}^{1}f(x,y)\,dy\bigg]dx + \displaystyle\int_{3}^{4}\bigg[\int_{x-3}^{1}f(x,y)\,dy\bigg]dx$  

  3.  $\displaystyle\int_{-1}^{1}\bigg[\int_{-\sqrt{1-y^{2}}}^{\sqrt{1-y^{2}}}f(x,y)\,dx\bigg]dy$


3019   

Esboce a região de integração para a integral iterada $\displaystyle\int_{\pi}^{2\pi}\!\!\int_{\sin{y}}^{\ln(y)}f(x,y)\,dx dy$.


A expressão 1 cossec x e o mesmo que


3132   

Encontre a massa da lâmina descrita como sendo a porção do cilindro circular \(x^2+z^2=4\) que fica diretamente acima do retângulo \(\displaystyle R=\{(x,y)\in\mathbb{R}^2;\ 0\leq x\leq 1,\ 0\leq y\leq 4\}\) e tem densidade \(\delta_0\) constante.


\(\dfrac{4}{3}\pi\delta_0\)


3011   

A fronteira de uma lâmina consiste nos semicírculos $y = \sqrt{1 - x^2}$ e   $y = \sqrt{4 - x^2}$, juntamente com as partes do eixo $x$ que os une. Encontre o centro de massa da lâmina se a densidade em qualquer ponto é proporcional à sua distância da origem.


$\displaystyle \left(0, \frac{45}{14\pi} \right).$


2627   

Utilizando o Teorema de Stokes, transforme a integral $\displaystyle\iint_{ S}\mbox{rot}{\bf F}\cdot{\bf n}dS$ numa integral de linha e calcule.

  • ${\bf F}(x,y,z) = y{\bf i}$, $S$ a superfície $z = x^2+y^2$ com $z \leq 1$, sendo ${\bf n}$ a normal com componente $z$ positiva.


$-\pi$.


2482   

Use a integral tripla para determinar o volume do sólido dado.

  1.  $x^{2}+y^{2}\leq 4$ e $x^{2}+y^{2}+z^{2}\leq 9.$

  2.  $x^{2}+4y^{2}+9z^{2}\leq 1.$

  3.  $\dfrac{x^{2}}{a^{2}}+\dfrac{y^{2}}{b^{2}}+\dfrac{z^{2}}{c^{2}}$,  $(a>0,\;b>0\;e\;c>0).$

  4.  $x^{2}+y^{2}\leq z \leq 4x+2y.$


  1.  $\left(36 - \dfrac{20\sqrt{5}}{3} \right)\pi.$

  2.  $\dfrac{2\pi}{9}.$

  3.  $\dfrac{4\pi abc}{3}.$

  4.  $\dfrac{25\pi}{4}$


2759   

Verifique que a função $f(x,y) = x \cos{(x^2 + y^2)}$ é diferenciável.


As derivadas parciais $\frac{\partial f}{\partial x}$ e $\frac{\partial f}{\partial y}$ de cada função $f$ existem e são contínuas em todos os pontos do domínio.


1927   

Determine o campo vetorial gradiente de  $f(x,y,z) = \sqrt{x^2+y^2+z^2}$.


$\nabla f(x,y,z) = \dfrac{x\textbf{i} + y\textbf{j} + z \textbf{k}}{\sqrt{x^{2} + y^{2} + z^{2}}}.$


2578   

Determine o conjunto dos pontos de continuidade da função $f(x,y) = 3x^2y^2 - 5xy + 6$. Justifique sua resposta.


$\mathbb{R}^{2}.$


2909   

Utilize coordenadas polares para determinar o volume do sólido dado: delimitado pelo paraboloide $z=9-x^2-y^2$ e pelo plano $z=5.$


$8\pi.$


2151   

Use o Teorema do Divergente para calcular o fluxo de ${\bf F}$ através de $S,$ onde ${\bf F}(x,y,z)=(\cos z+xy^{2})\,{\bf i}+xe^{-z}\,{\bf j}+(\sin y+x^{2}z)\,{\bf k}$ e $S$ é a superfície do sólido limitado pelo parabolóide $z=x^{2}+y^{2}$ e pelo plano $z=4.$


2935   

Utilize o resultado $\displaystyle \int_{-\infty}^{\infty}e^{-x^{2}}\,dx=\sqrt{\pi}$  para calcular as integrais:

  1. $\displaystyle\int_{0}^{\infty}   x^{2}e^{-x^{2}}\,dx$

  2. $\displaystyle\int_{0}^{\infty}\sqrt{x}e^{-x}\,dx$


  1. $\displaystyle \frac{\sqrt{\pi}}{4}.$

  2. $\displaystyle \frac{\sqrt{\pi}}{2}.$


3022   

Esboce a região de integração e calcule a integral $\displaystyle\int_{0}^{3}\!\!\int_{-2}^{0}(x^{2}y-2xy)\,dy dx$.


$0.$

A expressão 1 cossec x e o mesmo que


2622   

Utilizando o Teorema de Stokes, transforme a integral $\iint_{ S}\mbox{rot}{\bf F}\cdot{\bf n}dS$ numa integral de linha e calcule.

  • ${\bf F}(x,y,z) = y{\bf k}$, $S$ a superfície parametrizada por ${\bf R} (u,v) = (u,v,u^2+v^2)$, $u^2+v^2 \leq 1$, sendo ${\bf n}$ a normal apontando para cima.


 $0.$


2696   

Calcule todas as derivadas parciais de $2^{\underline{a}}$ ordem de $z=e^{x^{2}-y^{2}}$.


$\begin{aligned}[t]\frac{\partial^{2} z}{\partial x^{2}} &= 2e^{x^{2} - y^{2}}(1 + 2x^{2}),\;\;\;\;\; \frac{\partial^{2} z}{\partial y^{2}}= 2e^{x^{2} - y^{2}}(2y^{2} - 1) \;\;\;\;\;\text{e}\\\frac{\partial^{2} z}{\partial x\partial y} &= \frac{\partial^{2} z}{\partial y\partial x}= -4xye^{x^{2} - y^{2}}.\end{aligned}$


2694   

Seja  $w=f(x,y,z)$ uma função de três variáveis independentes. Escreva a definição formal de derivada parcial $\partial f/\partial z$ em $(x_{0},y_{0},z_{0})$. Use essa definição para encontrar $\partial f/\partial z$ em $(1,2,3)$ para $f(x,y,z)=x^{2}yz^{2}.$


$\displaystyle \frac{\partial f}{\partial z}(1,2,3) = 12$.


2770   

 Utilize coordenadas polares para combinar a soma $$\int_{\frac{1}{\sqrt{2}}}^{1} \int_{\sqrt{1-x^{2}}}^{x}xy\,dy dx+\int_{1}^{\sqrt{2}}   \int_{0}^{x}xy\,dy dx+\int_{\sqrt{2}}^{2} \int_{0}^{\sqrt{4-x^{2}}}xy\,dy dx$$ em uma única integral dupla. Em seguida, calcule essa integral dupla.



Queremos combinar a soma, abaixo, de integrais em uma única: $$\underbrace{\int_\frac{1}{\sqrt{2}}^{1} \int_\sqrt{1-x^{2}}^{x}xy\,dy dx}_{1}+\underbrace{\int_{1}^{\sqrt{2}}\int_{0}^{x}xy\,dy dx}_{2}+ \underbrace{\int_{\sqrt{2}}^{2} \int_{0}^{\sqrt{4-x^{2}}}xy\,dy dx}_{3}$$ Na figura abaixo, temos que a região da esquerda corresponde à região de integração da integral $(1)$, a região do meio corresponde à região de integração  da integral $(2)$ e a região da esquerda corresponde à região de integração da integral $(3)$.


A expressão 1 cossec x e o mesmo que


Notemos que com a junção das três regiões, podemos olhar como uma única região. Assim, em coordenadas  polares teremos que $0\leq \theta \leq \frac{\pi}{4}$ e $1\leq r \leq 2.$ Então: $$\int_{\frac{1}{\sqrt{2}}}^{1} \int_{\sqrt{1-x^{2}}}^{x}xy\,dy dx+\int_{1}^{\sqrt{2}}   \int_{0}^{x}xy\,dy dx+\int_{\sqrt{2}}^{2} \int_{0}^{\sqrt{4-x^{2}}}xy\,dy dx$$ $$=\int_{0}^{\frac{\pi}{4}}\int_{1}^{2}(r\,\cos \theta)\cdot (r\,\sin \theta)\,r\,dr\,d\theta=\int_{0}^{\frac{\pi}{4}}\int_{1}^{2}r^{3}\cos\theta \sin \theta\,dr\,d\theta$$ $$=\underbrace{\int_{0}^{\frac{\pi}{4}}\cos \theta\, \sin \theta\,d \theta}_{\substack{ u=\sin \theta\\ du=\cos\, d\theta}}\cdot \int_{1}^{2}r^{3}\,dr =\int_{0}^{\frac{\sqrt{2}}{2}}u\,du\cdot \frac{r^{4}}{4}\bigg|_{1}^{2}$$ $$=\frac{u^{2}}{2}\bigg|_{0}^{\frac{\sqrt{2}}{2}}\cdot \bigg(\frac{16}{4}-\frac{1}{4}\bigg)=\frac{1}{4}\cdot \frac{15}{4}=\frac{15}{16}.$$


2950   

Calcule utilizando coordenadas esféricas. $\displaystyle\iiint\limits_{E}xyz\,dV$, onde $E$ é o sólido limitado pelos paraboloides $z=x^{2}+y^{2}$ e $z=8-x^{2}-y^{2}.$


$0.$


2142   

Verifique que o Teorema do Divergente é verdadeiro para o campo vetorial ${\bf F}$ na região $E.$

${\bf F}(x,y,z)=x^{2}\,{\bf i}+xy\,{\bf j}+z\,{\bf k}$, $E$ é o sólido delimitado pelo paraboloide $z=4-x^{2}-y^{2}$ e pelo plano $xy.$


$\displaystyle\iint_{S} {\bf F} \cdot d{\bf S} = \iiint_{E} \mbox{div} {\bf F} dV = 8\pi.$


2063   

Dados ${\bf F}(x,y)=x^{2}\,{\bf i}+y^{2}\,{\bf j}$, $C$ é o arco da parábola $y=2x^{2}$ de $(-1,2)$ a $(2,8).$

  1. Determine uma função $f$ tal que ${\bf F}=\nabla f$.

  2. Use o resultado anterior para calcular $\int_{C}{\bf F}\cdot d{\bf r}$ sobre a curva $C$ dada.


  1. $f(x,y) = \dfrac{x^{3} + y^{3}}{3};$

  2. $171.$


1972   

Determine a equação da reta tangente à trajetória da função \newline${\bf r}(t)=\bigg(\dfrac{1}{t},
\dfrac{1}{t},t^{2}\bigg)$, no ponto ${\bf r}(2)$.



$${\bf r}(2)=\bigg(\frac{1}{2},\frac{1}{2},4\bigg)\,\,\,\,\,\, e \,\,\,\,\,\, \frac{d{\bf r}}{dt}=\bigg(-\frac{1}{t^{2}},-\frac{1}{t^{2}},2t\bigg).$$
Assim, 
$$\frac{d{\bf r}}{dt}(2)=\bigg(-\frac{1}{4},-\frac{1}{4},4\bigg).$$
Portanto, a equação da reta tangente em ${\bf r}(2)$ é:
$${\bf x}={\bf r}(2)+\lambda \frac{d{\bf r}}{dt}(2),\,\, \lambda \in \mathbb{R},$$
ou seja, 
$$(x,y,z)=\bigg(\frac{1}{2},\frac{1}{2},4\bigg)+\lambda \bigg(-\frac{1}{4},-\frac{1}{4},4\bigg),\,\, \lambda \in \mathbb{R}.$$


1980   

Mostre que a função vetorial
$$\textbf{r}(t) = (2\textbf{i} + 2\textbf{j} + \textbf{k}) + (\cos{t})\left( \dfrac{1}{\sqrt{2}}\textbf{i} - \dfrac{1}{\sqrt{2}}\textbf{j} \right) + (\sin{t})\left( \dfrac{1}{\sqrt{3}}\textbf{i} + \dfrac{1}{\sqrt{3}}\textbf{j} + \dfrac{1}{\sqrt{3}}\textbf{k} \right)$$
descreve o movimento de uma partícula no círculo de raio $1$ centrado no ponto $(2,2,1)$ e contido no plano $x + y - 2z = 2$.


2087   

Calcule $\displaystyle\int_{C}\dfrac{-y}{x^{2}+y^{2}}\,dx+\dfrac{x}{x^{2}+y^{2}}\,dy$, onde $C:[0,1]\rightarrow \mathbb{R}^{2}$ é uma curva de classe $C^{1}$ por partes, com imagem contida no conjunto $\Omega=\{(x,y)\in \mathbb{R}^{2}| y>0\}\cup\{(x,y)\in \mathbb{R}^{2}|\,x<0\}$, tal que $C(0)=(1,1)$ e $C(1)=(-1,-1).$


$0.$


1993   

Calcule.

  1. $\displaystyle\int_{0}^{1}(t{\bf i}+e^{t}{\bf j})\mathrm{d}t$
  2. $\displaystyle\int_{-1}^{1}\!\bigg(\sin(3t){\bf i}+\dfrac{1}{1+t^{2}}{\bf j}+{\bf k}\bigg)\mathrm{d}t$
  3. $\displaystyle\int_{1}^{2}(3{\bf i}+2{\bf j}+{\bf k})\mathrm{d}t$


2646   

Determine a derivada parcial indicada. $u=e^{r\theta}\sin{\theta}$; $\dfrac{\partial ^{3}u}{\partial r^{2}\partial \theta}$.


$\dfrac{\partial ^{3}u}{\partial r^{2}\partial \theta} = \theta e^{r\theta} (2\sin \theta + \theta \cos \theta + r\theta \sin \theta)$.


2604   

Vamos demonstrar a expressão geral para o volume de um cone circular de altura $h$ e raio da base $R$.

  1.  Representando o cone com vértice na origem e base no plano $z = h$, expresse $V$ por meio de uma integral dupla.

  2.  Calculando a integral, verifique que $V = \dfrac{\pi R^2 h}{3}$.


  1.  $V = 2 \displaystyle \int_{0}^{h} \int_{-\frac{R}{h}z}^{\frac{R}{h}z} \sqrt{\dfrac{R^{2}}{h^{2}} z^{2} - x^{2}} dx dz.$

  2.  Note que $\displaystyle \int_{0}^{h} \int_{-\frac{R}{h}z}^{\frac{R}{h}z} \sqrt{\dfrac{R^{2}}{h^{2}} z^{2} - x^{2}} dx dz = \dfrac{\pi R^{2}h}{6}$ é o volume da parte superior (ou inferior) do cone.


2814   

Determine os valores máximo e mínimo absolutos de $f$ no conjunto $D.$

$f(x,y)=xy^{2}$, $D=\{(x,y) \in \mathbb{R}^2: x\geq 0,\;y\geq 0,\;x^{2}+y^{2}\leq3\}.$


Valor máximo: $2;$ valor mínimo: $0.$


2376   

Seja $f$ uma função de duas variáveis que tenha derivadas parciais contínuas e considere os pontos $A = (1,3)$, $B = (3,3)$, $C = (1,7)$ e $D = (6,15)$. A derivada direcional em $A$ na direção do vetor $\overrightarrow{AB}$ é 3, e a derivada direcional em $A$ na direção $\overrightarrow{AC}$ é 26. Determine a derivada direcional de $f$ em $A$ na direção do vetor $\overrightarrow{AD}$.


 $\displaystyle \frac{327}{13}.$


2280   

Defina gradiente de uma função de três variáveis. Calcule $\nabla f(x,y,z)$.

$f(x,y,z) = x^2 + y^2 + z^2$


 $\displaystyle \nabla f(x,y,z) = (2x,2y,2z).$


2558   

Determine o limite, se existir, ou mostre que o limite não existe.

$\displaystyle \lim_{(x,y) \to (0,0)} \dfrac{x^2ye^y}{x^4 + 4y^2}$.

Não existe.


Não existe.


3114   

A tendência de uma lâmina de resistir a uma mudança no seu movimento de rotação em torno de um eixo é medida pelo seu momento de inércia em torno daquele eixo. Se a lâmina ocupar uma região \(R\) do plano \(xy\) e se sua densidade \(\delta(x,y)\) for uma função contínua em \(R\), então os momentos de inércia em torno dos eixos \(x\), \(y\) e \(z\) são denotados por \(I_x\), \(I_y\) e \(I_z\), respectivamente, e são definidos por \begin{align*} I_x & = \iint\limits_R y^2\delta(x,y)\,dA, \\ I_y & = \iint\limits_R x^2\delta(x,y)\,dA, \\ I_z & = \iint\limits_R (x^2+y^2)\delta(x,y)\,dA. \\ \end{align*} Considere a lâmina circular que ocupa a região descrita pelas desigualdades \(0\leq x^2+y^2\leq a^2\). Supondo que a lâmina tenha densidade \(\delta\) constante, mostre que \[  I_x= I_y=\dfrac{\delta\pi a^4}{4}, \quad  I_z= \dfrac{\delta\pi a^4}{2}.\]


2144   

Verifique que o Teorema do Divergente é verdadeiro para o campo vetorial ${\bf F}$ na região $E.$

${\bf F}(x,y,z)=x\,{\bf i}+y\,{\bf j}+z\,{\bf k}$, $E$ é a bola unitária $x^{2}+y^{2}+z^{2}\leq 1.$


$\displaystyle\iint_{S} {\bf F} \cdot d{\bf S} = \iiint_{E} \mbox{div} {\bf F} dV =  4\pi.$


2501   

Encontre uma equação para a curva de nível da função $f(x,y)=16-x^{2}-y^{2}$ que passa pelo ponto $(2\sqrt{2},\sqrt{2})$.


$x^{2} + y^{2} = 10.$


2923   

Utilize a integral dupla para determinar a área da região: limitada pelo eixo $x$ positivo e pela espiral $r=4\theta/3$, $0\leq \theta \leq 2\pi.$ A região se parece com uma concha de caracol.


$\dfrac{64\pi^3}{27}.$


3116   

Seja \(G\) a região sólida dentro da esfera de raio \(2\) centrada na origem e acima do plano \(z=1\). Mostre (ou verifique) os seguintes resultados:

  1.  O volume de \(G\) é dado por \[\iiint\limits_G\,dV = \int_0^{2\pi}\int_0^{\sqrt{3}}\int_1^{\sqrt{4-r^2}}r\,dzdrd\theta \]

  2.  \[\iiint\limits_G\dfrac{z}{x^2+y^2+z^2}\,dV = \int_0^{2\pi}\int_0^{\sqrt{3}}\int_1^{\sqrt{4-r^2}}\dfrac{rz}{r^2+z^2}\,dzdrd\theta \]


2264   

Calcule $\displaystyle\iint\limits_{B}f(x,y)\,dx dy$ sendo dados:

  1.  $f(x,y)=\dfrac{1}{\ln(y)}$ e $B=\bigg\{(x,y)\in \mathbb{R}^{2}|\;2\leq y\leq 3,\;0\leq x\leq \dfrac{1}{y}\bigg\}.$

  2.  $f(x,y) = xy\cos{x^{2}}$ e $B=\{(x,y) \in \mathbb{R}^{2}| \; 0 \leq x \leq 1, \; x^{2} \leq y \leq 1\}$.

  3.  $f(x,y) = \cos(2y)\sqrt{4-\sin^{2}{x}}$ e $B$ é o triângulo de vértices $(0,0)$, $\bigg(0,\dfrac{\pi}{2}\bigg)$ e $\bigg(\dfrac{\pi}{2},\dfrac{\pi}{2}\bigg).$

  4.  $f(x,y)=x+y$ e $B$ a região compreendida entre os gráficos das funções $y=x$ e $y=e^{x}$, com $0\leq x\leq 1.$


  1.  $\ln(\ln(3)) - \ln(\ln(2)).$

  2.  $\dfrac{\sin(1) - \cos(1)}{2}$.

  3.  $\dfrac{8}{3} - \sqrt{3}.$

  4.  $\dfrac{1 + e^{2}}{4}.$


2530   

Represente graficamente o domínio da função $z=f(x,y)$ dada por $z=\sqrt{y-x^{2}}+\sqrt{2x-y}$.


$\left\lbrace (x,y); x^{2} \leq y \leq 2x \right\rbrace$

A expressão 1 cossec x e o mesmo que


2999   

Esboce a região de integração e mude a ordem de integração. $\displaystyle\int_{1}^{2}\!\!\int_{0}^{\ln(x)} \! f(x,y)\,dy dx$.


Note que a região de integração é do tipo I, é dada por

$$\{(x,y) \in \mathbb{R}^2: 1 \leq x \leq 2 \mbox{ e } 0 \leq y \leq \ln(x)\}$$

e pode ser vista geometricamente como a região esboçada na figura abaixo.

A expressão 1 cossec x e o mesmo que

Além disso, ela pode ser descrita como uma região do tipo II da seguinte forma:
$$\{(x,y) \in \mathbb{R}^2: e^y \leq x \leq 2 \mbox{ e } 0 \leq y \leq \ln{2}\}.$$
Portanto, a integral pode ser reescrita como
$\displaystyle\int_{0}^{\ln{2}}\!\!\int_{e^y}^{2} \! f(x,y)\,dx dy$.


2313   

Determine a derivada direcional da função no ponto dado e na direção do vetor $\bf{v}$.

$f(x,y,z) = xe^y + ye^z + ze^x,  (0,0,0),  \bf{v} = \left(5,1,-2\right).$


$\displaystyle \frac{4}{\sqrt{30}}.$


2755   

Verifique que a função $f(x,y) = e^{x - y^2}$ é diferenciável.


As derivadas parciais $\frac{\partial f}{\partial x}$ e $\frac{\partial f}{\partial y}$ de cada função $f$ existem e são contínuas em todos os pontos do domínio.


2958   

Usando coordenadas esféricas, determine o volume do sólido que está acima do plano $z=2\sqrt{3}$ e abaixo da esfera $x^{2}+y^{2}+z^{2}=16.$


$\dfrac{88\pi}{3}.$


2966   

Dentre as coordenadas cilíndricas ou esféricas, utilize a que lhe parecer mais apropriada para determinaretermine o volume da região limitada abaixo pelo plano $z=0$, lateralmente pelo cilindro $x^{2}+y^{2}=1$ e acima pelo paraboloide $z=x^{2}+y^{2}$.


$\dfrac{\pi}{2}.$


2713   

Determine uma equação do plano tangente à superfície no ponto especificado.

$z = 3(x-1)^2 + 2(y+3)^2 + 7, \quad (2,-2,12)$.


$z = 6x + 4y + 8$.


1963   

Calcule a integral de linha $\displaystyle\int_{C}y^{2}\,dx+x\,dy -\,dz$, onde $C$ é a poligonal de vértices $A_{0}=(0,0,0)$, $A_{1}=(1,1,1)$, $A_{2}=(1,1,0)$, orientada de $A_{0}$ para $A_{2}.$


$\displaystyle \frac{5}{6}.$


2572   

Calcule $\displaystyle \lim_{(x,y) \to (0,0)} \dfrac{xy(x - y)}{x^4 + y^4}$, caso exista.


Não existe.


2161   

Use o Teorema de Green para calcular a integral de linha ao longo da curva dada com orientação positiva. $\displaystyle\int_{C}(y+e^{\sqrt{x}}) \, dx + (2x+\cos{y^2}) \, dy$, $C$ é a fronteira da região englobada pelas parábolas $y=x^2$ e $x=y^2$.


$\dfrac{1}{3}.$


2265   

Calcule $\displaystyle\iint\limits_{B}f(x,y)\,dx dy$ sendo dados:

  1.  $f(x,y)=y^{3}e^{xy^{2}}$ e $B$ o retângulo $0\leq x\leq 1$, $1\leq y\leq 2.$

  2.  $f(x,y)=x^{5}\cos{y^{3}}$ e $B=\{(x,y)\in \mathbb{R}^{2}|\;y\geq x^{2},\;x^{2}+y^{2}\leq 2\}.$

  3.  $f(x,y)= x^{2}$ e $B$ o conjunto de todos $(x,y)$ tais que $x\leq y\leq -x^{2}+2x+2.$

  4.  $f(x,y)=x$ e $B$ a região compreendida entre os gráficos de $y=\cos{x}$ e $y=1-\cos{x}$, com $0\leq x\leq \dfrac{\pi}{2}.$


  1.  $\dfrac{e^{4} - e - 3}{2}.$

  2.  $0.$

  3.  $\dfrac{63}{20}.$

  4.  $\left(\dfrac{5}{72} -\dfrac{ \sqrt{3}}{18}\right)\pi^{2} + \left( \dfrac{4\sqrt{3}}{3} - 1 \right) \pi.$


2092   

Calcule a integral de linha

$$\int_{C}{\bf F}\cdot d{\bf r}=\int_{C}{\bf F}\cdot r'(t)\,dt$$

onde ${\bf F}=(2xyz^{3},x^{2}z^{3},3x^{2}yz^{2})$ e $C$ é a curva dada por $r(t)=(\sin^{6}t,1-\cos t, e^{t(t-\pi/2)}$, $0\leq t\leq \pi/2.$ (Dica: verifique se ${\bf F}$ é conservativo.)


$1.$


2907   

Utilize coordenadas polares para determinar o volume do sólido dado: dentro do cilindro $x^2+y^2=4$ e do elipsoide $4x^2+4y^2+z^2=64.$


$\displaystyle \frac{8\pi}{3} (64 - 24\sqrt{3}).$


2062   

Determine se ${\bf F}(x,y,z)=e^{y+2z}({\bf i}+x\,{\bf j}+2x\,{\bf k})$ é ou não um campo vetorial conservativo. Se for, determine uma função $f$ tal que ${\bf F}=\nabla f.$


Sim. $f(x,y,z) = xe^{y + 2z} + K.$


2388   

Determine as equações do plano tangente e da reta normal à superfície dada, no ponto dado.

$ze^{x - y} + z^3 = 2$ em $(2,2,1)$.


 Plano tangente: $x - y + 4z = 4$,
Reta normal: $(x,y,z) = (2,2,1) + \lambda (1,-1,4),$ $\lambda \in \mathbb{R}.$


1960   

Calcule a integral de linha $\displaystyle\int_{C}{\bf F}\cdot d{\bf r}$, onde ${\bf F}(x,y,z)=(yz,2xz,xy+2z)$ e $C$ é o segmento de reta que liga o ponto $(1,0,1)$ ao ponto $(-2,2,2).$


$-7.$


2467   

Calcule a integral de superfície $\displaystyle\iint \limits_{ S}{\bf F}\cdot d{\bf S}$ para o campo vetorial ${\bf F}$ e superfície orientada $S$ dados abaixo. Em outras palavras, determine o fluxo de ${\bf F}$ através de $S$. Para superfícies fechadas, use a orientação positiva (para fora).

  • ${\bf F}(x,y,z)=x{\bf i}+y{\bf j}+z{\bf k}$ e $S$ é a parte no primeiro octante do plano $2x+3y+z=6.$


$18.$


2638   

Determine as derivadas parciais de primeira ordem da função $f(x,y)=x^{5}+3x^{3}y^{2}+3xy^{4}$.


$\displaystyle \frac{\partial f}{\partial x} = 5x^{4} + 9x^{2}y^{2} + 3y^{4}\;\;\;\text{e}\;\;\; \frac{\partial f}{\partial y} = 2x^{3}y + 12xy^{3}$.


3057   

Faça uma correspondência entre as funções $f$ e os desenhos de seus campos vetoriais gradientes (rotulados de I-IV). Justifique.

  1. $f(x,y) = x^2+y^2$
  2. $f(x,y) = (x+y)^2$.
  3. $f(x,y) = x(x+y)$.
  4. $f(x,y) = \sin{\sqrt{x^2+y^2}}$.

I

A expressão 1 cossec x e o mesmo que

II

A expressão 1 cossec x e o mesmo que

III

A expressão 1 cossec x e o mesmo que

IV

A expressão 1 cossec x e o mesmo que


  1. III.
  2. IV.
  3. II.
  4. I.


1949   

Calcule a integral de linha $\int_{C}{\bf F}\cdot d{\bf r}$, onde $C$ é dada pela função vetorial ${\bf r}(t).$

${\bf F}(x,y)=xy\,{\bf i}+3y^{2}\,{\bf j}$, ${\bf r}(t)=11t^{4}\,{\bf i}+t^{3}\,{\bf j}$, $0\leq t\leq 1.$


$45.$


3082   

De acordo com a lei dos gases ideais, a pressão, a temperatura e o volume de um gás confinado estão relacionados por \( P=kT/V\), onde \(k\) é uma constante. Use diferenciais para aproximar a variação percentual na pressão se a temperatura de um gás tiver crescido em \(3\%\) e o volume tiver crescido em \(5\%\).


2664   

Determine as derivadas parciais de $f(x,y)=\sqrt[3]{x^{3}+y^{2}+3}$.


$\displaystyle \frac{\partial f}{\partial x} = \frac{x^{2}}{\sqrt[3]{(x^{3} + y^{3} + 3)^{2}}}\;\;\;\;\;\;\text{e}\;\;\;\;\; \frac{\partial f}{\partial y} = \frac{2y}{3 \sqrt[3]{(x^{3} + y^{3} + 3)^{2}}} .$


2347   

Seja $f:K\rightarrow \mathbb{R}$ de classe $C^{1}$ no compacto $K$ com fronteira de conteúdo nulo e interior não-vazio. Mostre que a área da superfície $z=f(x,y)$ (isto é, da superfície ${\bf r}$ dada por $x=u$, $y=v$ e $z=f(u,v)$) é dada pela fórmula

$$\iint\limits_{ K}\sqrt{1+\bigg(\frac{\partial f}{\partial x}\bigg)^{2}+\bigg(\frac{\partial f}{\partial y}\bigg)^{2}}dxdy.$$


2698   

Calcule todas as derivadas parciais de $2^{\underline{a}}$ ordem de $g(x,y)=4x^{3}y^{4}+y^{3}$.


$\displaystyle \frac{\partial^{2} g}{\partial x^{2}}= 24xy^{2},\;\;\;\;\; \frac{\partial^{2} g}{\partial y^{2}}= 48x^{3}  y^{2} \;\;\;\;\;\text{e}\;\;\;\;\; \frac{\partial^{2} g}{\partial x\partial y}= \frac{\partial^{2} g}{\partial y\partial x}= 48x^{2}y^{3}.$


2498   

Esboce a região limitada pelos gráficos das equações e use uma integral tripla para calcular seu volume.

  1.  $z+x^{2}=4$, $y+z=4$, $y=0$ e $z=0.$

  2.  $y=2-z^{2}$, $y=z^{2}$, $x+z=4$ e $x=0.$

  3.  $y^{2}+z^{2}=1$, $x+y+z=2$ e $x=0.$


  1.  $\dfrac{128}{5}.$

  2.  $\dfrac{32}{3}.$

  3.  $2\pi.$


2422   

Calcule a integral iterada.

  1.  $\displaystyle\int_{0}^{1}\!\!\int_{0}^{z}\!\!\int_{0}^{x+z}6xz\;dy dx dz$

  2.  $\displaystyle\int_{0}^{3}\!\!\int_{0}^{1}\!\!\int_{0}^{\sqrt{1-z^{2}}}ze^{y}\;dx dz dy$

  3.  $\displaystyle\int_{0}^{\pi/2}\int_{0}^{y}\int_{0}^{x}\cos(x+y+z)\;dz dx dy$


  1.  $1.$

  2.  $\displaystyle \frac{e^3 - 1}{3}.$

  3.  $-\dfrac{1}{3}.$


2663   

Determine as derivadas parciais de $z=(x^{2}+y^{2})\ln(x^{2}+y^{2})$.


$\displaystyle \frac{\partial z}{\partial x} = 2x(1 + \ln(x^{2} + y^ {2}))\;\;\;\;\;\;\text{e}\;\;\;\;\; \frac{\partial z}{\partial y} = 2y(1 + \ln(x^{2} + y^ {2})).$


2109   

Sejam $f(x)$ e $g(x)$ duas funções contínuas, respectivamente, nos intervalos $[a,b]$ e $[c,d].$ Use o seguinte resultado $$\iint\limits_{R}f(x)g(y)\,dx dy=\bigg(\int_{a}^{b}f(x)\,dx\bigg)\bigg(\int_{c}^{d}g(y)\,dy\bigg),$$ onde $R$ é o retângulo $a\leq x\leq b$ e $c\leq y\leq d$, para calcular as integrais

  1.  $\displaystyle\iint\limits_{R} x\ln(y)\,dx dy$, onde $R$ é o retângulo $0\leq x\leq 2,\;1\leq y\leq 2.$

  2.  $\displaystyle\iint\limits_{R} xye^{x^{2}-y^{2}}\,dx dy$, onde $R$ é o retângulo $-1\leq x\leq 1,\;0\leq y\leq 3.$


  1.  $2(2\ln(2) - 1).$

  2.  $0.$


3006   

Calcule o centro de massa do quadrado $D$ dado por $0 \leq x \leq 1, \ 0 \leq y \leq 1$  e com densidade $\quad \rho(x,y) = y$.


$\displaystyle \left(\frac{1}{2}, \frac{2}{3} \right).$


2717   

Explique por que a função é diferenciável no ponto dado. A seguir, encontre a linearização $L(x,y)$ da função naquele ponto. $f(x,y) = \dfrac{x}{x+y}, \quad (2,1)$.


As derivadas $f_{x}$ e $f_{y}$ de cada $f$ existem e são contínuas no ponto dado, logo $f$ é diferenciável.

$L(x,y) = \frac{1}{9}x - \frac{2}{9}y + \frac{2}{3}$.


2780   

Calcule a integral dupla utilizando coordenadas polares: $\displaystyle\iint\limits_{R}(x^{2}+y^{2})\,dx dy$, onde $R=\{(x,y)\in \mathbb{R}^{2}| 1\leq x^{2}+y^{2}\leq 4\}.$


$\displaystyle \frac{15\pi}{2}.$


2339   

Determine a área da superfície com equações paramétricas $x=u^{2}$, $y=uv$, $z=\dfrac{1}{2}v^{2}$, $0\leq u\leq 1$, $0\leq v\leq 2.$


$4.$


2668   

Seja $\phi:\mathbb{R}\rightarrow \mathbb{R}$ uma função de uma variável real, diferenciável e tal que $\phi '(1)=4.$ Seja $g(x,y)=\phi\bigg(\dfrac{x}{y}\bigg).$ Verifique que, para todo $(x,y)\in \mathbb{R}^{2}$, com $y\neq 0$, temos que

$$x\;\dfrac{\partial g}{\partial x}(x,y)+y\;\dfrac{\partial g}{\partial y}(x,y)=0.$$


2854   

Determine a menor distância entre os planos paralelos $2x+3y-z=2$ e $2x+3y-z=4.$


$\displaystyle \frac{\sqrt{14}}{7}.$


2154   

Use o Teorema do Divergente para calcular o fluxo de ${\bf F}$ através de $S,$ onde ${\bf F}(x,y,z)=(x^{2}+z^{2})\,{\bf i}+(y^{2}-2xy)\,{\bf j}+(4z-2yz)\,{\bf k}$ e $S$ é a superfície da região delimitada pelo cone $x=\sqrt{y^{2}+z^{2}}$ e pelo plano $x=9.$


1936   

Calcule a integral de linha, onde $C$ é a curva dada.

$\displaystyle\int_{C}x\,dx+y\,dy$,   $C:\,x=t^{2},\,y=\sin t$, $0\leq t\leq \pi/2.$


$\displaystyle \frac{\pi^{4}}{32} + \frac{1}{2}.$


2940   

Determine a massa e o centro de massa da lâmina que ocupa a região $D$ e tem função densidade $\rho$, quando: $D$ é delimitada por $y = e^x$, $y = 0$, $x = 0$ e $x = 1; \quad \rho(x,y) = y$.


Massa: $\dfrac{1}{4}(e^{2} - 1);$ centro de massa: $\displaystyle \left(\frac{e^2 + 1}{2(e^2 - 1)},\frac{4(e^3 - 1)}{9 (e^2 - 1)} \right).$


2171   

Determine o trabalho $W = \int_{C}\mathbf{F}\cdot\, d\mathbf{r}$ realizado pelo campo de força

$$\mathbf{F}(x,y) = x\mathbf{i} + (x^3 + 3xy^2)\mathbf{j}$$

em uma partícula que inicialmente está no ponto $(-2,0)$, se move ao longo do eixo $x$ para $(2,0)$ e então se move ao longo da semicircunferência $y = \sqrt{4-x^2}$ até o ponto inicial.


$12\pi.$


2185   

No item abaixo: 

  1. expresse $\partial w/\partial u$ e $\partial w/ \partial v$ como funções de $u$ e $v$, usando a Regra da Cadeia e também expressando $w$ diretamente em termos e $u$ e $v$ antes de diferenciar; 
  2. calcule $\partial w/\partial u$ e $\partial w/ \partial v$ no ponto dado $(u,v)$.

$w=\ln(x^{2}+y^{2}+z^{2})$,  $x=ue^{v}\sin{u}$,  $y=ue^{v}\cos{u}$,  $z=ue^{v}$; $(u,v)=(-2,0).$


  1. $\displaystyle w(u,v) = \ln(2) + 2\ln(u) + 2v,$$\displaystyle \frac{\partial w}{\partial u}(u,v) = \frac{2}{u}$ e $\displaystyle \frac{\partial w}{\partial v}(u,v) = 2.$
  2. $\displaystyle \frac{\partial w}{\partial u}(-2,0) = -1$ e $\displaystyle \frac{\partial w}{\partial v}(-2,0) = 2.$


2496   

Para qual valor de $c$ o volume do elipsóide $x^{2}+(y/2)^{2}+(z/c)^{2}=1$ é igual a $8\pi$?


$3.$


3153   

Mude a ordem de integração para mostrar que:
$$ \int_0^a \left[ \int_0^y e^{m(a-x)} f(x) \, dx \right] dy = \int_0^a (a-x) e^{m(a-x)} f(x) \, dx,$$
onde $a$ e $m$ são constantes e $a>0$.


2955   

Usando coordenadas esféricas, determine o volume da porção da esfera sólida $\rho \leq a$ que está entre os cones $\phi=\pi/3$ e $\phi=2\pi/3.$


$\dfrac{2\pi a^{3}}{3}.$


2659   

Determine as derivadas parciais de $z=xye^{xy}$.


$\displaystyle \frac{\partial z}{\partial x} = ye^{xy} (1 + xy) \;\;\;\;\;\;\text{e}\;\;\;\;\; \frac{\partial z}{\partial y} = xe^{xy} (1 + xy).$


2594   

Mude as coordenadas de $(1,-1,4)$ de retangulares para cilíndricas.


 $\displaystyle (\sqrt{2}, \dfrac{7\pi}{4}, 4).$


2691   

Encontre $f_{x}$, $f_{y}$ e $f_{z}$ para $f(x,y,z)=\ln(x+2y+3z)$.


$\displaystyle f_{x} = \frac{1}{x + 2y + 3z},\;\;\;\; f_{y} = \frac{2}{x + 2y + 3z}\;\;\;\;\text{e}\;\;\;\; f_{z} = \frac{3}{x + 2y + 3z}$.


2705   

Considere a função

$$f(x,y)= \begin{cases}\dfrac{xy^{2}}{x^{2}+y^{4}}, & \quad \text{se } (x,y)\neq (0,0),\\0, & \quad \text{se } (x,y)=(0,0).\\\end{cases}$$

  1. A função é contínua em $(0,0)$? Justifique sua resposta.

  2. Determine as derivadas parciais $\dfrac{\partial f}{\partial x}(0,0)$ e $\dfrac{\partial f}{\partial y}(0,0)$.


  1. Não, pois $\lim_{(x,y) \to (0,0)} f(x,y)$ não existe.

  2. $\displaystyle \frac{\partial f}{\partial x}(0,0) = \frac{\partial f}{\partial y}(0,0) = 0$.


2856   

Determine as dimensões da caixa retangular de volume máximo, com faces paralelas aos planos coordenados, que possa ser inscrita no elipsóide $16x^{2}+4y^{2}+9z^{2}=144.$


$\displaystyle \frac{8}{\sqrt{3}} \times \frac{6}{\sqrt{3}} \times \frac{12}{\sqrt{3}}.$


2423   

A temperatura em um ponto $(x,y,z)$ em uma substância com condutividade $K=6,5$ é $u(x,y,z)=2y^{2}+2z^{2}.$ Determine a taxa de transmissão de calor nessa substância para dentro da superfície cilíndrica $y^{2}+z^{2}=6$, $0\leq x\leq 4.$



O fluxo de calor, com $u(x,y,z)=2y^{2}+2z^{2}$, é dado por

$${\bf F}(x,y,z)=-K \nabla u=-6,5(0{\bf i}+4y{\bf j}+4z{\bf k})=0{\bf i}-26y{\bf j}-26z{\bf k}.$$

Temos que $S$ é a superfície cilíndrica $y^{2}+z^{2}=6$ e $0\leq x \leq 4.$ As equações paramétricas de $S$ são:

$$x=x, y=\sqrt{6}\cos \theta \mbox{e} z=\sqrt{6}\sin \theta$$

onde $0\leq x \leq 4$ e $0\leq \theta \leq 2\pi.$

Então,

$${\bf r}(x,\theta)=x{\bf i}+\sqrt{6}\cos \theta{\bf j}+\sqrt{6}\sin \theta{\bf k}.$$

Como queremos o fluxo de calor para dentro de $S$ devemos calcular

$$\int \int\limits_{S}{\bf F}\cdot dS=\int \int\limits_{ D}{\bf F}({\bf r}(x,\theta))\cdot ({\bf r}_{x}\times {\bf r}_{\theta})dA.$$

Então,

$${\bf r}_{x}(x,\theta)={\bf i}+0{\bf j}+0{\bf k}$$

e

$${\bf r}_{\theta}(x,\theta)=0{\bf i}-\sqrt{6}\sin \theta{\bf j}-\sqrt{6}\cos \theta{\bf k}.$$

Logo,

$\begin{array}{rcl} {\bf r}_{x} \times {\bf r}_{\theta} &=& \left| \begin{array}{ccc}{\bf i} & {\bf j} & {\bf k}\\1 & 0 & 0\\0 & -\sqrt{6}\sin \theta & -\sqrt{6}\cos \theta \\ \end{array} \right| \\ &=& 0{\bf i}-\sqrt{6}\cos \theta{\bf j}-\sqrt{6}\sin \theta{\bf k}, \end{array}$

$${\bf F}({\bf r}(x,\theta))=(0{\bf i}-26\sqrt{6}\cos\theta{\bf j}-26\sqrt{6}\sin \theta{\bf k})$$

e

$${\bf F}({\bf r}(x,\theta))\cdot ({\bf r}_{x}\times {\bf r}_{\theta})=(0{\bf i}-26\sqrt{6}\cos\theta{\bf j}-26\sqrt{6}\sin \theta{\bf k}) \cdot (0{\bf i}-\sqrt{6}\cos \theta{\bf j}-\sqrt{6}\sin \theta{\bf k})=156$$

Assim, a taxa de fluxo de calor para dentro de $S$ é:

$$\int \int\limits_{S}{\bf F}\cdot dS=\int \int\limits_{ D}{\bf F}({\bf r}(x,\theta))\cdot ({\bf r}_{x}\times {\bf r}_{\theta})dA=\int \int\limits_{ D}156 dA=156\int \int\limits_{ D} 1 dA$$

$$=156\int_{0}^{2\pi}\int_{0}^{4}1dxd\theta=156\int_{0}^{2\pi}d\theta\cdot \int_{0}^{4}dx=156\cdot (\theta)\bigg|_{0}^{2\pi}\cdot (x)\bigg|_{0}^{4}=156\cdot 2\pi \cdot 4=1248 \pi.$$


2123   

Suponha que, para todo $t$, $f(t^{2},2t)=t^{3}-3t$. Mostre que 
$$\dfrac{\partial f}{\partial x}(1,2)=-\dfrac{\partial f}{\partial y}(1,2).$$


Tome $t = 1$ em $\displaystyle \frac{df}{dt}(t^{2},2t) = 2t \frac{\partial f}{\partial x}(t^{2},2t) + 2\frac{\partial f}{\partial y}(t^{2},2t) = 3t^{2} - 3.$


2204   

Suponha que $u=f(x,y)$ e $v=g(x,y)$ verifiquem as equações de Cauchy- Riemann $u_{x}=v_{y}$ e $u_{y}=-v_{x}$. Se $x=r\cos{\theta}$ e 
$y=r\sin{\theta}$, mostre que 
$$\frac{\partial u}{\partial r}=\frac{1}{r}\frac{\partial v}{\partial \theta}  \text{ e }  \frac{\partial v}{\partial r}=-\frac{1}{r}\frac{\partial u}{\partial \theta}.$$



Note que $\displaystyle \frac{\partial u}{\partial r} = \cos(\theta) u_{x} + \sin (\theta) u_{y},$ $\displaystyle \frac{\partial v}{\partial r} = \cos(\theta) v_{x} + \sin (\theta) v_{y},$ 
$\displaystyle \frac{\partial u}{\partial \theta} = -r\sin(\theta) u_{x} + r \cos(\theta) u_{y}$ e $\displaystyle \frac{\partial v}{\partial \theta} = - r\sin(\theta) v_{x} + r \cos(\theta) v_{y}$.


2871   

Utilize os multiplicadores de Lagrange para determinar os valores máximo e mínimo da função sujeita à(s) restrição(ões) dada(s).

$f(x,y) = 4x + 6y; \quad x^2 + y^2 = 13.$


Valor máximo: $26;$ valor mínimo: $-26.$


2670   

Seja $z=e^{y}\phi(x-y)$, onde $\phi$ é uma função diferenciável de uma variável real. Mostre que $$\dfrac{\partial z}{\partial x}+\dfrac{\partial z}{\partial y}=z.$$


$\displaystyle \frac{\partial z}{\partial x} = e^{y}\phi'(x-y) \;\;\;\;\;\text{e}\;\;\;\;\; \frac{\partial z}{\partial y} = e^{y} \phi(x-y) -e^{y}\phi' (x-y).$


3149   

Seja \(\displaystyle \mathbf{F}(x,y,z)=f(x,y,z)\mathbf{i}+ g(x,y,z)\mathbf{j} + h(x,y,z)\mathbf{k}\) e suponha que \(f\), \(g\) e \(h\) sejam contínuas e tenham derivadas parciais de primeira ordem contínuas numa região. Mostre que se \(\mathbf{F}\) é conservativo numa região esférica aberta então \(\mathrm{rot\,}\mathbf{F} = \mathbf{0}\) nessa região. [Sugestão: use que se \(\mathbf{F}\) for conservativo numa região, então \[ \dfrac{\partial f}{\partial y}=\dfrac{\partial g}{\partial x},\quad \dfrac{\partial f}{\partial z}=\dfrac{\partial h}{\partial x},\quad \dfrac{\partial g}{\partial z}=\dfrac{\partial h}{\partial y} \]  nessa mesma região.]


2176   

Calcule a área da região limitada pela astroide $x=\cos^3{t}$, $y = \sin^3{t}$,  $0 \leq t \leq 2\pi$.


$\dfrac{3\pi}{8}.$


2952   

Seja $E$ o sólido limitado pelos dois planos $z=1$ e $z=2$ e lateralmente pelo cone $z=\sqrt{x^{2}+y^{2}}$. Expresse o volume de $E$ como integral tripla em coordenadas esféricas (não é necessário calcular a integral).


$\displaystyle \int_{0}^{2\pi} \int_{0}^{\pi/4} \int_{\sec(\phi)}^{2\sec(\phi)} \rho^{2}\sin(\phi)\;d\rho d\phi d\theta.$


2402   

 Determine as equações paramétricas da reta tangente à curva formada pela intersecção do paraboloide $z = x^2 + y^2$ com o elipsoide $4x^2 + y^2 + z^2 = 9$ no ponto $(-1,1,2)$.


 $(x,y,z) = (-1,1,2) + \lambda (-10, -16, -12),$ $\lambda \in \mathbb{R}.$


3068   

Calcule a integral de linha $\displaystyle\int_{C}\sqrt[3]{x}\,dx+\dfrac{dy}{1+y^{2}}$, onde $C$ é a curva na figura abaixo.

A expressão 1 cossec x e o mesmo que


$0.$


2556   

Determine o limite, se existir, ou mostre que o limite não existe.

$\displaystyle \lim_{(x,y) \to (0,0)} \dfrac{xy \ \mbox{cos} \ y}{3x^2 + y^2}$.


2862   

Estude com relação a máximos e mínimos a função dada com as restrições dadas.

$f(x,y) = x^2 + 2y^2$ e $3x + y = 1.$


Ponto de mínimo:  $\displaystyle \left( \frac{6}{19}, \frac{1}{19} \right)$.


2832   

Determine as dimensões de uma caixa retangular de volume máximo tal que a soma dos comprimentos de suas 12 arestas seja uma constante $c$.


A caixa é um cubo com arestas de comprimento $\dfrac{c}{12}.$


3148   

Seja \(G\) um sólido com a superfície \(\sigma\) orientada por vetores normais unitários para fora, suponha que \(\phi\) tenha derivadas parciais de primeira e segunda ordens contínuas em algum conjunto aberto contendo \(G\) e seja \(D_{\mathbf{n}}\phi\) a derivada direcional de \(\phi\), onde \(\mathbf{n}\) é um vetor normal unitário para fora de \(\sigma\). Mostre que \[ \iint\limits_\sigma D_{\mathbf{n}}\phi\,dS = \iiint\limits_G\left[\dfrac{\partial^2\phi}{\partial x^2}+ \dfrac{\partial^2\phi}{\partial y^2} + \dfrac{\partial^2\phi}{\partial z^2} \right]\,dV. \]


2917   

Determine a imagem do conjunto $S$ sob a transformação dada. $S$ é o disco dado por $u^2 + v^2 \leq 1$;$x = au$, $y = bv$.



Suponha $a$ e $b$ não-nulos. Por essa mudança de coordenadas, temos que $u = x/a$ e $v = y/b$. Substituindo na equação dada, obtemos

$$\frac{x^2}{a^2}+\frac{y^2}{b^2} \leq 1,$$

isto é, o disco $S$ é transformado em uma elipse.


2844   

Passe para coordenadas polares e calcule: $\displaystyle\int_{0}^{1}  \int_{0}^{\sqrt{1-y^{2}}}(x^{2}+y^{2})\,dx dy$


$\displaystyle \frac{\pi}{8}.$


2493   

Dada a função $f(x,y)=xy$.

  1. Encontre o domínio da função.

  2. Encontre a imagem da função.

  3. Descreva as curvas de nível da função.


  1. $D_{f} = \mathbb{R}^{2}$.

  2. $Im(f) = \mathbb{R}.$

  3. As curvas de nível são as hipérboles $xy = C$ quando $C \neq 0$ e os eixos $x$ e $y$ quando $C = 0.$


2107   

Um campo vetorial inverso do quadrado é da forma:

$${\bf F}({\bf r})=\frac{c{\bf r}}{|{\bf r}|^{3}}$$

para alguma constante $c$, onde ${\bf r}=x\,{\bf i}+y\,{\bf j}+z\,{\bf k}$. Um exemplo de campo inverso do quadrado é o campo elétrico ${\bf F}=\epsilon q Q{\bf r}/|{\bf r}|^{3}$. Suponha que um elétron com carga de $-1,6\times 10^{-19}\, C$ esteja localizado na origem. Uma carga positiva unitária é colocada à distância de $10^{-12}\,m$ do elétron e se move para uma posição que está à metade da distância original do elétron. Determine o trabalho realizado pelo campo elétrico. (Use o valor $\epsilon=8,985\times 10^{9}$.)


$\approx 1,4 \times 10^{4}$ J.


3024   

Esboce a região de integração e calcule a integral $\displaystyle\int_{\pi}^{2\pi}\!\!\int_{0}^{\pi}(\sin{x}+\cos{y})\,dx dy$.


$2\pi.$

A expressão 1 cossec x e o mesmo que


2562   

Determine o maior conjunto no qual a função $F(x,y) = \dfrac{1}{x^2 - y}$ é contínua.


$\left\lbrace (x,y);\; y \neq x^{2} \right\rbrace.$


2971   

Mostre que

$$\int_{-\infty}^{\infty}\int_{-\infty}^{\infty}\int_{-\infty}^{\infty}\sqrt{x^{2}+y^{2}+z^{2}}\,e^{-(x^{2}+y^{2}+z^{2})}\,dxdydz=2\pi.$$

(A integral imprópria tripla é definida como o limite da integral tripla sobre uma esfera sólida quando o raio da esfera aumenta indefinidamente.)


Note que $$\begin{split}&\int_{-\infty}^{\infty}\int_{-\infty}^{\infty}\int_{-\infty}^{\infty}\sqrt{x^{2}+y^{2}+z^{2}}\,e^{-(x^{2}+y^{2}+z^{2})}\,dxdydz  \\&= \lim_{R \to \infty} \int_{0}^{2\pi}\int_{0}^{\pi}\int_{0}^{R} \rho e^{-\rho^2}\rho^2 \sin(\phi)\;d\rho d\phi d\theta.\end{split}$$


3084   

Suponha que \(f(x,y)\) seja uma função diferenciável no ponto \((x_0,y_0)\) e seja \(z_0=f(x_0,y_0)\). Mostre que a função \(\displaystyle g(x,y,z)=z-f(x,y)\) é diferenciável em \((x_0,y_0,z_0)\).


2153   

Use o Teorema do Divergente para calcular o fluxo de ${\bf F}$ através de $S,$ onde ${\bf F}(x,y,z)=yz\,{\bf i}+xz\,{\bf j}+xy\,{\bf k}$ e $S$ é o gráfico de $x^{2/3}+y^{2/3}+z^{2/3}=1.$


2841   

Passe para coordenadas polares e calcule: $\displaystyle\int_{0}^{1}  \int_{x^{2}}^{\sqrt{2-x^{2}}}\sqrt{x^{2}+y^{2}}\,dy dx$


$\displaystyle \frac{2}{45}(1 + \sqrt{2}) + \frac{\pi}{3\sqrt{2}}.$


2623   

Utilizando o Teorema de Stokes, transforme a integral $\iint_{ S}\mbox{rot}{\bf F}\cdot{\bf n}dS$ numa integral de linha e calcule.

  • ${\bf F}(x,y,z) = y{\bf i}-x^2{\bf j}+5{\bf k}$, $S$ a superfície parametrizada por ${\bf R}(u,v) = (u,v,1-u^2)$, $u \geq 0$, $v \geq 0$, $u+v\leq 1$, sendo ${\bf n}$ a normal apontando para cima.


$-\dfrac{5}{6}.$


2226   

Determine se o campo vetorial $\mathbf{F}(x,y,z) = 2xy\mathbf{i} + (x^2+2yz)\mathbf{j} + y^2\mathbf{k}$ é conservativo ou não. Se for conservativo, determine uma função $f$ tal que $\mathbf{F} = \nabla{f}$.


$\mathbf{F}$ é conservativo. $f(x,y,z) = x^2 y + y^2 z.$


2150   

Use o Teorema do Divergente para calcular o fluxo de ${\bf F}$ através de $S,$ onde ${\bf F}(x,y,z)=x^{3}y\,{\bf i}-x^{2}y^{2}\,{\bf j}-x^{2}yz\,{\bf k}$ e $S$ é a superfície do sólido delimitado pelo hiperbolóide $x^{2}+y^{2}-z^{2}=1$ e pelos planos $z=-2$ e $z=2.$


2739   

$2x + y + 3z = 6$ é a equação do plano tangente ao gráfico de $f(x,y)$ no ponto $(1,1,1)$.

  1. Calcule $\dfrac{\partial f}{\partial x}(1,1)$ e $\dfrac{\partial f}{\partial y}(1,1)$.

  2. Determine a equação da reta normal no ponto $(1,1,1).$


  1. $\displaystyle \frac{\partial f}{\partial x} (1,1) = -\frac{2}{3}$ e  $\displaystyle \frac{\partial f}{\partial y} (1,1) = -\frac{1}{3}.$

  2. $(x,y,z) = (1,1,1) + \lambda (2,1,3)$.


3050   

A figura mostra a região de integração da integral

$$\int_{0}^{1}\int_{\sqrt{x}}^{1}\int_{0}^{1-y}f(x,y,z)\;dz dy dx.$$

Reescreva essa integral como uma integral iterada equivalente nas cinco outras ordens.

A expressão 1 cossec x e o mesmo que


$\int_{0}^{1}\int_{\sqrt{x}}^{1}\int_{0}^{1-y}f(x,y,z)\;dz dy dx = \int_{0}^{1}\int_{0}^{y^2}\int_{0}^{1-y}f(x,y,z)\;dz dx dy $
$= \int_{0}^{1}\int_{0}^{1 - z}\int_{0}^{y^2}f(x,y,z)\;dx dy dz = \int_{0}^{1}\int_{0}^{1 - y}\int_{0}^{y^2}f(x,y,z)\;dx dz dy $
$= \int_{0}^{1}\int_{0}^{1 - \sqrt{x}}\int_{\sqrt{x}}^{1-z}f(x,y,z)\;dy dz dx = \int_{0}^{1}\int_{0}^{(1 - z)^2}\int_{\sqrt{x}}^{1-z}f(x,y,z)\;dy dx dz.$


1940   

Calcule a integral de linha, onde $C$ é a curva dada.

$\displaystyle\int_{C}(2x+9z)\,ds$,   $C:\,x=t,\, y=t^{2},\, z=t^{3},\, 0\leq t\leq 1.$


$\displaystyle \frac{1}{6}\left(14^{3/2} - 1\right).$


2274   

No item abaixo :

  1.  determine o gradiente de $f$; 
  2.  calcule o gradiente no ponto $P$; e 
  3.  determine a taxa de variação de $f$ em $P$ na direção do vetor $\bf{u}$.

$f(x,y,z) = \sqrt{x+yz},  P = (1,3,1),  \bf{u} = \left( \frac{2}{7}, \frac{3}{7}, \frac{6}{7} \right)$.


  1. $\displaystyle \nabla f(x,y,z) = \left(\frac{1}{2\sqrt{x + yz}}, \frac{z}{2\sqrt{x + yz}}, \frac{y}{2\sqrt{x + yz}} \right).$
  2. $\displaystyle \nabla f(1,3,1) = \left(\frac{1}{4}, \frac{1}{4}, \frac{3}{4}\right).$
  3. $\displaystyle \frac{23}{28}.$


2712   

Determine uma equação do plano tangente à superfície no ponto especificado.

$z = 4x^2 - y^2 + 2y, \quad (-1,2,4)$.


$z = -8x - 2y$.


2480   

Esboce o gráfico da função $f(x,y)=y$.


$z = y.$

A expressão 1 cossec x e o mesmo que


2781   

Calcule a integral dupla usando coordenadas polares: $\displaystyle\iint\limits_{R}e^{x^{2}+y^{2}}\,dx dy$, onde $R$ é o conjunto de todos os $(x,y)$ tais que  $1\leq x^{2}+y^{2}\leq 4$,  $-x\leq y\leq x$ e $x\geq 0.$


$\displaystyle \frac{\pi}{4}(e^4 - e).$


1950   

Calcule a integral de linha $\int_{C}{\bf F}\cdot d{\bf r}$, onde $C$ é dada pela função vetorial ${\bf r}(t).$

${\bf F}(x,y,z)=(x+y)\,{\bf i}+(y-z)\,{\bf j}+z^{2}\,{\bf k}$, ${\bf r}(t)=t^{2}\,{\bf i}+t^{3}\,{\bf j}+t^{2}\,{\bf k}$, $0\leq t\leq 1.$


$\dfrac{17}{15}.$


2938   

Determine a massa e o centro de massa da lâmina que ocupa a região $D$ e tem função densidade $\rho$, sendo:$D$ a região triangular com vértices $(0,0), (2,1), (0,3)$ e  $\rho(x,y) = x + y$.


Massa: $6;$ centro de massa: $\displaystyle \left(\frac{3}{4},\frac{3}{2} \right).$


2972   

Calcule a integral $\displaystyle\iint\limits_{R} \dfrac{e^{y - x^2}}{y - x^2} dA$, em que $R$ é o conjunto de todos $(x,y)$ tais que $1 + x^2 \leq y \leq 2 + x^2$, $y \geq x + x^2$ e $x \geq 0$, efetuando uma mudança de variáveis apropriada.


Olhando o integrando, é natural pensar que uma das novas variáveis introduzidas deva ser $y-x^2$, mas a outra, a princípio, não está pré-definida. Seja $u = y - x^2$ (escolheremos $v$ apropriadamente depois). Vamos analisar a região de integração dada.

\qquad Como $1 + x^2 \leq y \leq 2 + x^2, \text{ temos } 1 \leq y-x^2 \leq 2$, isto é, $1 \leq u \leq 2$;

\qquad Como $y \geq x + x^2$ e $x \geq 0$, temos $y-x^2 \geq x \geq 0$, isto é, $u \geq x \geq 0$.

Da análise acima, é natural pensar na outra variável como sendo $v = x$. Considere então a mudança de variáveis dada por

$$\begin{cases}x = v, \\y = u+v^2.\end{cases}$$

O Jacobiano dessa transformação é

$$\dfrac{\partial (x,y)}{\partial (u,v)} = \left| \begin{array}{cc} \dfrac{\partial x}{\partial u} & \dfrac{\partial x}{\partial v}  \\ \dfrac{\partial y}{\partial u} & \dfrac{\partial y}{\partial v} \end{array} \right| = \left| \begin{array}{cc} 0 & 1  \\ 1 & 2v \end{array} \right| = -1.$$

Como analisamos anteriormente, a nova região de integração é

$$S = \{(u,v) \in \mathbb{R}^2: 1 \leq u \leq 2 \mbox{ e } 0 \leq v \leq u\}.$$

Assim,

\begin{array}{rcl}\displaystyle\iint\limits_{R} \dfrac{e^{y - x^2}}{y - x^2} \, dA & = & \displaystyle \iint\limits_{S} \frac{e^u}{u} \left|\frac{\partial (x,y)}{\partial (u,v)}\right|\, dv du \\& = & \displaystyle\int_{1}^{2}\int_{0}^{u} \frac{e^u}{u} (1) \, dv du \\  & = & \displaystyle\int_{1}^{2} \left.\left(\frac{v e^u}{u}\right|_{v=0}^{v=u}\right) \, du \\    & = & \displaystyle\int_{1}^{2} e^u \, du \\    & = & e^u |_{1}^{2} = e^2 - e.\end{array}


2322   

Calcule a área da superfície dada por: ${\bf r}(u,v)=(u,v,1-u-v)$, $u\geq 0$,  $v\geq 0$ e $u+v\leq 1.$. (Sugerimos ao leitor desenhar a imagem da superfície.)


$\dfrac{\sqrt{3}}{2}.$


3066   

  1. Esboce o campo vetorial $\textbf{F}(x,y) = \textbf{i} + x\textbf{j}$ e algumas linhas de escoamento. Qual é o formato que essas linhas de escoamento parecem ter?
  2. Se as equações paramétricas das linhas de escoamento são $x=x(t)$ e $y=y(t)$, que equações diferenciais essas funções satisfazem? Deduza que $dy/dx = x$.
  3. Se uma partícula está na origem no instante inicial e o campo de velocidade é dado por $\textbf{F}$, determine uma equação para a trajetória percorrida por ela.


2143   

Verifique que o Teorema do Divergente é verdadeiro para o campo vetorial ${\bf F}$ na região $E.$
${\bf F}(x,y,z)=xy\,{\bf i}+yz\,{\bf j}+zx\,{\bf k}$, $E$ é o cilindro sólido $x^{2}+y^{2}\leq 1$, $0\leq z\leq 1.$


$\displaystyle\iint_{S} {\bf F} \cdot d{\bf S} = \iiint_{E} \mbox{div} {\bf F} dV =  \dfrac{\pi}{2}.$


2598   

Considere a integral tripla iterada $$\int_{-\sqrt{2}}^{\sqrt{2}}\int_{-\sqrt{2-x^2}}^{\sqrt{2-x^2}}\int_{x^2 + y^2}^{4-x^2-y^2} dz dy dx.$$

  1.  Transforme a integral utilizando coordenadas cilíndricas.

  2.  Calcule a integral.

  3.  Descreva o sólido cujo volume é dado por essa integral.


  1.  $\displaystyle \int_{0}^{2\pi}\int_{0}^{\sqrt{2}}\int_{ r^2}^{4-r^{2}} r dz dr d\theta.$

  2.  $4\pi.$

  3.  Região entre os parabolóides $z = x^2 + y^2$ e $z = 4 - x^2 - y^2$.


2336   

Determine a área da superfície dada pela parte da superfície $z=xy$ que está dentro do cilindro $x^{2}+y^{2}=1$.


$\dfrac{2\pi}{3}(2\sqrt{2} - 1)$.


2367   

Em que direção e sentido a função dada cresce mais rapidamente no ponto dado? E em que direção e sentido decresce mais rapidamente?

$f(x,y) = \ln{||(x,y)||}$ em $(1,-1)$.


 Cresce: $(1,-1)$; descresce: $(-1,1).$


2719   

Determine a aproximação linear da função $f(x,y) = \sqrt{20 - x^2 - 7y^2}$ em $(2,1)$ e use-a para aproximar $f(1,95; 1,08)$.


$L(x,y) = -\frac{2}{3}x - \frac{7}{3}y + \frac{20}{3}$  e $f(1,95; 1,08) \approx 2.847.$


1997   

Determine o trabalho realizado pelo campo  de força ${\bf F}(x,y)=x^{2}\,{\bf i}+xy\,{\bf j}$ sobre uma partícula que dá uma volta no círculo $x^{2}+y^{2}=4$ no sentido anti-horário.


$0.$


2079   

Sejam $f(x)$ e $g(x)$ duas funções contínuas, respectivamente, nos intervalos $[a,b]$ e $[c,d].$ Prove que $$\iint\limits_{R}f(x)g(y)\,dx dy=\bigg(\int_{a}^{b}f(x)\,dx\bigg)\bigg(\int_{c}^{d}g(y)\,dy\bigg),$$ onde $R$ é o retângulo $a\leq x\leq b$ e $c\leq y\leq d.$


Note que $$ \int_{c}^{d} \left[\int_{a}^{b}f(x)g(y)\,dx\right] \;dy = \int_{c}^{d} \left[\int_{a}^{b}f(x)\,dx\right]g(y)  \;dy = \left(\int_{a}^{b}f(x)\,dx\right) \int_{c}^{d} g(y)  \;dy.$$


2031   

Utilize a Regra da Cadeia para determinar $\mathrm{\partial}z/\mathrm{\partial} s$ e $\mathrm{\partial}z/ \mathrm{\partial}t.$

$z=x^{2}y^{3}$, $x=s\cos{t}$, $y=s\sin{t}$. 


$\displaystyle \frac{\partial z}{\partial s} = 2xy^{3} \cos(t) + 3x^{2}y^{2} \sin(t) $ e $\displaystyle \frac{\partial z}{\partial t} = -2sxy^{3} \sin(t) + 3 sx^{2}y^{2} \cos(t)$. 


2812   

Determine os valores máximo e mínimo absolutos de $f$ no conjunto $D.$

$f(x,y)=3+xy-x-2y$, $D$ é a região triangular fechada com vértices $(1,0)$, $(5,0)$ e $(1,4).$


Valor máximo: $2;$  valor mínimo: $-2.$


2285   

Identifique a superfície que tem equação paramétrica ${\bf r}(u,v)=(u+v)\,{\bf i}+(3-v)\,{\bf j}+(1+4u+5v)\,{\bf k}.$.


$4x - y - z = -4.$


2658   

Determine as derivadas parciais de $z=x^{2}\ln(1+x^{2}+y^{2})$.


$\displaystyle \frac{\partial z}{\partial x} = 2x\ln(1+ x^{2} + y^{2}) + \frac{2x^{3}}{1 + x^{2} + y^{2}}\;\;\;\;\;\;\text{e}\;\;\;\;\; \frac{\partial z}{\partial y} = \frac{2x^{2}y}{1 + x^{2} + y^{2}}.$


2863   

Estude com relação a máximos e mínimos a função dada com as restrições dadas.

$f(x,y) = x^2 + 4y^2$ e $xy = 1$, $x > 0$ e $y>0.$


Ponto de mínimo: $\displaystyle \left( \sqrt{2}, \frac{\sqrt{2}}{2} \right)$.


2368   

Em que direção e sentido a função dada cresce mais rapidamente no ponto dado? E em que direção e sentido decresce mais rapidamente?

$f(x,y) = \sqrt{4 - x^2 - 2y^2}$ em $\left(1,\dfrac{1}{2}\right)$.


 Cresce: $(-1,-1)$; descresce: $(1,1).$


3117   

Seja \(G\) a região sólida dentro da esfera de raio \(2\) centrada na origem e acima do plano \(z=1\). Mostre (ou verifique) os seguintes resultados:

  1.  O volume de \(G\) é dado por \[\iiint\limits_G\,dV = \int_0^{2\pi}\int_0^{\dfrac{\pi}{3}}\int_{\sec\phi}^{2}\rho^2\sin\phi\,d\rho d\phi d\theta \]

  2.  \[\iiint\limits_G\dfrac{z}{x^2+y^2+z^2}\,dV = \int_0^{2\pi}\int_0^{\pi/3}\int_{\sec\phi}^{2}\rho\cos\phi\sin\phi\,d\rho d\phi d\theta \]


2875   

Utilize os multiplicadores de Lagrange para determinar os valores máximo e mínimo da função sujeita à(s) restrição(ões) dada(s).

$f(x_1,x_2, \ldots, x_n) = x_1 + x_2 + \cdots + x_n; \quad x_1^2 + x_2^2 + \cdots + x_n^2 = 1.$


Valor máximo: $\sqrt{n};$ valor mínimo: $-\sqrt{n}.$


2090   

Considere o campo

$${\bf F}(x,y,z)=(e^{z},2yz, xe^{z}+y^{2}).$$

  1. Verifique se o campo ${\bf F}$ é conservativo.

  2. Se ${\bf F}$ for conservativo, calcule $f(x,y,z)$ tal que $\nabla f={\bf F}.$

  3. Calcule a integral de linha $\int_{C}{\bf F}\cdot d{\bf r}$ onde $C$ é dada por $r(t)=(\cos t, \sin t, t)$, $0\leq t\leq 2\pi.$


Sim.

$f(x,y) = x e^{z} + y^{2} z.$

$e^{2\pi} - 1.$


2386   

Determine as equações do plano tangente e da reta normal à superfície dada, no ponto dado.

$x^2 + 3y^2 + 4z^2 = 8$, em $(1,-1,1)$.


Plano tangente: $x - 3y + 4z = 8$
Reta normal: $(x,y,z) = (1,-1,1) + \lambda (2,-6,8),$ $\lambda \in \mathbb{R}.$


2989   

Calcule a integral, efetuando uma mudança de variáveis apropriada. $\displaystyle\iint\limits_{R} e^{x+y} \, dA$, em que $R$ é dada pela inequação $|x| + |y| \leq 1$.


$e - e^{-1}.$


2489   

Esboce o gráfico da função $f(x,y)=\sqrt{x^{2}+y^{2}}$.


$z = \sqrt{x^{2} + y^{2}}$

A expressão 1 cossec x e o mesmo que


2132   

Seja $g(t)=f(3t^{2},t^{3},e^{2t})$ e suponha $\dfrac{\partial f}{\partial z}(0,0,1)=4.$ 

  1.  Expresse $g^{'}(t)$ em termos das derivadas parciais de $f.$
  2. Calcule $g^{'}(0).$


  1. $\displaystyle g^{'}(t) = 6t \frac{\partial f}{\partial x}(3t^{2},t^{3},e^{2t}) +  3t^{2} \frac{\partial f}{\partial y}(3t^{2},t^{3},e^{2t}) + 2e^{2t} \frac{\partial f}{\partial z}(3t^{2},t^{3},e^{2t}).$ 
  2.  $g^{'}(0) = 8.$


2827   

Determine o valor máximo de $f(x,y)=x+5y$, onde $x$ e $y$ estão sujeitos às restrições: $5x+6y\leq 30$, $3x+2y\leq 12$, $x\geq 0$ e $y\geq 0.$


$25.$


2415   

Calcule a integral trocando a ordem de integração.

  1.  $\displaystyle\int_{0}^{4}\!\!\int_{\sqrt{x}}^{2}\dfrac{1}{y^{3}+1}\,dy dx$

  2.  $\displaystyle\int_{0}^{\pi}\!\!\int_{x}^{\pi}\dfrac{\sin{y}}{y}\,dy dx$

  3.  $\displaystyle\int_{0}^{2}\!\!\int_{x}^{2}2y^{2}\sin(xy)\,dy dx.$


  1.  $\dfrac{\ln(9)}{3}.$

  2.  $2.$

  3.  $4 - \sin(4).$


2041   

Utilize a Regra da Cadeia para determinar as derivadas parciais indicadas.
$u=\sqrt{r^{2}+s^{2}}$, $r=y+x\;\cos{t}$,  $s=x+y\;\sin{t}$;
$\dfrac{\partial u}{\partial x}$, $\dfrac{\partial u}{\partial y}$, $\dfrac{\partial u}{\partial t}$ quando $x=1$, $y=2$, $t=0$.


$\displaystyle \frac{\partial u}{\partial x} = \frac{4}{\sqrt{10}}$, $\displaystyle \frac{\partial u}{\partial y} = \frac{3}{\sqrt{10}}$, $\displaystyle \frac{\partial u}{\partial t}= \frac{2}{\sqrt{10}}.$


2377   

Mostre que a operação de calcular o gradiente de uma função tem a propriedade fornecida. Suponha que $u$ e $v$ sejam funções de $x$ e $y$, diferenciáveis, e $a$ e $b$ sejam constantes.

$\nabla (au + bv) = a \nabla u + b \nabla v$

$\nabla\left(\dfrac{u}{v}\right) = \dfrac{v \nabla u - u \nabla v}{v^2}$

$\nabla(uv) = u \nabla v + v \nabla u$

$\nabla u^n = nu^{n-1}\nabla u$


Pelas propriedades análogas para derivadas parciais e a linearidade de vetores, os quatro itens são válidos.


2692   

Encontre $f_{x}$, $f_{y}$ e $f_{z}$ para $f(x,y,z)=e^{-(x^{2}+y^{2}+z^{2})}$.


$\displaystyle f_{x} = -2xe^{-(x^{2} + y^{2} + z^{2})},\;\;\;\; f_{y} = -2ye^{-(x^{2} + y^{2} + z^{2})}\;\;\;\;\text{e}\;\;\;\; f_{z} = -2ze^{-(x^{2} + y^{2} + z^{2})}$.


2126   

Admita que, para todo $(x,y)$, 

$$4y\frac{\partial f}{\partial x}(x,y)-x\frac{\partial f}{\partial y}(x,y)=0.$$

Prove que $f$ é constante sobre a elipse $\dfrac{x^{2}}{4}+y^{2}=1.$



Note que $\displaystyle \frac{dz}{dt} \left(t \right) = 0,$ para $z = f(x,y),$ $x = t$ e $\displaystyle y = \pm \sqrt{1 - \frac{t^{2}}{4}}.$


2443   

Calcule a integral de superfície $\displaystyle\iint\limits_{S}y^{2}dS$, onde $S$ é a parte da esfera $x^{2}+y^{2}+z^{2}=4$ que está dentro
do cilindro $x^{2}+y^{2}=1$ e acima do plano $xy.$


$\pi\left( \dfrac{32}{3} - 6\sqrt{3}\right).$


2032   

Utilize a Regra da Cadeia para determinar $\mathrm{\partial}z/\mathrm{\partial} s$ e $\mathrm{\partial}z/ \mathrm{\partial}t.$

$z=\arcsin(x-y)$, $x=s^{2}+t^{2}$,  $y=1-2st$.


$\displaystyle \frac{\partial z}{\partial s} = \displaystyle \frac{\partial z}{\partial t} = \frac{2s + 2t}{\sqrt{1 - (x - y)^{2}}}$.


2830   

Determine três números positivos cuja soma é $100$ e cujo produto é máximo.


$\displaystyle x = y = z = \frac{100}{3}.$


2575   

Calcule $\displaystyle \lim_{(x,y) \to (0,0)} \dfrac{xy^2}{x^2 - y^2}$, caso exista.


Não existe.


2101   

Considere o campo vetorial

$${\bf F}(x,y)=(1+ye^{xy})\,{\bf i}+(2y+xe^{xy})\,{\bf j}.$$

  1. Determine se ${\bf F}$ é ou não um campo conservativo. Em caso afirmativo, encontre uma função potencial para ${\bf F}.$

  2. Determine o trabalho realizado pelo campo vetorial ${\bf F}$ ao mover uma partícula sobre a hipérbole $x^{2} - y^{2} = 1,$ desde o ponto $(3,-\sqrt{8})$ até o ponto $(3,\sqrt{8}).$


  1. Sim. Função potencial: $f(x,y) = x + e^{xy} + y^{2}.$

  2. $e^{3\sqrt{8}} - e^{-3\sqrt{8}}.$


2998   

Seja $f$ uma função contínua em $[0,1]$ e seja $R$ a região triangular com vértices $(0,0), (1,0)$ e $(0,1)$. Mostre que

$$\iint\limits_{R} f(x,y) \, dA = \int_0^1 uf(u) \, du.$$


Utilize a mudança de variáveis $u = x + y$ e $v = y.$


2855   

Determine os pontos do gráfico de $xy^{3}z^{2}=16$ mais próximos da origem.


$\displaystyle \left( \frac{2}{\sqrt[4]{12}}, \sqrt[4]{12}, \frac{2 \sqrt{2}}{\sqrt[4]{12}}\right),$ $\displaystyle \left( \frac{2}{\sqrt[4]{12}}, \sqrt[4]{12}, - \frac{2 \sqrt{2}}{\sqrt[4]{12}}\right),$ $\displaystyle \left( -\frac{2}{\sqrt[4]{12}}, \sqrt[4]{12}, \frac{2 \sqrt{2}}{\sqrt[4]{12}} \right)$ e $\displaystyle \left( -\frac{2}{\sqrt[4]{12}}, \sqrt[4]{12}, - \frac{2 \sqrt{2}}{\sqrt[4]{12}} \right).$


3043   

Calcule

$$\oint_{C} \dfrac{-y}{x^2+y^2} \, dx + \dfrac{x}{x^2 + y^2} \, dy,$$

em que $C$ é a curva

A expressão 1 cossec x e o mesmo que



Podemos escrever $C$ como $C_1 \cup C_2$, em que $C_1$ e $C_2$ são as curvas dadas abaixo.

A expressão 1 cossec x e o mesmo que

Seja $A$ um aberto simplesmente conexo que contém $C_1$ e não contém a origem. O campo $\mathbf{F}$ restrito a $A$ é conservativo, pois $A$ é aberto e simplesmente conexo, $P(x,y) = \dfrac{-y}{x^2 + y^2}$ e $Q(x,y) = \dfrac{x}{x^2 + y^2}$ possuem derivadas de primeira ordem contínuas em $A$ e $P$ e $Q$ satisfazem a relação $\frac{\partial Q}{\partial x} = \frac{\partial P}{\partial y}$. Então,
$$\oint_{C_1}\mathbf{F} \cdot\, d\mathbf{r} = 0.$$
Não podemos proceder de maneira análoga em $C_2$, já que todo aberto $B$ que contém a curva $C_2$ e não contém a origem não será simplesmente conexo. Com isso, não conseguimos garantir que o campo $\mathbf{F}$ restrito a $B$ é conservativo (observe que, a princípio, não podemos afirmar que o campo é não conservativo).
A ideia para contornar esse problema é ``isolar" a origem com uma curva fechada $C_3$, a princípio arbitrária. Vamos escolher essa curva $C_3$ de maneira conveniente para que consigamos resolver o problema. Seja $\varepsilon > 0$ pequeno o suficiente para que a curva $C_3$ parametrizada por $r(t) = (\varepsilon \cos{t}, \varepsilon \sin{t})$, com $t$ variando de $2\pi$ a $0$, não intercepte a curva $C_2$ e esteja entre a curva $C_2$ e a origem.

A expressão 1 cossec x e o mesmo que

Considere $D_1 = \{(x,y) \in \mathbb{R}^2: (x,y) \mbox{ está entre } C_2 \mbox{ e } C_3 \mbox{ e } y \geq 0\}$ e $D_2 = \{(x,y) \in \mathbb{R}^2: (x,y) \mbox{ está entre } C_2 \mbox{ e } C_3 \mbox{ e } y \leq 0\}$. As curvas que delimitam $D_1$ e $D_2$ são $C_{D_1}= C_{2}^+\cup C_{a}\cup C_{3}^+\cup C_{b}$ e $C_{D_2}=C_{2}^-\cup -C_{b}\cup C_{3}^- \cup -C_{a}$, respectivamente, e estão ilustradas a seguir.

A expressão 1 cossec x e o mesmo que


Note que
$$\oint_{C_{D_1}} \mathbf{F} \cdot d\mathbf{r} = \int_{C^+_2} \mathbf{F} \cdot d\mathbf{r} + \int_{C_a} \mathbf{F} \cdot d\mathbf{r} + \int_{C^+_3} \mathbf{F} \cdot d\mathbf{r} + \int_{C_b} \mathbf{F} \cdot d\mathbf{r} \qquad (\star)$$
e
$$\oint_{C_{D_2}} \mathbf{F} \cdot d\mathbf{r} = \int_{C^-_2} \mathbf{F} \cdot d\mathbf{r} + \int_{-C_a} \mathbf{F} \cdot d\mathbf{r} + \int_{C^-_3} \mathbf{F} \cdot d\mathbf{r} + \int_{-C_b} \mathbf{F} \cdot d\mathbf{r}  \qquad (\star \star).$$
Como $\frac{\partial Q}{\partial x} = \frac{\partial P}{\partial y}$, temos, pelo Teorema de Green,
$$\displaystyle\oint_{C_{D_1}} \mathbf{F} \cdot d\mathbf{r} = \iint\limits_{D_1} 0 \, dA = 0$$
e
$$\displaystyle\oint_{C_{D_2}} \mathbf{F} \cdot d\mathbf{r} = \iint\limits_{D_2} 0 \, dA = 0.$$
Somando as equações ($\star$) e ($\star \star$), obtemos
$$\int_{C^+_2} \mathbf{F} \cdot d\mathbf{r} + \int_{C^+_3} \mathbf{F} \cdot d\mathbf{r} + \int_{C^-_2} \mathbf{F} \cdot d\mathbf{r} + \int_{C^-_3} \mathbf{F} \cdot d\mathbf{r} = 0,$$
isto é,
$$\int_{C_2} \mathbf{F} \cdot d\mathbf{r} = -\int_{C_3} \mathbf{F} \cdot d\mathbf{r} = \int_{-C_3} \mathbf{F} \cdot d\mathbf{r}.$$
Assim, basta determinar $\int_{-C_3} \mathbf{F} \cdot d\mathbf{r}$. A parametrização de $-C_3$ é $r(t) = (\varepsilon \cos{t}, \varepsilon \sin{t})$, com $t$ variando de $0$ a $2\pi$. Daí,
$$\begin{array}{rcl}\displaystyle \int_{-C_3} \mathbf{F} \cdot d\mathbf{r} & = & \displaystyle \int_{0}^{2\pi} \left(\frac{-\varepsilon \sin{t}}{\varepsilon^2},\frac{\varepsilon \cos{t}}{\varepsilon^2}\right) \cdot (-\varepsilon \sin{t}, \varepsilon \cos{t}) \, dt \\& = & \displaystyle \int_{0}^{2\pi} 1 \, dt =  2\pi.\end{array}$$
Portanto,
$$\oint_{C} \mathbf{F} \cdot d\mathbf{r} = \oint_{C_1} \mathbf{F} \cdot d\mathbf{r} + \oint_{C_2} \mathbf{F} \cdot d\mathbf{r} = 0 + 2\pi = 2\pi.$$


2235   

Seja $\mathbf{r} = x\mathbf{i} + y\mathbf{j} + z\mathbf{k}$ e $r=|\mathbf{r}|$. Verifique a identidade $\nabla \times \mathbf{r} = \mathbf{0}$.


$\nabla \times \mathbf{r} = \left[\dfrac{\partial}{\partial y} (z) - \dfrac{\partial}{\partial z}(y) \right]\mathbf{i} + \left[\dfrac{\partial}{\partial z} (x) - \dfrac{\partial}{\partial x}(z) \right]\mathbf{j} + \left[\dfrac{\partial}{\partial x} (y) - \dfrac{\partial}{\partial y}(x) \right]\mathbf{k}.$ (Note que: $r = \sqrt{x^{2} + y^{2} + z^{2}}.$)


2395   

Seja $g(x,y) = f(x^2 + y^2)$, em que $f$ é uma função diferenciável. Sabendo que $f'(2) = 1$, determine a equação da reta tangente à curva de nível de $g$ que passa pelo ponto $(1,1)$.


 $x + y = 2.$


3130   

Use o Teorema de Green para determinar o trabalho realizado pelo campo de forças \(\displaystyle\textbf{F}(x,y)=xy\textbf{i}+(\dfrac{1}{2}x^2+xy)\textbf{j}\) sobre uma partícula que se move ao longo do caminho que começa em \((5,0)\), percorre o semicírculo superior \(x^2+y^2=25\) e retorna ao seu ponto de partida ao longo do eixo \(x\).


\(\dfrac{250}{3}\)


2010   

Um homem pesando $160$ lb carrega uma lata de tinta de $25$ lb por uma escada helicoidal em torno de um silo com raio de $20$ pés. Se o silo tem $90$ pés de altura e o homem dá três voltas completas em torno do silo, quanto trabalho é realizado pelo homem contra a gravidade para subir ao topo?


$16650$ ft-lb.


2710   

Determine o plano que é paralelo ao plano $z = 2x + 3y$ e tangente ao gráfico de $f(x,y) = x^2 + xy$.


Considere

$$z-f(x_{0},y_{0})=\frac{\partial f}{\partial x}(x_{0},y_{0})(x-x_{0})+\frac{\partial f}{\partial y}(x_{0},y_{0})(y-y_{0})$$

o plano tangente ao gráfico de $f$. Assim,

$$z=\frac{\partial f}{\partial x}(x_{0},y_{0})\cdot x+\frac{\partial f}{\partial y}(x_{0},y_{0})\cdot y+\bigg[ f(x_{0},y_{0})-\frac{\partial f}{\partial x}(x_{0},y_{0})\cdot x_{0}-\frac{\partial f}{\partial y}(x_{0},y_{0})\cdot y_{0}\bigg].$$

Como tal plano é paralelo ao plano $z=2x+3y$, obtemos que

$$\frac{\partial f}{\partial x}(x_{0},y_{0})=2\;\;\;\;\;\;\; \mbox{e}\;\;\;\;\;\;\; \frac{\partial f}{\partial y}(x_{0},y_{0})=3.$$

Notemos que

$$\frac{\partial f}{\partial x}(x,y)=2x+y\;\;\;\;\;\;\; \mbox{e} \;\;\;\;\;\; \frac{\partial f}{\partial y}(x,y)=x.$$

Assim, temos o seguinte sistema de equações

$$\left \{\begin{array}{cc}2x_{0}+y_{0}=2 \\x_{0}=3\\\end{array}\right.$$

Logo, $x_{0}=3$ e $y_{0}=-4.$ A partir desses valores temos que $f(x_{0},y_{0})=-3$, $\dfrac{\partial f}{\partial x}(x_{0},y_{0})\cdot x_{0}=6$ e

$\dfrac{\partial f}{\partial y}(x_{0},y_{0})\cdot y_{0}=-12.$ Portanto, o plano desejado tem equação

$$z=2x+3y-3-6+12,$$

ou seja,

$$z=2x+3y+3.$$


2248   

Seja

$$\mathbf{F}(x,y) = \dfrac{x}{(x^2+y^2)^5}\mathbf{i} + \dfrac{y}{(x^2+y^2)^5}\mathbf{j}$$

e $\mathbf{n}$ a normal unitária exterior ao círculo $x^2 + y^2 \leq 1$. Calcule $\int_{C} \mathbf{F} \cdot \mathbf{n} \, ds$, em que $C$ é dada por $\mathbf{r}(t) = (\cos{t},\sin{t})$, $0 \leq t \leq \pi$. (Sugestão: Verifique que $\mathbf{F} \cdot \mathbf{n}$ é constante.)


$\pi.$


2661   

Determine as derivadas parciais de $z=\arctan \dfrac{x}{y}$.


$\displaystyle \frac{\partial z}{\partial x} = \frac{y}{x^{2} + y^{2}}\;\;\;\;\;\;\text{e}\;\;\;\;\; \frac{\partial z}{\partial y} = \frac{-x}{x^{2} + y^{2}}.$


3046   

O campo vetorial $\mathbf{F}$ é mostrado no plano $xy$ e é o mesmo em todos os planos horizontais (em outras palavras, $\mathbf{F}$ é independente de $z$ e sua componente $z$ é 0).

  1. O $\text{div }{\mathbf{F}}$ será positivo, negativo ou nulo? Justifique.

  2. Determine se o $\text{rot }{\mathbf{F}} = 0$. Se não, em que direção rot $\mathbf{F}$ aponta?

A expressão 1 cossec x e o mesmo que


  1. Negativo.

  2. $\text{rot } \bf{F} = \bf{0}.$


2284   

Determine se os pontos $P(3,-1,5)$ e $Q(-1,3,4)$ estão na superfície ${\bf r}(u,v)=(u+v,u^{2}-v,u+v^{2})$.


$P$ está na superfície; $Q$ não está na superfície.


2180   

Se $\mathbf{F}(x,y) = (-y\mathbf{i} + x\mathbf{j})/(x^2+y^2)$, mostre que $\int_{C}\mathbf{F} \cdot d\mathbf{r} = 0$ para todo caminho fechado simples que não passe pela origem e nem a circunde.


Dica: como $C$ é um caminho fechado simples que não passa pela origem e não circunda a origem, então existe uma região aberta $A$ que ainda não contém a origem, mas contém $D,$ a região limitada por $C.$ Em $A,$ tanto $-y/(x^{2} + y^{2})$ quanto $x/(x^{2} + y^{2})$ possuem derivadas parciais contínuas e podemos aplicar o Teorema de Green.


2687   

Encontre $f_{x}$, $f_{y}$ e $f_{z}$ para $f(x,y,z)=x-\sqrt{y^{2}+z^{2}}$.


$\displaystyle f_{x} = 1,\;\;\;\; f_{y} = -\frac{y}{\sqrt{y^{2} + z^{2}}}\;\;\;\;\text{e}\;\;\;\; f_{z} =  -\frac{z}{\sqrt{y^{2} + z^{2}}}$.


2159   

Calcule a integral de linha $\displaystyle\oint_{C} xy \, dx + x^2y^3 \, dy$, $C$ é o triângulo com vértices $(0,0)$, $(1,0)$ e $(1,2)$ por dois métodos:

  1. diretamente; e

  2. utilizando o Teorema de Green.


$\dfrac{2}{3}.$


2307   

Calcule $D_{\bf{u}}f(x_0,y_0)$, sendo dados

$f(x,y) = x^2 - 3y^2$, $(x_0,y_0) = (1,2)$ e $\bf{u}$ o versor de $2\bf{i} + \bf{j}.$


$\displaystyle D_{\bf{(2,1)}}f(1,2) = -\frac{8}{5}.$ 


3041   

Faça uma correspondência entre as equações e os gráficos identificados a seguir,  enumerador respectivamente por $I-VI$, e justifique sua resposta. Determine quais famílias de curvas da grade têm $u$ constante e quais têm $v$ constante.

  1. ${\bf r}(u,v)=u\cos v{\bf i}+u\sin v{\bf j}+v{\bf k}.$

  2. ${\bf r}(u,v)=u\cos v{\bf i}+u\sin v{\bf j}+\sin u{\bf k}$, $-\pi\leq u\leq \pi.$

  3. ${\bf r}(u,v)=\sin v{\bf i}+\cos u\sin 2v{\bf j}+\sin u\sin 2v{\bf k}.$

  4. $x=(1-u)(3+\cos v)\cos 4\pi u$, $y=(1-u)(3+\cos v)\sin 4\pi u$,$z=3u+(1-u)\sin v.$

  5. $x=\cos^{3}u\cos^{3}v$,  $y=\sin^{3}u\cos^{3}v$, $z=\sin^{3}v.$

  6. $x=(1-|u|)\cos v$, $y=(1-|u|)\sin v$, $z=u.$

A expressão 1 cossec x e o mesmo que

A expressão 1 cossec x e o mesmo que

A expressão 1 cossec x e o mesmo que

A expressão 1 cossec x e o mesmo que

A expressão 1 cossec x e o mesmo que

A expressão 1 cossec x e o mesmo que


  1. IV.
  2. I.
  3. II.
  4. V.
  5. III.
  6. VI


2411   

Inverta a ordem de integração.

  1.  $\displaystyle\int_{0}^{\dfrac{\pi}{4}}\bigg[\int_{\sin{x}}^{\cos{x}}f(x,y)\,dy\bigg]dx$

  2.  $\displaystyle\int_{-1}^{2}\bigg[\int_{\sqrt{\frac{7+5y^{2}}{3}}}^{\frac{y+7}{3}}f(x,y)\,dx\bigg]dy$

  3.  $\displaystyle\int_{0}^{3}\bigg[\int_{x^{2}-2x}^{\sqrt{3x}}f(x,y)\,dy\bigg]dx$


  1.    $\displaystyle \int_{0}^{\frac{\sqrt{2}}{2}}\bigg[\int_{0}^{\arcsin{y}}f(x,y)\,dx\bigg]dy + \int_{\frac{\sqrt{2}}{2}}^{1}\bigg[\int_{0}^{\arccos{y}}f(x,y)\,dx\bigg]dy$

  2.  $\displaystyle\int_{2}^{3}\bigg[\int_{3x - 7}^{\sqrt{\frac{3x^2 - 7}{5}}}f(x,y)\,dy\bigg]dx$

  3.  $\displaystyle\int_{-1}^{0}\bigg[\int_{1 - \sqrt{1 + y}}^{1 + \sqrt{1 + y}}f(x,y)\,dx\bigg]dy + \int_{0}^{3}\bigg[\int_{\dfrac{y^{2}}{3}}^{1 + \sqrt{1 + y}}f(x,y)\,dx\bigg]dy$


2815   

Determine os valores máximo e mínimo absolutos de $f$ no conjunto $D.$

$f(x,y)=2x^{3}+y^{4}$, $D=\{(x,y) \in \mathbb{R}^2: x^{2}+y^{2}\leq 1\}.$


Valor máximo: $2;$  valor mínimo: $-2.$


2625   

Utilizando o Teorema de Stokes, transforme a integral $\displaystyle\iint_{ S}\mbox{rot}{\bf F}\cdot{\bf n}dS$ numa integral de linha e calcule.

  • ${\bf F}(x,y,z) = y{\bf i} + x^2{\bf j} + z{\bf k}$, $S$ a superfície $x^2+y^2 = 1$, $0\leq z \leq 1$ e $y\geq 0$, sendo ${\bf n}$ a normal com componente $y\geq 0$.


$0$.


2806   

Determine os valores máximos e mínimos locais e pontos de sela da função $f(x,y)=x^{2}-4xy+4y^{2}-x+3y+1$.


Não há pontos críticos.


2915   

Um sólido está acima do cone $z=\sqrt{x^{2}+y^{2}}$ e abaixo da esfera $x^{2}+y^{2}+z^{2}=z.$ Escreva uma descrição do sólido em termos de desigualdades envolvendo coordenadas esféricas.



A mudança de coordenadas retangulares para coordenadas cartesianas é dada por

$$\begin{cases}x = \rho \cos{\theta} \sin{\phi} \\y = \rho \sin{\theta} \sin{\phi}\\z = \rho \cos{\phi},\end{cases}$$

em que $\rho \geq 0$, $\theta \in [0,2\pi]$ e $\phi \in [0,\pi]$. Observe que $\sin{\phi} \geq 0$ quando $\phi \in [0,\pi]$. Logo, a equação do cone em coordenadas esféricas pode ser escrita como $\rho \cos{\phi} = \sqrt{\rho^2 \sin^2{\phi}} = \rho\sin{\phi}$. A origem $(0,0,0)$ pertence ao cone e é dada por $\rho = 0$. Nos demais pontos, $\rho \neq 0$, donde $\phi = \pi/4$.

A equação da esfera em coordenadas esféricas pode ser escrita como $\rho^2=\rho\cos{\phi}$. A origem $(0,0,0)$ pertence à esfera e é dada por $\rho=0$. Nos demais pontos, $\rho \neq 0$, donde $\rho = \cos{\phi}$.

Portanto, o sólido pode ser descrito em coordenadas esféricas por

$$E = \left\{(\rho, \theta, \phi): 0 \leq \rho \leq \cos{\phi}, 0 \leq \theta \leq 2\pi \mbox{ e } 0 \leq \phi \leq \frac{\pi}{4}\right\}.$$

A expressão 1 cossec x e o mesmo que


2461   

Calcule a integral de superfície $\displaystyle\iint \limits_{ S}{\bf F}\cdot d{\bf S}$ para o campo vetorial ${\bf F}$ e superfície orientada $S$ dados abaixo. Em outras palavras, determine o fluxo de ${\bf F}$ através de $S$. Para superfícies fechadas, use a orientação positiva (para fora).

  • ${\bf F}(x,y,z)=xy{\bf i}+yz{\bf j}+zx{\bf k}$ e $S$ é a parte do parabolóide $z=4-x^{2}-y^{2}$ que está acima do quadrado $0\leq x\leq 1$, $0\leq y\leq 1$, com orientação para cima.


$\dfrac{713}{180}.$


2790   

Determine os valores máximos e mínimos locais e pontos de sela da função $f(x,y)=x^{2}+3xy+4y^{2}-6x+2y$.


Ponto de mínimo: $\displaystyle \left( \frac{54}{7}, -\frac{22}{7} \right).$


2824   

Determine a menor distância entre o ponto $(2,1,-1)$ e o plano $x+y-z=1$.


$\sqrt{3}.$



A distância entre um ponto qualquer $(x,y,z)$ e o ponto $(2,1,-1)$ é \[ d=\sqrt{(x-2)^2+(y-1)^2+(z+1)^2} \] mas, se $(x,y,z)$ pertence ao plano $x+y-z=1$, então $z=x+y-1$, e assim temos \[ d=\sqrt{(x-2)^2+(y-1)^2+(x+y)^2}. \] Podemos minimizar $d$ minimizando a expressão mais simples \[ d^2=f(x,y)=(x-2)^2+(y-1)^2+(x+y)^2. \] Vamos encontrar os pontos críticos de $f$. Temos que $f_x(x,y)=2(x-2)+2(x+y)=4x+2y-4$ e $f_y(x,y)=2(y-1)+2(x+y)=4y+2x-2$. Fazendo $f_x=0$ obtemos \[ 4x+2y-4=0\Rightarrow 2x+y-2=0 \Rightarrow y=2-2x. \] Agora, fazendo $f_y=0$ e substituindo $y=2-2x$ obtemos: \[ 4y+2x-2=0\Rightarrow 4(2-2x)+2x-2=0\Rightarrow 8-8x+2x-2=0 \] \[ \Rightarrow -6x+6=0 \Rightarrow x=1 \] e, portanto, $y=2-2x=2-2\cdot 1=0$ é o único ponto crítico de $f$. Note que \[ f_{xx}(1,0)=4, f_{xy}(1,0)=2 \text{ e } f_{yy}(1,0)=4 \] assim $f_{xx}(1,0)f_{yy}(1,0)-(f_{xy}(1,0)) ^{2}=4\cdot 4-2^{2}=14-4=12>0$ e $f_{xx}(1,0)>0$. Portanto, pelo Teste da Derivada Segunda, $f$ tem um mínimo em $(1,0)$. Assim, a distância menor distância entre o ponto $(2,1,-1)$ e o plano $x+y-z=1$ é \[ d=\sqrt{(1-2)^2+(0-1)^2+(1+0)^2}=\sqrt{3}.

\]


2543   

Encontre o fluxo exterior do campo ${\bf F}(x,y,z)=z^{2}{\bf i}+x{\bf j}-3z{\bf k}$ através da superfície cortada do cilindro parabólico $z=4-y^{2}$ pelos planos $x=0$, $x=1$ e $z=0.$


$-32.$


2809   

Determine os valores máximos e mínimos locais e pontos de sela da função $f(x,y)=4+x^{3}+y^{3}-3xy$.


Ponto de mínimo: $(1,1);$ ponto de sela: $(0,0).$


3062   

Esboce o campo vetorial $\textbf{F}= \dfrac{y\textbf{i} + x\textbf{j}}{\sqrt{x^2+y^2}}$, desenhando um diagrama.


2813   

Determine os valores máximo e mínimo absolutos de $f$ no conjunto $D.$

$f(x,y)=x^{2}+y^{2}+x^{2}y+4$, $D=\{(x,y) \in \mathbb{R}^2: |x|\leq 1, \; |y|\leq 1\}.$


Valor máximo: $7;$ valor mínimo: $4.$


2568   

Calcule $\displaystyle \lim_{(x,y) \to (0,0)} x \ \sin{\dfrac{1}{x^2 + y^2}}$, caso exista.


$0.$


2356   

Calcule o volume do conjunto dado.

  1.  $x^{2}+y^{2}\leq z\leq 2x.$

  2.  $x\leq z\leq1-y^{2}$ e $x\geq 0.$


  1.  $\dfrac{\pi}{2}.$

  2.  $\dfrac{8}{15}.$


2108   

Sejam $f(x)$ e $g(x)$ duas funções contínuas, respectivamente, nos intervalos $[a,b]$ e $[c,d].$ Use o seguinte resultado $$\iint\limits_{R}f(x)g(y)\,dx dy=\bigg(\int_{a}^{b}f(x)\,dx\bigg)\bigg(\int_{c}^{d}g(y)\,dy\bigg),$$ onde $R$ é o retângulo $a\leq x\leq b$ e $c\leq y\leq d$, para calcular as integrais

  1.  $\displaystyle\int\!\!\!\!\int\limits_{\!\!\!\!\!\! R} xy^{2}\,dx dy$, onde $R$ é o retângulo $1\leq x\leq 2,\;2\leq y\leq 3.$

  2.  $\displaystyle\int\!\!\!\!\int\limits_{\!\!\!\!\!\! R} x\cos(2y)\,dx dy$, onde $R$ é o retângulo $0\leq x\leq 1,\;-\dfrac{\pi}{4}\leq y\leq \dfrac{\pi}{4}.$


  1.  $\dfrac{19}{2}.$

  2.  $\dfrac{1}{2}.$


2007   

A força em um ponto $(x,y)$ de um plano coordenado é ${\bf F}(x,y)=(x^{2}+y^{2})\,{\bf i}+xy\,{\bf j}$. Ache o trabalho realizado por ${\bf F}(x,y)$ ao longo do gráfico de $y=x^{3}$ de $(0,0)$ a $(2,8).$


$\dfrac{1592}{21}.$


2978   

Determine a imagem do conjunto $S$ sob a transformação dada. $S = \{(x,y) \in \mathbb{R}^2: 0 \leq u \leq 3, \, 0 \leq v \leq 2\}$;$x = 2u + 3v$, $y = u - v$.


O paralelogramo com vértices $(0,0),$ $(6,3),$ $(12,1),$ $(6,-2).$


3031   

No cálculo de uma integral dupla sobre uma região $D$, obtivemos uma soma de integrais iteradas como a que segue:

$$\int\!\!\!\!\int\limits_{\!\!\!\!\!\! D} \! f(x,y)\,dA=\int_{0}^{1}\!\!\int_{0}^{2y} \! f(x,y)\,dx dy+\int_{1}^{3}\!\!\int_{0}^{3-y} \! f(x,y)\,dx dy.$$

Esboce a região $D$ e expresse a integral dupla como uma integral iterada com ordem de integração contrária.


$\displaystyle \int_{0}^{2}\!\!\int_{\frac{x}{2}}^{3-x} \! f(x,y)\,dx dy.$

A expressão 1 cossec x e o mesmo que


2006   

Determine o trabalho realizado pelo campo de força ${\bf F}(x,y)=x\,{\bf i}+(y+2)\,{\bf j}$ sobre um objeto que se move sobre um arco de cicloide ${\bf r}(t)=(t-\sin t)\,{\bf i}+(1-\cos t)\,{\bf j}$, $0\leq t\leq 2\pi.$


$2\pi^{2}.$


2141   

Verifique que o Teorema do Divergente é verdadeiro para o campo vetorial ${\bf F}$ na região $E.$

${\bf F}(x,y,z)=3x\,{\bf i}+xy\,{\bf j}+2xz\,{\bf k}$, $E$ é o cubo limitado pelos planos $x=0$, $x=1$, $y=0$, $y=1$,  $z=0$ e $z=1.$


$\displaystyle\iint_{S} {\bf F} \cdot d{\bf S} = \iiint_{E} \mbox{div} {\bf F} dV = \dfrac{9}{2}.$


2756   

Verifique que a função $f(x,y) = x^4 + y^3$ é diferenciável.


As derivadas parciais $\frac{\partial f}{\partial x}$ e $\frac{\partial f}{\partial y}$ de cada função $f$ existem e são contínuas em todos os pontos do domínio.


2921   

Utilize a integral dupla para determinar a área da região: dentro da cardióide $r=1+\cos{\theta}$ e fora do círculo $r=3\cos{\theta}.$


$\displaystyle \frac{\pi}{4}.$


2269   

Determine uma equação do plano tangente à superfície parametrizada dada no ponto especificado. ${\bf r}(u,v)=u^{2}\,{\bf i}+2u\,\sin v\,{\bf j}+u\,\cos v\,{\bf k}$; $u=1$, $v=0.$



Temos que ${\bf r}(u,v)=\underbrace{u^{2}}_{x(u,v)}\,{\bf i}+\underbrace{2u\,\sin v}_{y(u,v)}\,{\bf j}+\underbrace{u\,\cos v}_{z(u,v)}\,{\bf k}$

Primeiro, vamos calcular os vetores tangentes:

$$\begin{array}{rcl}{\bf r}_{u}&=&\frac{\partial x(u,v)}{\partial u}\,{\bf i}+\frac{\partial y(u,v)}{\partial u}\,{\bf j}+\frac{\partial z(u,v)}{\partial u}\,{\bf k}\\&=& 2u\,{\bf i}+2\,\sin v\,{\bf j}+\cos v\,{\bf k}\end{array}$$

e

$$\begin{array}{rcl}{\bf r}_{v}&=&\frac{\partial x(u,v)}{\partial v}\,{\bf i}+\frac{\partial y(u,v)}{\partial v}\,{\bf j}+\frac{\partial z(u,v)}{\partial v}\,{\bf k}\\&=& 0\,{\bf i}+2u\,\cos v\,{\bf j}-u\sin v\,{\bf k}\end{array}$$


Assim, o vetor normal ao plano tangente é:

$$\begin{array}{rcl}{\bf r}_{u}\times {\bf r}_{v}&=&\left|\begin{array}{ccc}{\bf i} & {\bf j} & {\bf k}\\2u & 2\sin v & \cos v\\0 & 2u\cos v & -u\sin v\\\end{array}\right|\\&=&(-2u\,\sin^{2}v-2u\cos^{2}v)\,{\bf i}+(2u^{2}\,\sin v)\,{\bf j}+(4u^{2}\,\cos v)\,{\bf k}\end{array}$$


Como $u=1$ e $v=0$ temos que o vetor normal é $-2\,{\bf i}+0\,{\bf j}+4\,{\bf k}.$

Portanto, uma equação do plano tangente no ponto ${\bf r}(1,0)=(1,0,1)$ é

$$-2\cdot(x-1)+0\cdot(y-0)+4\cdot (z-1)=0$$

$$-2x+2+4z-4=0$$

$$-2x+4z-2=0    \mbox{ou}     x-2z+1=0$$


2642   

Determine as derivadas parciais de primeira ordem da função $u=x^{y/z}$.


$\displaystyle \frac{\partial u}{\partial x} = \frac{y}{z} x^{(y/z) - 1},\;\;\; \frac{\partial u}{\partial y} = x^{y/z} \ln x \;\;\;\text{e}\;\;\; \frac{\partial u}{\partial z} = - \frac{yx^{y/z}}{z^{2}} \ln x$.


2979   

Determine a imagem do conjunto $S$ sob a transformação dada. $S$ é o quadrado limitado pelas retas $u = 0$, $u = 1$, $v = 0$, $v = 1$;$x = v$, $y = u(1+v^2)$.


A região limitada por $y = 1 + x^2,$ pelo eixo $x$ e pelas retas $x = 0$ e $x = 1.$


2888   

Encontre os pontos da curva $x^2 - 6xy - 7y^2 + 80 = 0$ mais próximos da origem. Desenhe a curva.


$(1,3)$ e $(-1,-3).$ Realizando a mudança de coordenadas $x = \frac{1}{\sqrt{10}} u - \frac{3}{\sqrt{10}} v$ e $y = \frac{3}{\sqrt{10}} u + \frac{1}{\sqrt{10}} v,$ a equação da curva inicial é transformada em $\frac{u^{2}}{10} - \frac{v^{2}}{40} = 1,$.


2860   

Estude com relação a máximos e mínimos a função dada com as restrições dadas.

$f(x,y) = 3x + y$ e $x^2 + 2y^2 = 1.$


Ponto de máximo: $\displaystyle \left( \frac{6}{\sqrt{38}}, \frac{1}{\sqrt{38}} \right)$; ponto de mínimo: $\displaystyle \left( -\frac{6}{\sqrt{38}}, -\frac{1}{\sqrt{38}} \right)$.


2256   

Demonstre a identidade abaixo, supondo que $S$ e $E$ satisfaçam as condições do Teorema do Divergente e que as funções escalares e as componentes dos campos vetoriais tenham derivadas parciais de segunda ordem contínuas.

  1. $\displaystyle\iint\limits_{S}(f\nabla g-g\nabla f)\cdot {\bf n}\,dS=\displaystyle\iiint\limits_{E}(f\nabla^{2} g-g\nabla^{2} f)\,dV.$



Use o Teorema da Divergência e que $\nabla f \cdot \nabla g = \nabla g \cdot \nabla f.$


2291   

Determine uma representação paramétrica para a superfície descrita a seguir. A parte do cilindro $y^{2}+z^{2}=16$ que está entre os planos $x=0$ e $x=5.$


$x = u,$ $y = 4\cos (\theta),$ $z = 4\sin(\theta),$ onde $0 \leq u \leq 5,$ $0 \leq \theta \leq 2\pi.$


2361   

Considere a superfície parametrizada por

$${\bf r}(u,v)=(uv,u+v,u-v).$$

  1. Determine o valor de $c$ de forma que o ponto $(c,1,0)$ pertença à superfície.

  2. Calcule a área da parte da superfície correspondente à variação $u^{2}+v^{2}\leq 1.$


  1. $\dfrac{1}{4}.$

  2. $\left(\sqrt{6} - \dfrac{4}{3} \right)2\pi.$


2725   

O comprimento e a largura de um retângulo foram medidos como $30$ cm e $24$ cm, respectivamente, com um erro de medida de, no máximo, $0,1$ cm. Utilize as diferenciais para estimar o erro máximo cometido no cálculo da área do retângulo.


$\Delta A \approx 5.4$ cm$^{2}$.


1944   

Calcule a integral de linha, onde $C$ é a curva dada.

$\displaystyle\int_{C}x\,dx+dy+2\,dz$, $C$ é a interseção do paraboloide $z=x^{2}+y^{2}$ com o plano $z=2x+2y-1$; caminhe no sentido anti-horário.


$0.$


2438   

Calcule a integral de superfície $\displaystyle\iint\limits_{S}yz dS$, onde $S$ é a parte do plano $x+y+z=1$ que está no primeiro octante.


$\dfrac{\sqrt{3}}{24}.$


2608   

Use o Teorema de Stokes para calcular $\displaystyle\iint\limits_{ S}\mbox{rot}{\bf F}\cdot d{\bf S}.$

  • ${\bf F}(x,y,z) = (e^{xy}\cos{z},(x^2+1)z,-y)$, $S$ é o hemisfério $x^2+y^2+z^2 = 1$, $x \geq 0$, orientado na direção positiva do eixo $x$.


$-2\pi$.


2821   

Determine os valores máximo e mínimo absolutos de $f$ no conjunto $D.$

$f(x,y)=xy$ em $D=\{(x,y) \in \mathbb{R}^2: x\geq 0,\;y\geq 0\;\text{e}\;2x+y\leq 5\}.$


Valor máximo: $\displaystyle \frac{25}{8};$ valor mínimo: $\displaystyle 0.$


2330   

Determine a taxa de variação máxima de $f$ no ponto dado e a direção em que isso ocorre.
$f(x,y) = \dfrac{y^2}{x},  (2,4).$


$4\sqrt{2}.$


3075   

  1.  Esboce o gráfico da função $f(x,y)=e^{-(x^2+y^2)}$.

  2.  Descreva em palavras como o gráfico da função \(\displaystyle g(x,y)= e^{-a(x^2+y^2)}\) está relacionado com o gráfico de \(f\), sendo \(a>0\). Mostre (verifique) que o valor de \(a\) influencia na "largura" do pico presente no gráfico da função.


2485   

Use a integral tripla para determinar o volume do sólido dado.

  1.  O tetraedro limitado pelos planos coordenados e o plano $2x+y+z=4.$

  2.  O sólido limitado pelo paraboloide $x=y^{2}+z^{2}$ e pelo plano $x=16.$

  3.  O sólido delimitado pelo cilindro $x=y^{2}$ e pelos planos $z=0$ e $x+z=1$.


  1.  $\dfrac{16}{3}.$

  2.  $128\pi.$

  3.  $\dfrac{8}{15}.$


2340   

Determine a área da superfície dada pela parte do plano $x+2y+z=4$ que está dentro do cilindro $x^{2}+y^{2}=4$.


$4\sqrt{6}\pi.$


2292   

Determine uma representação paramétrica para a superfície descrita a seguir. A parte do plano $z=x+3$ que está dentro do cilindro $x^{2}+y^{2}=1.$


$x = r \cos(\theta),$ $y = r \sin(\theta),$ $z = 3 + r \cos(\theta),$ onde $0 \leq r \leq 1$ e $0\leq \theta \leq 2\pi.$


2417   

Determine o volume do sólido.

  1.  Abaixo do paraboloide $z=x^{2}+y^{2}$ e acima da região delimitada por $y=x^{2}$ e $x=y^{2}.$

  2.  Abaixo do paraboloide $z=3x^{2}+y^{2}$ e acima da região delimitada por $y=x$ e $x=y^{2}-y.$

  3.  $ $ Abaixo da superfície $z=xy$ e acima do triângulo com vértices $(1,1)$, $(4,1)$ e $(1,2).$

  4.  Limitado pelo cilindro $y^{2}+z^{2}=4$ e pelos planos $x=2y$, $x=0$ e $z=0$, no primeiro octante.


  1.  $\dfrac{6}{35}.$

  2.  $\dfrac{144}{35}.$

  3.  $\dfrac{31}{8}.$

  4.  $\dfrac{16}{3}.$


2514   

Faça uma correspondência entre a função: (i) e seu gráfico; (ii) e seus mapas de contorno. Justifique sua escolha.

  1. $z=\sin(xy)$

  2. $z=\sin(x-y)$

  3. $z=(1-x^{2})(1-y^{2})$

  4. $z=e^{x} \; \cos{y}$

  5. $z=\sin{x}-\sin{y}$

  6. $z=\dfrac{x-y}{1+x^{2}+y^{2}}$

A expressão 1 cossec x e o mesmo que

A expressão 1 cossec x e o mesmo que


3127   

Verifique que para o vetor posição \(\mathbf{r}=x\mathbf{i}+y\mathbf{j}+z\mathbf{k}\) valem as seguintes propriedades

  1.  \(\displaystyle \mathrm{div\,}\mathbf{r} = 3\)

  2.  \(\displaystyle \nabla\dfrac{1}{\|\mathbf{r}\|} = -\dfrac{\mathbf{r}}{\|\mathbf{r}\|^3} \)


3064   

Determine o campo vetorial gradiente $\nabla f$ de $f(x,y) = \sqrt{x^2+y^2}$ e o esboce.


2047   

Determine se o conjunto $\{(x,y)|\,x>0,\,y>0\}$ é ou não:

  1. aberto;

  2. conexo; e

  3. simplesmente conexo.



Temos que o conjunto $D=\{(x,y)|\,x>0,\,y>0\}$ representa o primeiro quadrante, excluindo os eixos. Então:

  1. $D$ é aberto, pois em torno de cada ponto em $D$, podemos colocar um disco que se encontra em $D.$

  2. $D$ é conexo, pois o segmento de reta que une dois pontos quaisquer de $D$ encontra-se em $D.$

  3. $D$ é simplesmente conexo, pois ele é conexo e não tem buracos.


2723   

Se $z = 5x^2 + y^2$ e $(x,y)$ varia de $(1,2)$ a $(1,05; 2,1)$, compare os valores de $\Delta z$ e $dz$.


$\Delta z = 0.9225$ e $dz = 0.9$.


3003   

Determine a massa e o centro de massa da lâmina que ocupa a região $D$ e tem função densidade $\rho$, sendo: $\displaystyle D = \{(x,y) \in\mathbb{R}^2: 0 \leq y \leq \sin{(\pi x/L)}, \ 0 \leq x \leq L\}; \quad \rho(x,y) = y$.


Massa: $\dfrac{L}{4};$ centro de massa: $\displaystyle \left(\frac{L}{2},\frac{16}{9\pi} \right).$


2644   

Use a definição de derivadas parciais como limites para encontrar $f_{x}(x,y)$ e $f_{y}(x,y)$, sendo $f(x,y)=x^{2}y-x^{3}y$.


$\displaystyle f_{x} = y^{2} - 3x^{2}y \;\;\;\text{e}\;\;\; f_{y} = 2xy - x^{3}$.


3096   

  1.  Faça um esboço do sólido no primeiro octante compreendido pelos planos \(x=0\), \(z=0\), \(x=5\), \(z-y=0\) e \(z=-2y+6\).

  2.  Calcule o volume do sólido dividindo-o em duas partes.


2298   

Determine uma representação paramétrica para a superfície descrita a seguir. A porção do cilindro $(x-2)^{2}+z^{2}=4$ entre os planos $y=0$ e $y=3.$


$x = 4\cos^{2}(v),$ $y = u,$ $z = 4\cos(v)\sin(v),$ onde $-\dfrac{\pi}{2}\leq v \leq \dfrac{\pi}{2}$ e $0 \leq u \leq 3.$


2172   

Calcule $\int_{C}\mathbf{F}\cdot\, d\mathbf{r}$, em que

$$\mathbf{F}(x,y) = (x^2+y)\mathbf{i} + (3x-y^2)\mathbf{j}$$

e $C$ é a fronteira orientada positivamente de uma região $D$ que tem área 6.


$12.$


2537   

Suponha que o sólido tenha densidade constante $k$. Encontre os momentos de inércia para um cubo com comprimento do lado $L$ se um vértice está localizado na origem e três arestas estão nos eixos coordenados.


$\displaystyle I_{x} = I_{y} = I_{z} = \dfrac{2kL^5}{3}.$


1965   

Verifique que
$$\int_{C} P\,dx+Q\,dy=\iint\limits_{B}\bigg(\frac{\partial Q}{\partial x}-\frac{\partial P}{\partial y}\bigg)\,dxdy,$$

onde $B$ é o triângulo de vértices $(0,0)$, $(1,0)$ e $(1,1)$, $C$ é a fronteira de $B$ orientada no sentido anti-horário, $P(x,y)=x^{2}-y$ e $Q(x,y)=x^{2}+y.$


$\displaystyle \int_{C} P\,dx+Q\,dy = \dfrac{7}{6} =  \iint\limits_{B}\bigg(\frac{\partial Q}{\partial x}-\frac{\partial P}{\partial y}\bigg)\,dxdy.$


2735   

Determine o plano que passa pelos pontos $(1,1,2)$ e $(-1,1,1)$ e que seja tangente ao gráfico de $f(x,y) = xy$.


$x + 6y - 2z = 3$.


2987   

Calcule a integral, efetuando uma mudança de variáveis apropriada. $\displaystyle\iint\limits_{R}\cos{\left(\dfrac{y - x}{y + x}\right)} \, dA$, em que $R$ é a região trapezoidal com vértices $(1,0)$, $(2,0)$, $(0,2)$ e $(0,1)$.


$\dfrac{3}{2} \sin(1).$


2276   

Calcule $\nabla f(x,y)$.

$f(x,y) = \dfrac{x}{y}$


 $\displaystyle \nabla f(x,y) = \left(\frac{1}{y}, -\frac{x}{y^{2}} \right).$


2931   

Calcule a integral em coordenadas esféricas. $\displaystyle\int_{0}^{2\pi}\int_{0}^{\pi/4}\int_{0}^{2}(\rho\cos{\phi})\rho^{2}\sin{\phi}\,d\rho d\phi d\theta$.


$2\pi.$


2337   

Determine a área da superfície dada pela parte da superfície $y=4x+z^{2}$ que está entre os planos $x=0$, $x=1$, $z=0$ e $z=1.$


$\dfrac{\sqrt{21}}{2} + \dfrac{17}{4} \left( \ln(2 + \sqrt{21}) - \ln(\sqrt{17}) \right).$


2760   

Verifique que a função $f(x,y) = \arctan{xy}$ é diferenciável.


As derivadas parciais $\frac{\partial f}{\partial x}$ e $\frac{\partial f}{\partial y}$ de cada função $f$ existem e são contínuas em todos os pontos do domínio.


3055   

 Faça o esboço do sólido cujo volume é dado pela integral e calcule essa integral.

$\displaystyle \int_0^4 \int_0^{2\pi}\int_r^4 r \, dz d\theta dr$


A expressão 1 cossec x e o mesmo que


2945   

Calcule utilizando coordenadas esféricas. $\displaystyle\iiint\limits_{B}x\,dxdydz$, onde $B$ é o conjunto $x\geq 0$ e $x^{2}+y^{2}+z^{2}\leq 4.$


$4\pi$.


1941   

Calcule a integral de linha, onde $C$ é a curva dada.

$\displaystyle\int_{C}xyz\,ds$, onde $C$ é a hélice ${\bf r}(t)=(\cos t,\sin t,3t)$, $0\leq t\leq 4\pi.$


$-3\sqrt{10}\pi.$


2614   

Use o Teorema de Stokes para calcular $\displaystyle\int_C {\bf F}\cdot  d{\bf R}$. $C$ é orientada no sentido anti-horário quando vista de cima.

  • ${\bf F}(x,y,z) = (2xyz-2y,x^2+2x,x^2+2y)$, $C$ é a circunferência $y^2+z^2=1$, $x=2$.


$2\pi$.


2540   

Um cubo sólido de $2$ unidades de lado é limitado pelos planos $x=\pm 1$, $z=\pm 1$, $y=3$ e $y=5.$ Encontre o centro de massa e os momentos de inércia desse cubo.


Centro de massa: $\displaystyle \left(0,4,0 \right),$ $I_{x} = \dfrac{400}{3},$ $I_{y} = \dfrac{16}{3},$ $I_{z} = \dfrac{400}{3}.$


2523   

Esboce o gráfico da função $f(x,y)=\ln(\sqrt{x^{2}+y^{2}})$. Em geral, se $g$ é uma função de uma variável, como saber o gráfico de $f(x,y)=g(\sqrt{x^{2}+y^{2}})$ a partir do gráfico de $g$?


O gráfico de $f(x,y) = g(\sqrt{x^{2} + y^{2}})$ pode ser obtido rotacionando o gráfico de $g$ no plano $xz$ ao redor do eixo $z.$

A expressão 1 cossec x e o mesmo que


2081   

 O comprimento $l$, a largura $w$ e a altura $h$ de uma caixa variam com o tempo. Em certo instante, as dimensões da caixa são $l=1m$ e $w=h=2m$. $l$ e $w$ aumentam a uma taxa de $2m/s$, ao passo que $h$ diminui a uma taxa de $3m/s$. Nesse instante, determine as taxas nas quais as seguintes quantidades estão variando.

  1. O volume.
  2. A área da superfície.
  3. O comprimento da diagonal.


  1. $6$ m$^3$/s.
  2. $10$ m$^2$/s.
  3. $0$ m/s.


2165   

Use o Teorema de Green para calcular $\int_{C}\mathbf{F} \cdot d\mathbf{r}$, onde $\mathbf{F}(x,y) = (e^x+x^2y,e^y-xy^2)$, $C$ é a circunferência $x^2+y^2=25$, orientada no sentido horário. (Verifique a orientação da curva antes de aplicar o Teorema.)


$\dfrac{625\pi}{2}.$


2074   

Seja $R$ o retângulo $1\leq x\leq 2$, $0\leq y\leq 1$. Calcule $\iint\limits_{R} f(x,y)\,dxdy$, sendo $f(x,y)$ dada por

  1. $y\cos(xy)$

  2. $x\sin(\pi y)$


  1.  $\cos(1) - \dfrac{(1 + \cos(2))}{2}$

  2.  $\ln\left(\dfrac{4}{3}\right).$


2181   

Utilize o Teorema de Green para demonstrar a fórmula de mudança de variáveis para as integrais duplas para o caso em que $f(x,y) = 1$:

$$\iint\limits_{R} dxdy = \iint\limits_{R}\left|\dfrac{\partial(x,y)}{\partial(u,v)}\right|\, dudv.$$

Aqui, $R$ é a região do plano $xy$ que corresponde à região $S$ do plano $uv$ sob a transformação dada por $x=g(u,v)$, $y=h(u,v)$. (Sugestão: observe que o lado esquerdo é $A(R)$. Converta a integral de linha sobre $\partial R$ para uma integral de linha sobre $\partial S$ e aplique o Teorema de Green no plano $uv$.)


Dica: pelo Teorema de Green, $A(R) = \displaystyle \iint_{R} dxdy = \int_{\partial R} x dy.$ Escolhendo a orientação positiva em $\partial S$ correspondente a orientação positiva em $\partial R,$ segue que

$$\displaystyle \int_{\partial R} x dy = \int_{\partial S} g(u,v) \dfrac{\partial h}{\partial u} du + g(u,v) \frac{\partial h}{\partial v} dv.$$

Conclua utilizando o Teorema de Green no plano $uv$ e a Regra da Cadeia.


2250   

Verifique que $\mbox{div} {\bf E}=0$ para o campo elétrico ${\bf E}({\bf x})=\dfrac{\epsilon Q}{|{\bf x}|^{3}}{\bf x}.$


2167   

Use o Teorema de Green para calcular $\int_{C}\mathbf{F} \cdot d\mathbf{r}$, onde $\mathbf{F}(x,y) = 4x^3y^3\mathbf{i} + (3x^4y^2+5x)\mathbf{j}$, $C$ é a fronteira do quadrado de vértices $(-1,0)$, $(0,-1)$, $(1,0)$ e $(0,1)$. (Verifique a orientação da curva antes de aplicar o Teorema.)


$10.$


2996   

Calcule a integral, efetuando uma mudança de variáveis apropriada. $\displaystyle\iint\limits_{R} \dfrac{y - 2x}{3y + 2x} \, dA$, em que $R$ é a paralelogramo de vértices $(1,2)$, $(2,4)$, $(5,2)$ e $(4,0)$.


$-4\ln(2).$


3095   

Cada integral iterada abaixo representa o volume de um sólido. Faça um esboço do sólido. (Não é necessário calcular o volume.)

  1.  \(\displaystyle \int_0^1\int_0^1 (2-x-y)\, dydx\)

  2.  \(\displaystyle \int_{-2}^2\int_{-2}^2(x^2+y^2)\,dxdy\)


2410   

Inverta a ordem de integração.

  1.   $\displaystyle\int_{0}^{1}\bigg[\int_{\sqrt{x-x^{2}}}^{\sqrt{2x}}f(x,y)\,dy\bigg]dx$

  2.   $\displaystyle\int_{0}^{3a}\bigg[\int_{\frac{\sqrt{3}}{3}x}^{\sqrt{4ax-x^{2}}}f(x,y)\,dy\bigg]dx, \; a> 0.$

  3.   $\displaystyle\int_{0}^{\pi}\bigg[\int_{0}^{\sin{x}}f(x,y)\,dy\bigg]dx$


  1.  $\  \\ \begin{array}{ll} \displaystyle\int_{0}^{\frac{1}{2}}\bigg[\int_{\frac{y^{2}}{2}}^{\frac{1}{2} - \sqrt{\frac{1}{4} - y^{2}}}f(x,y)\,dx\bigg]dy &+ \displaystyle\int_{0}^{\frac{1}{2}}\bigg[\int_{\frac{1}{2}+ \sqrt{\frac{1}{4} - y^{2}}}^{1}f(x,y)\,dx\bigg]dy\\ &+ \displaystyle\int_{\frac{1}{2}}^{\sqrt{2}}\bigg[\int_{\frac{y^{2}}{2}}^{1}f(x,y)\,dx\bigg]dy \end{array} $

  2.   $\displaystyle\int_{0}^{\sqrt{3}a} \bigg[\int_{2a + \sqrt{4a^2 - y^{2}}}^{\sqrt{3} y}f(x,y)\,dx\bigg]dy.$

  3.   $\displaystyle\int_{0}^{1}\bigg[\int_{\arcsin(y)}^{\pi-\arcsin(y)}f(x,y)\,dx\bigg]dy$


3133   

Encontre a massa da lâmina descrita como sendo a porção do parabolóide \(2z=x^2+y^2\) que fica dentro do cilindro \(x^2+y^2=8\) e tem densidade \(\delta_0\) constante.


2262   

Calcule $\displaystyle\iint\limits_{B} y\,dx dy$, onde $B$ é o conjunto dado.

  1.  $B$ é a região compreendida entre os gráficos de $y=x$ e $y=x^{2}$, com $0\leq x\leq 2.$

  2.  $B$ é o paralelogramo de vértices $(-1,0)$, $(0,0)$, $(1,1)$ e $(0,1).$

  3.  $B$ é o semicírculo $x^{2}+y^{2}\leq 4$, $y\geq 0.$

  4.  $B=\{(x,y)\in \mathbb{R}^{2}|\;x\geq 0,\;x^{5}-x\leq y \leq 0\}.$


  1.  $2$.

  2.  $\dfrac{1}{2}$.

  3.  $\dfrac{16}{3}$.

  4.  $-\dfrac{16}{231}$.


3054   

Esboce o sólido cujo volume é dado pela integral iterada.

$\displaystyle\int_{0}^{2}\int_{0}^{2-y}\int_{0}^{4-y^{2}}\;dx dz dy$


A expressão 1 cossec x e o mesmo que


2077   

Calcule a integral dupla.

  1. $\displaystyle\iint\limits_{R} (6x^{2}y^{3}-5y^{4})\,dA, \quad R=\{(x,y) \in \mathbb{R}^2:0\leq x\leq 3,\;0\leq y\leq 1\}.$

  2. $\displaystyle\iint\limits_{R} \dfrac{xy^{2}}{x^{2}+1}\,dA, \quad R=\{(x,y)\in \mathbb{R}^2:0\leq x\leq 1,\;-3\leq y\leq 3\}.$


  1. $\dfrac{21}{2}.$

  2. $9 \ln(2).$


2345   

Mostre que as equações paramétricas $x=a \cosh u\cos v$, $y=b\cosh u \sin v$, $z=c\sinh u$, representam um hiperboloide de uma folha.


Note que $\dfrac{x^{2}}{a^{2}} + \dfrac{y^{2}}{b^{2}} - \dfrac{z^{2}}{c^{2}} = 1$.


2175   

Calcule a área da região limitada pela elipse $x = a\cos{t}$, $y=b\sin{t}$,  $0\leq t \leq \pi/2$, em que $a > 0$ e $b > 0$.


$\pi ab.$


2536   

Calcule a massa do sólido $x+y+z\leq 1$, $x\geq 0$, $y\geq 0$ e $z\geq 0$, sendo a densidade dada por $\rho(x,y,z)=x+y.$


$\dfrac{1}{12}.$


2967   

Dentre as coordenadas cilíndricas ou esféricas, utilize a que lhe parecer mais apropriada para determinaretermine o volume da região limitada acima pelo paraboloide  $z=5-x^{2}-y^{2}$ e abaixo pelo paraboloide $z=4x^{2}+4y^{2}.$


$\dfrac{5\pi}{2}.$


2722   

Determine a diferencial da função $R = \alpha\beta^2 \cos{\lambda}$.


$dR = \beta^{2} \cos(\gamma) d\alpha + 2\gamma \beta \cos (\gamma) d\beta - \alpha \beta^{2} \sin(\gamma) d\gamma$.


2589   

A água do mar tem densidade $1025 kg/m^{3}$ e escoa em um campo de velocidade ${\bf v}=y{\bf i}+x{\bf j}$, onde $x$, $y$ e $z$ são medidos em metros e as componentes de ${\bf v}$ em metros por segundo. Encontre a vazão para fora do hemisfério $x^{2}+y^{2}+z^{2}=9$, $z\geq 0.$


2338   

Determine a área da superfície $z=\frac{2}{3}(x^{3/2}+y^{3/2})$, $0\leq x \leq 1$ e $0\leq y\leq 1.$


$\dfrac{4}{15}(3^{5/2} - 2^{7/2} + 1).$


2774   

Determine o volume máximo da maior caixa retangular no primeiro octante com três faces nos planos coordenados e com um vértice no plano $x+2y+3z=6.$



Vamos maximizar a função:

$$f(x,y)=x\cdot y\cdot \bigg(\dfrac{6-x-2y}{3}\bigg)=\dfrac{6xy-x^{2}y-2xy^{2}}{3},$$

então o volume máximo é $V=x \cdot y \cdot z.$

Para encontrar os pontos críticos devemos encontrar as derivadas parciais $f_{x}$ e $f_{y}.$ Assim,

$$f_{x}(x,y)=\frac{6y-2xy-2y^{2}}{3}\;\;\;\;\;\;\;\; \mbox{e}\;\;\;\;\;\;\;\; f_{y}(x,y)=\frac{6x-x^{2}-4xy}{3}.$$

Fazendo $f_{x}=0$ e $f_{y}=0$, obtemos o seguinte sistema de equações

$$\left \{\begin{array}{cc}6y-2xy-2y^{2}=0\\6x-x^{2}-4xy=0\\\end{array}\right.$$

Da primeira equação obtemos

$$y=0 \;\;\;\;\;\;\;\; \mbox{ou}\;\;\;\;\;\;\;\; y=3-x.$$

Como, $y=0$ não satifaz as condicões, vamos analisar o caso onde $y=3-x.$ Substituindo esse valor na segunda equação obtemos

$$x=0\;\;\;\;\;\;\;\; \mbox{ou}\;\;\;\;\;\;\;\; 3x^{2}-6x=0.$$

Novamente, como $x=0$ não satisfaz as condições, vamos analisar o caso onde $3x^{2}-6=0$. Logo, obtemos

$$x=0\;\;\;\;\;\;\;\; \mbox{ou} \;\;\;\;\;\;\;\; x=2.$$

Novamente, $x=0$ não nos interessa. Assim, sendo $x=2$ obtemos que $y=1$ e $z=\dfrac{2}{3}.$ Portanto, o volume máximo da maior caixa, nas condições do exercício, será

$$V=(2)\cdot(1)\cdot \frac{2}{3}=\frac{4}{3}.$$


2676   

Calcule as derivadas parciais de $f(x,y,z) = \sin{(x^2 + y^2 + z^2)}$.


$\begin{aligned}[t]\frac{\partial f}{\partial x} &= 2x \cos (x^{2} + y^{2} + z^{2}),\;\;\;\;  \frac{\partial f}{\partial y} = 2y \cos (x^{2} + y^{2} + z^{2}) \;\;\;\;\;\text{e}\\\frac{\partial f}{\partial z} &= 2z \cos (x^{2} + y^{2} + z^{2}).\end{aligned}$


3063   

Determine o campo vetorial gradiente $\nabla f$ de $f(x,y) = x^2-y$ e o esboce.


2124   

Suponha que, para todo $x$,$f(3x,x^{3})=\arctan(x)$.

  1.  Calcule $\dfrac{\partial f}{\partial x}(3,1)$ admitindo $\dfrac{\partial f}{\partial y}(3,1)=2$.
  2.  Determine a equação do plano tangente ao gráfico de $f$ no ponto $(3,1,f(3,1))$.


  1.  $\dfrac{\partial f}{\partial x}(3,1) = -\frac{11}{6}.$
  2.  $\displaystyle z - \frac{\pi}{4} = -\frac{11}{6}(x - 3) + 2(y - 1).$


2139   

Use o Teorema do Divergente para calcular $\displaystyle\iint \limits_{S}(2x+2y+z^{2})\,dS$ onde $S$ é a esfera $x^{2}+y^{2}+z^{2}=1.$



A superfície $S$ em questão é a esfera unitária, que é a fronteira da bola unitária $B$ dada por $x^2+y^2+z^2 \leq 1$ e tem vetor normal num ponto $(x,y,z)$ igual a $(x,y,z)$ (o qual aponta para ``fora").

A expressão 1 cossec x e o mesmo que

Observe que podemos transformar o integrando $2x+2y+z^{2}$ em $(2,2,z) \cdot (x,y,z)$ e essa escrita é interessante, já que o segundo vetor é exatamente o vetor normal a $S$. Agora estamos em condições de aplicar o Teorema do Divergente quando tomamos o campo ${\bf F}(x,y,z) = (2,2,z)$. Assim,
\begin{array}{rcl}\displaystyle\iint\limits_{S}(2x+2y+z^{2})\,dS & = & \iint\limits_{ S}(2,2,z) \cdot (x,y,z)\,dS \\& = & \int\int\int \limits_{ S}{\bf F} \cdot {\bf n}\,dS \\& = & \iiint\limits_{ B}\text{div } F\,dV \\& = & \iiint\limits_{ B}(0+0+1)\,dV \\& = & V(B) = \frac{4\pi}{3}.\end{array}


3142   

Considere o campo vetorial \(\mathbf{F}(x,y,z)=(x-y)\mathbf{i} + (y-z)\mathbf{j}+(z-x)\mathbf{k}\) e a superfície \(\sigma\)
descrita como sendo a porção do plano \(x+y+z=1\) no primeiro octante e orientada para cima. Verifique o Teorema de Stokes
calculando, separadamente, a integral de linha e a integral dupla e, em seguida, comparando os valores.


\(\dfrac{3}{2}\)


2349   

Calcule o volume do conjunto dado.

  1.   $\{(x,y,z)\in \mathbb{R}^{3}|  0\leq x\leq 1, 0\leq y\leq 1,0\leq z\leq x+2y\}$

  2.  $\{(x,y,z)\in \mathbb{R}^{3}| 0\leq x\leq 2,  1\leq y\leq 2, 0\leq z\leq \sqrt{xy}\}$

  3.  $\{(x,y,z)\in \mathbb{R}^{3}| 0\leq x\leq 1,  0\leq y\leq 1, 0\leq z\leq xye^{x^{2}-y^{2}}\}$

  4.  $ \{(x,y,z)\in \mathbb{R}^{3}| 0\leq x\leq 1,  0\leq y\leq 1, x^{2}+y^{2}\leq z\leq 2\}$

  5.  $\{(x,y,z)\in \mathbb{R}^{3}| 1\leq x\leq 2, 0\leq y\leq 1,\;x+y\leq z\leq x+y+2\}$

  6.  $\{(x,y,z)\in \mathbb{R}^{3}|\;0\leq x\leq 1, 0\leq y\leq 1,1\leq z\leq e^{x+y}\}$


  1.   $\dfrac{3}{2}.$

  2.   $\dfrac{8\sqrt{2}(2\sqrt{2} - 1)}{9}.$

  3.   $\dfrac{(e - 1)(1 - e^{-1})}{4}.$

  4.   $\dfrac{4}{3}.$

  5.   $2.$

  6.   $e^{2}-2e.$


1957   

Calcule a integral de linha $\int_{C}{\bf F}\cdot d{\bf r}$, onde $C$ é dada pela função vetorial ${\bf r}(t).$

${\bf F}(x,y)=(e^{-y}-2x,-xe^{-y}-\sin y)$, ${\bf r}(t)=(t,\tan t)$, $0\leq t\leq \pi/4.$


$\displaystyle \cos(1) - \frac{\pi}{4}e^{-1} - \frac{\pi^{2}}{16} - 1.$


2263   

Calcule $\displaystyle\iint\limits_{B}f(x,y)\,dx dy$ sendo dados:

  1.  $f(x,y)=x\cos{y}$ e $B=\{(x,y)\in \mathbb{R}^{2}|\;x\geq 0,\;x^{2}\leq y\leq \pi\}.$

  2.  $f(x,y)=xy$ e $B=\{(x,y)\in \mathbb{R}^{2}|\;x^{2}+y^{2}\leq 2,\;y\leq x\;e\;x\geq 0\}.$

  3.  $f(x,y)=x$ e $B$ o triângulo de vértices $(0,0)$, $(1,1)$ e $(2,0).$

  4.  $f(x,y)=xy\sqrt{x^{2}+y^{2}}$ e $B$ o retângulo $0\leq x\leq 1$, $0\leq y\leq 1.$

  5.  $f(x,y)=x+y$ e $B$ o paralelogramo de vértices $(0,0)$, $(1,1)$, $(3,1)$ e $(2,0).$


  1.  $-1.$

  2.  $-\dfrac{1}{4}$.

  3.  $1.$

  4.  $\dfrac{2(2\sqrt{2} - 1)}{15}.$

  5.  $4.$


1947   

Calcule a integral de linha, onde $C$ é a curva dada.

$\displaystyle\int_{C}\dfrac{-y}{4x^{2}+y^{2}}\,dx+\dfrac{x}{4x^{2}+y^{2}}\,dy$, $C$ tem por imagem a elipse $4x^{2}+y^{2}=9$ e o sentido de percurso é o anti-horário.


$\displaystyle \pi.$


2325   

Calcule a área da superfície dada por: ${\bf r}(u,v)=(u,v,4-u^{2}-v^{2})$, $(u,v)\in K$, onde $K$ é o conjunto no plano $uv$ limitado pelo eixo $u$ e pela curva (em coordenadas polares) $\rho=e^{-\theta}$,$0\leq \theta \leq \pi.$. (Sugerimos ao leitor desenhar a imagem da superfície.)


$\displaystyle \dfrac{1}{72} \left( \ln\left(3\dfrac{\sqrt{e^{2\pi} + 4} + e^{\pi}}{\sqrt{e^{2\pi} + 4} - e^{\pi}} \right) + 3 \ln\left(\dfrac{\sqrt{5} - 1 }{\sqrt{5} + 1 }\right) - 8e^{3\pi} \sqrt{e^{2\pi} + 4}(e^{2\pi} + 1) + 16\sqrt{5} - 6\pi \right).$


2878   

Determine os valores extremos de $f(x,y) = 2x^2 + 3y^2 - 4x - 5$ na região descrita por $x^2 + y^2 \leq 16$.


Valor máximo: $f(-2, \pm 2 \sqrt{3}) = 47$ e valor mínimo $f(1,0) = -7.$


1933   

Calcule a integral de linha, onde $C$ é a curva dada.

$\displaystyle\int_{C}x\,dx-y\,dy$,   $C$ é o segmento de extremidades $(1,1)$ e $(2,3)$, percorrido no sentido de $(1,1)$ para $(2,3).$


$\displaystyle -\frac{5}{2}.$


3112   

Encontre a área da parte da superfície \(z=\sqrt{4-x^2}\) que fica acima do retângulo \(R\) do plano \(xy\) cujas coordenadas satisfazem \(0\leq x\leq 1\) e \(0\leq y\leq 4\).



A superfície é uma parte do cilindro \(x^2+z^2=4\) localizada no primeiro octante. Neste caso, como \(z=f(x,y)\), podemos tomar \(x=u\) e \(y=v\) como parâmetros. Assim, teremos que \(\displaystyle \mathbf{r}=u\mathbf{i}+v\mathbf{j}+f(u,v)\mathbf{k} \) e \[ \|\dfrac{\partial\mathbf{r}}{\partial u}\times \dfrac{\partial \mathbf{r}}{\partial v}\| = \sqrt{\left(\dfrac{\partial z}{\partial x}\right)^2+\left(\dfrac{\partial z}{\partial y}\right)^2+1}.\] Segue para a área que \begin{align*} S & = \iint\limits_R\sqrt{\left(\dfrac{\partial z}{\partial x}\right)^2+\left(\dfrac{\partial z}{\partial y}\right)^2+1}\,dA \\  & = \iint\limits_R\sqrt{\left(-\dfrac{x}{\sqrt{4-x^2}}\right)^2+ 0 + 1}\,dA = \int_0^4\int_0^1\dfrac{2}{\sqrt{4-x^2}}\,dxdy \\   & = 2\int_0^4\left[\arcsin\left(\dfrac{1}{2}x\right)\right]_{x=0}^1\,dy = 2\int_0^4\dfrac{\pi}{6}\,dy = \dfrac{4}{3}\pi. \end{align*}


2170   

Use o Teorema de Green para achar o trabalho realizado pela força $\mathbf{F}(x,y) = x(x+y)\mathbf{i} + xy^2\mathbf{j}$ ao mover uma partícula da origem ao longo do eixo $x$ até $(1,0)$, em seguida ao longo de um segmento de reta até $(0,1)$ e então de volta à origem ao longo do eixo $y$.


$-\dfrac{1}{12}.$


2258   

Um sólido ocupa a região $E$ com superfície $S$ e está imerso em um líquido com densidade constante $\rho$. Escolhemos um sistema de
coordenadas de modo que o plano $xy$ coincida com a superfície do líquido e valores positivos de $z$ sejam medidos para baixo, adentrando o líquido. Então, a pressão na profundidade $z$ é $p=\rho g z$, onde $g$ é a aceleração da gravidade. A força de empuxo total sobre o sólido devida $\grave{a}$ distribuição de pressão é dada pela integral de superfície
${\bf F}=-\displaystyle\iint\limits_{S} p{\bf n}\,dS$ onde ${\bf n}$ é o vetor normal unitário apontando para fora. Use o resultado do exercício anterior para mostrar que ${\bf F}=-W{\bf k}$, onde $W$ é o peso do líquido deslocado pelo sólido. (Observe que ${\bf F}$ é orientado para cima porque $z$ está orientado para baixo.) O resultado é o Princípio de Arquimedes: a força de empuxo sobre um objeto é igual ao

peso do líquido deslocado.



Note que $\displaystyle {\bf F}=-\int_{S} p {\bf n} \,dS = -\iiint_{E} \nabla p\,dV = -\iiint_{E} \nabla p\,dV = - \iiint_{E} \nabla (\rho g z)\,dV.$

Conclua usando que $W = \rho g V(E),$ onde $V(E)$ é o volume de $E.$


1967   

Seja $C: {\bf r}(t)=(R\,\cos t, R\,\sin t)$, $0\leq t \leq 2\pi$\,$(R>0).$ Mostre que

$$\int_{C}\frac{-y}{x^{2}+y^{2}}\,dx+\frac{x}{x^{2}+y^{2}}\,dy$$

não depende de $R.$


Note que o valor da integral é $2\pi,$ independente de $R.$


3017   

Esboce a região de integração para a integral iterada $\displaystyle\int_{-1}^{2}\!\int_{-\sqrt{4-x^{2}}}^{4-x^{2}}f(x,y)\,dy dx$.


A expressão 1 cossec x e o mesmo que


2611   

Use o Teorema de Stokes para calcular $\displaystyle\int_C {\bf F}\cdot  d{\bf R}$. $C$ é orientada no sentido anti-horário quando vista de cima.

  • ${\bf F}(x,y,z) = (x^2-y){\bf i} + 4z{\bf j} + x^2{\bf k}$, $C$ é a curva de interseção do plano $z=2$ com o cone $z=\sqrt{x^2+y^2}$.


$4\pi$.


2744   

Considere a função $f(x,y) = x \ g(x^2 - y^2)$, em que $g(u)$ é uma função derivável de uma variável. Mostre que o plano tangente ao gráfico de $f$ no ponto $(a,a,f(a,a))$ passa pela origem.


Note que $a \frac{\partial f}{\partial x} (a,a) + a \frac{\partial f}{\partial y}(a,a) = f(a,a).$


2683   

Encontre $\partial f/\partial x$ e $\partial f/\partial y$ para $f(x,y)=e^{-x}\;\sin(x+y)$.


$\displaystyle \frac{\partial f}{\partial x} = -e^{-x} \sin(x + y) + e^{-x}\cos(x + y) \;\;\;\;\text{e}\;\;\;\; \frac{\partial f}{\partial y} = e^{-x}\cos(x + y)$.


2513   

Uma placa fina de metal, localizada no plano $xy$, tem temperatura $T(x,y)$ no ponto $(x,y)$. As curvas de nível de $T$ são chamadas isotérmicas porque todos os pontos em uma isotérmica têm a mesma temperatura. Faça o esboço de algumas isotérmicas se a função temperatura for dada por

$$T(x,y)=\dfrac{100}{1+x^{2}+2y^{2}}.$$


As isotérmicas são dadas pela família de elipses: $x^{2} + 2y^{2} = \frac{100}{C} - 1,$ com $0 < C \leq 100.$

A expressão 1 cossec x e o mesmo que


2799   

Determine os valores máximos e mínimos locais e pontos de sela da função $f(x,y)=(x^{2}+y^{2})e^{y^{2}-x^{2}}$.


Ponto de mínimo: $(0,0);$ pontos de sela: $(1,0)$ e $(-1,0).$


2430   

Calcule a integral tripla.

  1.  $\displaystyle\iiint\limits_{  E}\sqrt{1-z^{2}}\;dx dy dz$, onde $E$ é o cubo $0\leq x\leq 1$, $0\leq y\leq 1$ e $0\leq z\leq 1.$

  2.  $\displaystyle\iiint\limits_{  E}\;dx dy dz$, onde $E$ é o conjunto $x^{2}+y^{2}\leq z \leq 2x.$


  1.  $\dfrac{\pi}{4}.$

  2.  $\dfrac{\pi}{2}.$


2983   

Utilize a transformação dada para calcular a integral. $\displaystyle\iint\limits_{R} (x^2 - xy + y^2) \, dA$, em que $R$ é a região delimitada pela elipse  $x^2 - xy + y^2 = 2$; $x = \sqrt{2}u - \sqrt{\dfrac{2}{3}}v$, $y = \sqrt{2}u + \sqrt{\dfrac{2}{3}}v$.


$\dfrac{4\pi}{\sqrt{3}}.$


2099   

Determine o trabalho realizado pelo campo de força ${\bf F}(x,y)=2y^{3/2}\,{\bf i}+3x\sqrt{y}\,{\bf j}$ ao mover um objeto de $P(1,1)$ a $Q(2,4).$


$30.$


2370   

Seja $f(x,y) = x \arctan{\dfrac{x}{y}}$. Calcule $D_{\bf{u}}f(1,1)$, em que $\bf{u}$ aponta na direção e sentido de máximo crescimento de $f$, no ponto $(1,1)$.


$\displaystyle D_{\bf{u}}f(1,1) = \sqrt{\left( \frac{\pi}{4} + \frac{1}{2}\right)^{2} + \frac{1}{4}}.$


2853   

Considere a função $f(x,y)=x^{2}+y^{2}+2xy-x-y+1$ no quadrado $0\leq x\leq 1$ e $0\leq y\leq 1$.

  1. Mostre que $f$ tem um mínimo absoluto ao longo do segmento de reta $2x+2y=1$ nesse quadrado. Qual é o valor mínimo absoluto?

  2. Encontre o valor máximo absoluto de $f$ no quadrado.


  1. $\displaystyle \frac{3}{4}.$

  2. $f(1,1) = 3.$


2297   

Determine uma representação paramétrica para a superfície descrita a seguir. A superfície cortada do cilindro parabólico $z=4-y^{2}$ pelos planos $x=0$, $x=2$ e $z=0.$


$x = u,$ $y = v,$ $z = 4 - v^2,$ onde $0\leq u \leq 2$ e $-2 \leq v \leq 2.$


2305   

Identifique e faça um esboço da imagem da superfície parametrizada dada por ${\bf r}(u,v)=(u,v,1-u^{2})$, $u\geq 0$, $v\geq 0$ e $u+v\leq 1.$


${\bf r}(u,v)=(u,v,1-u^{2})$, $u\geq 0$,\, $v\geq 0$ e $u+v\leq 1.$


2257   

Suponha que $S$ e $E$ satisfaçam as condições do Teorema do Divergente e que $f$ seja uma função escalar com derivadas parciais contínuas. Demonstre que $\displaystyle\iint\limits_{S}f{\bf n}\,dS=\iiint\limits_{E}\nabla f\,dV.$ Estas integrais de superfície e triplas de funções vetoriais são vetores definidos integrando cada função componente. [Sugestão: comece aplicando o Teorema do Divergente a ${\bf F}=f{\bf c}$, onde ${\bf c}$ é um vetor constante arbitrário.]



Note que se ${\bf n} = n_{1} {\bf i} + n_{2} {\bf j} + n_{3} {\bf k},$ então

\begin{align*} &\iint_{S} f \cdot {\bf n}\,dS \\ &= \left( \iint_{S} f n_{1}\,dS \right) {\bf i} + \left( \iint_{S} fn_{2}\,dS\right) {\bf j} + \left( \iint_{S} fn_{3}\,dS\right) {\bf k}\\ &= \left( \iiint_{E} \dfrac{\partial f}{\partial x}\,dV \right) {\bf i}+ \left( \iiint_{E} \dfrac{\partial f}{\partial y}\,dV\right) {\bf j} + \left( \iiint_{E} \dfrac{\partial f}{\partial z}\,dV \right) {\bf k}. \end{align*}


3113   

A tendência de uma lâmina de resistir a uma mudança no seu movimento de rotação em torno de um eixo é medida pelo seu momento de inércia em torno daquele eixo. Se a lâmina ocupar uma região \(R\) do plano \(xy\) e se sua densidade \(\delta(x,y)\) for uma função contínua em \(R\), então os momentos de inércia em torno dos eixos \(x\), \(y\) e \(z\) são denotados por \(I_x\), \(I_y\) e \(I_z\), respectivamente, e são definidos por \begin{align*} I_x & = \iint\limits_R y^2\delta(x,y)\,dA, \\ I_y & = \iint\limits_R x^2\delta(x,y)\,dA, \\ I_z & = \iint\limits_R (x^2+y^2)\delta(x,y)\,dA. \\ \end{align*} Considere a lâmina retangular que ocupa a região descrita pelas desigualdades \(0\leq x\leq a\) e \( 0\leq y\leq b\). Supondo que a lâmina tenha densidade \(\delta\) constante, mostre que \[ \begin{array}{lll} I_x= \dfrac{\delta ab^3}{3}, & I_y= \dfrac{\delta a^3b}{3}, & I_z= \dfrac{\delta ab(a^2+b^2)}{3}. \end{array} \]


2049   

Determine se o conjunto $\{(x,y)|\,1<x^{2}+y^{2}<4\}$ é ou não:

  1. aberto;

  2. conexo; e

  3. simplesmente conexo.



Temos que o conjunto $D=\{(x,y)|\,1<x^{2}+y^{2}<4\}$ representa a região anelar entre os círculos com centro $(0,0)$ e raio $1$ e $2$. Então:

  1. $D$ é aberto pois, em torno de cada ponto em $D$, podemos colocar um disco que se encontra inteiramente em $D$.

  2. $D$ é conexo pois quaisquer dois pontos de $D$ podem ser conectados por um caminho em $D$.

  3. $D$ não é simplesmente conexo pois,  por exemplo, a região delimitada pela curva simples e fechada $x^{2}+y^{2}=(3/2)^2$ possui pontos que não estão em $D$, por exemplo, a origem $(0,0)$.


3107   

Encontre a área da superfície descrita como sendo a parte do plano \(2x+2y+z=8\) no primeiro octante.


3016   

Esboce o sólido cujo volume é dado pela integral iterada

$$\int_{0}^{1} \!\! \int_{0}^{1}(4-x-2y)\, dx dy.$$


A expressão 1 cossec x e o mesmo que


2468   

Calcule a integral de superfície $\displaystyle\iint \limits_{ S}{\bf F}\cdot d{\bf S}$ para o campo vetorial ${\bf F}$ e superfície orientada $S$ dados abaixo. Em outras palavras, determine o fluxo de ${\bf F}$ através de $S$. Para superfícies fechadas, use a orientação positiva (para fora).

  • ${\bf F}(x,y,z)=(x^{2}+z){\bf i}+y^{2}z{\bf j}+(x^{2}+y^{2}+z){\bf k}$ e $S$ é a parte no primeiro octante do parabolóide $z=x^{2}+y^{2}$ intersectada pelo plano $z=4.$


$4\pi - \dfrac{320}{7}.$


2803   

Determine os valores máximos e mínimos locais e pontos de sela da função $f(x,y)=x^{4}+y^{4}+4x+4y$.


Ponto de mínimo : $(-1,-1).$


2499   

Dada a função $f(x,y)=\ln (x^{2}+y^{2})$.

  1. Encontre o domínio da função.

  2. Encontre a imagem da função.

  3. Descreva as curvas de nível da função.


  1. $D_{f} = \left\lbrace (x,y);\; (x,y) \neq (0,0) \right\rbrace$.

  2. $Im(f) = \mathbb{R}.$

  3. As curvas de nível são os círculos $x^{2} + y^{2} = C$ com $C > 0.$


2539   

Encontre o centróide e os momentos de inércia $I_{x}$, $I_{y}$ e $I_{z}$ do tetraedro cujos vértices são os pontos $(0,0,0)$, $(1,0,0)$, $(0,1,0)$ e $(0,0,1).$


Centróide: $\displaystyle \left(\frac{1}{4},\frac{1}{4},\frac{1}{4} \right),$ $I_{x} = I_{y} = I_{z} = \dfrac{1}{30}.$


2997   

Calcule a integral, efetuando uma mudança de variáveis apropriada. $\displaystyle\iint\limits_{R} \dfrac{\cos{(x-y)}}{\sin{(x+y)}} \, dA$, em que $R$ é a região trapezoidal com vértices $(1,0)$, $(2,0)$, $(0,2)$ e $(0,1)$.


 $1.$


2115   

Mostre que qualquer função da forma 
$$z=f(x+at)+g(x-at)$$
é uma solução da equação de onda
$$\frac{\partial^{2} z}{\partial t^{2}}=a^{2}\frac{\partial^{2}z}{\partial x^{2}}.$$
(Sugestão: Tome $u=x+at$, $v=x-at$.)



Note que se $u = x + at$ e $v = x - at,$ então $\displaystyle \frac{\partial^{2} z}{\partial t^{2}} = a^{2}f''(u) + a^{2} g''(v)$e\\$\displaystyle \frac{\partial^{2} z}{\partial x^{2}} = f''(u) + g''(v).$


2329   

Considere o vetor unitário $\bf{u} = (\sqrt{3}/2,1/2)$ e a função

$$f(x,y) = \begin{cases}
\dfrac{xy^2}{x^2 + y^4}, &  \text{se } (x,y) \neq (0,0),\\ 0, &  \text{se } (x,y) = (0,0).\end{cases}$$

  1. Determine a derivada direcional $D_{\bf{u}}f(0,0)$.
  2.  Explique por que o produto escalar $\nabla f(0,0) \cdot \bf{u}$ não fornece a derivada direcional de $f$ em $(0,0)$ na direção de $\bf{u}$.


  1. $\displaystyle \frac{\sqrt{3}}{6}.$
  2.  Pois $f$ não é diferenciável em $(0,0),$ já que não é contínua nesse ponto.


3025   

Esboce a região de integração e calcule a integral $\displaystyle\int_{1}^{2}\!\!\int_{y}^{y^{2}} \,dx dy$.


$\frac{5}{6}.$

A expressão 1 cossec x e o mesmo que


2834   

Três alelos (versões alternativas de um gene) $A$, $B$ e $O$ determinam os quatro tipos de sangue: $A$ ($AA$ ou $AO$), $B$ ($BB$ ou $BO$), $O$ ($OO$) e $AB$. A Lei de Hardy-Weinberg afirma que a proporção de indivíduos em uma população que carregam dois alelos diferentes é $P=2pq+2pr+2rq$, onde $p$, $q$ e $r$ são as proporções de $A$, $B$ e $O$ na população. Use o fato de que $p+q+r=1$ para mostrar que $P$ é no máximo $\dfrac{2}{3}$.


É preciso maximizar de $P = 2q - 2q^{2} + 2r - 2r^{2} -2rq$ no conjunto delimitado pelas retas $q = 0,$ $r = 0$ e $q + r = 1.$ O ponto de máximo ocorre em $\displaystyle \left( \frac{1}{3}, \frac{1}{3} \right),$ no qual o valor de $P$ é justamente $\dfrac{2}{3}.$


2534   

Determine a massa e o centro de massa do cubo dado por $0\leq x\leq a$, $0\leq y\leq a$, $0\leq z\leq a$ e com função densidade:

  1.  $\rho(x,y,z)=x^{2}+y^{2}+z^{2}.$

  2.  $\rho(x,y,z)=x+y+z.$


  1.  Massa: $a^5;$ centro de massa: $\displaystyle \left(\frac{7a}{12},\frac{7a}{12},\frac{7a}{12} \right).$

  2.  Massa: $\dfrac{3a^4}{2};$ centro de massa: $\displaystyle \left(\frac{5a}{9},\frac{5a}{9},\frac{5a}{9} \right).$


2549   

$f(x,y) = \begin{cases}\dfrac{xy^2}{x^2 + y^2}, & \quad \text{se } (x,y) \neq (0,0), \\0, & \quad \text{se } (x,y) = (0,0), \end{cases}$ é contínua em (0,0)? Justifique.



Notemos que para $(x,y)\neq (0,0)$ a função $f$ é contínua, pois $xy^{2}$ e $x^{2}+y^{2}$ são funções contínuas e $x^{2}+y^{2}\neq 0.$ Agora, estudemos a continuidade da função $f$ no ponto $(0,0).$ Temos que

$$\lim_{(x,y)\to (0,0)}f(x,y)=\lim_{(x,y)\to (0,0)}\frac{xy^{2}}{x^{2}+y^{2}}=\lim_{(x,y)\to (0,0)}x\cdot \frac{y^{2}}{x^{2}+y^{2}}.$$

Como

$$\lim_{(x,y)\to (0,0)}x=0\,\,\,\,\,\, \mbox{e}\,\,\,\,\,\, \bigg| \dfrac{y^{2}}{x^{2}+y^{2}}\bigg|\leq 1,\, \forall (x,y)\neq (0,0),$$

obtemos que

$$\lim_{(x,y)\to (0,0)}f(x,y)=0.$$

Assim,

$$\lim_{(x,y)\to (0,0)}f(x,y)=0=f(0,0).$$

Portanto, $f$ é contínua em $(0,0).$


2061   

Determine se ${\bf F}(x,y,z)=(y\,\sin z)\,{\bf i}+(x\,\sin z)\,{\bf j}+(xy\,\cos z)\,{\bf k}$ é ou não um campo vetorial conservativo. Se for, determine uma função $f$ tal que ${\bf F}=\nabla f.$


Sim. $f(x,y,z) = xy\sin(z) + K.$


2720   

Determine a diferencial da função $z = x^3 \ln{y^2}$.


$dz = 3x^{2} \ln (y^{2})dx + \frac{2x^{3}}{y} dy$.


2476   

Determine e faça o esboço do domínio da função $\bigstar$ $f(x,y)=\dfrac{\sqrt{y-x^{2}}}{1-x^{2}}$.


$\left\lbrace (x,y);\; y \geq x^{2},\; x\neq \pm 1 \right\rbrace.$

A expressão 1 cossec x e o mesmo que


3029   

Esboce a região de integração e mude a ordem de integração. $\displaystyle\int_{0}^{3}\!\!\int_{-\sqrt{9-y^{2}}}^{\sqrt{9-y^{2}}}f(x,y)\,dx dy$.


A expressão 1 cossec x e o mesmo que


2060   

Determine se ${\bf F}(x,y,z)=(e^{x}\,\cos y)\,{\bf i}-(e^{x}\,\sin y)\,{\bf j}+z\,{\bf k}$ é ou não um campo vetorial conservativo. Se for, determine uma função $f$ tal que ${\bf F}=\nabla f.$


Sim. $f(x,y,z) = e^{x}\cos(y) + \dfrac{z^{2}}{2}  + K.$


2805   

Determine os valores máximos e mínimos locais e pontos de sela da função $f(x,y)=x^{3}-12xy+8y^{3}$.


Ponto de mínimo : $\displaystyle \left( 2,1\right);$ ponto de sela: $\displaystyle \left(0,0\right).$


2102   

Seja ${\bf F}=\nabla f$, onde $f(x,y)=\sin(x-2y)$. Determine curvas $C_{1}$ e $C_{2}$ que não sejam fechadas e satisfaçam a equação.

  1. $\displaystyle\int_{C_{1}}{\bf F}\cdot d{\bf r}=0$

  2. $\displaystyle\int_{C_{2}}{\bf F}\cdot d{\bf r}=1$


  1. $\mathbf{r}(t) = \pi t \mathbf{i} + \pi t \mathbf{j},$ $0 \leq t \leq 1.$

  2. $\mathbf{r}(t) = \dfrac{\pi}{2} t \mathbf{i},$ $0 \leq t \leq 1.$


2147   

Use o Teorema do Divergente para calcular o fluxo de ${\bf F}$ através de $S,$ onde ${\bf F}(x,y,z)=e^{x}\,\sin y\,{\bf i}+e^{x}\,\cos y\,{\bf j}+yz^{2}\,{\bf k}$ e $S$ é a superfície da caixa delimitada pelos planos $x=0$, $x=1$, $y=0$, $y=1$, $z=0$ e $z=2.$


3008   

Calcule o centro de massa da região: $D$ o triângulo de vértices $(0,0), (0,1)$ e $(1,1)$ e a densidade é proporcional à distância do ponto à origem.


$\displaystyle \left(\frac{3}{4}, \frac{2\sqrt{2} - 1}{2\sqrt{2} + 2\ln(1 + \sqrt{2})} \right).$


2674   

Calcule as derivadas parciais de $w = x^2 \arcsin{\dfrac{y}{z}}$.


$\displaystyle \frac{\partial w}{\partial x} = 2x \arcsin \left( \frac{t}{z}\right),\;\;\;\;  \frac{\partial w}{\partial y} = \frac{x^{2}|z|}{z\sqrt{z^{2} - y^{2}}} \;\;\;\;\;\text{e}\;\;\;\;\;\frac{\partial w}{\partial z} = - \frac{x^{2}y}{|z|\sqrt{z^{2} - y^{2}}}.$


1981   

 Uma partícula se move no plano $xy$ de tal maneira que sua posição no instante $t$ é
$$\textbf{r}(t) = (t - \sin{t} )\textbf{i} + (1 - \cos{t})\textbf{j}.$$
Trace o gráfico de $\textbf{r}(t)$. A curva resultante é chamada de ciclóide.


2052   

Determine se ${\bf F}(x,y)=-y\,{\bf i}+x\,{\bf j}$ é ou não um campo vetorial conservativo. Se for, determine uma função $f$ tal que ${\bf F}=\nabla f.$


Não.


2816   

Determine os valores máximo e mínimo absolutos de $f$ no conjunto $D.$

$f(x,y)=x^{3}-3x-y^{3}+12y$, $D$ é o quadrilátero cujos vértices são $(-2,3)$, $(2,3)$, $(2,2)$ e $(-2,-2).$


Valor máximo: $18;$  valor mínimo: $-18.$


2229   

Mostre que qualquer campo vetorial da forma

$$\mathbf{F}(x,y,z) = f(x)\mathbf{i} + g(y)\mathbf{j} + h(z)\mathbf{k},$$

em que $f,g$ e $h$ são diferenciáveis, é irrotacional.


Note que $\text{rot } \mathbf{F} = \bf{0}.$


2315   

Determine a derivada direcional de $f(x,y,z) = xy + yz + zx$ em $P = (1,-1,3)$ na direção de $Q = (2,4,5)$.


$\displaystyle \frac{22}{\sqrt{30}}.$


2067   

Dados ${\bf F}(x,y,z)=e^{y}\,{\bf i}+xe^{y}\,{\bf j}+(z+1)e^{z}\,{\bf k}$, $C: {\bf r}(t)=t\,{\bf i}+t^{2}\,{\bf j}+t^{3}\,{\bf k}$, $0\leq t\leq 1.$

  1. Determine uma função $f$ tal que ${\bf F}=\nabla f$.

  2. Use o resultado anterior para calcular $\int_{C}{\bf F}\cdot d{\bf r}$ sobre a curva $C$ dada.


  1. $f(x,y,z) = x e^{y} + ze^{z};$

  2. $2e.$


2445   

Calcule a integral de superfície $\displaystyle\iint\limits_{S}xy dS$, onde $S$ é a superfície com equações paramétricas $x=u-v$, $y=u+v$, $z=2u+v+1$, $0 \leq u \leq 1$, $0 \leq v \leq u.$


2208   

Calcule a integral dupla.

  1.  $\displaystyle\iint\limits_{ D}x^{3}y^{2}\,dA, \quad D=\{(x,y) \in \mathbb{R}^2|\;0\leq x\leq 2,\;-x\leq y\leq x\}.$

  2.  $\displaystyle\iint\limits_{D}x\,dA, \quad D=\{(x,y) \in \mathbb{R}^2|\;0\leq x\leq \pi,\;0\leq y\leq \sin{x}\}.$

  3. $\displaystyle\iint\limits_{D}x^{3}\,dA, \quad D=\{(x,y) \in \mathbb{R}^2|\;1\leq x\leq e,\;0\leq y\leq \ln(x)\}.$

  4. $\displaystyle\iint\limits_{D}y^{2}e^{xy}\,dA, \quad D=\{(x,y) \in \mathbb{R}^2|\;0\leq y\leq 4,\;0\leq x\leq y\}.$

  5. $\displaystyle\iint\limits_{D}y^{3}\,dA, \quad D$ região com vértices $(0,2)$, $(1,1)$ e $(3,2).$


  1.  $\dfrac{256}{21}.$

  2.  $\pi.$

  3.  $\dfrac{3e^{4} + 1}{16}.$

  4.  $\dfrac{e^{16} - 17}{2}.$

  5.  $\dfrac{147}{20}.$


2189   

Use o Teorema do Divergente para calcular o fluxo de ${\bf F}$ através de $S,$ onde ${\bf F}(x,y,z)=(5x^{3}+12xy^{2})\,{\bf i}+(y^{3}+e^{y}\,\sin z)\,{\bf j}+(5z^{3}+e^{y}\,\cos z)\,{\bf k}$ e $S$ é a superfície do sólido entre as esferas $x^{2}+y^{2}+z^{2}=1$ e $x^{2}+y^{2}+z^{2}=2.$


2531   

Represente graficamente o domínio da função $z=f(x,y)$ dada por $z=\ln(2x^{2}+y^{2}-1)$.


$\left\lbrace (x,y); 2x^{2} + y^{2} > 1 \right\rbrace$

A expressão 1 cossec x e o mesmo que


2776   

Encontre os pontos da elipse $x^2 + xy + y^2 = 3$ mais próximos e mais distantes da origem.



A distância entre um ponto $(x,y)$ e a origem $(0,0)$ é

$$d=\sqrt{(x-0)^{2}+(y-0)^{2}}=\sqrt{x^{2}+y^{2}}.$$

Mas a álgebra fica mais simples se maximizarmos e minimizarmos o quadrado da distância:

$$d^{2}=f(x,y)=x^{2}+y^{2}.$$

A restrição é que os pontos pertencem a elipse, ou seja,

$$g(x,y)=x^{2}+xy+y^{2}=3$$

De acordo com os multiplicadores de Lagrange, resolvemos $\nabla f=\lambda \nabla g$ e $g=3.$ Então

$$\nabla f(x,y)=(2x,2y)$$

e

$$\lambda \nabla g(x,y)=\lambda(2x+y,x+2y)=(2x\lambda+y\lambda,2y\lambda+x \lambda).$$

Logo temos,

\begin{array}{rcl}2x=2x\lambda+y\lambda\\2y=2y\lambda+x\lambda\\x^{2}+xy+y^{2}=3\\end{array}

Se $\lambda=0$ teremos que $x=0$ e $y=0$, mas esses valores não satisfazem equação $(3)$. Logo $\lambda \neq 0$ e multiplicando

ambos os lados  da equação $(1)$ por $\dfrac{y}{\lambda}$ e ambos os lados da equação $(2)$ por $\dfrac{x}{\lambda}$, obtemos que

$$\frac{2xy}{y}=2xy+y^{2}\;\;\;\;\;\;\;\; \mbox{e}\;\;\;\;\;\;\;\; \frac{2xy}{y}=2xy+x^{2}.$$

Logo,

$$y^{2}=x^{2}\Rightarrow y=x\;\;\;\; \mbox{ou}\;\;\;\; y=-x.$$

Se $y=x$ temos que da equação $(3)$ que $x^{2}+x^{2}+x^{2}=3\Rightarrow x^{2}=1\Rightarrow x=\pm 1.$

Logo temos os pontos $(1,1)$ e $(-1, -1).$

Se $y=-x$ temos que da equação $(3)$ que $x^{2}-x^{2}+x^{2}=3\Rightarrow x^{2}=3\Rightarrow  x=\pm \sqrt{3}.$

Logo temos os pontos $(\sqrt{3},-\sqrt{3})$ e $(-\sqrt{3},\sqrt{3}).$

Os valores de $f$ nesses pontos são:

$$f(1,1)=f(-1,-1)=2\;\;\;\;\;\;\;\; \mbox{e}\;\;\;\;\;\;\;\; f(\sqrt{3},-\sqrt{3})=f(-\sqrt{3},\sqrt{3})=6.$$

Portanto, $(1,1)$ e $(-1, -1)$ são os pontos mais próximos e $(\sqrt{3},-\sqrt{3})$ e $(-\sqrt{3},\sqrt{3})$ os pontos mais afastados da origem $(0,0).$


1942   

Calcule a integral de linha, onde $C$ é a curva dada.

$\displaystyle\int_{C}x^{2}y\sqrt{z}\,dz$,   $C:\,x=t^{3},\, y=t,\, z=t^{2},\, 0\leq t\leq 1.$


$\dfrac{1}{5}.$


2729   

Determine as equações do plano tangente e da reta normal ao gráfico da função dada, no ponto dado. $f(x,y) = 2x^2y$ em $(1,1,f(1,1))$.


Plano tangente: $z = 4x + 2y - 4$

Reta normal: $(x,y,z) = \left(1,1,2 \right) + \lambda \left(4,2,-1 \right)$.


3077   

Determine \(\displaystyle \lim_{(x,y)\to (0,0)}(x^2+y^2)\ln(x^2+y^2). \)



Usando coordenadas polares, teremos que: \[\begin{array}{lll} x=r\cos\theta, & y=r\sin\theta, & r^2=x^2+y^2. \end{array} \] Além disso, como \(r=\sqrt{x^2+y^2}\geq 0\), temos que \( r\rightarrow 0^+\) se, e somente se, \( (x,y)\rightarrow (0,0) \). Assim, segue para o limite dado que \begin{align*} \lim_{(x,y)\to (0,0)}(x^2+y^2)\ln(x^2+y^2) & = \lim_{r\to 0^+} r^2\ln r^2 \\     & = \lim_{r\to 0^+}\underbrace{\dfrac{2\ln r}{1/r^2}}_{\text{do tipo}\ \infty/\infty} \\     & \stackrel{\text{L'Hospital}}{=} \lim_{r\to 0^+} \dfrac{2/r}{-2/r^3} \\    & = \lim_{r\to 0^+} (-r^2) = 0.\end{align*}


2564   

Determine o maior conjunto no qual a função $f(x,y) = \begin{cases}\dfrac{x^2y^3}{2x^2 + y^2}, & \quad \text{se } (x,y) \neq (0,0), \\1, & \quad \text{se } (x,y) = (0,0) \end{cases}$ é contínua.


$\left\lbrace (x,y);\; (x,y) \neq (0,0) \right\rbrace.$


2699   

Seja $f(x,y)=\dfrac{1}{x^{2}+y^{2}}$. Verifique que

  1. $x\;\dfrac{\partial ^{2}f}{\partial x^{2}}(x,y)+y\;\dfrac{\partial^{2} f}{\partial y \partial x}(x,y)=-3\dfrac{\partial f}{\partial x}(x,y)$

  2. $\dfrac{\partial ^{2}f}{\partial x^{2}}(x,y)+\dfrac{\partial^{2} f}{\partial y^{2}}(x,y)=\dfrac{4}{(x^{2}+y^{2})^{2}}$


$\begin{aligned}[t]\frac{\partial f}{\partial x} &= -\frac{2x}{(x^{2} + y^{2})^{2}},\;\;\;\;\; \frac{\partial^{2} f}{\partial x^{2}}= \frac{6 x^{2} - 2y^{2}}{(x^{2} + y^{2})^{3}},\;\;\;\;\; \frac{\partial^{2} f}{\partial y^{2}}= \frac{6 y^{2} - 2x^{2}}{(x^{2} + y^{2})^{3}} \;\;\;\;\;\text{e}\\ \frac{\partial^{2} f}{\partial y\partial x} &= \frac{8xy}{(x^{2} + y^{2})^{3}}.\end{aligned}$


2742   

Determine os planos que são tangentes ao gráfico de $f(x,y) = x^2 + y^2$ e que contenham a interseção dos planos $x + y + z = 3$ e $z = 0$.


$z = 0$ e $z = 6x + 6y - 18.$


2899   

Determine o valor máximo de $f(x,y,z) = 6x + z$ sobre a curva de interseção das superfícies $x^2 + y^2 = 4$ e $z = x^2 - 2y^2$.


$16.$


1926   

Determine o campo vetorial gradiente de  $f(x,y) = \ln(x + 2y)$.


$\nabla f(x,y) = \dfrac{\textbf{i} + 2\textbf{j}}{x + 2y}.$


2025   

Suponha que em uma certa região do espaço o potencial elétrico $V$ seja dado por $V(x,y,z) = 5x^2 - 3xy + xyz.$

  1. Determine a taxa de variação do potencial em $P = (3,4,5)$ na direção do vetor $\bf{v} = \bf{i} + \bf{j} - \bf{k}$.
  2. Em que direção $V$ varia mais rapidamente em $P$?
  3. Qual a taxa máxima de variação em $P$?



  1. Queremos determinar o valor de $D_\bf{u}f(P)$, em que $\bf{u}$ é o vetor unitário que tem mesma direção de $\bf{v}$, isto é, $\bf{u} = \frac{1}{\sqrt{3}}(1,1,-1)$. Como $V$ é diferenciável, segue que  $D_\bf{u}f(P) = \nabla V(P) \cdot \mathbf{u}$. Observe que \linebreak $\nabla V(x,y,z) = (10x-3y+yz,-3x+xz,xy)$. Logo $\nabla V(P) = (38,6,12)$. Portanto, $$D_\bf{u}f(P) = \nabla V(P) \, \cdot \, \mathbf{u} = (38,6,12) \, \cdot \, \dfrac{1}{\sqrt{3}}(1,1,-1) = \dfrac{32\sqrt{3}}{3}.$$
  2. A direção em que $V$ varia mais rapidamente no ponto $P$ é a direção do gradiente de $V$ no ponto $P$, isto é, na direção de $\nabla V(P) = (38,6,12)$. Observe que aqui não é necessário normalizar o vetor, pois o exercício pede apenas a direção.
  3. A taxa de variação máxima é $|\nabla V(P)| = 2\sqrt{406}$.


2505   

Faça um esboço do diagrama de contorno da função cujo gráfico é mostrado.

A expressão 1 cossec x e o mesmo que


$y = 2x \pm \sqrt{C},$ $C \geq 0.$

A expressão 1 cossec x e o mesmo que


2545   

Seja $S$ a superfície $z=f(x,y)$, $(x,y)\in K$, de classe $C^{1}$ num aberto contendo $K$. (Observação: trata-se da superfície dada por $x=u$, $y=v$ e $z=f(u,v)$). Seja ${\bf n}$ a normal a $S$ com componente $z>0$ e seja ${\bf F}=P{\bf i}+Q{\bf j}+R{\bf k}$ um campo vetorial contínuo na imagem de $S$. Mostre que $\displaystyle\iint\limits_{S}{\bf F}\cdot {\bf n}dS=\displaystyle\iint\limits_{K}\left[ -P\dfrac{\partial f}{\partial x}(x,y)-Q\dfrac{\partial f}{\partial y}+R\right]dx dy,$ onde $P$, $Q$ e $R$ são calculadas em $(x,y,f(x,y)).$



Veja a subseção "Integrais de superfície de campos vetoriais"' da seção 16.7 do livro do Stewart.


2788   

Utilize as curvas de nível da figura para predizer a localização dos pontos críticos de $f(x,y)=3x-x^{3}-2y^{2}+y^{4}$ e se $f$ tem um ponto de sela ou um máximo ou mínimo local em cada um desses pontos. Explique seu raciocínio. Em seguida, empregue o Teste da Segunda Derivada para confirmar suas predições.

A expressão 1 cossec x e o mesmo que


$f$ possui um ponto de máximo local em $(1,0),$ pontos de sela em $(1,1),$ $(1,-1)$ e $(-1,0)$ e pontos de mínimo local em $(-1,1)$ e $(-1,-1).$


3036   

Uma região $R$ é mostrada na figura. Decida se você deve usar coordenadas polares ou retangulares e escreva $\iint \limits_{R}f(x,y)\,dA$ como uma integral iterada, onde $f$ é uma função qualquer contínua em $R.$


A expressão 1 cossec x e o mesmo que


$\displaystyle \int_{-1}^{1} \int_{0}^{\frac{(x + 1)}{2}} f(x,y)  dy dx .$


2974   

Determine o jacobiano da transformação dada por: $x = uv, \quad y = \dfrac{u}{v}$.


$-\dfrac{2u}{v}.$


2403   

  1.  Duas superfícies são ditas \textbf{ortogonais} em um ponto de intersecção se suas normais são perpendiculares nesse ponto. Mostre que superfícies com equação $F(x,y,z) = 0$ e $G(x,y,z) = 0$ são ortogonais em um ponto $P$, em que $\nabla F \neq 0$ e $\nabla G \neq 0$, se, e somente se, em $P$, $$F_xG_x + F_yG_y + F_zG_z = 0.$$
  2. Use a parte 1. para mostrar que as superfícies $z^2 = x^2 + y^2$ e $x^2 + y^2 + z^2 = r^2$ são ortogonais em todo ponto de intersecção. Você pode ver isso sem fazer os cálculos?



  1.  Note que a direção da normal de $F$ é dada por $\nabla F,$ a de $G$ por $\nabla G$  e que duas normais em $P$ são perpendiculares se $\nabla F \cdot \nabla G = 0.$
  2.  Tome $F = x^2 + y^2 - z^2,$ $G = x^2 + y^2 + z^2 - r^2$ e verifique $(a).$ Para "ver" isso sem calcular, note que $F = 0$ é a equação de um cone circular com vértice na origem e $G = 0$ é a equação de uma esfera centrada na origem.


2494   

Dada a função $f(x,y)=x^{2}-y^{2}$.

  1. Encontre o domínio da função.

  2. Encontre a imagem da função.

  3. Descreva as curvas de nível da função.


  1. $D_{f} = \mathbb{R}^{2}.$

  2. $Im(f) = \mathbb{R}.$

  3. As curvas de nível são as hipérboles $x^{2} - y^{2} = C$ com foco no eixo $x$ se $C > 0;$ com foco no eixo $y$ se $C < 0$ e as retas $y = \pm x$ se $C = 0.$


2177   

  1. Se $C$ é o segmento de reta ligando o ponto $(x_1,y_1)$ ao ponto $(x_2,y_2)$, mostre que

    $$\int_{C}x \, dy - y \, dx = x_1y_2-x_2y_1.$$

  2. Se os vértices de um polígono, na ordem anti-horária, são

    $$ (x_1,y_1), (x_2,y_2), \ldots, (x_n,y_n), $$

    mostre que a área do polígono é

    $$A=\dfrac{1}{2}[(x_1y_2-x_2y_1) + (x_2y_3-x_3y_2) + \cdots + (x_{n-1}y_n - x_ny_{n-1}) + (x_ny_1-x_1y_n)].$$

  3. Determine a área do pentágono com vértices $(0,0)$, $(2,1)$, $(1,3)$, $(0,2)$ e $(-1,1)$.


  1. Use as equações paramétricas do segmento de reta: $x = (1-t)x_{1} + tx_{2}$ e $y = (1-t)y_{1} + ty_{2},$ $0 \leq t \leq 1.$

  2. Aplique o Teorema de Green ao caminho $C = C_{1} \cup C_{2} \cup \cdots \cup C_{n},$ onde $C_{i}$ é o segmento ligando o ponto $(x_{i},y_{i})$ ao ponto $(x_{i + 1},y_{i + 1}),$ para cada $i = 1,\cdots, n-1.$

  3. $\dfrac{9}{2}.$


2985   

  1. Calcule $\iiint\limits_{E} dV$, em que $E$ é o sólido delimitado pelo elipsoide  $x^2/a^2 + y^2/b^2 + z^2/c^2 = 1$. Utilize a transformação $x = au$, $y = bv$ e $z = cw$.

  2. A Terra não é perfeitamente esférica; como resultado da rotação, os polos foram achatados. Assim, seu formato pode ser aproximado por um elipsoide com $a = b = 6.378$ km e $c = 6.356$ km. Use o item anterior para estimar o volume da Terra.


  1. $\dfrac{4\pi a b c}{3}.$

  2. $\dfrac{4\pi (6378) (6378) (6356)}{3} \approx 1.083 \times 10^{12}$ km$^{3}.$


3125   

Determine a equação do plano tangente à superfície descrita parametricamente por \(x=u\cosh v\), \(y=u\sinh v\), \(z=u^2\) no ponto \((-3,0,9)\).


2435   

Calcule a integral tripla.

  1. $\displaystyle\iiint\limits_{  E}2z\;dx dy dz$, onde $E$ é o conjunto $x^{2}+y^{2}+z^{2}\leq 4$ e $z\geq 0.$

  2. $\displaystyle\iiint\limits_{  E}2z\;dx dy dz$, onde $E$ é o conjunto $4x^{2}+9y^{2}+z^{2}\leq 4$ e $z\geq 0.$


  1.  $8\pi.$

  2.  $0.$


2757   

Verifique que a função $f(x,y) = x^2y$ é diferenciável.


As derivadas parciais $\frac{\partial f}{\partial x}$ e $\frac{\partial f}{\partial y}$ de cada função $f$ existem e são contínuas em todos os pontos do domínio.


1943   

Calcule a integral de linha, onde $C$ é a curva dada.

$\displaystyle\int_{C}(x+yz)\,dx+2x\,dy+xyz\,dz$,   $C$ consiste nos segmentos de reta de $(1,0,1)$ a $(2,3,1)$ e de $(2,3,1)$ a $(2,5,2).$


$\dfrac{97}{3}.$


2633   

Determine as derivadas parciais de primeira ordem da função $f(x,y)=\displaystyle\int_{y}^{x}\cos^2t \ \mathrm{d}t$.



Sendo $f(x,y)=\displaystyle\int_{y}^{x}\cos (t^{2})\,dt$, temos que as derivadas parciais em relação a $x$ e $y$, respectivamente, são:

$\bullet \dfrac{\partial}{\partial x}f(x,y)=\dfrac{\partial}{\partial x}\bigg(\displaystyle\int_{y}^{x}\cos(t^{2})\bigg)=\cos(x^{2}).$

$\bullet \dfrac{\partial}{\partial y}f(x,y)=\dfrac{\partial}{\partial y}\bigg(\displaystyle\int_{y}^{x}\cos(t^{2})\bigg)=\dfrac{\partial}{\partial y}\bigg(-\displaystyle\int_{x}^{y}\cos(t^{2})\bigg)=-\cos(y^{2}).$

Notemos que nas soluções das derivadas parciais acima utilizamos o Teorema Fundamental do Cálculo.


2838   

Calcule a integral dupla usando coordenadas polares: $\displaystyle\iint\limits_{R}y\,dA$, onde $R$ é a região no primeiro quadrante limitada pelo semi-círculo $x^{2}+y^{2}=2x.$


$\displaystyle \frac{2}{3}.$


2797   

Determine os valores máximos e mínimos locais e pontos de sela da função $f(x,y)=e^{x}\cos{y}$.


Não há pontos críticos.


2675   

Calcule as derivadas parciais de $w = \dfrac{xyz}{x + y + z}$.


$\displaystyle \frac{\partial w}{\partial x} = \frac{yz(y+z)}{(x+y+z)^{2}},\;\;\;\;  \frac{\partial w}{\partial y} = \frac{xz(x+z)}{(x+y+z)^{2}}\;\;\;\;\;\text{e}\;\;\;\;\;\frac{\partial w}{\partial z} = \frac{xy(x+y)}{(x+y+z)^{2}}.$


2341   

Determine a área da superfície dada pela porção do cone $z=2\sqrt{x^{2}+y^{2}}$ entre os planos $z=2$ e $z=6.$


$8\sqrt{5}\pi.$


2357   

Calcule o volume do conjunto dado.

  1.  $4x+2y\geq z\geq 3x+y+1$, $x\geq 0$ e $y\geq 0.$

  2.  $0\leq z\leq \sin{y^{3}}$ e $\sqrt{x}\leq y\leq \sqrt[3]{\pi}.$


  1.  $\dfrac{1}{6}.$

  2.  $\dfrac{2}{3}.$


2773   

Determine os valores máximos e mínimos locais e pontos de sela da função $f(x,y)=y\cos{x}$.



Sendo $f(x,y)=y\,\cos x$, vamos inicialmente localizar seus pontos críticos:

$$f_{x}(x,y)=-y\,\sin x \;\;\;\;\;\;\;\; \mbox{e} \;\;\;\;\;\;\;\; f_{y}(x,y)=\cos x.$$

Igualando essas derivadas parciais a zero, obtemos as equações

$$y\,\sin x=0 \;\;\;\;\;\;\;\; \mbox{e} \;\;\;\;\;\;\;\; \cos x=0.$$

Da segunda equação obtemos que $x=\bigg(\dfrac{\pi}{2}+n\pi\bigg)$, $n\in \mathbb{Z}.$ Da primeira equação temos que $y=0$ para todos essas $x$-valores.

Assim, os pontos críticos são $\bigg(\dfrac{\pi}{2}+n\pi,0\bigg).$ Agora,

$$f_{xx}(x,y)=-y\,\cos x,\;\;\;\;\;\; f_{xy}(x,y)=-\sin x\;\;\;\;\;\;\; \mbox{e}\;\;\;\;\;\; f_{yy}(x,y)=0.$$

Então

\begin{array}{rcl}D(x,y)&=&(f_{xx}(x,y))\cdot (f_{yy}(x,y))-(f_{xy}(x,y))^{2}\\&\Rightarrow& D\bigg(\dfrac{\pi}{2}+n\pi,0\bigg)=0-\sin^{2}x=-\sin^{2}x<0.\end{array}

Portanto, cada ponto crítico é ponto de sela.


2884   

Determine o ponto da reta $x + 2y = 1$ cujo produto das coordenadas seja máximo.


$\displaystyle \left( \frac{1}{2}, \frac{1}{4} \right).$


2490   

Faça uma correspondência entre a função e seu gráfico (indicado por I-VI). Dê razões para sua escolha.

  1. $f(x,y)=|x|+|y|$.

  2. $f(x,y)=\dfrac{1}{1+x^{2}+y^{2}}$.

  3. $f(x,y)=(x-y)^{2}$.

  4. $f(x,y)=|xy|$.

  5. $f(x,y)=(x^{2}-y^{2})^{2}$.

  6. $f(x,y)=\sin(|x|+|y|)$.

A expressão 1 cossec x e o mesmo que


2957   

Usando coordenadas esféricas, determine o volume da região cortada do cilindro sólido $x^{2}+y^{2}\leq 1$ pela esfera $x^{2}+y^{2}+z^{2}=4.$


$\dfrac{4\pi(8 - 3\sqrt{3})}{3}.$


2553   

Determine o limite, se existir, ou mostre que o limite não existe.

$\displaystyle \lim_{(x,y) \to  (5,-2)}(x^5 + 4x^3y - 5xy^2)$.


$2025.$


2503   

Encontre uma equação para a curva de nível da função $f(x,y)=\displaystyle \int_{x}^{y}\dfrac{dt}{1+t^{2}}$ que passa pelo ponto $(-\sqrt{2},\sqrt{2})$.


$\arctan(y) - \arctan(x) = 2\arctan(\sqrt{2}).$


2188   

Use o Teorema do Divergente para calcular o fluxo de ${\bf F}$ através de $S,$ onde ${\bf F}(x,y,z)=2xz\,{\bf i}+xyz\,{\bf j}+yz\,{\bf k}$ e $S$ é a superfície da região delimitada pelos planos coordenados e os planos $x+2z=4$ e $y=2.$


3137   

Calcule o trabalho realizado pelo campo vetorial \[ \mathbf{F}(x,y,z) = x^2\mathbf{i}+4xy^3\mathbf{j}+y^2x\mathbf{k}\] sobre uma partícula que percorre o caminho \(C\) definido como o bordo da superfície \(\sigma\) contida no plano \(z=y\) e cuja projeção no plano \(xy\) corresponde ao retângulo \(R=\{(x,y)\in\mathbb{R}^2; 0\leq x\leq 1\),\ \(0\leq y\leq 3\}\). O sentido de percurso é tal que a fronteira de \(R\) é percorrida no sentido horário.



Note que calcular o trabalho \(\displaystyle W= \oint_C\mathbf{F}\cdot\,d\mathbf{r}\) assim diretamente exigiria quatro integrações separadas, uma para cada lado do retângulo. Entretanto, usando o Teorema de Stokes podemos, em vez disso, calcular uma (única!) integral de superfície \[ W= \iint\limits_\sigma\mathrm{rot\,}\mathbf{F}\cdot\mathbf{n}\,dS \] na qual \(\sigma\) é tomada com a orientação para baixo, como requerido pelo Teorema de Stokes. Como a superfície \(\sigma\) está contida no plano \(z=y\) e \[\mathrm{rot\,}\mathbf{F} = \left|\begin{array}{ccc} \mathbf{i} & \mathbf{j} & \mathbf{k} \\ \dfrac{\partial}{\partial x} & \dfrac{\partial}{\partial y} & \dfrac{\partial}{\partial z} \\ x^2 & 4xy^3 & xy^2 \end{array}\right| = 2xy\mathbf{i}-y^2\mathbf{j}+4y^3\mathbf{k}, \] segue então que \begin{align*} W= \iint\limits_\sigma\mathrm{rot\,}\mathbf{F}\cdot\mathbf{n}\,dS & = \iint\limits_R\mathrm{rot\,}\mathbf{F}\cdot\left( \dfrac{\partial z}{\partial x}\mathbf{i} +\dfrac{\partial z}{\partial y}\mathbf{j} - \mathbf{k}\right)\,dA \\   & = \iint\limits_R\left(2xy\mathbf{i}-y^2\mathbf{j}+4y^3\mathbf{k}\right)\cdot\left(0\mathbf{i}+\mathbf{h}-\mathbf{k}\right)\,dA \\   & = \int_0^1\int_0^3(-y^2-4y^3)\,dydx \\   & = - \int_0^1\left[\dfrac{y^3}{3}+y^4\right]_{y=0}^3\,dx \\   & = -\int_0^1 90\,dx = -90. \end{align*}


2566   

Utilize coordenadas polares $x=r\cos \theta$ e $y=r\sin \theta$, com $r \geq 0$ e $0 \leq \theta < 2 \pi$, e o teorema do confronto para calcular o limite

$$\displaystyle \lim_{(x,y) \to (0,0)}\dfrac{x^3 + y^3}{x^2 + y^2}.$$

Dica: Note que, se $(r, \theta)$ são as coordenadas polares do ponto $(x,y)$, com $r \geq 0$, então $r \to 0^+$ quando $(x,y) \to (0,0)$.


$0.$


2889   

Passe para coordenadas polares e calcule: $\displaystyle\int_{0}^{a}  \int_{0}^{\sqrt{a^{2}-x^{2}}}\sqrt{a^{2}-x^{2}-y^{2}}\,dy dx$, em que $a>0.$


$\displaystyle \frac{\pi a^3}{6}.$


2893   

Passe para coordenadas polares e calcule: $\displaystyle\int_{-1}^{0} \int_{-\sqrt{1-x^{2}}}^{0}\frac{2}{1+\sqrt{x^{2}+y^{2}}}\,dy dx$


$(1 - \ln(2))\pi.$


2590   

Use a Lei de Gauss para achar a carga contida no hemisfério sólido $x^{2}+y^{2}+z^{2} \leq a^{2}$, $z\geq 0$, se o campo elétrico for ${\bf E}(x,y,z)=x{\bf i}+y{\bf j}+2z{\bf k}$.


$\dfrac{8\pi a^3 \epsilon_{0}}{3}$.


3045   

Se uma circunferência $C$ de raio $1$ rola ao longo do interior da circunferência $x^2+y^2=16$, um ponto fixo $P$ de $C$ descreve uma curva chamada epicicloide, com equações paramétricas $x = 5\cos{t}-\cos{5t}$, $y = 5\sin{t} - \sin{5t}$. Faça o gráfico da epicicloide e calcule a área da região que ela envolve.


$30\pi.$

A expressão 1 cossec x e o mesmo que


2743   

Determine os planos tangentes ao gráfico de $f(x,y) = 2 + x^2 + y^2$ e que contenham o eixo $x$.


$z = 2\sqrt{2} y$ e $z = -2\sqrt{2} y.$


3004   

Determine a massa e o centro de massa da lâmina que ocupa a região $D$ e tem função densidade $\rho$, sendo: $D$ delimitada pelas parábolas $y = x^2$ e $x = y^2; \quad \rho(x,y) = \sqrt{x}$.


Massa: $\dfrac{3}{14};$ centro de massa: $\displaystyle \left(\frac{14}{27},\frac{28}{55} \right).$


2458   

O índice I de temperatura-umidade (ou simplesmente humidex) é a temperatura aparente do ar quando a temperatura real é $T$ e a umidade relativa é $h$, de modo que podemos escrever $I=f(T,h)$. A tabela seguinte com valores de $I$ foi extraída de uma tabela do Environment Canada.

A expressão 1 cossec x e o mesmo que

  1. Qual é o valor de $f(35,60)$? Qual é o seu significado?

  2. Para que valor de $h$ temos $f(30,h)=36$?

  3. Para que valor de $T$ temos $f(T,40)=42$?

  4. Qual o significado de $I=f(20,h)$ e $I=f(40,h)$? Compare o comportamento dessas duas funções de $h.$


  1. 48, o que significa que quando a temperatura real é $35^\circ$C e a umidade relativa é $60\%,$ o humidex é $48^\circ$C.

  2. $50\%.$

  3. $35^\circ$C.

  4. $I = f(20,h)$ e $I = f(40,h)$ são funções de $h$ que fornecem os valores do humidex quando a temperatura real é $20^\circ$C e $40^\circ$C, respectivamente. Ambas as funções crescem com $h,$ porém $f(20,h)$ cresce aproximadamente a taxa constante, enquanto $f(40,h)$ cresce mais rapidamente a uma taxa crescente.


2112   

 Suponha que a equação $F(x,y,z)=0$ defina implicitamente cada uma das três variáveis $x$,$y$ e $z$ como função das outras duas: 
$z=f(x,y)$, $y=g(x,y)$ e $x=h(y,z).$ Se $F$ for diferenciável e $F_{x}$,$F_{y}$ e $F_{z}$ forem todas não nulas, mostre que
$$\frac{\partial z}{\partial x} \frac{\partial x}{\partial y}\frac{\partial y}{\partial z}=-1.$$



Note que$\displaystyle \frac{\partial z}{\partial x} = -\frac{F_{x}}{F_{z}},$$\displaystyle \frac{\partial x}{\partial y} = -\frac{F_{y}}{F_{x}}$e$\displaystyle \frac{\partial y}{\partial z} = -\frac{F_{z}}{F_{y}}.$


2324   

Calcule a área da superfície dada por: ${\bf r}(u,v)=(u,v,u^{2}+v^{2})$ e $u^{2}+v^{2}\leq 4.$. (Sugerimos ao leitor desenhar a imagem da superfície.)


$\dfrac{\pi}{6}(17 \sqrt{17} - 1).$


2954   

Usando coordenadas esféricas, determine o volume do elipsoide $\dfrac{x^{2}}{a^{2}}+\dfrac{y^{2}}{b^{2}}+\dfrac{z^{2}}{c^{2}}\leq 1.$


$\dfrac{4 \pi abc}{3}.$


2671   

Seja $\phi:\mathbb{R}\rightarrow \mathbb{R}$ uma função diferenciável de uma variável real e seja $f(x,y)=(x^{2}+y^{2})\phi \bigg(\dfrac{x}{y}\bigg).$

Mostre que

$$x\;\frac{\partial f}{\partial x}+y\;\frac{\partial f}{\partial y}=2f.$$


$\displaystyle \frac{\partial f}{\partial x} = 2x \phi \left( \frac{x}{y} \right) + \frac{(x^{2} + y^{2})}{y} \phi'\left( \frac{x}{y} \right)\ \;\;\;\;\;\text{e}\;\;\;\;\; \frac{\partial f}{\partial y} = 2y \phi \left( \frac{x}{y} \right) - \frac{x(x^{2} + y^{2})}{y^{2}} \phi'\left( \frac{x}{y} \right).$


2183   

Nos item abaixo: 

  1. expresse $\mathrm{d} w/\mathrm{d} t$ como uma função de $t$, usando a Regra da Cadeia, expressando $w$ em termos de $t$ e diferenciando em relação a $t$;
  2. calcule $\mathrm{d} w/\mathrm{d} t$ no valor dado de $t$.

$w=x^{2}+y^{2}$,  $x=\cos{t}+\sin{t}$,  $y=\cos{t}-\sin{t}$;  $t=0.$


  1. $\displaystyle \frac{dw}{dt}(t) = 0.$
  2. $\displaystyle \frac{dw}{dt}(0) = 0.$


2155   

Use o Teorema de Green para calcular a integral de linha ao longo da curva dada com orientação positiva.

$\displaystyle\int_{C}y^3 \, dx - x^3 \, dy$, $C$ é o círculo $x^2 + y^2 = 4$.



Observe que a curva $C$ com orientação positiva está nas hipóteses do Teorema de Green, assim como o campo $\mathbf{F}(x,y) = (y^3, -x^3)$. Logo,

$$\displaystyle\int_{C}y^3 \, dx - x^3 \, dy  =  \iint\limits_{D} \left(\frac{\partial}{\partial x}(-x^3) - \frac{\partial}{\partial y}(y^3)\right) \, dA = -3 \iint\limits_{D} (x^2 + y^2) \, dA,$$

em que $D = \{(x,y) \in \mathbb{R}^2: x^2 + y^2 \leq 4\}$. Usando coordenadas polares

$$\begin{cases}x = r \cos{\theta} \\y = r \sin{\theta}, \\\end{cases}$$

temos que a região de integração $D$ pode ser escrita como $$\{(r,\theta) \in \mathbb{R}^2: 0 \leq r \leq 2, 0 \leq \theta \leq 2 \pi\}$$ e o jacobiano dessa mudança de coordenadas é igual a $r$. Logo,

$$\iint\limits_{       D} (x^2 + y^2) dA = \displaystyle\int_{0}^{2\pi} \int_{0}^{2}r^2 \cdot r\,dr d\theta = 8\pi.$$

Portanto, $\displaystyle\int_{C}y^3 \, dx - x^3 \,dy = -24\pi$.


3109   

Encontre a área da região descrita como sendo a parte do cone \(z=\sqrt{x^2+y^2}\) dentro do cilindro \(x^2+y^2=2x\).


2571   

Calcule $\displaystyle \lim_{(x,y) \to (0,0)} \dfrac{xy}{x^2 + y^2}$, caso exista.


Não existe.


3098   

Como não há antiderivada elementar da função \(e^{x^2}\), a integral \[ \int_0^2\int_{y/2}^1 e^{x^2}\, dxdy \] não pode ser calculada integrando-se primeiro em relação a \(x\). Calcule essa integral expressando-a como uma integral iterada equivalente com ordem de integração invertida.



A região de integração é dada por \(\displaystyle R=\{(x,y)\in\mathbb{R}^2;\ 0\leq y\leq 2,\ y/2\leq x\leq 1\}\). Vamos inverter a ordem de integração sobre a região \(R\):\begin{align*} \int_0^2\int_{y/2}^1 e^{x^2}\, dxdy  & = \iint\limits_R e^{x^2}\,dA = \int_0^1\int_0^{2x} e^{x^2}\,dydx= \int_0^1\left[e^{x^2}y\right]_{y=0}^{2x}\,dx \\     & = \int_0^1 2xe^{x^2}\,dx = \left.e^{x^2}\right]_0^1 = e-1 \end{align*}


2886   

Determine o ponto do elipsóide $x^2 + 4y^2 + z^2 = 1$ que maximiza a soma $x + 2y + z$.


$\displaystyle \left( \frac{1}{\sqrt{3}}, \frac{1}{2\sqrt{3}}, \frac{1}{\sqrt{3}} \right).$


2173   

Calcule o trabalho realizado pela força $\mathbf{F}(x,y) = xy\mathbf{i}+y^2\mathbf{j}$ ao mover uma partícula da origem ao longo da reta $y=x$ até $(1,1)$ e então de volta à origem ao longo da curva $y=x^2$.


$\dfrac{1}{12}.$


2887   

Encontre o ponto da curva $x^2 - 2xy + y^2 - 2x - 2y + 1 = 0$ mais próximo da origem.


$\displaystyle \left( \frac{1}{4}, \frac{1}{4} \right).$


2385   

Expresse a integral dupla, sobre a região $R$ indicada, como uma integral iterada e ache seu valor.

  1. $\displaystyle\iint\limits_{R}(y+2x)\,dA; \quad R$ região retangular de vértices $(-1,-1)$, $(2,-1)$, $(2,4)$ e $(-1,4).$

  2. $\displaystyle\iint\limits_{R}(x-y)\,dA; \quad R$ região triangular de vértices $(2,9)$, $(2,1)$ e $(-2,1).$

  3. $\displaystyle\iint\limits_{R}xy^{2}\,dA; \quad R$ região triangular de vértices $(0,0)$, $(3,1)$ e $(-2,1).$

  4. $\displaystyle\iint\limits_{R}e^{x/y}\,dA; \quad R$ região limitada pelos gráficos de $y=2x$, $y=-x$ e $y=4.$


  1.  $\displaystyle\int_{-1}^{4} \int_{-1}^{2} (y+2x)\,dx;dy = \dfrac{75}{2}.$

  2.  $\displaystyle\int_{-2}^{2} \int_{1}^{2x + 5} x - y\,dy;dx = -48.$

  3.  $\displaystyle\int_{0}^{1} \int_{-2y}^{3y} xy^{2}\,dx;dy = \dfrac{1}{2}.$

  4.  $\displaystyle\int_{0}^{4} \int_{-y}^{y/2} e^{x/y}\,dx;dy = 8(e^{1/2} - e^{-1}).$


2304   

Identifique e faça um esboço da imagem da superfície parametrizada dada por ${\bf r}(u,v)=\bigg(v\cos u,v\sin u,\dfrac{1}{v^{2}}\bigg)$, $0\leq u\leq 2\pi$, $v>0.$


Gráfico de $f(x,y) = \dfrac{1}{x^2 + y^2}.$


2371   

Determine todos os pontos nos quais a direção de maior variação da função $f(x,y) = x^2 + y^2 - 2x - 4y$ é $\bf{i} + \bf{j}$.


 $\left\lbrace (x,y) \in \mathbb{R}^{2}; y =x + 1 \right\rbrace.$


1959   

Calcule a integral de linha $\displaystyle\int_{C}{\bf F}\cdot d{\bf r}$, onde ${\bf F}(x,y)=(y,3x)$ e $C$ é a elipse $\dfrac{x^{2}}{a^{2}}+\dfrac{y^{2}}{b^{2}}=1$, percorrida no sentido anti-horário.


$-2\pi ab.$


2951   

Seja $D$ a região limitada abaixo pelo plano $z=0$, acima pela esfera  $x^{2}+y^{2}+z^{2}=4$ e dos lados pelo cilindro $x^{2}+y^{2}=1$. Monte as integrais triplas em coordenadas esféricas que dão o volume de $D$ usando as ordens de integração a seguir.

  1. $d\rho\,d\phi\,d\theta$;

  2. $d\phi\,d\rho\,d\theta$.


  1. $\displaystyle \int_{0}^{2\pi}\int_{0}^{\pi/6}\int_{0}^{2} \rho^{2}\sin(\phi)\; d\rho d\phi d\theta + \int_{0}^{2\pi}\int_{\pi/6}^{\pi/2}\int_{0}^{\csc(\phi)} \rho^{2}\sin(\phi)\; d\rho d\phi d\theta.$

  2. $\displaystyle \int_{0}^{2\pi}\int_{1}^{2}\int_{\pi/6}^{\arcsin(1/\rho)} \rho^{2}\sin(\phi)\; d\phi d\rho d\theta + \int_{0}^{2\pi}\int_{0}^{2}\int_{0}^{\pi/6} \rho^{2}\sin(\phi)\; d\phi d\rho d\theta.$


2654   

Determine as derivadas parciais de $f(x,y)=5x^{4}y^{2}+xy^{3}+4$.


$\displaystyle \frac{\partial f}{\partial x} = 20x^{3}y^{2} + y^{3}\;\;\;\;\;\;\text{e}\;\;\;\;\; \frac{\partial f}{\partial y} = 10x^{4}y + 3xy^{2}.$


2152   

Use o Teorema do Divergente para calcular o fluxo de ${\bf F}$ através de $S,$ onde ${\bf F}(x,y,z)=x^{4}\,{\bf i}-x^{3}z^{2}\,{\bf j}+4xy^{2}z\,{\bf k}$ e $S$ é a superfície do sólido limitado pelo cilindro $x^{2}+y^{2}=1$ e pelos planos $z=x+2$ e $z=0.$


2110   

Sejam $f(x)$ e $g(x)$ duas funções contínuas, respectivamente, nos intervalos $[a,b]$ e $[c,d].$ Use o seguinte resultado $$\iint\limits_{R}f(x)g(y)\,dx dy=\bigg(\int_{a}^{b}f(x)\,dx\bigg)\bigg(\int_{c}^{d}g(y)\,dy\bigg),$$ onde $R$ é o retângulo $a\leq x\leq b$ e $c\leq y\leq d$, para calcular as integrais

  1. $\displaystyle\iint\limits_{R} \dfrac{\sin^{2}{x}}{1+4y^{2}}\,dx dy$, onde $R$ é o retângulo $0\leq x\leq \dfrac{\pi}{2},\;0\leq y\leq \dfrac{1}{2}.$

  2. $\displaystyle\iint\limits_{R} \dfrac{xy\sin{x}}{1+4y^{2}}\,dx dy$, onde $R$ é o retângulo $0\leq x\leq \dfrac{\pi}{2},\;0\leq y\leq 1.$


  1.  $\dfrac{\pi^{2}}{32}.$

  2.  $\dfrac{\ln(5)}{8}.$


2766   

Use a derivação implicíta para determinar $\partial z/\partial x$ e $\partial z/\partial y$ na expressão $\sin(xyz)=x+2y+3z$.


$\displaystyle \frac{\partial z}{\partial x} = \frac{1 - yz \cos(xyz)}{xy\cos(xyz) - 3}$

$\displaystyle \frac{\partial z}{\partial y} = \frac{2 - xz \cos(xyz)}{xy\cos(xyz) - 3} $.


2351   

Calcule o volume do conjunto dado.

  1.  $x^{2}+y^{2}\leq 1$ e $x+y+2\leq z \leq 4.$

  2.  $x\geq 0$, $y \geq 0$, $x+y\leq 1$ e $0\leq z\leq x^{2}+y^{2}.$


  1.  $2\pi.$

  2.  $\dfrac{1}{6}.$


3010   

Uma lâmina ocupa parte do disco $x^2 + y^2 \leq 1$ no primeiro quadrante. Determine o centro de massa se a densidade em qualquer ponto for proporcional à distância do ponto ao eixo $x$.


$\displaystyle \left(\frac{3}{8}, \frac{3\pi}{16} \right).$


2617   

Seja $C$ uma curva fechada, simples e lisa que está no plano $x+y+z=1$. Mostre que a integral de linha $\displaystyle\int_C zdx - 2xdy + 3ydz$ depende apenas da área da região englobada por $C$ e não da forma de $C$ ou de sua posição no plano.


$\displaystyle\int_C zdx - 2xdy + 3ydz = \dfrac{2}{\sqrt{3}} \times $ (área da região englobada por $C$).


2528   

Represente graficamente o domínio da função $z=f(x,y)$ dada por $x+y-1+z^{2}=0$, $z\geq 0$.


$\left\lbrace (x,y); x + y \leq 1 \right\rbrace$

A expressão 1 cossec x e o mesmo que


2607   

Use o Teorema de Stokes para calcular $\displaystyle\iint\limits_{ S}\mbox{rot}{\bf F}\cdot d{\bf S}.$

  • ${\bf F}(x,y,z) = x{\bf i} - z{\bf j} + y{\bf k}$, $S$ é a parte do plano $x+z=1$ dentro do cilindro $x^2+y^2 = 1$, com orientação para cima.


$2\pi.$


2121   

Expresse $\partial z/\partial t$ em termos das derivadas parciais de $f$, sendo $z=f(x,y)$ e $x=t^{2}$ e $y=3t.$


$\displaystyle \frac{dz}{dt} (t) = 2t \frac{\partial f}{\partial x}(t^{2},3t) + 3 \frac{\partial f}{\partial y}(t^{2},3t).$


2736   

Determine a equação dos planos tangentes ao gráfico de $f(x,y) =  - x^2 - y^2$ que passam por ambos os pontos $(1,0,7)$ e $(3,0,3)$.


$2x + 2y + z = 9$ e $2x - 2y + z = 9.$


2201   

O raio $r$ e a altura $h$ de um cilindro circular reto aumentam à razão de $0,01cm/min$ e $0,02cm/min$, respectivamente.

  1.  Ache a taxa de variação do volume quando $r=4cm$ e $h=7cm.$
  2.  A que taxa a área da superfície curva está variando nesse instante?


  1.  $0,88\pi$ cm$^{3}/$min.
  2.  $0,3\pi$ cm$^{2}/$min.


2606   

Use o Teorema de Stokes para calcular $\displaystyle\iint\limits_{ S}\mbox{rot}{\bf F}\cdot d{\bf S}.$

  • ${\bf F}(x,y,z) = xyz{\bf i} + xy{\bf j} + x^2yz{\bf k}$ e $S$ é formada pelo topo e pelos quatro lados (mas não pelo fundo) do cubo com vértices $(\pm 1,\pm 1,\pm 1)$, com orientação para fora.


$0.$


2186   

Encontre os valores de $\partial z/ \partial x$ e $\partial z/\partial y$ no ponto indicado.
$z^{3}-xy+yz+y^{3}-2=0$,  $(1,1,1).$


 $\displaystyle \frac{\partial z}{\partial x}(1,1,1) = \frac{1}{4}$ e $\displaystyle \frac{\partial z}{\partial x}(1,1,1) = -\frac{3}{4}.$


2030   

Use a Regra da Cadeia para determinar $\mathrm{d}z/\mathrm{d} t$ ou $\mathrm{d}w/ \mathrm{d}t.$

$w=xe^{y/z}$, $x=t^{2}$, $y=1-t$, $z=1+2t$.


$\displaystyle \frac{dw}{dt} = e^{\frac{y}{z}} \left(2t - \frac{x}{z} - \frac{2xy}{z^{2}} \right).$


2425   

Se $S$ é uma esfera e ${\bf F}$ satisfaz as hipóteses do Teorema de Stokes, mostre que $\displaystyle\iint\limits_{S}\mbox{rot}{\bf F} \cdot d{\bf S} = 0$.


2439   

Calcule a integral de superfície $\displaystyle\iint\limits_{S}yz dS$, onde $S$ é a superfície com equações paramétricas $x=u^{2}$, $y=u \sin v$, $z=u\cos v$, $0 \leq u \leq 1$, $0 \leq v \leq \pi/2.$


$\dfrac{5\sqrt{5}}{48} + \dfrac{1}{240}.$


3048   

O campo vetorial $\mathbf{F}$ é mostrado no plano $xy$ e é o mesmo em todos os planos horizontais (em outras palavras, $\mathbf{F}$ é independente de $z$ e sua componente $z$ é 0).

  1. O $\text{div }{\mathbf{F}}$ será positivo, negativo ou nulo? Justifique.

  2. Determine se o $\text{rot }{\mathbf{F}} = 0$. Se não, em que direção rot $\mathbf{F}$ aponta?

A expressão 1 cossec x e o mesmo que


  1. Nulo.

  2. $\text{rot } \bf{F}$ aponta na direção negativa do eixo $z.$


2133   

Mostre que cada a equação a seguir define implicitamente pelo  menos uma função diferenciável $y=y(x).$ 
Expresse $\mathrm{d} y/\mathrm{d} x$ em termos de $x$ e $y.$
$x^{2}y+\sin(y)=x$


 $\displaystyle \frac{d y}{d x} = -\frac{2xy - 1}{x^{2} + \cos(y)}.$


1971   

Encontre ${\bf r}(t)$ se ${\bf r}'(t)=2t\;{\bf i}+3t^{2}\;{\bf j}+\sqrt{t}\;{\bf k}$ e ${\bf r}(1)={\bf i}+{\bf j}.$



Como ${\bf r}'(t)=2t\;{\bf i}+3t^{2}\;{\bf j}+\sqrt{t}\;{\bf k}$, temos que 
$${\bf r}(t)=t^{2}\,{\bf i}+t^{3}\,{\bf j}+\frac{2}{3}t^{3/2}\,{\bf k}+C.$$
Mas, ${\bf r}(1)={\bf i}+{\bf j}$, logo
$${\bf i}+{\bf j}={\bf i}+{\bf j}+\frac{2}{3}{\bf k}+C$$
implicando que $C=\dfrac{2}{3}\,{\bf k}.$
Portanto 
$${\bf r}(t)=t^{2}\,{\bf i}+t^{3}\,{\bf j}+\frac{2}{3}(t^{3/2}-1)\,{\bf k}.$$


2028   

Use a Regra da Cadeia para determinar $\mathrm{d}z/\mathrm{d} t$ ou $\mathrm{d}w/ \mathrm{d}t.$

$z=\sin{x}\cos{y}$, $x=\pi t$, $y=\sqrt{t}$.


$\displaystyle \frac{dz}{dt} = \pi \cos(x) \cos(y) - \frac{1}{2\sqrt{t}} \sin(x) \sin(y).$


1986   

Determine a equação da reta tangente à trajetória da função dada, no ponto dado.

  1. ${\bf r}(t)=(\cos (t),\sin (t),t)$ e ${\bf r}(\pi/3).$
  2. ${\bf r}(t)=(t^{2},t)$ e ${\bf r}(1).$
  3. ${\bf r}(t)=\bigg(\dfrac{1}{t},\dfrac{1}{t},t^{2}\bigg)$ e ${\bf r}(2).$


2397   

Determine uma reta que seja tangente à elipse $2x^2 + y^2 = 3$ e paralela à reta $2x + y = 5$.


 $\displaystyle y = -2x + 3$ ou$\displaystyle y = -2x - 3.$


2149   

Use o Teorema do Divergente para calcular o fluxo de ${\bf F}$ através de $S,$ onde ${\bf F}(x,y,z)=3xy^{2}\,{\bf i}+xe^{z}\,{\bf j}+z^{3}\,{\bf k}$ e $S$ é a superfície do sólido delimitado pelo cilindro $y^{2}+z^{2}=1$ e pelos planos $x=-1$ e $x=2.$


2456   

Dada $f(x,y)=\dfrac{1}{\sqrt{16-x^{2}-y^{2}}}$.

  1. Encontre o domínio da função;

  2. Encontre a imagem da função;

  3. Descreva as curvas de nível da função.



  1. O domínio de $f$ é

    $$D=\{(x,y)|\, 16-x^{2}-y^{2}>0\}=\{(x,y)|\,x^{2}+y^{2}<16\}.$$

    A expressão 1 cossec x e o mesmo que

  2. A imagem de $f$ é

    $$\bigg\{z|\, z=\frac{1}{\sqrt{16-x^{2}-y^{2}}},\,(x,y)\in D\bigg\}.$$

    Mas,

    $$z=\frac{1}{\sqrt{16-x^{2}-y^{2}}}\geq \frac{1}{\sqrt{16}}=\frac{1}{4}.$$

    Assim, a imagem de $f$ é $\bigg\{z|\, z \geq \dfrac{1}{4}\bigg\}.$

  3. s curvas de níveis de $f$ são da forma $f(x,y)=c$, isto é,

    $$\frac{1}{\sqrt{16-x^{2}-y^{2}}}=c\Leftrightarrow \sqrt{16-x^{2}-y^{2}}=\frac{1}{c}\Leftrightarrow 16-x^{2}-y^{2}=\frac{1}{c^{2}}$$

    $$\Leftrightarrow x^{2}+y^{2}=16-\frac{1}{c^{2}}.$$

    Assim, as curvas de níveis de $f$ são circunferências com centro na origem e raio menor do que $4.$


3083   

A resistência total \(R\) de três resistores \(R_1\), \(R_2\) e \(R_3\) ligados em paralelo é dada por \[ \frac{1}{R}=\frac{1}{R_1}+\frac{1}{R_2}+\frac{1}{R_3}. \] Suponha que \(R_1\), \(R_2\) e \(R_3\) tenham sido medidos como \(100\ \Omega\), \(200\  \Omega\) e \(300\  \Omega\), respectivamente, com um erro máximo de \(10\%\) em cada um e sendo \(\Omega\)(Ohm) a unidade de medida no sistema internacional de unidades. Use diferenciais para aproximar o erro percentual máximo no valor calculado de \(R\).


1999   

Calcule o trabalho realizado por uma partícula andando sobre a espiral dada por $C:\,x=t\,\cos t$, $y=t\,\sin t$, com $0\leq t\leq 2\pi$, sob a ação do campo ${\bf F}(x,y)=(x,y)$, ou seja, calcule a integral $\int_{C}x\,dx+y\,dy.$


$2\pi^{2}.$


2270   

Determine uma equação do plano tangente à superfície parametrizada dada no ponto especificado. ${\bf r}(u,v)=(u-v,u^{2}+v^{2},uv)$, no ponto ${\bf r}(1,1).$



Temos que ${\bf r}(u,v)=\underbrace{(u-v)}_{x(u,v)}\,{\bf i}+\underbrace{(u^{2}+v^{2})}_{y(u,v)}\,{\bf j}+\underbrace{uv}_{z(u,v)}\,{\bf k}$

Primeiro, vamos calcular os vetores tangentes:

$$\begin{array}{rcl}{\bf r}_{u}&=&\frac{\partial x(u,v)}{\partial u}\,{\bf i}+\frac{\partial y(u,v)}{\partial u}\,{\bf j}+\frac{\partial z(u,v)}{\partial u}\,{\bf k}\\&=& \,{\bf i}+2u\,{\bf j}+v\,{\bf k}\end{array}$$

e

$$\begin{array}{rcl}{\bf r}_{v}&=&\frac{\partial x(u,v)}{\partial v}\,{\bf i}+\frac{\partial y(u,v)}{\partial v}\,{\bf j}+\frac{\partial z(u,v)}{\partial v}\,{\bf k}\\&=& -\,{\bf i}+2v\,{\bf j}+u\,{\bf k}\end{array}$$


Assim, o vetor normal ao plano tangente é:

$$\begin{array}{rcl}{\bf r}_{u}\times {\bf r}_{v}&=&\left|\begin{array}{ccc}{\bf i}& {\bf j}&{\bf k}\\1 & 2u & v\\-1 & 2v & u\\\end{array}\right|\\&=&(-2u^{2}-2v^{2})\,{\bf i}-(u+v)\,{\bf j}+(2u+2v)\,{\bf k}\end{array}$$


Como $u=1$ e $v=1$ temos que o vetor normal é $-4\,{\bf i}-2\,{\bf j}+4\,{\bf k}.$

Portanto, uma equação do plano tangente no ponto ${\bf r}(1,1)=(0,2,1)$ é

$$-4\cdot(x-0)-2\cdot(y-2)+4\cdot (z-1)=0$$

$$-4x-2y+4+4z-4=0$$

$$-4x-2y+4z=0    \mbox{ou}     2x+y-2z=0$$


2333   

Determine a taxa de variação máxima de $f$ no ponto dado e a direção em que isso ocorre.

$f(x,y,z) = \sqrt{x^2 + y^2 + z^2},  (3,6,-2).$


$1.$


1989   

Calcule a integral $\displaystyle\int_{0}^{\pi/2}(3\sin^{2}(t) \cos(t){\bf i}+3\sin(t) \cos^{2}(t){\bf j}+2\sin(t)\cos(t){\bf k})\mathrm{d}t$.


2593   

Marque o ponto cujas coordenadas cilíndricas são $(2, \pi/4,1)$ e $(4, -\pi/3,5)$. Em seguida, encontre as coordenadas retangulares do ponto.


Para $(2, \pi/4,1):$ $(\sqrt{2},\sqrt{2},1)$ e para $(4, -\pi/3,5):$ $(2, -2\sqrt{3},5)$.


2913   

Suponha que a área de uma região no plano de coordenadas polares seja  $$A=\int_{\pi/4}^{3\pi/4}   \int_{\mathrm{cosec\,}{\theta}}^{2\sin{\theta}}r\,dr d\theta.$$ Esboce a região e encontre sua área.


$A = \dfrac{\pi}{2};$ região:

A expressão 1 cossec x e o mesmo que


2680   

Encontre $\partial f/\partial x$ e $\partial f/\partial y$ para $f(x,y)=(x^{2}-1)(y+2)$.


$\displaystyle \frac{\partial f}{\partial x} = 2x(y + 2) \;\;\;\;\text{e}\;\;\;\; \frac{\partial f}{\partial y} = x^{2} - 1$.


2096   

Seja $\Omega=\{(x,y)\in \mathbb{R}^{2}|\,(x,y)\notin A\}$, onde $A$ é a semirreta $\{(x,y)\in \mathbb{R}^{2}|\,y=0\,e\,x\geq 0\}$. Calcule

$$\int_{C}\frac{-y}{x^{2}+y^{2}}\,dx+\frac{x}{x^{2}+y^{2}}\,dy,$$

onde $C:[0,1]\rightarrow \mathbb{R}^{2}$ é uma curva de classe $C^{1}$ por partes, com imagem contida em $\Omega$, tal que $C(0)=(1,1)$ e $C(1)=(1,-1).$


$\dfrac{3\pi}{2}.$


2497   

Esboce o sólido cujo volume é dado pela integral iterada.

  1.  $\displaystyle\int_{0}^{1}\int_{0}^{1-x}\int_{0}^{2-2z}\;dy dz dx$

  2.  $\displaystyle\int_{0}^{2}\int_{0}^{2-y}\int_{0}^{4-y^{2}}\;dx dz dy$

  3.  $\displaystyle\int_{0}^{1}\int_{\sqrt{1-z}}^{\sqrt{4-z}}\int_{2}^{3}\;dx dy dz$

  4.  $\displaystyle\int_{0}^{2}\int_{x^{2}}^{2x}\int_{0}^{x+y}\;dz dy dx$


  1. (... fig)

  2. (... fig.)

  3.  $\displaystyle \left\lbrace (x,y,z); 2 \leq x \leq 3,  \sqrt{1 - z} \leq y \leq \sqrt{4 - z} , 0 \leq z \leq 1\right\rbrace.$

  4.  $\displaystyle \left\lbrace (x,y,z); 0 \leq x \leq 2,  x^{2} \leq y \leq 2x , 0 \leq z \leq x + y\right\rbrace.$


2570   

Calcule $\displaystyle \lim_{(x,y) \to (0,0)} \dfrac{x^2}{\sqrt{x^2 + y^2}}$, caso exista.


$0.$


2193   

Calcule $\displaystyle\iint \limits_{S}{\bf u}\cdot {\bf n}\,dS$, sendo $B=\{(x,y,z)\in \mathbb{R}^{3}|\, x^{2}+y^{2}\leq 1,\,x^{2}+y^{2}\leq z \leq 5-x^{2}-y^{2}\}$ e ${\bf u}=3xy\,{\bf i}-\dfrac{3}{2}y^{2}\,{\bf j}+z\,{\bf k}.$


$36\pi.$


2636   

O índice de sensação térmica $W$ é a temperatura sentida quando a temperatura real é $T$ e a velocidade do vento, $v$. Portanto, podemos escrever $W=f(T,v)$. Considerando a tabela abaixo:

A expressão 1 cossec x e o mesmo que

  1. Estime os valores de $f_{T}(-15,30)$ e $f_{v}(-15,30)$. Quais são as nterpretações práticas desses valores?

  2. Em geral, o que se pode dizer sobre o sinal de $\partial W/\partial T$ e $\partial W/\partial v$?

  3. Qual parece ser o valor do seguinte limite

    $$\lim_{v\rightarrow \infty}\frac{\partial W}{\partial v}?$$


  1. $f_{T}(-15,30) \approx 1.3$ Isto significa que quando a temperatura real é $-15º$C e a velocidade do vento é $30$km/h, a temperatura aparente aumenta cerca de $1.3º$C para cada $1º$C que a temperatura real aumenta;\\

  2. $f_{v}(-15,30) \approx -0.15$ Isto significa que quando a temperatura real é $-15º$C e a velocidade do vento é $30$km/h, a temperatura aparente diminui cerca de $0.15º$C para cada $1$km/h que a velocidade do vento aumenta.  

  3. $\frac{\partial W}{\partial T} > 0$ e  $\frac{\partial W}{\partial v} \leq 0.$

    $\lim_{v \to \infty} \frac{\partial W}{\partial v} = 0.$


2391   

Calcule a integral de superfície $\displaystyle\iint\limits_{S}x^{2}z^{2}dS$, onde $S$ é a parte do cone $z^{2}=x^{2}+y^{2}$ que está entre os planos $z=1$ e $z=3.$



Temos que $S$ é a porção do cone $z^{2}=x^{2}+y^{2}$ para $1 \leq z \leq 3$, ou equivalentemente, $S$ é a parte da superfície $z=\sqrt{x^{2}+y^{2}}$ sobre a região $D=\{(x,y)| 1 \leq x^{2}+y^{2} \leq 9\}.$ Assim,
$\displaystyle\iint\limits_{S}x^{2}z^{2}dS=\displaystyle\iint\limits_{D}x^{2}(x^{2}+y^{2})\sqrt{\left(\dfrac{\partial z}{\partial x}\right)^{2}
+\left(\frac{\partial z}{\partial y}\right)^{2}+1}dA$
$=\displaystyle\iint\limits_{D}x^{2}(x^{2}+y^{2})\sqrt{\left(\frac{x}{\sqrt{x^{2}+y^{2}}}\right)^{2}+\left(\frac{y}{\sqrt{x^{2}+y^{2}}}\right)^{2}+1}dA$
$=\displaystyle\iint\limits_{D}x^{2}(x^{2}+y^{2})\sqrt{\frac{x^{2}+y^{2}}{x^{2}+y^{2}}+1}dA=\iint\limits_{D}\sqrt{2}x^{2}(x^{2}+y^{2})dA$
$=\sqrt{2}\displaystyle\iint\limits_{D}x^{2}(x^{2}+y^{2})dA.$
Por coordenadas polares, temos que $x=r\cos \theta, y=r\sin \theta, 1\leq r\leq 3 , 0\leq \theta \leq 2\pi \,\mbox{e} \, dA=r dr d\theta.$
Logo,
$\displaystyle\iint\limits_{S}x^{2}z^{2}dS=\sqrt{2}\int_{0}^{2\pi}\int_{1}^{3}(r^{2}\cos^{2}\theta)(r^{2})r dr d\theta =\sqrt{2}\int_{0}^{2\pi}\cos^{2}\theta d\theta \cdot \int_{1}^{3}r^{5}dr$

$=\sqrt{2}\cdot (\theta)\bigg|_{0}^{2\pi}\cdot \bigg(\frac{r^{6}}{6}\bigg)\bigg|_{1}^{3}=\sqrt{2}\cdot \pi \cdot \frac{1}{6}\cdot (3^{6}-1)=\frac{364\sqrt{2}}{3}\pi$


2215   

Determine o rotacional e o divergente do campo vetorial $\mathbf{F}(x,y,z) = e^x\sin{y}\mathbf{i} + e^x\cos{y}\mathbf{j} + z\mathbf{k}$.


$\text{rot } \mathbf{F} = \bf{0}.$ $\text{div } \mathbf{F} = 1.$


2879   

A produção total $P$ de certo produto depende da quantidade $L$ de trabalho empregado e da quantidade $K$ de capital investido. Nas Seções 14.1 e 14.3 do livro do Stewart, foi discutido o modelo Cobb-Douglas $P = bL^\alpha K^{1-\alpha}$ seguido de certas hipóteses econômicas, em que $b$ e $\alpha$ são constantes positivas e $\alpha < 1$. Se o custo por unidade de trabalho for $m$ e o custo por unidade de capital for $n$, e uma companhia puder gastar somente uma quantidade $p$ de dinheiro como despesa total, então a maximização da produção $P$ estará sujeita à restrição $mL + nK = p$. Mostre que a produção máxima ocorre quando

$$L = \dfrac{\alpha p}{m} \quad \text{e} \quad K = \dfrac{(1 - \alpha)p}{n}.$$


2374   

A temperatura $T$ em uma bola de metal é inversamente proporcional à distância do centro da bola, que tomamos como a origem. A temperatura no ponto $(1,2,2)$ é de 120°.

  1.  Determine a taxa de variação de $T$ em $(1,2,2)$ em direção ao ponto $(2,1,3)$.
  2.  Mostre que em qualquer ponto da bola a direção de maior crescimento na temperatura é dada por um vetor que aponta para a origem.


  1. $\displaystyle -\frac{40}{3\sqrt{3}}.$
  2.  Note que $\nabla T = -360 (x^{2} + y^{2} + z^{2})^{-3/2} (x,y,z)$ sempre aponta para a origem. 


2576   

Seja $f(x,y) = \dfrac{2xy^2}{x^2 + y^4}$.

  1. Considere a reta $\gamma(t) = (at, bt)$, com $a^2 + b^2 > 0$; mostre que, quaisquer que sejam $a$ e $b$,

    $$\displaystyle \lim_{t \to 0} f(\gamma(t)) = 0.$$

    Tente visualizar este resultado através das curvas de nível de $f$.

  2. Calcule $\displaystyle \lim_{t \to 0} f(\delta(t))$, onde $\delta(t) = (t^2,t).$ (Antes de calcular o limite, tente prever o resultado olhando para as curvas de nível de $f$.)

  3. $\displaystyle \lim_{(x,y) \to (0,0)}\dfrac{2xy^2}{x^2 + y^4}$ existe? Por quê?


  1. Demonstração.

  2. $1.$

  3. Não existe.


2082   

Se $z=f(x,y)$, onde $x=r\cos{\theta}$ e $y=r\sin{\theta}$,

  1. Determine $\dfrac{\partial z}{\partial r}$ e $\dfrac{\partial z}{\partial \theta}.$
  2. Mostre que $\bigg(\dfrac{\partial z}{\partial x}\bigg)^{2}+ \bigg(\dfrac{\partial z}{\partial y}\bigg)^{2}=\bigg(\dfrac{\partial z}{\partial r}\bigg)^{2}+\dfrac{1}{r^{2}}\bigg(\dfrac{\partial z}{\partial \theta}\bigg)^{2}$.


  1. $\displaystyle \frac{\partial z}{\partial r} = \cos(\theta) \frac{\partial z}{\partial x}  + \sin(\theta) \frac{\partial z}{\partial y} $e$\displaystyle \frac{\partial z}{\partial \theta} = -r \sin(\theta)\frac{\partial z}{\partial x}  + r\cos(\theta) \frac{\partial z}{\partial y}.$
  2. Use $(a)$ para calcular $\bigg(\dfrac{\partial z}{\partial r}\bigg)^{2}+\dfrac{1}{r^{2}}\bigg(\dfrac{\partial z}{\partial \theta}\bigg)^{2}$.


2620   

Suponha que  $S$ e $C$ satisfaçam as hipóteses do Teorema de Stokes e $f$ e $g$ tenham derivadas parciais de segunda ordem contínuas. Demonstre que $\displaystyle\int_C (f\nabla f)\cdot d{\bf R} = 0$


Note que $\mbox{rot} (f\nabla f) = {\bf 0}.$


2465   

Calcule a integral de superfície $\displaystyle\iint \limits_{ S}{\bf F}\cdot d{\bf S}$ para o campo vetorial ${\bf F}$ e superfície orientada $S$ dados abaixo. Em outras palavras, determine o fluxo de ${\bf F}$ através de $S$. Para superfícies fechadas, use a orientação positiva (para fora).

  • ${\bf F}(x,y,z)=x{\bf i}+2y{\bf j}+3z{\bf k}$ e $S$ é o cubo com vértices $(\pm 1, \pm 1,\pm 1).$


$48.$


2128   

Seja $z=f(u-v,v-u)$. Verifique que 
$$\frac{\partial z}{\partial u}+\frac{\partial z}{\partial v}=0.$$



Note que $\displaystyle \frac{\partial z}{\partial u}(u,v) = \frac{\partial f}{\partial x}(u-v,v-u) - \frac{\partial f}{\partial y}(u-v,v - u)$ e $\displaystyle \frac{\partial z}{\partial v}(u,v) = -\frac{\partial f}{\partial x}(u-v,v-u) + \frac{\partial f}{\partial y}(u-v,v - u).$


2474   

Determine e faça o esboço do domínio da função $f(x,y)=\ln(9-x^{2}-9y^{2})$.


$\left\lbrace (x,y);\; \frac{x^{2}}{9} + y^{2} < 1 \right\rbrace.$


A expressão 1 cossec x e o mesmo que


2029   

Use a Regra da Cadeia para determinar $\mathrm{d}z/\mathrm{d} t$ ou $\mathrm{d}w/ \mathrm{d}t.$

$z=\tan^{-1}(x/y)$, $x=e^{t}$, $y=1-e^{-t}$.


$\displaystyle \frac{dz}{dt} = \frac{xe^{-t} - ye^{t}}{x^{2} + y^{2}}.$


2976   

Determine o jacobiano da transformação dada por: $x = \alpha \sin{\beta}, \quad y = \alpha \cos{\beta}$.


$-\alpha.$


2843   

Passe para coordenadas polares e calcule: $\displaystyle\int_{-1}^{1} \int_{0}^{\sqrt{1-x^{2}}}\,dy dx$


$\displaystyle \frac{\pi}{2}.$


2098   

Suponha que ${\bf F}=\nabla f$ seja um campo vetorial conservativo e

$$g(x,y,z)=\int_{(0,0,0)}^{(x,y,z)}{\bf F}\cdot d{\bf r}.$$

Mostre que $\nabla g={\bf F}.$


Como $g(x,y,z) = f(x,y,z) - f(0,0,0),$ segue que $\nabla g = \nabla f = \mathbf{F}.$


3000   

Calcule a integral trocando a ordem de integração. $\displaystyle\int_{0}^{1}\!\!\int_{x}^{1}e^{x/y}\,dy dx$.


A região de integração é do tipo I, é dada por

$$\{(x,y) \in \mathbb{R}^2: 0 \leq x \leq 1 \mbox{ e } x \leq y \leq 1\}$$

e pode ser vista geometricamente como a região esboçada na figura abaixo.

A expressão 1 cossec x e o mesmo que

Essa região pode ser descrita como uma região do tipo II da seguinte forma:
$$\{(x,y) \in \mathbb{R}^2: 0 \leq x \leq y \mbox{ e } 0 \leq y \leq 1\}.$$
Assim,
\begin{array}{rcl}\displaystyle\int_{0}^{1}\!\!\int_{x}^{1}e^{x/y}\,dy dx & = & \displaystyle\int_{0}^{1}\!\!\int_{0}^{y} \! e^{x/y}\,dx dy \\  & = & \displaystyle\int_{0}^{1} \! \left. ye^{x/y} \right|_{x=0}^{x=y}\,dx \\    & = & \displaystyle\int_{0}^{1} \! \left. y(e-1) \right|_{x=0}^{x=y}\,dx  \\    & = & \left.(e-1) \frac{y^2}{2}\right|_{0}^{1} = \frac{e-1}{2}.\end{array}


2001   

Uma partícula move-se no plano de modo que no instante $t$ sua posição é dada por ${\bf r}(t)=(t,t^{2})$. Calcule o trabalho realizado pelo campo de forças ${\bf F}(x,y)=(x+y)\,{\bf i}+(x-y)\,{\bf j}$ no deslocamento da partícula de ${\bf r}(0)$ até ${\bf r}(1).$


$1.$


2179   

Seja $D$ a região limitada por um caminho fechado e simples $C$ no plano $xy$. As coordenadas do centroide $(\bar{x},\bar{y})$ de $D$ são

$$\bar{x} = \dfrac{1}{2A}\oint_{C}x^2 \, dy \quad \quad\quad\quad \bar{y} = -\dfrac{1}{2A}\oint_{C}y^2 \, dx,$$

em que $A$ é a área de $D$. Encontre o centroide de um quarto de uma região circular de raio $a$.


$\displaystyle \left(\frac{4a}{3\pi},\frac{4a}{3\pi} \right),$ se a região for a parte do disco $x^{2} + y^{2} = a^{2}$ no primeiro quadrante.


1929   

Uma partícula se move em um campo de velocidade $\textbf{V}(x,y) = (x^2,x+y^2)$. Se ela está na posição $(2,1)$ no instante $t=3$, estime sua posição no instante $t=3,01$.


$(2,04;1,03).$


2732   

Determine as equações do plano tangente e da reta normal ao gráfico da função dada, no ponto dado. $f(x,y) = xe^{x^2 - y^2}$ em $(2,2,f(2,2))$.


Plano tangente: $z = 9x - 8y$

Reta normal: $(x,y,z) = \left(2,2,2 \right) + \lambda \left(9,-8,-1 \right)$.


3131   

Use o Teorema de Green para determinar o trabalho realizado pelo campo de forças \(\displaystyle\mathbf{F}(x,y)=\sqrt{y}\textbf{i}+\sqrt{x}\textbf{j}\) sobre uma partícula que percorre uma vez, no sentido anti-horário, a curva fechada dada pelas equações \(y=0\), \(x=2\) e \(y=x^3/4\).


2097   

Mostre que a integral de linha $\int_{C}2x\,\sin y\,dx+(x^{2}\,\cos y-3y^{2})\,dy$, onde $C$ é qualquer caminho entre $(-1,0)$ a $(5,1)$, é independente do caminho e calcule a integral.


$\mathbf{F} (x,y) = 2x \sin(y) \mathbf{i} + x^{2} \cos(y) - 3y^{2} \bf j$ é um campo conservativo com uma função potencial $f(x,y) = x^{2} \sin(y) - y^{3};$ o valor da integral é $25 \sin(1) - 1.$


2839   

Calcule a integral dupla usando coordenadas polares: $\displaystyle\iint\limits{R}\sin(x^{2}+y^{2})\,dA$, onde $R$ é a região acima do eixo $x$ e dentro da circunferência $x^{2}+y^{2}=9.$


$\displaystyle \frac{\pi}{2}(1 - \cos(9).$


2437   

Calcule a integral de superfície $\displaystyle\iint\limits_{S}x^{2}yz dS$, onde $S$ é a parte do plano $z=1+2x+3y$ que está acima do retângulo $[0,3]\times [0,2].$


2146   

Aplique o Teorema da Divergência para achar $\displaystyle\iint \limits_{S}{\bf F}\cdot {\bf n}\,dS,$  sendo ${\bf F}(x,y,z)=(x^{2}+\sin yz)\,{\bf i}+(y-xe^{-z})\,{\bf j}+z^{2}\,{\bf k}$ e $S$ a superfície da região delimitada pelo cilindro $x^{2}+y^{2}=4$ e os planos $x+z=2$ e $z=0.$


$20\pi.$


2091   

Calcule a integral de linha

$$\int_{C}e^{2y}\,dx+(1+2xe^{2y})\,dy,$$

onde $C$ é a curva dada por $r(t)=(te^{t},1+\sin(\pi t/2))$, $0\leq t\leq 1.$ (Sugestão: verifique se o campo é conservativo.)


$e^{5} + 1.$


2117   

Calcule $\mathrm{d} z/\mathrm{d} t$ por dois processos:

  1. substituindo as expressões para $x(t)$ e $y(t)$ em $z$ e depois derivando diretamente com relação a $t$
  2. aplicando a Regra da Cadeia: $\frac{dz}{dt}=\frac{\partial z}{\partial x}\frac{dx}{dt}+\frac{\partial z}{\partial y }\frac{dy}{dt}$.

$z=\sin(xy)$, $x=3t$ e $y=t^{2}.$


$\displaystyle \frac{dz}{dt} (t) =  9t^{2}\cos(3t^{3}).$


2753   

A função $f(x,y) = \begin{cases}\dfrac{x^2y}{x^2 + y^2}, & \quad \text{se } (x,y) \neq (0,0),\\0, & \quad \text{se } (x,y) = (0,0)\\\end{cases}$ é diferenciável em $(0,0)$? Justifique.


Não.


1934   

Calcule a integral de linha, onde $C$ é a curva dada.

$\displaystyle\int_{C}(x^{2}y^{3}-\sqrt{x})\,dy$,   $C$ é o arco da curva $y=\sqrt{x}$ de $(1,1)$ a $(4,2).$


$\dfrac{243}{8}.$


2164   

Use o Teorema de Green para calcular $\int_{C}\mathbf{F} \cdot d\mathbf{r}$, onde $\mathbf{F}(x,y) = (\sqrt{x} + y^3,x^2+\sqrt{y})$, $C$ consiste no arco da curva $y = \sin{x}$ de $(0,0)$ a $(\pi,0)$ e no segmento de reta $(\pi,0)$ a $(0,0)$. (Verifique a orientação da curva antes de aplicar o Teorema.)


$\dfrac{4}{3} - 2\pi.$


3072   

A figura mostra uma curva $C$ e um mapa de contorno de uma função $f$ cujo gradiente é contínuo. Determine $\int_{C}\nabla f\cdot d{\bf r}.$

A expressão 1 cossec x e o mesmo que


$40.$


3007   

Calcule o centro de massa da região: $D = \{(x,y) \in \mathbb{R}^2: x^2 + 4y^2 \leq 1, \ y \geq 0\}$ e a densidade é proporcional à distância do ponto ao eixo $x$.


$\displaystyle \left(0, \frac{3\pi}{32} \right).$


2053   

Determine se ${\bf F}(x,y)=(2x-3y)\,{\bf i}+(-3x+4y-8)\,{\bf j}.$ é ou não um campo vetorial conservativo. Se for, determine uma função $f$ tal que ${\bf F}=\nabla f.$


Sim. $f(x,y) = x^2 - 3xy + 2y^2 -8y + K.$


2247   

Prove que se $\mathbf{F} \cdot \mathbf{n}$ for constante sobre $Im\,\mathbf{r}$, então o fluxo de $\mathbf{F}$ sobre $\mathbf{r}$ é o produto de $\mathbf{F} \cdot \mathbf{n}$ pelo comprimento de $\mathbf{r}$, em que $\mathbf{n}$ é normal a $\mathbf{r}$.


Direto da definição do fluxo de $\mathbf{F}$ através de $\bf{r}$ na direção $\bf{n}.$


2277   

Calcule $\nabla f(x,y)$.
$f(x,y) = e^{x^2 - y^2}$


$\displaystyle \nabla f(x,y) = e^{x^{2} - y^{2}}(2x,-2y).$


2481   

Use a integral tripla para determinar o volume do sólido dado.

  1.  $0\leq x \leq 1$, $0\leq y \leq 1$ e $0\leq z \leq 5-x^{2}-3y^{2}.$

  2.  $0\leq x \leq 1$, $0\leq y \leq x^{2}$ e $0\leq z \leq x+y^{2}.$

  3. $x^{2}+y^{2}\leq z \leq 4.$

  4.  $x^{2}+4y^{2}\leq z \leq 1.$


  1.  $\dfrac{11}{3}.$

  2. $\dfrac{25}{84}.$

  3. $8\pi.$

  4.  $\dfrac{\pi}{4}.$


2472   

Seja $g(x,y,z)=\ln(25-x^{2}-y^{2}-z^{2}).$

  1. Calcule $g(2,-2,4).$

  2. Determine o domínio de $g$.

  3. Determine a imagem de $g$.


  1. $0.$

  2. $\left\lbrace (x,y,z): x^{2} + y^{2} + z^{2} < 25 \right\rbrace.$

  3. $(-\infty, \ln(25)].$


3033   

Dados um hemisfério $H$ e uma parte $P$ de um paraboloide, suponha que ${\bf F}$ seja um campo vetorial sobre $\mathbb{R}^3$ cujas componentes tenham derivadas parciais contínuas. Explique por que

$$\displaystyle\iint\limits_{H}\mbox{rot}{\bf F}\cdot{\bf S} = \iint\limits_{P}\mbox{rot}{\bf F}\cdot{\bf S}.$$

A expressão 1 cossec x e o mesmo que

A expressão 1 cossec x e o mesmo que


Note que $H$ e $P$ satisfazem as hipóteses do Teorema de Stokes. Logo,
$$\displaystyle \iint \limits_{H} \mbox{rot } {\bf F} \cdot {\bf S} = \int \limits_{C} {\bf F} \cdot d{\bf r} = \iint \limits_{P} \mbox{rot }{\bf F}\cdot{\bf S},$$

onde $C$ é a curva de fronteira.


2209   

Calcule a integral dupla.

  1. $\displaystyle\iint\limits_{D}(2x-y)\,dA, \quad D$ limitada pelo círculo de centro na origem e raio 2.

  2. $\displaystyle\iint\limits_{D}\dfrac{x}{y}\,dA, \quad D$ região no primeiro quadrante limitada pelas retas $y=x$, $y=2x$, $x=1$ e $x=2.$

  3. $\displaystyle\iint\limits_{D}\dfrac{1}{xy}\,dA, \quad D$ o quadrado $1\leq x\leq 2$, \;$1\leq y\leq 2.$

  4. $\displaystyle\iint\limits_{D}(x-\sqrt{y})\,dA, \quad D$ região triangular cortado do primeiro quadrante do plano $xy$ pela reta $x+y=1.$


  1.  $0.$

  2.  $\dfrac{3\ln(2)}{2}.$

  3.  $(\ln(2))^{2}.$

  4. $-\dfrac{1}{10}.$


2145   

Aplique o Teorema da Divergência para achar $\displaystyle\iint \limits_{S}{\bf F}\cdot {\bf n}\,dS,$  sendo ${\bf F}(x,y,z)=y\,\sin x\,{\bf i}+y^{2}z\,{\bf j}+(x+3z)\,{\bf k}$ e $S$ é a superfície da região delimitada pelos planos $x=\pm 1$, $y=\pm 1$ e $z=\pm 1.$


$24.$


2792   

Determine os valores máximos e mínimos locais e pontos de sela da função $f(x,y)=x^{3}+2xy+y^{2}-5x$.


Ponto de mínimo: $\displaystyle \left( \frac{5}{3}, -\frac{5}{3} \right);$ ponto de sela: $\displaystyle \left(-1,1\right).$


2626   

Utilizando o Teorema de Stokes, transforme a integral $\displaystyle\iint_{ S}\mbox{rot}{\bf F}\cdot{\bf n}dS$ numa integral de linha e calcule.

  • ${\bf F}(x,y,z) = x{\bf j}$, $S$ a superfície $\{(x,y,z) \in \mathbb{R}^3; 0\leq z\leq 1, x^2+y^2=1,$$x\geq 0, y\geq 0\}$, sendo ${\bf n}$ a normal com componente $x$ positiva.


 $0$.


2927   

Escreva a equação $z^{2}=x^{2}+y^{2}$ em coordenadas esféricas.


$\cos^2 \phi = \sin^2 \phi.$


2390   

A função diferenciável $z = f(x,y)$ é dada implicitamente pela equação $x^3 + y^3 + z^3 = 10$. Determine a equação do plano tangente ao gráfico de $f$ no ponto $(1,1,f(1,1))$.


 $x + y + 4z = 10.$


2817   

Determine os valores máximo e mínimo absolutos de $f$ no conjunto $D.$

$f(x,y)=(2x-x^{2})(2y-y^{2})$, $D$ é a região do plano $xy$ dada por $0\leq y\leq 2(2x-x^{2})$.


Valor máximo: $1;$  valor mínimo: $0.$


2831   

Encontre o volume máximo de uma caixa retangular que está inscrita em uma esfera de raio $r.$


$\displaystyle \frac{8}{3\sqrt{3}} r^{3}.$


2106   

Um campo vetorial inverso do quadrado é da forma:

$${\bf F}({\bf r})=\frac{c{\bf r}}{|{\bf r}|^{3}}$$

para alguma constante $c$, onde ${\bf r}=x\,{\bf i}+y\,{\bf j}+z\,{\bf k}$. Um exemplo de um campo inverso do quadrado é o campo gravitacional ${\bf F}=-(mMG){\bf r}/|{\bf r}|^{3}$. Determine o trabalho realizado pelo campo gravitacional quando a Terra se move do afélio (em uma distância máxima de $1,52\times 10^{8}\,km$ do Sol) ao periélio (em uma distância mínima de $1,47\times 10^{8}\,km)$. (Use os valores $m=5,97\times 10^{24}\,kg$, $M=1,99\times 10^{30}\,kg$ e $G=6,67\times 10^{-11}\,N\cdot m^{2}/kg^{2}.$)


$\approx 1,77 \times 10^{35}$ J.


2359   

Seja $S$ a parte do cone $x^{2}=y^{2}+z^{2}$ que está dentro do cilindro $x^{2}+y^{2}=a^{2}$ e no primeiro octante. Determine a área da superfície $S.$


$\dfrac{\pi a^2}{4}$.


2275   

Calcule $\nabla f(x,y)$.
$f(x,y) = x^2y$


$\displaystyle \nabla f(x,y) = (2xy,x^{2}).$


2434   

Calcule a integral tripla.

  1.  $\displaystyle\iiint\limits_{  E}e^{x^{2}}\;dx dy dz$, onde $E$ é o conjunto $0\leq x \leq 1$, $0\leq y \leq x$ e $0\leq z \leq 1.$

  2.  $\displaystyle\iiint\limits_{  E}x\;dx dy dz$, onde $E$ é o conjunto $x^{2}\leq y\leq x$, $0\leq z\leq x+y.$


  1.  $\dfrac{e - 1}{2}.$

  2.  $\dfrac{11}{120}.$


3110   

A parte da superfície \[ z= \dfrac{h}{a}\sqrt{x^2+y^2}\quad\left(a,\ h>0\right) \] entre o plano \(xy\) e o plano \(z=h\) é um cone circular reto de altura \(h\) e raio \(a\). Use uma integral dupla para mostrar que a área da superfície lateral desse cone é dada por \(\displaystyle S=\pi a\sqrt{a^2+h^2}\).


2588   

Seja $C$ o cilindro de base circular e eixo $(Oz)$, com raio $2$ e altura $3$, com base na origem e densidade inversamente proporcional $\grave{a}$ distância ao eixo.

  1. Determine o momento de inércia de $C$ com relação ao eixo $(Oz)$.

  2. Se $C$ gira em torno do eixo $(Oz)$ com energia cinética $K$, qual a velocidade instantânea nos pontos de sua superfície lateral? (Fórmulas: $\bullet$ Momento de inércia: $I=\iiint\limits_{C}\rho\cdot l^{2}\,dV$, onde $\rho$ é a densidade e $l$  é a distância ao eixo; $\bullet$ Energia cinética de rotação: $K=\dfrac{1}{2}I\omega^{2}.$)


  1.  $6\pi.$

  2.  $\displaystyle \sqrt{\frac{K}{3\pi}}.$


2194   

Use o Teorema do Divergente para calcular $\displaystyle\iint \limits_{S}{\bf F}\cdot dS$, onde ${\bf F}(x,y,z)=z^{2}x\,{\bf i}+(\frac{1}{3}y^{3}+tg z)\,{\bf j}+(x^{2}z+y^{2})\,{\bf k}$ e $S$ é a metade de cima da esfera $x^{2}+y^{2}+z^{2}=1.$
[Sugestão: observe que $S$ não é uma superfície fechada. Calcule primeiro as integrais sobre $S_{1}$ e $S_{2}$, onde $S_{1}$ é o círculo $x^{2}+y^{2}\leq 1$, orientado para baixo, e $S_{2}=S\cup S_{1}.$]



Note que $\dfrac{\partial}{\partial x} \left( \dfrac{x}{|{\bf x}|^3} \right) = \dfrac{|{\bf x}|^2 - 3x^2}{|{\bf x}|^5},$ $\dfrac{\partial}{\partial y} \left( \dfrac{y}{|{\bf x}|^3} \right) = \dfrac{|{\bf x}|^2 - 3y^2}{|{\bf x}|^5}$ e $\dfrac{\partial}{\partial z} \left( \dfrac{x}{|{\bf x}|^3} \right) = \dfrac{|{\bf x}|^2 - 3z^2}{|{\bf x}|^5}.$


2591   

Seja ${\bf F}$ um campo inverso do quadrado, ou seja, ${\bf F}(r)=cr/|r|^{3}$ para alguma constante $c$, onde $r=x{\bf i}+y{\bf j}+z{\bf k}.$ Mostre que o fluxo de ${\bf F}$ por uma esfera $S$ com centro na origem é independente do raio de $S.$


$\displaystyle \iint\limits_{S}{\bf F}\cdot d \bf S = 4\pi c.$


2017   

Determine o volume do sólido limitado pelos planos coordenados e pelo plano $3x+2y+z=6.$



O sólido cujo volume deve ser calculado é $$E = \{(x,y,z) \in \mathbb{R}^3; (x,y) \in R \mbox{ e } 0 \leq z \leq 6 - 3x - 2y\},$$ em que $R$ é a projeção de $E$ no plano $xy$. Assim, o volume é dado por $$V = \displaystyle\int\!\!\!\!\int\limits_{R}(6-3x-2y)\,dA.$$ A região $R$ é tanto do tipo I como do tipo II, então é possível escrevê-la de pelo menos duas formas. Escrevendo como uma região do tipo I, obtemos: $$R = \left\{(x,y) \in \mathbb{R}^2: 0 \leq x \leq 2 \mbox{ e } 0 \leq y \leq \frac{6-3x}{2}\right\}.$$ Portanto, \begin{eqnarray*} V & = & \displaystyle\int_{0}^{2}\!\int_{0}^{\frac{6-3x}{2}}(6-3x-2y)\,dy dx \\   & = & \displaystyle\int_{0}^{2} \left.\left(6y-3xy-y^2 \right|_{y=0}^{y=\frac{6-3x}{2}} \right) \,dx \\     & = & \displaystyle\int_{0}^{2} \left(9-9x+\frac{9x^2}{4}\right) \,dx \\     & = & \left.9x-\frac{9x^2}{2}+\frac{9x^3}{12} \right|_{x=0}^{x=2} = 6. \end{eqnarray*} Observe que podemos escrever $R$ como uma região do tipo II, obtendo: $$R = \left\{(x,y) \in \mathbb{R}^2: 0 \leq x \leq \frac{6-2y}{3} \text{ e } 0 \leq y \leq 3\right\}.$$ Então, uma outra expressão para $V$ é $$V = \displaystyle\int_{0}^{3}\!\int_{0}^{\frac{6-2y}{3}}(6-3x-2y)\,dx dy = 6.$$


2288   

Determine uma representação paramétrica para a superfície descrita a seguir. A parte do hiperboloide $x^{2}+y^{2}-z^{2}=1$ que está à direita do plano $xz.$


$x =u,$ $z = v,$ $y = \sqrt{1 - u^2 + v^2}.$


2162   

Use o Teorema de Green para calcular a integral de linha ao longo da curva dada com orientação positiva. $\displaystyle\int_{C} \sin{y} \, dx + x\cos{y} \, dy$, $C$ é a elipse $x^2 + xy + y^2 = 1$.


$0.$


3027   

Esboce o sólido cujo volume é dado pela integral iterada

$$\int_{0}^{1}\!\!\int_{0}^{1-x}(1-x-y)\,dy dx.$$


A expressão 1 cossec x e o mesmo que


2509   

Ache $\displaystyle\iint \limits_{ S}{\bf F}\cdot {\bf n} dS$ se ${\bf n}$ é uma normal unitária superior de $S.$

  • ${\bf F}=x{\bf i}+y{\bf j}+z{\bf k}$; $S$ é o hemisfério superior de $x^{2}+y^{2}+z^{2}=a^{2}.$


$2\pi a^3.$


2372   

Seja
$$f(x,y) = x - y\sin{(\pi(x^2 + y^2))}.$$

  1. Calcule a derivada direcional de $f$ no ponto $(0,0)$ na direção de $\bf{v} = (1/2,\sqrt{3}/2)$.

  2. Em que direção a taxa de variação de $f$ no ponto $(0,0)$ é máxima? Qual é o valor da taxa máxima nesse ponto?


  1. $ \displaystyle \frac{1}{2}.$
  2.  Na direção do vetor $(1,0).$ O valor da taxa máxima é $1.$


2563   

Determine o maior conjunto no qual a função $G(x,y) = \ln{(x^2 + y^2 - 4)}$ é contínua.


$\left\lbrace (x,y);\;x^{2} + y^{2} > 4 \right\rbrace.$


2848   

Passe para coordenadas polares e calcule: $\displaystyle\iint\limits_{ D}xy\,dA$, onde $D$ é o disco com centro na origem e raio 3.


$0.$


2459   

Verifique que, para a função de produção de Cobb-Douglas

$$P(L,K)=1,01L^{0,75}K^{0,25}$$

discutida no Exemplo 3 da Seção 14.1 do Stewart, a produção dobrará se as quantidades de trabalho e a de capital investido forem dobradas. Determine se isto também é verdade para uma função de produção genérica

$$P(L,K)=bL^{\alpha}K^{1-\alpha}.$$


Sim.


2618   

Uma partícula se move ao longo de segmentos de reta da origem aos pontos $(1,0,0)$, $(1,2,1)$, $(0,2,1)$ e de volta para a origem sob a influência do campo de forças ${\bf F}(x,y,z) = z^2{\bf i} + 2xy{\bf j} + 4y^2{\bf k}.$ Encontre o trabalho feito.


$3$.


2734   

Determine as equações do plano tangente e da reta normal ao gráfico da função dada, no ponto dado. $f(x,y) = xy$ em $\left(\dfrac{1}{2}, \dfrac{1}{2}, f\left(\dfrac{1}{2}, \dfrac{1}{2}\right)\right)$.


Plano tangente: $4z = 2x + 2y - 1$\\

Reta normal: $(x,y,z) = \left(\frac{1}{2}, \frac{1}{2}, \frac{1}{4} \right) + \lambda \left(\frac{1}{2},\frac{1}{2},-1 \right)$.


2163   

Use o Teorema de Green para calcular a integral de linha ao longo da curva dada com orientação positiva. $\displaystyle\int_{C}\dfrac{-y}{x^2+y^2} \, dx + \dfrac{ x}{x^2+y^2} \, dy$, $C$ curva fechada, $C^1$ por partes, simples e fronteira de um conjunto $B$ cujo interior contém o círculo $x^2 + y^2 \leq 1$. (Sugestão: Aplique o Teorema de Green à região $K$ compreendida entre a curva $C$ e a circunferência.)



$2\pi.$


2138   

Seja $S$ o gráfico de $f(x,y)=x^{2}+y^{2}$, $x^{2}+y^{2}\leq 1$ e seja ${\bf n}$ a normal a $S$ com componete $z\leq 0$. Seja ${\bf F}(x,y,z)=x^{2}y\,{\bf i}-xy^{2}\,{\bf j}+{\bf k}$. Calcule $\iint \limits_{S}{\bf F}\cdot {\bf n}\, dS.$



Observe que $S$ não é uma superfície fechada (isto é, $S$ não é a fronteira de um sólido $E$). Para que possamos utilizar o Teorema do Divergente, vamos considerar a superfície $S_2$ constituída pelo parabolóide $S$ e pelo círculo $S_1$ dado por $x^2+y^2 \leq 1$ em $z=1$. Como $S_2$ é uma superfície fechada, usamos a escolha da normal ${\bf n_2}$ em $S_2$ que está apontando ``para fora". Sejam ${\bf n_1}$ a normal a $S_1$ (apontando para cima) e ${\bf n}$ a normal a $S$ (apontando para fora).

A expressão 1 cossec x e o mesmo que

Temos
$\displaystyle\iint\limits_{S_2}{\bf F}\cdot {\bf n_2}\,dS = \iint\limits_{S}{\bf F}\cdot {\bf n}\,dS + \iint \limits_{S_1}{\bf F}\cdot {\bf n_1}\,dS,$
isto é,
$\displaystyle\iint\limits_{S}{\bf F}\cdot {\bf n}\,dS = \iint\limits_{S_2}{\bf F}\cdot {\bf n_2}\,dS - \iint \limits_{ S_1}{\bf F}\cdot {\bf n_1}\,dS.$
Pelo Teorema do Divergente,
$$\iint\limits_{S_2}{\bf F}\cdot {\bf n_2}\,dS = \iiint\limits_{E}(2xy-2xy+0)\,dV = 0,$$
em que $E$ é o sólido que possui $S_2$ como fronteira.
Para determinar $\displaystyle\iint\limits_{S_1}{\bf F}\cdot {\bf n_1}\,dS$, devemos encontrar uma parametrização para $S_1$ e determinar o vetor normal ${\bf n_1}$. Considere a seguinte parametrização de $S_1$: $r(u,v) = (u,v,1)$, com $u^2+v^2 \leq 1$. Daí, $r_u(u,v) = (1,0,0)$ e $r_v(u,v) = (0,1,0)$. Logo, $r_u \times r_v = (0,0,1)$ é um vetor normal a $S_1$. Devemos tomar ${\bf n_1} = (0,0,1)$ para que aponte para cima. Então,
$\displaystyle\iint \limits_{S_1}{\bf F}\cdot {\bf n_1}\,dS = \iint\limits_{D}(u^2v,-uv^2,1)\cdot(0,0,1)\,dA,$
em que $D = \{(u,v) \in \mathbb{R}^2; u^2+v^2 \leq 1\}$. Portanto,
$\displaystyle\iint \limits_{S_1}{\bf F}\cdot {\bf n_1}\,dS =  \iint\limits_{D}1\,dA = A(D) = \pi,$
donde concluímos que
$\displaystyle\iint \limits_{S}{\bf F}\cdot {\bf n}\,dS = 0 - \pi = -\pi.$


2042   

Utilize a Regra da Cadeia para determinar as derivadas parciais indicadas.
$Y=w\tan^{-1}(uv)$, $u=r+s$,  $v=s+t$; $w=t+r$
$\dfrac{\partial Y}{\partial r}$, $\dfrac{\partial Y}{\partial s}$, $\dfrac{\partial Y}{\partial t}$ quando $r=1$, $s=0$, $t=1$.


$\displaystyle \frac{\partial Y}{\partial r} = 1 + \frac{\pi}{4}$ ,$\dfrac{\partial Y}{\partial s} = 2$, $\displaystyle \dfrac{\partial Y}{\partial t} = 1 + \frac{\pi}{4}.$


2005   

Se um arame com densidade linear $\rho(x,y)$ está sobre uma curva plana $C$, seus momentos de inércia em relação aos eixos $x$ e $y$ são definidos por

$$I_{x}=\int_{C}y^{2}\rho(x,y)\,ds        I_{y}=\int_{C}x^{2}\rho(x,y)\,ds.$$

Determine os momentos de inércia de um arame com o formato de um semicírculo $x^{2}+y^{2}=1$, $y\geq 0$, que é mais grosso perto da base do que perto do topo, se a função densidade linear em qualquer ponto for proporcional à sua distância à reta $y=1.$


$I_{x} = k\left(\dfrac{\pi}{2} - \dfrac{4}{3} \right)$ e $I_{y} = k\left(\dfrac{\pi}{2} - \dfrac{2}{3} \right).$


2137   

A função diferenciável $z=z(x,y)$ é dada implicitamente pela equação  $f\bigg(\dfrac{x}{y},z\bigg)=0$, onde 
$f(u,v)$ é suposta diferenciável e $\dfrac{\partial f}{\partial v}(u,v)\neq 0$. Verifique que 
$$x\frac{\partial z}{\partial x}+y\dfrac{\partial z}{\partial y}=0.$$



Note que $\displaystyle \frac{\partial z}{\partial x} =  - \frac{1}{y} \frac{\partial f}{\partial u} \left(\frac{x}{y},z \right)\left(\frac{\partial f}{\partial v}\left(\frac{x}{y},z \right)\right)^{-1}$  e $\displaystyle \frac{\partial z}{\partial y} =  \frac{x}{y^{2}} \frac{\partial f}{\partial u} \left(\frac{x}{y},z \right)\left(\frac{\partial f}{\partial v}\left(\frac{x}{y},z \right)\right)^{-1}$.


2103   

Mostre que, se um campo vetorial ${\bf F}=P\,{\bf i}+Q\,{\bf j}+R\,{\bf k}$ é conservativo e $P$, $Q$, $R$ têm derivadas parciais de primeira ordem contínuas, então

$$\frac{\partial P}{\partial y}=\frac{\partial Q}{\partial x}, \,\,\,\,\,\frac{\partial P}{\partial z}=\frac{\partial R}{\partial x},\,\,\,\,\,\frac{\partial Q}{\partial z}=\frac{\partial R}{\partial y}$$


Se $f$ é uma função potencial de $\mathbf{F},$ então $f_{x} = P,$ $f_{y} = Q$ e $f_{z} = R.$ Como $P, Q$ e $R$ possuem derivadas parciais de primeira ordem contínuas, então pelo Teorema de Clairaut, temos $f_{xy} = f_{yx},$ $f_{yz} = f_{zy}$ e $f_{xz} = f_{zx}.$


2412   

Determine o volume do sólido que se encontra abaixo do plano $3x+2y+z=12$ e acima do retângulo $R=\{(x,y) \in \mathbb{R}^2|\;0\leq x\leq 1,\;-2\leq y\leq 3\}.$


$\dfrac{95}{2}.$


2290   

Determine uma representação paramétrica para a superfície descrita a seguir. A parte da esfera $x^{2}+y^{2}+z^{2}=4$ que está acima do cone $z=\sqrt{x^{2}+y^{2}}.$


$x = 2\sin(\phi)\cos(\theta),$ $y = 2\sin(\phi)\sin(\theta),$ $z = 2\cos(\phi),$ onde $0\leq \phi \leq \frac{\pi}{4}$ e $0 \leq \theta \leq 2\pi.$


2546   

Determine uma fórmula para $\displaystyle\iint \limits_{ S}{\bf F}\cdot d{\bf S}$ semelhante à fórmula

$\displaystyle\iint\limits_{S}{\bf F}\cdot d{\bf S}=\displaystyle\iint\limits_{D}\left(-P\dfrac{\partial f}{\partial x}-Q\dfrac{\partial f}{\partial y}+R\right)dA$ para o caso onde $S$ é dada por $y=h(x,z)$ e ${\bf n}$ é o vetor normal unitário que aponta para a esquerda.


$\displaystyle \iint\limits_{S}{\bf F}\cdot d{\bf S}=\iint\limits_{D}\left(P -Q\dfrac{\partial k}{\partial y}-R\frac{\partial k}{\partial z} \right)dA.$


2873   

Utilize os multiplicadores de Lagrange para determinar os valores máximo e mínimo da função sujeita à(s) restrição(ões) dada(s).

$f(x,y,z) = 2x + 6y + 10z; \quad x^2 + y^2 + z^2 = 35.$  


Valor máximo: $70;$ valor mínimo: $-70.$


2842   

Passe para coordenadas polares e calcule: $\displaystyle\int_{0}^{1}  \int_{0}^{\sqrt{x-x^{2}}}x\,dy dx$


$\displaystyle \frac{\pi}{16}.$


3067   

Determine o trabalho $W=\int_{C}{\bf F}\cdot d{\bf r}$ realizado pelo campo de força ${\bf F}(x,y)=x\,{\bf i}+(x^{3}+3xy^{2})\,{\bf j}$ em uma partícula que inicialmente está no ponto $(-2,0)$, se move ao longo do eixo $x$ para $(2,0)$ e ao longo da semicircunferência $y=\sqrt{4-x^{2}}$ até o ponto inicial.


2120   

Seja $g(t)=f(3t,2t^{2}-1).$

  1. Expresse $g^{'}(t)$ em termos das derivadas parciais de $f$.
  2. Calcule $g^{'}(0)$ admitindo $\dfrac{\partial f}{\partial x}(0,-1)=\dfrac{1}{3}.$


  1. $\displaystyle g'(t) =  3\frac{\partial f}{\partial x}(3t,2t^{2} - 1) + 4t \frac{\partial f}{\partial y}(3t,2t^{2} - 1).$
  2. $g'(0) = 1.$


2994   

Calcule a integral, efetuando uma mudança de variáveis apropriada. $\displaystyle\iint\limits_{R} x \, dA$, em que $R$ é o círculo $x^2 + y^2 - x \leq 0$.


$\dfrac{\pi}{8}.$


2444   

Calcule a integral de superfície $\displaystyle\iint\limits_{S}x dS$, onde $S$ é a superfície com equações paramétricas $x=u$, $y=v$, $z=u^{2}+v$, $0 \leq u \leq 1$, $u^{2} \leq v \leq 1.$


$\dfrac{\sqrt{2}}{10}(3\sqrt{3} - 2).$


2851   

Passe para coordenadas polares e calcule: $\displaystyle\iint\limits_{R}x\,dx dy$, onde $R$ é a região, no plano $xy$, limitada pela curva (dada em coordenadas polares) $\rho=\cos(3\theta)$, $-\dfrac{\pi}{6}\leq \theta \leq \dfrac{\pi}{6}.$


$\displaystyle \frac{81\sqrt{3}}{320}.$


1998   

Determine o trabalho $W=\int_{C}{\bf F}\cdot d{\bf r}$ realizado pelo campo de força ${\bf F}(x,y)=x^{2}(x-y)\,{\bf i}+xy^{2}\,{\bf j}$ em uma partícula que se move da origem ao longo do eixo $x$ para $(1,0)$, em seguida ao longo de um segmento de arco de circunferência $x^{2}+y^{2}=1$ até $(0,1)$ e então volta à origem ao longo do eixo $y.$


$\dfrac{\pi}{8}.$


2428   

Calcule a integral tripla.

  1.  $\displaystyle\iiint\limits_{  E}z\,dV$, onde $E$ é limitado pelo cilindro $y^{2}+z^{2}=9$ e pelos planos $x=0$, $y=3x$ e $z=0$ no primeiro octante.

  2.  $\displaystyle\iiint\limits_{  E}xyz\;dx dy dz$, onde $E$ é o paralelepípedo $0\leq x\leq 2$, $0\leq y\leq 1$, e $1\leq z\leq 2.$


  1.  $\dfrac{27}{8}.$

  2.  $\dfrac{3}{2}.$


2835   

Suponha que um cientista tenha razões para acreditar que duas quantidades $x$ e $y$ estejam relacionadas linearmente, ou seja, $y=mx+b$, pelo menos aproximadamente, para algum valor de $m$ e de $b$. O cientista realiza uma experiência e coleta os dados na forma de pontos $(x_{1},y_{1}), (x_{2},y_{2}), \ldots, (x_{n},y_{n})$, e então coloca-os em um gráfico. Os pontos não estão todos alinhados, de modo que o cientista quer determinar as constantes $m$ e $b$ para que a reta $y=mx+b$ ``ajuste" os pontos tanto quanto possível (veja a figura). Seja $d_{i}=y_{i}-(mx_{i}+b)$ o desvio vertical do ponto $(x_{i},y_{i})$ da reta. O {\bf método dos mínimos quadrados} determina $m$ e $b$ de modo a minimizar $\sum_{i=1}^{n}d_{i}^{2}$, a soma dos quadrados dos desvios. Mostre que, de acordo com esse método, a reta de melhor ajuste é obtida quando

$$m\sum_{i=1}^{n}x_{i}+bn=\sum_{i=1}^{n}y_{i}$$

$$m\sum_{i=1}^{n}x_{i}^{2}+b\sum_{i=1}^{n}x_{i}=\sum_{i=1}^{n}x_{i}y_{i}$$

Assim, a reta é determinada resolvendo esse sistema linear de duas equações nas incógnitas $m$ e $b.$

A expressão 1 cossec x e o mesmo que


As duas equações são obtidas como pontos críticos da função $\displaystyle \sum^{n}_{i = 1} d_{i}^{2} = \sum^{n}_{i = 1} \left(y_{i} - (mx_{i} + b) \right)^{2} = f(m,b).$ Note que de fato pontos satisfazendo as equações são pontos de mínimo de $f.$


2612   

Use o Teorema de Stokes para calcular $\displaystyle\int_C {\bf F}\cdot d{\bf R}$. Em cada caso, $C$ é orientada no sentido anti-horário quando vista de cima.

  • ${\bf F}(x,y,z) = x^2z{\bf i} + xy^2{\bf j} + z^2{\bf k}$, $C$ é a curva de interseção do plano $x+y+z=1$ com o cilindro $x^2+y^2 = 9$.


$\dfrac{81\pi}{2}.$


2610   

Use o Teorema de Stokes para calcular $\displaystyle\int_C {\bf F}\cdot  d{\bf R}$. $C$ é orientada no sentido anti-horário quando vista de cima.

  • ${\bf F}(x,y,z) = xy{\bf i} + 2z{\bf j} + 3y{\bf k}$, $C$ é a curva de interseção do plano $x+z=5$ com o cilindro $x^2+y^2=9$.


2018   

  1.  Estime o volume do sólido que está abaixo da superfície $z = x + 2y^2$ e acima do retângulo $R = [0,2] \times [0,4]$. Use a soma de Riemann com $m = n = 2$ e escolha os pontos amostrais como os cantos inferiores direitos.

  2.  Use a Regra do Ponto Médio para dar uma estimativa da integral do item anterior.


  1.  $\approx 44.$

  2.  $\approx 88.$


2628   

Utilizando o Teorema de Stokes, transforme a integral $\displaystyle\iint_{ S}\mbox{rot}{\bf F}\cdot{\bf n}dS$ numa integral de linha e calcule.

  • ${\bf F}(x,y,z) = y{\bf i}$, $S$ a superfície $x^2+y^2+z^2 = 2$, $x^2+y^2\leq 1$ e $z \geq 0$, sendo ${\bf n}$ a normal apontando para cima.


$-\pi$.


2949   

Calcule utilizando coordenadas esféricas. $\displaystyle\iiint\limits_{B}\sqrt{x^{2}+y^{2}+z^{2}}\,dxdydz$, onde $B$ é a interseção da semi-esfera  $x^{2}+y^{2}+z^{2}\leq 4$, $z\geq 0$, com o cilindro $x^{2}+y^{2}\leq 1.$


$\displaystyle \dfrac{\pi}{4}\left( 32- 14\sqrt{3} + \ln(2 + \sqrt{3})\right).$


2898   

Determine os pontos da elipse $\mathcal{D} = \left\{ (x,y) \in \mathbb{R}^2: \frac{x^2}{8} + \frac{y^2}{2} = 1 \right\}$ que fornecem o maior e o menor valor da função $f(x,y) = xy$.


Pontos de máximo: $(2,1)$ e $(-2,-1);$ pontos de mínimo: $(-2,1)$ e $(2,-1).$


3101   

Seja \(R\) a região triangular de vértices \((0,0)\), \((3,3)\) e \((0,4)\) do plano \(xy\). Expressa como uma integral dupla, qual é área de \(R\)?


\(\displaystyle A(R)=\int_0^3\int_x^{-\frac{1}{3}x+4}\,dydx \)


1955   

Calcule a integral de linha $\int_{C}{\bf F}\cdot d{\bf r}$, onde $C$ é dada pela função vetorial ${\bf r}(t).$

${\bf F}(x,y)=x^{2}\,{\bf i}+(x-y)\,{\bf j}$, ${\bf r}(t)=(t,\sin t)$, $0\leq t\leq \pi.$


$\displaystyle \frac{\pi^{3}}{3} - 2.$


2667   

Seja $\phi:\mathbb{R}\rightarrow \mathbb{R}$ uma função de uma variável real, diferenciável e tal que $\phi '(1)=4.$ Seja $g(x,y)=\phi\bigg(\dfrac{x}{y}\bigg).$ Calcule

  1. $\dfrac{\partial g}{\partial x}(1,1)$.

  2. $\dfrac{\partial g}{\partial y}(1,1)$.


  1. $\displaystyle \frac{\partial g}{\partial x} = \frac{1}{y} \phi'\left( \frac{x}{y} \right)$

  2. $\displaystyle \frac{\partial g}{\partial y} = -\frac{x}{y^{2}} \phi' \left( \frac{x}{y} \right).$


2869   

Estude com relação a máximos e mínimos a função dada com as restrições dadas.

$f(x,y) = x^2 - 2xy + 3y^2$ e $x^2 + 2y^2 = 1.$


Pontos de máximo: $\displaystyle \left( \frac{1}{\sqrt{3}}, -\frac{1}{\sqrt{3}} \right)$ e $\displaystyle \left( -\frac{1}{\sqrt{3}}, -\frac{1}{\sqrt{3}} \right)$; pontos de mínimo:  $\displaystyle \left( \frac{2}{\sqrt{6}}, \frac{1}{\sqrt{6}} \right)$ e $\displaystyle \left( -\frac{2}{\sqrt{6}}, -\frac{1}{\sqrt{6}} \right)$.


1932   

Calcule a integral de linha, onde $C$ é a curva dada.

$\displaystyle\int_{C}x\,\sin{y}\,ds$,   $C$ é o segmento de reta que liga $(0,3)$ a $(4,6).$


$\displaystyle \frac{20}{6} \left(\sin(6) - 3\cos(6) - \sin(3) \right).$


2200   

Suponha que substituamos coordenadas polares $x=r\cos{\theta}$ e $y=r\sin{\theta}$ em uma função diferenciável $w=f(x,y).$

  1.  Mostre que $$\frac{\partial w}{\partial r}=f_{x}\cos{\theta}+f_{y}\sin{\theta}$$ e $$\frac{1}{r}\frac{\partial w}{\partial \theta}=-f_{x}\sin{\theta}+f_{y}\cos{\theta}.$$
  2.  Resolva as equações no item 1. para expressar $f_{x}$ e $f_{y}$ em termos de $\partial w/ \partial r$ e $\partial w/\partial \theta$.
  3.  Mostre que  $$(f_{x})^{2}+(f_{y})^{2}=\bigg(\frac{\partial w}{\partial r}\bigg)^{2}+\frac{1}{r^{2}}\bigg(\frac{\partial w}{\partial \theta}\bigg)^{2}.$$



  1.  $\displaystyle f_{x} = \cos(\theta) \frac{\partial w}{\partial r} - \frac{\sin (\theta)}{r} \frac{\partial w}{\partial \theta}$ e $\displaystyle f_{y} = \sin(\theta) \frac{\partial w}{\partial r} + \frac{\cos (\theta)}{r} \frac{\partial w}{\partial \theta}.$


2479   

Esboce o gráfico da função $f(x,y)=3$.


$z = 3.$

A expressão 1 cossec x e o mesmo que


2342   

Determine a área da superfície dada pela porção do cilindro $x^{2}+y^{2}=1$ entre os planos $z=1$ e $z=4.$


$6\pi.$


2396   

Mostre que a equação do plano tangente ao elipsoide $x^2/a^2 + y^2/b^2 + z^2/c^2 = 1$ no ponto $(x_0,y_0,z_0)$ pode ser escrita como
$$\dfrac{xx_0}{a^2} + \dfrac{yy_0}{b^2} + \dfrac{zz_0}{c^2} = 1.$$



 Note que se $F(x,y,z) = x^2/a^2 + y^2/b^2 + z^2/c^2 - 1,$ então 
$$
\nabla F(x_{0},y_{0},z_{0}) = 2 \left(\frac{x_{0}}{a^{2}},\frac{y_{0}}{b^{2}},\frac{z_{0}}{c^{2}} \right)
$$
e a equação do plano tangente em $(x_{0},y_{0},z_{0})$ é 
$$
\nabla F(x_{0},y_{0},z_{0}) \cdot (x,y,z) = \nabla F(x_{0},y_{0},z_{0}) \cdot (x_{0},y_{0},z_{0}) = 2.
$$


2260   

Escreva a integral dupla $$\iint\limits_{R}x\cos{y}\;dA,$$ onde $R$ é limitada pelas retas $y=0$, $x=\pi/4$ e $y=x$, das duas formas possíveis (mudando a ordem de integração). Escolha uma dessas formas e calcule o valor dessa integral.


$\displaystyle \int_{0}^{\pi/4} \int_{0}^{x} x \cos(y)\;dy\;dx = \int_{0}^{\pi/4} \int_{y}^{\pi / 4} x \cos(y)\;dx\;dy = -\frac{\pi - 4}{4\sqrt{2}}.$


2894   

Passe para coordenadas polares e calcule: $\displaystyle\iint\limits_{R}\arctan\left(\dfrac{y}{x}\right)\,dA$, onde $R=\{(x,y) \in \mathbb{R}^2| 1\leq x^{2}+y^{2}\leq 4, 0\leq y\leq x\}.$


$\displaystyle \frac{3\pi^2}{64}.$


2768   

De acordo com o triângulo abaixo:

A expressão 1 cossec x e o mesmo que

  1. Expresse $A$ implicitamente como uma função de $a$, $b$ e $c$ e calcule $\partial A/\partial a$ e $\partial A/ \partial b.$

  2. Expresse $a$ implicitamente como uma função de $A$, $b$ e $B$ e calcule $\partial a/ \partial A$ e $\partial a/ \partial B.$


  1. $\displaystyle a^{2} = b^{2} + c^{2} -2bc\cos(A),\;\;\;\;\frac{\partial A}{\partial a} = \frac{a}{bc \sin (A)}\;\;\;\text{e}\;\;\;\frac{\partial A}{\partial b} = \frac{c \cos(A) - b}{bc \sin(A)}.$

  2. $\displaystyle \frac{a}{\sin(A)} = \frac{b}{\sin(B)},\;\;\;\;\frac{\partial a}{\partial A} = \frac{a\cos(A)}{\sin(A)}\;\;\;\text{e}\;\;\;\frac{\partial a}{\partial B} = - b\csc(B) \cot(B)\sin(A).$


2877   

Embora $\nabla f = \lambda \nabla g$ seja uma condição necessária para a ocorrência de um valor extremo de $f(x,y)$ sujeito à restrição $g(x,y) = 0$, ela não garante por si só que ele exista. Como um exemplo, tente usar o método dos multiplicadores de Lagrange para encontrar um valor máximo de $f(x,y) = x + y$ sujeito à restrição $xy = 16$. O método identificará os dois pontos $(4,4)$ e $(-4,-4)$ como candidatos para a localização dos valores extremos. Ainda assim, a soma $x + y$ não tem valor máximo sobre a hipérbole. Quanto mais distante você está da origem nessa hipérbole no primeiro quadrante, maior se torna a soma $f(x,y) = x + y$.


Note que quando $x \to 0,$ tem-se $y \to \infty$ e $f(x,y) \to \infty;$ e quando $x \to -\infty,$ tem-se $y \to 0$ e $f(x,y) \to -\infty,$ logo não há valores máximo e mínimo de $f$ sujeito a esta restrição.


1985   

 Determine o vetor tangente unitário ${\bf T}(t)$ no ponto com valor de parâmetro $t$ dado, sendo 
${\bf r}(t)=\cos(t){\bf i}+3t{\bf j}+2\sin(2t){\bf k}$ e $t=0.$


3035   

Uma região $R$ é mostrada na figura abaixo. Decida se você deve usar coordenadas polares ou retangulares e escreva $\iint\limits_{R}f(x,y)\,dA$ como uma integral iterada, onde $f$ é uma função qualquer contínua em $R.$


A expressão 1 cossec x e o mesmo que


$\displaystyle \int_{-1}^{1} \int_{0}^{1 - x^2} f(x,y)  dy dx .$


2009   

Determine o trabalho realizado pelo campo de força ${\bf F}(x,y,z)=(y+z)\,{\bf i}+(x+z)\,{\bf j}+(x+y)\,{\bf k}$ sobre uma partícula que se move ao longo do segmento de reta $(1,0,0)$ a $(3,4,2).$


$26.$


2520   

Dada a expressão $g(x,y)=2-f(x,y)$, escreva como o gráfico de $g$ é obtido a partir do gráfico de $f.$


Gráfico de $f$ refletido sobre o plano $xy$ e deslocado para cima por duas unidades.


2302   

Identifique e faça um esboço da imagem da superfície parametrizada dada por ${\bf r}(u,v)=(u,\sqrt{1-u^{2}-v^{2}},v)$, $u^{2}+v^{2}\leq 1.$


Semi superfície esférica $x^2 + y^2 + z^2 = 1,$ $y  \geq 0.$


2002   

Uma partícula desloca-se em um campo de forças dado por ${\bf F}(x,y,z)=-y\,{\bf i}+x\,{\bf j}+z\,{\bf k}.$ Calcule o trabalho realizado por ${\bf F}$ no deslocamento da partícula de ${\bf r}(a)$ até ${\bf r}(b)$, sendo dados:

  1. ${\bf r}(t)=(\cos t, \sin t,t)$, $a=0$ e $b=2\pi.$

  2. ${\bf r}(t)=(2t+1,t-1,t)$, $a=1$ e $b=2.$

  3. ${\bf r}(t)=(\cos t,0, \sin t)$, $a=0$ e $b=2\pi.$


  1. $2\pi(1 + \pi).$

  2. $\dfrac{9}{2}.$

  3. $0.$


2043   

Aplique o Teorema da Divergência para achar $\displaystyle\iint\limits_{S}{\bf F}\cdot {\bf n}\,dS.$, sendo ${\bf F}(x,y,z)=y^{3}e^{z}\,{\bf i}-xy\,{\bf j}+x \cdot \arctan y\,{\bf k}$ e $S$ a superfície da região delimitada pelos planos coordenados e o plano $x+y+z=1.$



Pelo Teorema do Divergente, temos
$$\iint\limits_{S}{\bf F}\cdot {\bf n}\,dS = \displaystyle\iiint\limits_{E}\text{div }{\bf F}\,dV,$$
em que $E$ é o sólido

A expressão 1 cossec x e o mesmo que

que pode ser escrito como
$E = \{(x,y,z) \in \mathbb{R}^3: 0 \leq x \leq 1, 0 \leq y \leq 1-x \mbox{ e } 0 \leq z \leq 1-x-y\}.$
Observe que
\begin{array}{rcl}\text{div }{\bf F} & = & \dfrac{\partial}{\partial x}(y^3e^z) + \dfrac{\partial}{\partial y}(-xy) + \dfrac{\partial}{\partial z}(x\arctan{y}) \\& = & 0 - x + 0 \\& = & -x.\end{array}
Assim,
\begin{array}{rcl}\iint\limits_{S}{\bf F}\cdot {\bf n}\,dS & = & \displaystyle\iiint\limits_{E}{\bf F}\,dV \\& = & \iiint\limits_{E}-x\,dV \\& = & \int_{0}^{1}\int_{0}^{1-x}\int_{0}^{1-x-y}-x\,dz dy dx \\& = & \int_{0}^{1}\int_{0}^{1-x}-x(1-x-y)\,dy dx \\& = & \int_{0}^{1}\left(-\frac{x}{2}+x^2-\frac{x^3}{3}\right)\,dx \\& = & -\frac{1}{12}.\end{array}


2125   

Admita que, para todo $(x,y)$, 

$$4y\frac{\partial f}{\partial x}(x,y)-x\frac{\partial f}{\partial y}(x,y)=2.$$

Calcule $g^{'}(t)$, sendo $g(t)=f(2\cos{t},\sin{t})$.


$g^{'}(t) = -1.$